Как ΠΏΠ΅Ρ€Π΅ΠΏΡ€ΠΎΠΉΡ‚ΠΈ мСдкомиссию Π² Π²ΠΎΠ΅Π½ΠΊΠΎΠΌΠ°Ρ‚Π΅: Как ΠΏΠΎΠΌΠ΅Π½ΡΡ‚ΡŒ ΠΊΠ°Ρ‚Π΅Π³ΠΎΡ€ΠΈΡŽ годности Π² Π²ΠΎΠ΅Π½Π½ΠΎΠΌ Π±ΠΈΠ»Π΅Ρ‚Π΅

Π‘ΠΎΠ΄Π΅Ρ€ΠΆΠ°Π½ΠΈΠ΅

Как ΠΏΠΎΠΌΠ΅Π½ΡΡ‚ΡŒ ΠΊΠ°Ρ‚Π΅Π³ΠΎΡ€ΠΈΡŽ годности Π² Π²ΠΎΠ΅Π½Π½ΠΎΠΌ Π±ΠΈΠ»Π΅Ρ‚Π΅

На протяТСнии Π½Π΅ΡΠΊΠΎΠ»ΡŒΠΊΠΈΡ… послСдних Π»Π΅Ρ‚ руководство страны ΠΏΡ€ΠΈΠΊΠ»Π°Π΄Ρ‹Π²Π°Π΅Ρ‚ ΠΎΠ³Ρ€ΠΎΠΌΠ½Ρ‹Π΅ усилия для ΠΏΠΎΠ²Ρ‹ΡˆΠ΅Π½ΠΈΡ прСстиТности армСйской слуТбы, Π² особСнности это касаСтся слуТбы ΠΏΠΎ ΠΊΠΎΠ½Ρ‚Ρ€Π°ΠΊΡ‚Ρƒ. Π’ Ρ†Π΅Π»ΠΎΠΌ сущСствСнно возросла Π·Π°Ρ€ΠΏΠ»Π°Ρ‚Π° воСннослуТащих, ΡƒΠ»ΡƒΡ‡ΡˆΠΈΠ»ΠΈΡΡŒ условия слуТбы, Π° Ρ‚Π°ΠΊΠΆΠ΅ ΠΏΠΎΠ²Ρ‹ΡΠΈΠ»ΠΎΡΡŒ финансированиС всСй Π²ΠΎΠ΅Π½Π½ΠΎΠΉ отрасли государства. Π’ Ρ‚Π°ΠΊΠΈΡ… условиях срСди Π³Ρ€Π°ΠΆΠ΄Π°Π½ Π½Π°Ρ‡Π°Π»Π° Π½Π°Π±Π»ΡŽΠ΄Π°Ρ‚ΡŒΡΡ тСндСнция, ΠΊΠΎΠ³Π΄Π° всС большС людСй Ρ…ΠΎΡ‡Π΅Ρ‚ ΠΏΠΎΡΠ²ΡΡ‚ΠΈΡ‚ΡŒ свою Тизнь Π°Ρ€ΠΌΠΈΠΈ. ΠŸΠΎΠΏΠ°ΡΡ‚ΡŒ Π½Π° ΠΊΠΎΠ½Ρ‚Ρ€Π°ΠΊΡ‚Π½ΡƒΡŽ слуТбу достаточно ΠΏΡ€ΠΎΠ±Π»Π΅ΠΌΠ°Ρ‚ΠΈΡ‡Π½ΠΎ, вСдь всС ΠΊΠ°Π½Π΄ΠΈΠ΄Π°Ρ‚Ρ‹ проходят ΡƒΠ³Π»ΡƒΠ±Π»Π΅Π½Π½ΠΎΠ΅ мСдицинскоС обслСдованиС, Π° Ρ‚Π°ΠΊΠΆΠ΅ физичСскиС испытания. Π˜ΡΡ…ΠΎΠ΄Ρ ΠΈΠ· Π΄Π°Π½Π½Ρ‹Ρ… Ρ„Π°ΠΊΡ‚ΠΎΠ², ΠΌΡƒΠΆΡ‡ΠΈΠ½Π΅ ΠΏΠΎΠΏΠ°ΡΡ‚ΡŒ Π² Π°Ρ€ΠΌΠΈΡŽ, имСя ΠΊΠ°Ρ‚Π΅Π³ΠΎΡ€ΠΈΡŽ годности Π’, Π±ΡƒΠ΄Π΅Ρ‚ просто Π½Π΅Ρ€Π΅Π°Π»ΡŒΠ½ΠΎ.

ΠžΠ΄Π½ΠΎΠ²Ρ€Π΅ΠΌΠ΅Π½Π½ΠΎ с этим стоит ΠΎΡ‚ΠΌΠ΅Ρ‚ΠΈΡ‚ Ρ‚ΠΎΡ‚ Ρ„Π°ΠΊΡ‚, Ρ‡Ρ‚ΠΎ для слуТбы Π² Π±ΠΎΠ»ΡŒΡˆΠΈΠ½ΡΡ‚Π²Π΅ государствСнных ΠΎΡ€Π³Π°Π½ΠΎΠ² трСбуСтся Π½Π°Π»ΠΈΡ‡ΠΈΠ΅ Π²ΠΎΠ΅Π½Π½ΠΎΠ³ΠΎ Π±ΠΈΠ»Π΅Ρ‚Π° с ΠΎΡ‚ΠΌΠ΅Ρ‚ΠΊΠΎΠΉ ΠΎ ΠΏΡ€ΠΎΡ…ΠΎΠΆΠ΄Π΅Π½ΠΈΠΈ срочной армСйской слуТбы. Π’Π°ΠΊΠΈΠ΅ Ρ„Π°ΠΊΡ‚ΠΎΡ€Ρ‹ Π²Ρ‹Π½ΡƒΠΆΠ΄Π°ΡŽΡ‚ ΠΌΠΎΠ»ΠΎΠ΄Ρ‹Ρ… людСй с ΠΊΠ°Ρ‚Π΅Π³ΠΎΡ€ΠΈΠ΅ΠΉ Π’, ΠΈΠΌΠ΅ΡŽΡ‰ΠΈΡ… Π°ΠΌΠ±ΠΈΡ†ΠΈΠΈ ΠΏΠΎΡΡ‚Ρ€ΠΎΠΈΡ‚ΡŒ ΠΊΠ°Ρ€ΡŒΠ΅Ρ€Ρƒ Π² государствСнных силовых структурах, ΠΈΡΠΊΠ°Ρ‚ΡŒ ΠΏΡƒΡ‚ΠΈ, ΠΊΠ°ΠΊ ΠΏΠΎΠΏΠ°ΡΡ‚ΡŒ Π² Π°Ρ€ΠΌΠΈΡŽ. Для Ρ‚ΠΎΠ³ΠΎ, Ρ‡Ρ‚ΠΎΠ± это ΠΎΡΡƒΡ‰Π΅ΡΡ‚Π²ΠΈΡ‚ΡŒ придСтся ΠΏΡ€ΠΎΠΉΡ‚ΠΈ слоТный ΠΏΡƒΡ‚ΡŒ ΠΏΠΎ измСнСнию ΠΊΠ°Ρ‚Π΅Π³ΠΎΡ€ΠΈΠΈ годности Π² Π²ΠΎΠ΅Π½Π½ΠΎΠΌ Π±ΠΈΠ»Π΅Ρ‚Π΅.

ΠšΡ€ΠΎΠΌΠ΅ Ρ‚ΠΎΠ³ΠΎ, Π±Ρ‹Π²Π°ΡŽΡ‚ ситуации, ΠΊΠΎΠ³Π΄Π° ΠΏΠ°Ρ€Π΅Π½ΡŒ ΠΏΠΎΠ»ΡƒΡ‡Π°Π΅Ρ‚ ΠΊΠ°Ρ‚Π΅Π³ΠΎΡ€ΠΈΡŽ Π’ ΠΏΠΎ ΠΊΠ°ΠΊΠΎΠΉ-Ρ‚ΠΎ ΠΏΠ°Ρ‚ΠΎΠ»ΠΎΠ³ΠΈΠΈ, ΠΎ ΠΊΠΎΡ‚ΠΎΡ€ΠΎΠΉ ΠΎΠ½ Π½Π΅ Ρ…ΠΎΡ‚Π΅Π» Π±Ρ‹ Ρ€Π°ΡΠΏΡ€ΠΎΡΡ‚Ρ€Π°Π½ΡΡ‚ΡŒΡΡ. Π­Ρ‚ΠΎ Π·Π°Ρ‡Π°ΡΡ‚ΡƒΡŽ касаСтся Π·Π°Π±ΠΎΠ»Π΅Π²Π°Π½ΠΈΠΉ психики ΠΈΠ»ΠΈ наркотичСской зависимости. Π‘Ρ‚ΠΎΠΈΡ‚ ΠΏΠΎΠΌΠ½ΠΈΡ‚ΡŒ ΠΎ Ρ‚ΠΎΠΌ, Ρ‡Ρ‚ΠΎ Ρ‡Π΅Π»ΠΎΠ²Π΅ΠΊ ΠΏΠΎΠ»ΡƒΡ‡ΠΈΠ²ΡˆΠΈΠΉ Π²ΠΎΠ»ΡŒΠ½ΡƒΡŽ ΠΎΡ‚ Π°Ρ€ΠΌΠΈΠΈ с ΠΏΠΎΠΌΠΎΡ‰ΡŒΡŽ Ρ‚Π°ΠΊΠΈΡ… Π΄ΠΈΠ°Π³Π½ΠΎΠ·ΠΎΠ² Π² Π±ΡƒΠ΄ΡƒΡ‰Π΅ΠΌ Π½Π°Ρ‡ΠΈΠ½Π°Π΅Ρ‚ ΠΈΡΠΏΡ‹Ρ‚Ρ‹Π²Π°Ρ‚ΡŒ ΡΠ΅Ρ€ΡŒΠ΅Π·Π½Ρ‹Π΅ трудности ΠΊΠ°ΠΊ с трудоустройством, Ρ‚Π°ΠΊ ΠΈ с ΠΏΠΎΠ»ΡƒΡ‡Π΅Π½ΠΈΠ΅ΠΌ Ρ‚Π΅Ρ… ΠΆΠ΅ Π²ΠΎΠ΄ΠΈΡ‚Π΅Π»ΡŒΡΠΊΠΈΡ… ΠΏΡ€Π°Π². Π­Ρ‚ΠΎ всС ΠΏΡ€ΠΈΠ²ΠΎΠ΄ΠΈΡ‚ ΠΊ сущСствСнному ΡƒΡ…ΡƒΠ΄ΡˆΠ΅Π½ΠΈΡŽ ΠΌΠ°Ρ‚Π΅Ρ€ΠΈΠ°Π»ΡŒΠ½ΠΎΠ³ΠΎ полоТСния ΠΌΡƒΠΆΡ‡ΠΈΠ½Ρ‹, Ρ‡Ρ‚ΠΎ Π½Π΅Π³Π°Ρ‚ΠΈΠ²Π½ΠΎ влияСт Π½Π° всю Π΅Π³ΠΎ Тизнь. По этой ΠΏΡ€ΠΈΡ‡ΠΈΠ½Π΅ ΠΌΠΎΠ»ΠΎΠ΄ΠΎΠΉ Ρ‡Π΅Π»ΠΎΠ²Π΅ΠΊ ΠΌΠΎΠΆΠ΅Ρ‚ ΠΈΠΌΠ΅Ρ‚ΡŒ вСскиС основания для Ρ‚ΠΎΠ³ΠΎ, Ρ‡Ρ‚ΠΎΠ± ΠΏΠΎΠΌΠ΅Π½ΡΡ‚ΡŒ ΠΊΠ°Ρ‚Π΅Π³ΠΎΡ€ΠΈΡŽ годности Π² своСм Π΄ΠΎΠΊΡƒΠΌΠ΅Π½Ρ‚Π΅ воинского ΡƒΡ‡Π΅Ρ‚Π°.

Π’ ΠΊΠ°ΠΊΠΈΡ… ситуациях Π²ΠΎΠ·ΠΌΠΎΠΆΠ½ΠΎ ΠΏΠΎΠΌΠ΅Π½ΡΡ‚ΡŒ ΠΊΠ°Ρ‚Π΅Π³ΠΎΡ€ΠΈΡŽ

Если ΠΌΡƒΠΆΡ‡ΠΈΠ½Π° ΡΠ΅Ρ€ΡŒΠ΅Π·Π½ΠΎ Π½Π°ΠΌΠ΅Ρ€Π΅Π½ ΠΈΠ·ΠΌΠ΅Π½ΠΈΡ‚ΡŒ свою ΠΊΠ°Ρ‚Π΅Π³ΠΎΡ€ΠΈΡŽ годности, Ρ‡Ρ‚ΠΎ ΡƒΠΊΠ°Π·Π°Π½Π° Π² Π΅Π³ΠΎ Π²ΠΎΠ΅Π½Π½ΠΎΠΌ Π±ΠΈΠ»Π΅Ρ‚Π΅, Π΅ΠΌΡƒ Π½Π΅ΠΎΠ±Ρ…ΠΎΠ΄ΠΈΠΌΠΎ ΠΏΠΎΠ»ΡƒΡ‡ΠΈΡ‚ΡŒ ΠΈΠ½Ρ„ΠΎΡ€ΠΌΠ°Ρ†ΠΈΡŽ ΠΎ Ρ‚ΠΎΠΌ, ΠΊΠ°ΠΊ ΠΆΠ΅ это ΡΠ΄Π΅Π»Π°Ρ‚ΡŒ.

Π‘ΠΏΠ΅Ρ€Π²Π° Π½Π΅ΠΎΠ±Ρ…ΠΎΠ΄ΠΈΠΌΠΎ ΠΏΡ€ΠΈΠ½ΡΡ‚ΡŒ Ρ‚ΠΎΡ‚ Ρ„Π°ΠΊΡ‚, Ρ‡Ρ‚ΠΎ ΠΏΠΎΠΌΠ΅Π½ΡΡ‚ΡŒ ΠΊΠ°Ρ‚Π΅Π³ΠΎΡ€ΠΈΡŽ согласно Π΄Π΅ΠΉΡΡ‚Π²ΡƒΡŽΡ‰Π΅Π³ΠΎ Π·Π°ΠΊΠΎΠ½ΠΎΠ΄Π°Ρ‚Π΅Π»ΡŒΡΡ‚Π²Π° Π²ΠΎΠ·ΠΌΠΎΠΆΠ½ΠΎ. ΠŸΡ€ΠΈ этом Π²ΠΎΠ΅Π½ΠΊΠΎΠΌΠ°Ρ‚Ρ‹ вСсьма Π½Π΅ΠΎΡ…ΠΎΡ‚Π½ΠΎ ΠΏΠΎΠΌΠΎΠ³Π°ΡŽΡ‚ Π² этом ΠΌΠΎΠ»ΠΎΠ΄Ρ‹ΠΌ людям. Π­Ρ‚ΠΎ связано с Ρ‚Π΅ΠΌ, Ρ‡Ρ‚ΠΎ начиная с 2005 Π³ΠΎΠ΄Π°, Π±Ρ‹Π»ΠΎ ΠΎΡ‚ΠΌΠ΅Π½Π΅Π½ΠΎ ΠΎΠ±ΡΠ·Π°Ρ‚Π΅Π»ΡŒΠ½ΠΎ мСдицинскоС ΠΏΠ΅Ρ€Π΅ΠΎΡΠ²ΠΈΠ΄Π΅Ρ‚Π΅Π»ΡŒΡΡ‚Π²ΠΎΠ²Π°Π½ΠΈΠ΅, ΠΊΠΎΡ‚ΠΎΡ€ΠΎΠ΅ Ρ€Π°Π½Π΅Π΅ ΠΏΡ€ΠΎΠ²ΠΎΠ΄ΠΈΠ»ΠΎΡΡŒ ΠΊΠ°ΠΆΠ΄Ρ‹Π΅ Ρ‚Ρ€ΠΈ Π³ΠΎΠ΄Π°. ΠžΠ΄Π½ΠΎΠ²Ρ€Π΅ΠΌΠ΅Π½Π½ΠΎ с этим, всС Π΅Ρ‰Π΅ ΠΌΠΎΠΆΠ½ΠΎ Π΄ΠΎΠ±Ρ€ΠΎΠ²ΠΎΠ»ΡŒΠ½ΠΎ ΠΏΡ€ΠΎΠΉΡ‚ΠΈ ΠΏΠ΅Ρ€Π΅ΠΎΡΠ²ΠΈΠ΄Π΅Ρ‚Π΅Π»ΡŒΡΡ‚Π²ΠΎΠ²Π°Π½ΠΈΠ΅.

РСбята Π½Π° слуТбС

Для этого, ΠΊΠ°ΠΆΠ΄Ρ‹ΠΉ ΠΆΠ΅Π»Π°ΡŽΡ‰ΠΈΠΉ Π΄ΠΎΠ»ΠΆΠ΅Π½ Π½Π°ΠΏΡ€Π°Π²ΠΈΡ‚ΡŒΡΡ Π² свой Π²ΠΎΠ΅Π½ΠΊΠΎΠΌΠ°Ρ‚ для написания заявлСния Π½Π° имя Π²ΠΎΠ΅Π½Π½ΠΎΠ³ΠΎ комиссара ΠΎ ΠΏΡ€ΠΎΠ²Π΅Π΄Π΅Π½ΠΈΠΈ ΠΏΠΎΠ²Ρ‚ΠΎΡ€Π½ΠΎΠ³ΠΎ мСдицинского ΠΎΡΠ²ΠΈΠ΄Π΅Ρ‚Π΅Π»ΡŒΡΡ‚Π²ΠΎΠ²Π°Π½ΠΈΡ. К Ρ‚Π°ΠΊΠΎΠΌΡƒ заявлСнию рСкомСндуСтся ΠΏΡ€ΠΈΠ»ΠΎΠΆΠΈΡ‚ΡŒ ΠΊΠΎΠΏΠΈΠΈ мСдицинских Π΄ΠΎΠΊΡƒΠΌΠ΅Π½Ρ‚ΠΎΠ², Ρ‡Ρ‚ΠΎ Π±ΡƒΠ΄Ρƒ ΠΏΠΎΠ΄Ρ‚Π²Π΅Ρ€ΠΆΠ΄Π°Ρ‚ΡŒ Ρ„Π°ΠΊΡ‚ Ρ‚ΠΎΠ³ΠΎ, Ρ‡Ρ‚ΠΎ ΠΏΠ°Ρ€Π½ΡŽ ΡƒΠ΄Π°Π»ΠΎΡΡŒ ΠΏΠΎΠ»Π½ΠΎΡΡ‚ΡŒΡŽ Π²Ρ‹Π»Π΅Ρ‡ΠΈΡ‚ΡŒ своС Π·Π°Π±ΠΎΠ»Π΅Π²Π°Π½ΠΈΠ΅, ΠΈΠ·-Π·Π° ΠΊΠΎΡ‚ΠΎΡ€ΠΎΠ³ΠΎ Π΅ΠΌΡƒ Ρ€Π°Π½Π΅Π΅ Π±Ρ‹Π»Π° поставлСна Π½Π΅ ΡƒΠ΄ΠΎΠ²Π»Π΅Ρ‚Π²ΠΎΡ€ΡΡŽΡ‰Π°Ρ катСгория.

Π’Π°ΠΆΠ½ΠΎ ΠΏΠΎΠΌΠ½ΠΈΡ‚ΡŒ ΠΎ Ρ‚ΠΎΠΌ, Ρ‡Ρ‚ΠΎ ΠΏΡ€ΠΈ отсутствии вСских оснований ΠΈ ΠΏΠ°ΠΊΠ΅Ρ‚Π° мСдицинских Π·Π°ΠΊΠ»ΡŽΡ‡Π΅Π½ΠΈΠΉ Π½Π΅Ρ‚ Π½ΠΈΠΊΠ°ΠΊΠΎΠ³ΠΎ смысла Π΄Π°ΠΆΠ΅ ΠΈΠ½ΠΈΡ†ΠΈΠΈΡ€ΠΎΠ²Π°Ρ‚ΡŒ ΠΏΡ€ΠΎΡ†Π΅Π΄ΡƒΡ€Ρƒ смСны разряда годности. Π’ Ρ‚Π°ΠΊΠΎΠΌ случаС Π² Π²ΠΎΠ΅Π½ΠΊΠΎΠΌΠ°Ρ‚Π΅ навСрняка ΠΎΡ‚ΠΊΠ°ΠΆΡƒΡ‚ Π² ΠΏΠΎΠ΄ΠΎΠ±Π½ΠΎΠΌ ΠΏΡ€ΠΎΡˆΠ΅Π½ΠΈΠΈ.

Π˜Π½Π°Ρ‡Π΅ говоря, ΠΏΠ΅Ρ€Π΅Π΄ Ρ‚Π΅ΠΌ ΠΊΠ°ΠΊ направится Π² Π²ΠΎΠ΅Π½ΠΊΠΎΠΌΠ°Ρ‚ ΠΊΡ€Π°ΠΉΠ½Π΅ Π²Π°ΠΆΠ½ΠΎ ΠΏΠΎΡΠ΅Ρ‚ΠΈΡ‚ΡŒ всСвозмоТных Π²Ρ€Π°Ρ‡Π΅ΠΉ для провСдСния ΡƒΠ³Π»ΡƒΠ±Π»Π΅Π½Π½ΠΎΠ³ΠΎ обслСдования. ΠšΡ€ΠΎΠΌΠ΅ Ρ‚ΠΎΠ³ΠΎ, потрСбуСтся ΠΏΡ€ΠΈΠ»ΠΎΠΆΠΈΡ‚ΡŒ усилия для Ρ‚ΠΎΠ³ΠΎ, Ρ‡Ρ‚ΠΎΠ± ΡΠ½ΡΡ‚ΡŒΡΡ с ΡƒΡ‡Π΅Ρ‚Π° Π² Ρ€Π°Π·Π½ΠΎΠ³ΠΎ Ρ€ΠΎΠ΄Π° диспансСрах, ΠΈΠ·-Π·Π° Ρ‡Π΅Π³ΠΎ Ρ€Π°Π½Π΅Π΅ ΠΏΡ€ΠΈΠ·Ρ‹Π²Π½ΠΈΠΊΡƒ Π±Ρ‹Π»Π° присвоСна Ρ‚Π° ΠΈΠ»ΠΈ иная катСгория.

ПослС Ρ‚ΠΎΠ³ΠΎ ΠΊΠ°ΠΊ Π±ΡƒΠ΄Π΅Ρ‚ ΠΏΠΎΠ΄Π°Π½ΠΎ заявлСниС, Ρƒ ΠΌΠΎΠ»ΠΎΠ΄ΠΎΠ³ΠΎ Ρ‡Π΅Π»ΠΎΠ²Π΅ΠΊΠ° ΠΌΠΎΠ³ΡƒΡ‚ Π²ΠΎΠ·Π½ΠΈΠΊΠ½ΡƒΡ‚ΡŒ ΠΎΠΏΡ€Π΅Π΄Π΅Π»Π΅Π½Π½Ρ‹Π΅ ΠΏΡ€ΠΎΠ±Π»Π΅ΠΌΡ‹. Π’Π°ΠΊ, ΠΊ ΠΏΡ€ΠΈΠΌΠ΅Ρ€Ρƒ, часто Π²ΠΎΠ΅Π½ΠΊΠΎΠΌΠ°Ρ‚Ρ‹ просто ΠΎΡ‚ΠΊΠ°Π·Ρ‹Π²Π°ΡŽΡ‚ΡΡ ΠΏΡ€ΠΈΠ½ΠΈΠΌΠ°Ρ‚ΡŒ ΠΊ Ρ€Π°ΡΡΠΌΠΎΡ‚Ρ€Π΅Π½ΠΈΡŽ Ρ‚Π°ΠΊΠΈΠ΅ ΠΏΡ€ΠΎΡˆΠ΅Π½ΠΈΡ ΠΎΡ‚ Π³Ρ€Π°ΠΆΠ΄Π°Π½. Π’ ΠΏΠΎΠ΄ΠΎΠ±Π½Ρ‹Ρ… ситуациях потрСбуСтся ΠΎΡ‚ΠΏΡ€Π°Π²ΠΈΡ‚ΡŒ Π΄Π°Π½Π½ΠΎΠ΅ заявлСниС Ρ‡Π΅Ρ€Π΅Π· ΠΏΠΎΡ‡Ρ‚Ρƒ с ΡƒΠ²Π΅Π΄ΠΎΠΌΠ»Π΅Π½ΠΈΠ΅ получатСля. ПослС Ρ‚ΠΎΠ³ΠΎ ΠΊΠ°ΠΊ Π²ΠΎΠ΅Π½ΠΊΠΎΠΌΠ°Ρ‚ ΠΏΠΎΠ»ΡƒΡ‡ΠΈΡ‚ заявлСниС ΠΎΠ½ ΠΈΠΌΠ΅Π΅Ρ‚ ΠΏΡ€Π°Π²ΠΎ Π΅Π³ΠΎ Ρ€Π°ΡΡΠΌΠ°Ρ‚Ρ€ΠΈΠ²Π°Ρ‚ΡŒ Π½Π° протяТСнии 30 Π΄Π½Π΅ΠΉ.

БущСствуСт всСго Π΄Π²Π° Π²Π°Ρ€ΠΈΠ°Π½Ρ‚Π° ΠΏΠΎΡΠ»Π΅Π΄ΡƒΡŽΡ‰Π΅Π³ΠΎ развития событий. Π’ΠΎΠ΅Π½ΠΊΠΎΠΌΠ°Ρ‚ ΠΌΠΎΠΆΠ΅Ρ‚ ΡƒΠ΄ΠΎΠ²Π»Π΅Ρ‚Π²ΠΎΡ€ΠΈΡ‚ΡŒ ΠΏΡ€ΠΎΡΡŒΠ±Ρƒ заявитСля ΠΈ Π½Π°Π·Π½Π°Ρ‡ΠΈΡ‚ΡŒ Π² Π΅Π³ΠΎ ΠΎΡ‚Π½ΠΎΡˆΠ΅Π½ΠΈΠΈ ΠΏΠΎΠ²Ρ‚ΠΎΡ€Π½ΡƒΡŽ ΠΌΠ΅Π΄ΠΈΡ†ΠΈΠ½ΡΠΊΡƒΡŽ комиссию ΠΈΠ»ΠΈ ΠΆΠ΅ ΠΌΠΎΠΆΠ΅Ρ‚ Π΅Π³ΠΎ просто Π½Π΅ Ρ€Π°ΡΡΠΌΠ°Ρ‚Ρ€ΠΈΠ²Π°Ρ‚ΡŒ. Π’ΠΎ Π²Ρ‚ΠΎΡ€ΠΎΠΌ случаС СдинствСнным Π²Ρ‹Ρ…ΠΎΠ΄ΠΎΠΌ Π±ΡƒΠ΄Π΅Ρ‚ ΠΎΠ±Ρ€Π°Ρ‰Π΅Π½ΠΈΠ΅ Π² ΠΏΡ€ΠΎΠΊΡƒΡ€Π°Ρ‚ΡƒΡ€Ρƒ с заявлСниСм ΠΎ бСздСйствии сотрудников Π²ΠΎΠ΅Π½ΠΊΠΎΠΌΠ°Ρ‚Π°. ΠŸΡ€Π°ΠΊΡ‚ΠΈΡ‡Π΅ΡΠΊΠΈΠΉ ΠΎΠΏΡ‹Ρ‚ дСмонстрируСт, Ρ‡Ρ‚ΠΎ Ссли ΠΏΡ€ΠΈΠ»ΠΎΠΆΠΈΡ‚ΡŒ всСвозмоТныС усилия, Ρ‚ΠΎ Π²ΠΏΠΎΠ»Π½Π΅ Ρ€Π΅Π°Π»ΡŒΠ½ΠΎ Π΄ΠΎΠ±ΠΈΡ‚ΡŒΡΡ ΠΎΡ‚ Π²ΠΎΠ΅Π½ΠΊΠΎΠΌΠ°Ρ‚Π° ΠΏΠΎΠ²Ρ‚ΠΎΡ€Π½ΠΎ ΠΎΡΠ²ΠΈΠ΄Π΅Ρ‚Π΅Π»ΡŒΡΡ‚Π²ΠΎΠ²Π°Π½ΠΈΡ.

ΠŸΡ€ΠΎΡ†Π΅ΡΡ смСны ΠΊΠ°Ρ‚Π΅Π³ΠΎΡ€ΠΈΠΈ пригодности Π² Π²ΠΎΠ΅Π½ΠΊΠΎΠΌΠ°Ρ‚Π΅

Π’ зависимости ΠΎΡ‚ Ρ€Π°Π½Π΅Π΅ присвоСнной ΠΊΠ°Ρ‚Π΅Π³ΠΎΡ€ΠΈΠΈ годности ΠΏΡ€ΠΎΡ†Π΅Π΄ΡƒΡ€Π° Π΅Π΅ смСны Π±ΡƒΠ΄Π΅Ρ‚ нСсколько ΠΎΡ‚Π»ΠΈΡ‡Π°Ρ‚ΡŒΡΡ, хотя ΠΈ ΠΏΡ€ΠΎΠΉΠ΄Π΅Ρ‚ ΠΏΠΎ ΠΎΠ΄Π½ΠΎΠΎΠ±Ρ€Π°Π·Π½ΠΎΠΉ схСмС.

Как ΠΈΠ·ΠΌΠ΅Π½ΠΈΡ‚ΡŒ ΠΊΠ°Ρ‚Π΅Π³ΠΎΡ€ΠΈΡŽ годности с Π‘ Π½Π° А:

  • ΠŸΠ΅Ρ€Π΅Π΄ ΠΏΠΎΡ…ΠΎΠ΄ΠΎΠΌ Π² Π²ΠΎΠ΅Π½ΠΊΠΎΠΌΠ°Ρ‚ ΠΏΠΎΡΠ΅Ρ‚ΠΈΡ‚ΡŒ мСдицинскиС учрСТдСния для прохоТдСния ΡƒΠ³Π»ΡƒΠ±Π»Π΅Π½Π½ΠΎΠ³ΠΎ обслСдования ΠΎΡ€Π³Π°Π½ΠΈΠ·ΠΌΠ° с ΠΏΠΎΡΠ»Π΅Π΄ΡƒΡŽΡ‰ΠΈΠΌ ΠΏΠΎΠ»ΡƒΡ‡Π΅Π½ΠΈΠ΅ΠΌ ΠΌΠ΅Π΄Π·Π°ΠΊΠ»ΡŽΡ‡Π΅Π½ΠΈΡ ΠΎΠ± идСальном Π·Π΄ΠΎΡ€ΠΎΠ²ΡŒΠ΅;
  • Π”Π°Π»Π΅Π΅, потрСбуСтся Π½Π°ΠΏΡ€Π°Π²ΠΈΡ‚ΡŒΡΡ Π² Π²ΠΎΠ΅Π½ΠΊΠΎΠΌΠ°Ρ‚ с этими Π΄ΠΎΠΊΡƒΠΌΠ΅Π½Ρ‚Π°ΠΌΠΈ ΠΈ Π½Π°ΠΏΠΈΡΠ°Ρ‚ΡŒ заявлСниС ΠΎ ΠΏΠΎΠ²Ρ‚ΠΎΡ€Π½ΠΎΠΌ мСдицинском ΠΎΡΠ²ΠΈΠ΄Π΅Ρ‚Π΅Π»ΡŒΡΡ‚Π²ΠΎΠ²Π°Π½ΠΈΠΈ;
  • ПослС этого ΠΏΡ€ΠΎΠΉΡ‚ΠΈ ΠΌΠ΅Π΄ΠΈΡ†ΠΈΠ½ΡΠΊΡƒΡŽ комиссию ΠΏΡ€ΠΈ Π²ΠΎΠ΅Π½ΠΊΠΎΠΌΠ°Ρ‚Π΅ ΠΏΠΎ Ρ€Π΅Π·ΡƒΠ»ΡŒΡ‚Π°Ρ‚Π°ΠΌ, ΠΊΠΎΡ‚ΠΎΡ€ΠΎΠΉ Π±ΡƒΠ΄Π΅Ρ‚ принято Ρ€Π΅ΡˆΠ΅Π½ΠΈΠ΅ ΠΎ смСнС ΠΊΠ°Ρ‚Π΅Π³ΠΎΡ€ΠΈΠΈ Π² Π²ΠΎΠ΅Π½Π½ΠΎΠΌ Π±ΠΈΠ»Π΅Ρ‚Π΅.

Π’ΠΎΠ΅Π½Π½ΠΈΠΊΠΈ

Как ΠΈΠ·ΠΌΠ΅Π½ΠΈΡ‚ΡŒ ΠΊΠ°Ρ‚Π΅Π³ΠΎΡ€ΠΈΡŽ годности с Π‘ Π½Π° Π’:

  • ΠŸΠΎΠ»ΡƒΡ‡ΠΈΡ‚ΡŒ мСдицинскиС выписки ΠΈ ΠΏΡ€ΠΎΠΉΡ‚ΠΈ мСдобслСдованиС ΠΏΠ΅Ρ€Π΅Π΄ посСщСниСм Π²ΠΎΠ΅Π½ΠΊΠΎΠΌΠ°Ρ‚Π° для получСния подтвСрТдСния наличия ΡΠ΅Ρ€ΡŒΠ΅Π·Π½ΠΎΠ³ΠΎ заболСвания;
  • ΠΠ°ΠΏΠΈΡΠ°Ρ‚ΡŒ Π² Π²ΠΎΠ΅Π½ΠΊΠΎΠΌΠ°Ρ‚Π΅ заявлСниС ΠΎ ΠΏΠΎΠ²Ρ‚ΠΎΡ€Π½ΠΎΠΌ ΠΏΡ€ΠΎΡ…ΠΎΠΆΠ΄Π΅Π½ΠΈΠΈ мСдкомиссии;
  • ΠŸΡ€ΠΎΠΉΡ‚ΠΈ мСдобслСдованиС ΠΈ ΠΏΠΎΠ»ΡƒΡ‡ΠΈΡ‚ΡŒ Π½ΠΎΠ²ΡƒΡŽ ΠΊΠ°Ρ‚Π΅Π³ΠΎΡ€ΠΈΡŽ.

Как ΠΈΠ·ΠΌΠ΅Π½ΠΈΡ‚ΡŒ ΠΊΠ°Ρ‚Π΅Π³ΠΎΡ€ΠΈΡŽ годности с Π’ Π½Π° А:

  • Π£Π³Π»ΡƒΠ±Π»Π΅Π½Π½ΠΎΠ΅ обслСдованиС Π² мСдицинском ΡƒΡ‡Ρ€Π΅ΠΆΠ΄Π΅Π½ΠΈΠΈ для получСния выписки ΠΎΠ± отсутствии ΠΊΠ°ΠΊΠΈΡ…-Π»ΠΈΠ±ΠΎ Π·Π°Π±ΠΎΠ»Π΅Π²Π°Π½ΠΈΠΉ ΠΈ ΠΏΠ°Ρ‚ΠΎΠ»ΠΎΠ³ΠΈΠΉ;
  • БоставлСниС заявлСния ΠΎ ΠΏΠΎΠ²Ρ‚ΠΎΡ€Π½ΠΎΠΉ мСдкомиссии;
  • ΠŸΡ€ΠΎΡ…ΠΎΠΆΠ΄Π΅Π½ΠΈΠ΅ ΠΌΠ΅Π΄ΠΎΡΠ²ΠΈΠ΄Π΅Ρ‚Π΅Π»ΡŒΡΡ‚Π²ΠΎΠ²Π°Π½ΠΈΡ ΠΈ ΠΏΠΎΠ»ΡƒΡ‡Π΅Π½ΠΈΠ΅ Π½ΠΎΠ²ΠΎΠΉ ΠΊΠ°Ρ‚Π΅Π³ΠΎΡ€ΠΈΠΈ.

Как ΠΈΠ·ΠΌΠ΅Π½ΠΈΡ‚ΡŒ ΠΊΠ°Ρ‚Π΅Π³ΠΎΡ€ΠΈΡŽ годности с Π’ Π½Π° Π‘:

  • Π‘ΠΎΠ±Ρ€Π°Ρ‚ΡŒ выписки ΠΈ мСдицинскиС Π·Π°ΠΊΠ»ΡŽΡ‡Π΅Π½ΠΈΡ ΠΎ Π²Ρ‹Π·Π΄ΠΎΡ€ΠΎΠ²Π»Π΅Π½ΠΈΠΈ ΠΈ отсутствии ΠΏΠ°Ρ‚ΠΎΠ»ΠΎΠ³ΠΈΠΈ, ΠΈΠ·-Π·Π° ΠΊΠΎΡ‚ΠΎΡ€ΠΎΠΉ Ρ€Π°Π½Π΅Π΅ Π±Ρ‹Π»Π° присвоСна катСгория Π’;
  • ΠΠ°ΠΏΠΈΡΠ°Ρ‚ΡŒ заявлСниС ΠΎ ΠΏΡ€ΠΎΠ²Π΅Π΄Π΅Π½ΠΈΠ΅ мСдицинского ΠΎΡΠ²ΠΈΠ΄Π΅Ρ‚Π΅Π»ΡŒΡΡ‚Π²ΠΎΠ²Π°Π½ΠΈΡ;
  • УспСшно ΠΏΡ€ΠΎΠΉΡ‚ΠΈ мСдкомиссию ΠΈ ΠΏΠΎΠ΄Ρ‚Π²Π΅Ρ€Π΄ΠΈΡ‚ΡŒ Ρ„Π°ΠΊΡ‚ выздоровлСния для получСния ΠΊΠ°Ρ‚Π΅Π³ΠΎΡ€ΠΈΠΈ Π‘.

Как ΠΈΠ·ΠΌΠ΅Π½ΠΈΡ‚ΡŒ ΠΊΠ°Ρ‚Π΅Π³ΠΎΡ€ΠΈΡŽ годности с Π” Π½Π° А

  • ΠŸΡ€ΠΎΡ…ΠΎΠΆΠ΄Π΅Π½ΠΈΠ΅ Π³Π»ΡƒΠ±ΠΎΠΊΠΎΠΉ диагностики ΠΎΡ€Π³Π°Π½ΠΈΠ·ΠΌΠ° для получСния ΠΎΡ„ΠΈΡ†ΠΈΠ°Π»ΡŒΠ½Ρ‹Ρ… выписок ΠΎΠ± отсутствии ΠΊΠ°ΠΊΠΈΡ…-Π»ΠΈΠ±ΠΎ ΠΏΡ€ΠΎΠ±Π»Π΅ΠΌ со Π·Π΄ΠΎΡ€ΠΎΠ²ΡŒΠ΅ΠΌ;
  • ΠΠ°ΠΏΠΈΡΠ°Ρ‚ΡŒ заявлСниС Π½Π° имя Π²ΠΎΠ΅Π½Π½ΠΎΠ³ΠΎ комиссара с ΠΏΡ€ΠΎΡΡŒΠ±ΠΎΠΉ ΠΎ ΠΏΠΎΠ²Ρ‚ΠΎΡ€Π½ΠΎΠΌ ΠΏΡ€ΠΎΡ…ΠΎΠΆΠ΄Π΅Π½ΠΈΠΈ мСдкомиссии;
  • ΠŸΠΎΠ΄Ρ‚Π²Π΅Ρ€Π΄ΠΈΡ‚ΡŒ Ρ„Π°ΠΊΡ‚ отсутствия ΠΏΡ€ΠΎΠ±Π»Π΅ΠΌ со Π·Π΄ΠΎΡ€ΠΎΠ²ΡŒΠ΅ΠΌ Ρ‡Π»Π΅Π½Π°ΠΌ мСдицинской комиссии ΠΏΡ€ΠΈ Π²ΠΎΠ΅Π½ΠΊΠΎΠΌΠ°Ρ‚Π΅.

Как ΠΈΠ·ΠΌΠ΅Π½ΠΈΡ‚ΡŒ ΠΊΠ°Ρ‚Π΅Π³ΠΎΡ€ΠΈΡŽ годности с Π” Π½Π° Π‘:

  • ΠŸΠΎΠ»ΡƒΡ‡ΠΈΡ‚ΡŒ выписки ΠΈ справки с мСдучрСТдСния ΠΎΠ± отсутствии ΡΠ΅Ρ€ΡŒΠ΅Π·Π½Ρ‹Ρ… Π·Π°Π±ΠΎΠ»Π΅Π²Π°Π½ΠΈΠΉ, ΠΈΠ·-Π·Π° ΠΊΠΎΡ‚ΠΎΡ€Ρ‹Ρ… Ρ€Π°Π½Π΅Π΅ Π±Ρ‹Π»Π° присвоСна катСгория Π”;
  • ΠΠ°ΠΏΠΈΡΠ°Ρ‚ΡŒ заявлСниС ΠΎ ΠΏΠΎΠ²Ρ‚ΠΎΡ€Π½ΠΎΠΉ мСдкомиссии;
  • ΠŸΡ€ΠΎΠΉΡ‚ΠΈ мСдицинскоС обслСдованиС ΠΈ ΠΏΠΎΠ΄Ρ‚Π²Π΅Ρ€Π΄ΠΈΡ‚ΡŒ Ρ„Π°ΠΊΡ‚ ΡƒΠ΄ΠΎΠ²Π»Π΅Ρ‚Π²ΠΎΡ€ΠΈΡ‚Π΅Π»ΡŒΠ½ΠΎΠ³ΠΎ состояния Π·Π΄ΠΎΡ€ΠΎΠ²ΡŒΡ.

Π’Π°ΠΆΠ½ΠΎ ΠΏΠΎΠ½ΠΈΠΌΠ°Ρ‚ΡŒ! Π˜Π·ΠΌΠ΅Π½ΠΈΡ‚ΡŒ ΠΊΠ°Ρ‚Π΅Π³ΠΎΡ€ΠΈΡŽ годности Π” Π±ΡƒΠ΄Π΅Ρ‚ ΠΊΡ€Π°ΠΉΠ½Π΅ слоТно, Ρ‚Π°ΠΊ ΠΊΠ°ΠΊ ΠΎΠ½Π° присваиваСтся людям с ΠΎΡ‡Π΅Π½ΡŒ ΡΠ΅Ρ€ΡŒΠ΅Π·Π½Ρ‹ΠΌΠΈ заболСваниями. Как ΠΏΡ€Π°Π²ΠΈΠ»ΠΎ Π΅Π΅ удаСтся ΡƒΡΠΏΠ΅ΡˆΠ½ΠΎ ΠΏΠΎΠΌΠ΅Π½ΡΡ‚ΡŒ Ρ‚ΠΎΠ»ΡŒΠΊΠΎ ΠΏΡ€ΠΈ условии, Ρ‡Ρ‚ΠΎ Ρ€Π°Π½Π΅Π΅ Π±Ρ‹Π» присвоСн Π½Π΅ Π²Π΅Ρ€Π½Ρ‹ΠΉ Π΄ΠΈΠ°Π³Π½ΠΎΠ·.

Π˜Ρ‚ΠΎΠ³ΠΈ

НСсмотря Π½Π° всю ΡΠ»ΠΎΠΆΠ½ΠΎΡΡ‚ΡŒ ΠΏΡ€ΠΎΡ†Π΅Π΄ΡƒΡ€Ρ‹ смСны ΠΊΠ°Ρ‚Π΅Π³ΠΎΡ€ΠΈΠΈ пригодности ΠΈ частом Π½Π΅ΠΆΠ΅Π»Π°Π½ΠΈΠΈ Π²ΠΎΠ΅Π½ΠΊΠΎΠΌΠ°Ρ‚Π° ΠΈΠ΄Ρ‚ΠΈ Π½Π° уступки ΠΏΡ€ΠΈΠ·Ρ‹Π²Π½ΠΈΠΊΠ°ΠΌ, практичСский ΠΎΠΏΡ‹Ρ‚ ΠΏΠΎΠΊΠ°Π·Ρ‹Π²Π°Π΅Ρ‚, Ρ‡Ρ‚ΠΎ ΠΏΠΎΠ΄ΠΎΠ±Π½ΠΎΠ΅ всС ΠΆΠ΅ Ρ€Π΅Π°Π»ΡŒΠ½ΠΎ ΠΎΡΡƒΡ‰Π΅ΡΡ‚Π²ΠΈΡ‚ΡŒ.

ΠŸΠ΅Ρ€Π΅ΠΎΡΠ²ΠΈΠ΄Π΅Ρ‚Π΅Π»ΡŒΡΡ‚Π²ΠΎΠ²Π°Π½ΠΈΠ΅ Π² Π²ΠΎΠ΅Π½ΠΊΠΎΠΌΠ°Ρ‚Π΅ 2020 | Как ΡΠΎΡΡ‚Π°Π²ΠΈΡ‚ΡŒ заявлСниС

Π›ΡŽΠ±ΠΎΠΉ Π³Ρ€Π°ΠΆΠ΄Π°Π½ΠΈΠ½, ΠΊΠΎΡ‚ΠΎΡ€Ρ‹ΠΉ ΠΏΡ€ΠΎΡˆΠ΅Π» мСдкомиссию послС ΠΏΡ€ΠΈΠ·Ρ‹Π²Π°, Π² 2020 Π³ΠΎΠ΄Ρƒ ΠΈΠΌΠ΅Π΅Ρ‚ ΠΏΡ€Π°Π²ΠΎ Π½Π° мСдицинскоС ΠΏΠ΅Ρ€Π΅ΠΎΡΠ²ΠΈΠ΄Π΅Ρ‚Π΅Π»ΡŒΡΡ‚Π²ΠΎΠ²Π°Π½ΠΈΠ΅. Π’Π°ΠΊΡƒΡŽ Π²ΠΎΠ·ΠΌΠΎΠΆΠ½ΠΎΡΡ‚ΡŒ Π·Π°ΠΊΠΎΠ½ Π΄Π°Π΅Ρ‚ Π»ΠΈΡ†Π°ΠΌ, Π΄ΠΎΡΡ‚ΠΈΠ³ΡˆΠΈΠΌ ΡΠΎΠ²Π΅Ρ€ΡˆΠ΅Π½Π½ΠΎΠ»Π΅Ρ‚ΠΈΡ ΠΈ Π²ΠΏΠ»ΠΎΡ‚ΡŒ Π΄ΠΎ 27 Π»Π΅Ρ‚. ΠšΠ°Ρ‚Π΅Π³ΠΎΡ€ΠΈΡ годности опрСдСляСтся Π²ΠΎΠ΅Π½Π½ΠΎ-Π²Ρ€Π°Ρ‡Π΅Π±Π½ΠΎΠΉ комиссиСй. ΠŸΡ€ΠΎΡ†Π΅ΡΡ ΠΎΡΠ²ΠΈΠ΄Π΅Ρ‚Π΅Π»ΡŒΡΡ‚Π²ΠΎΠ²Π°Π½ΠΈΡ ΠΏΡ€ΠΎΡ…ΠΎΠ΄ΠΈΡ‚ Π²Β Ρ‚Ρ€ΠΈ шага:
  • Π€Π»ΡŽΠΎΡ€ΠΎΠ³Ρ€Π°Ρ„ΠΈΡ. Она трСбуСтся, Ρ‡Ρ‚ΠΎΠ±Ρ‹ ΠΎΠΏΡ€Π΅Π΄Π΅Π»ΠΈΡ‚ΡŒ, Π² ΠΊΠ°ΠΊΠΎΠΌ состоянии Ρƒ ΠΏΡ€ΠΈΠ·Ρ‹Π²Π½ΠΈΠΊΠ° находится систСма дыхания.
  • Анализы для Π²ΠΎΠ΅Π½ΠΊΠΎΠΌΠ°Ρ‚Π°, Π° ΠΈΠΌΠ΅Π½Π½ΠΎ ΠΊΡ€ΠΎΠ²ΡŒ ΠΈ ΠΌΠΎΡ‡Π°. На основании ΠΏΠΎΠ»ΡƒΡ‡Π΅Π½Π½Ρ‹Ρ… Ρ€Π΅Π·ΡƒΠ»ΡŒΡ‚Π°Ρ‚ΠΎΠ² Π±ΡƒΠ΄ΡƒΡ‚ Π΄Π΅Π»Π°Ρ‚ΡŒΡΡ Π²Ρ‹Π²ΠΎΠ΄Ρ‹, ΡΡƒΡ‰Π΅ΡΡ‚Π²ΡƒΡŽΡ‚ Π»ΠΈ Ρƒ ΠΏΡ€ΠΈΠ·Ρ‹Π²Π½ΠΈΠΊΠ° ΠΊΠ°ΠΊΠΈΠ΅-Π»ΠΈΠ±ΠΎ ΡΠ΅Ρ€ΡŒΠ΅Π·Π½Ρ‹Π΅ отклонСния.
  • ΠŸΡ€ΠΎΠ²Π΅Ρ€ΠΊΠ° сСрдСчно-сосудистой систСмы. Для этого проводится элСктрокардиографичСский осмотр.Β 

Π Π΅Π·ΡƒΠ»ΡŒΡ‚Π°Ρ‚Ρ‹ опрСдСлят, ΠΊ ΠΊΠ°ΠΊΠΎΠΉ ΠΈΠ· ΠΊΠ°Ρ‚Π΅Π³ΠΎΡ€ΠΈΠΉ слСдуСт отнСсти ΠΏΡ€ΠΈΠ·Ρ‹Π²Π½ΠΈΠΊΠ°. На сСгодняшний дСнь ΡΡƒΡ‰Π΅ΡΡ‚Π²ΡƒΡŽΡ‚ ΠΊΠ°Ρ‚Π΅Π³ΠΎΡ€ΠΈΠΈ пригодности ΠΎΡ‚ А Π΄ΠΎ Π”:

  • А. ΠŸΠΎΠ΄Ρ€Π°Π·ΡƒΠΌΠ΅Π²Π°Π΅Ρ‚, Ρ‡Ρ‚ΠΎ ΠΏΡ€ΠΈΠ·Ρ‹Π²Π½ΠΈΠΊ ΠΎΠ±Π»Π°Π΄Π°Π΅Ρ‚ ΠΎΡ‚ΠΌΠ΅Π½Π½Ρ‹ΠΌ Π·Π΄ΠΎΡ€ΠΎΠ²ΡŒΠ΅ΠΌ ΠΈ ΠΌΠΎΠΆΠ΅Ρ‚ ΡΠ»ΡƒΠΆΠΈΡ‚ΡŒ Π² любом Ρ€ΠΎΠ΄Ρƒ войск.
  • Π‘. Π“ΠΎΠ΄Π½ΠΎΡΡ‚ΡŒ ΠΊ слуТбС с нСсущСствСнными ограничСниями (ΠΏΠ»ΠΎΡ…ΠΎΠ΅ Π·Ρ€Π΅Π½ΠΈΠ΅ ΠΈ ΠΏΡ€ΠΎΡ‡ΠΈΠ΅ отклонСния).
  • Π’. ΠŸΡ€ΠΈΠ·Ρ‹Π²Π½ΠΈΠΊ сразу отправляСтся Π² запас. Π’Π΅ΠΌ Π½Π΅ ΠΌΠ΅Π½Π΅Π΅, ΠΏΡ€ΠΈ Π²Π²Π΅Π΄Π΅Π½ΠΈΠΈ Π² странС Π²ΠΎΠ΅Π½Π½ΠΎΠ³ΠΎ полоТСния ΠΎΠ½ Ρ‚Π°ΠΊΠΆΠ΅ ΠΏΠΎΠ΄Π»Π΅ΠΆΠΈΡ‚ ΠΏΡ€ΠΈΠ·Ρ‹Π²Ρƒ, ΠΏΡ€ΠΈΡ‡Π΅ΠΌ ΠΎΠ½ Π΄ΠΎΠ»ΠΆΠ΅Π½ ΠΏΡ€ΠΎΠΉΡ‚ΠΈ ΠΏΠΎΠ΄Π³ΠΎΡ‚ΠΎΠ²ΠΊΡƒ ΠΏΠΎ граТданской ΡΠΏΠ΅Ρ†ΠΈΠ°Π»ΡŒΠ½ΠΎΡΡ‚ΠΈ.
  • Π“. ΠšΠ°Ρ‚Π΅Π³ΠΎΡ€ΠΈΡ Π²Ρ€Π΅ΠΌΠ΅Π½Π½ΠΎΠΉ нСпригодности, Ρ‚. Π΅. Π½Π°Π»ΠΈΡ‡ΠΈΠ΅ Π½Π΅Π΄ΡƒΠ³Π°, вслСдствиС ΠΊΠΎΡ‚ΠΎΡ€ΠΎΠ³ΠΎ ΠΏΡ€ΠΈΠ·Ρ‹Π²Π½ΠΈΠΊ ΠΈΠΌΠ΅Π½Π½ΠΎ Π² ΠΌΠΎΠΌΠ΅Π½Ρ‚ ΠΏΡ€ΠΈΠ·Ρ‹Π²Π° Π½Π΅ ΠΌΠΎΠΆΠ΅Ρ‚ Π±Ρ‹Ρ‚ΡŒ взят Π² Π°Ρ€ΠΌΠΈΡŽ. Π’ Π΄Π°Π½Π½ΠΎΠΌ случаС прСдставляСтся отсрочка, которая, Π²ΠΏΡ€ΠΎΡ‡Π΅ΠΌ, ΠΌΠΎΠΆΠ΅Ρ‚ Π±Ρ‹Ρ‚ΡŒ ΠΏΡ€ΠΎΠ΄Π»Π΅Π½Π° ΠΈ послС ΡΠ»Π΅Π΄ΡƒΡŽΡ‰Π΅Π³ΠΎ ΠΏΡ€ΠΈΠ·Ρ‹Π²Π° – это опрСдСляСтся Ρ…Π°Ρ€Π°ΠΊΡ‚Π΅Ρ€ΠΎΠΌ ΠΈ Π΄Π»ΠΈΡ‚Π΅Π»ΡŒΠ½ΠΎΡΡ‚ΡŒΡŽ Π±ΠΎΠ»Π΅Π·Π½ΠΈ.
  • Π”. Полная Π½Π΅ΠΏΡ€ΠΈΠ³ΠΎΠ΄Π½ΠΎΡΡ‚ΡŒ ΠΊ нСсСнию воинской слуТбы. НапримСр, ΠΏΠΎΠ΄ эту ΠΊΠ°Ρ‚Π΅Π³ΠΎΡ€ΠΈΡŽ ΠΏΠΎΠΏΠ°Π΄Π°ΡŽΡ‚ ΠΈΠ½Π²Π°Π»ΠΈΠ΄Ρ‹, Π»ΠΈΡ†Π°, ΡΡ‚Ρ€Π°Π΄Π°ΡŽΡ‰ΠΈΠ΅ хроничСскими болСзнями ΠΈ сущСствСнными Π½Π°Ρ€ΡƒΡˆΠ΅Π½ΠΈΡΠΌΠΈ Π² Ρ€Π°Π±ΠΎΡ‚Π΅ Ρ€Π°Π·Π»ΠΈΡ‡Π½Ρ‹Ρ… систСм ΠΎΡ€Π³Π°Π½ΠΈΠ·ΠΌΠ°.
ΠŸΠΎΠ΄Ρ€ΠΎΠ±Π½Π΅Π΅ ΠΎ Ρ‚ΠΎΠΌ, ΠΊΠΎΠ³Π΄Π° начинаСтся ΠΏΡ€ΠΈΠ·Ρ‹Π² Π² Π°Ρ€ΠΌΠΈΡŽ, Ρ‡ΠΈΡ‚Π°ΠΉΡ‚Π΅ Π½Π° нашСм сайтС.

ΠŸΠ΅Ρ€Π΅ΠΎΡΠ²ΠΈΠ΄Π΅Ρ‚Π΅Π»ΡŒΡΡ‚Π²ΠΎΠ²Π°Π½ΠΈΠ΅ Π² Π²ΠΎΠ΅Π½ΠΊΠΎΠΌΠ°Ρ‚Π΅

Π‘ΠΌΠ΅Π½Π° ΠΊΠ°Ρ‚Π΅Π³ΠΎΡ€ΠΈΠΈ годности

Π’Π΅ΠΌ Π½Π΅ ΠΌΠ΅Π½Π΅Π΅, Π·Π°ΠΊΠΎΠ½ Ρ€Π°Π·Ρ€Π΅ΡˆΠ°Π΅Ρ‚ ΡΠΌΠ΅Π½ΠΈΡ‚ΡŒ ΠΊΠ°Ρ‚Π΅Π³ΠΎΡ€ΠΈΡŽ воинской годности на Π΄Ρ€ΡƒΠ³ΡƒΡŽ. Для этого ΠΌΠΎΠ»ΠΎΠ΄ΠΎΠΉ Ρ‡Π΅Π»ΠΎΠ²Π΅ΠΊ Π΄ΠΎΠ»ΠΆΠ΅Π½ ΡΠΎΡΡ‚Π°Π²ΠΈΡ‚ΡŒ заявлСниС Π½Π° имя руководитСля Π²ΠΎΠ΅Π½ΠΊΠΎΠΌΠ°Ρ‚Π°, Π³Π΄Π΅ Π±ΡƒΠ΄Π΅Ρ‚ ΠΎΡ‚Ρ€Π°ΠΆΠ΅Π½Π° ΠΏΡ€ΠΎΡΡŒΠ±Π° ΠΎ смСнС ΠΊΠ°Ρ‚Π΅Π³ΠΎΡ€ΠΈΠΈ годности ΠΈ ΠΏΡ€ΠΎΡ…ΠΎΠΆΠ΄Π΅Π½ΠΈΠΈ ΠΏΠ΅Ρ€Π΅ΠΎΡΠ²ΠΈΠ΄Π΅Ρ‚Π΅Π»ΡŒΡΡ‚Π²ΠΎΠ²Π°Π½ΠΈΡ. ΠŸΡ€ΠΈ этом Π²Π°ΠΆΠ½ΠΎ Π½Π΅ Π·Π°Π±Ρ‹Ρ‚ΡŒ ΠΏΡ€ΠΈΠ»ΠΎΠΆΠΈΡ‚ΡŒ ΠΊ ΠΏΠΈΡΡŒΠΌΡƒ всС Π·Π°ΠΊΠ»ΡŽΡ‡Π΅Π½ΠΈΡ ΠΌΠ΅Π΄ΠΈΠΊΠΎΠ², справки ΠΈ ΠΏΡ€ΠΎΡ‡ΠΈΠ΅ Π΄ΠΎΠΊΡƒΠΌΠ΅Π½Ρ‚Ρ‹, ΠΊΠΎΡ‚ΠΎΡ€Ρ‹Π΅ ΠΎΡ‚Ρ€Π°ΠΆΠ°ΡŽΡ‚ Ρ€Π΅Π°Π»ΡŒΠ½ΠΎΠ΅ ΡƒΠ»ΡƒΡ‡ΡˆΠ΅Π½ΠΈΠ΅ Π·Π΄ΠΎΡ€ΠΎΠ²ΡŒΡ. Для сбора Ρ‚Π°ΠΊΠΈΡ… справок потрСбуСтся ΠΎΠ±ΠΎΠΉΡ‚ΠΈ Π½Π΅ ΠΎΠ΄Π½ΠΎΠ³ΠΎ спСциалиста. Π’Π°ΠΊ, ΠΎΠ±ΡΠ·Π°Ρ‚Π΅Π»ΡŒΠ½ΠΎ придСтся ΠΎΠ±Ρ€Π°Ρ‚ΠΈΡ‚ΡŒΡΡ ΠΊ Ρ…ΠΈΡ€ΡƒΡ€Π³Ρƒ, Π²Π·ΡΡ‚ΡŒ ΠΏΠΎΠ»ΠΎΠΆΠΈΡ‚Π΅Π»ΡŒΠ½ΠΎΠ΅ Π·Π°ΠΊΠ»ΡŽΡ‡Π΅Π½ΠΈΠ΅ Ρƒ психиатра, Ρ‚Π΅Ρ€Π°ΠΏΠ΅Π²Ρ‚Π°. Π’Π°ΠΊΠΆΠ΅ Π½Π΅ΠΎΠ±Ρ…ΠΎΠ΄ΠΈΠΌΠΎ Π±ΡƒΠ΄Π΅Ρ‚ ΠΏΡ€ΠΎΠΉΡ‚ΠΈ обслСдованиС Π² Π›ΠžΠ -ΠΊΠ°Π±ΠΈΠ½Π΅Ρ‚Π΅ ΠΈ осмотр Ρƒ ΠΎΡ„Ρ‚Π°Π»ΡŒΠΌΠΎΠ»ΠΎΠ³Π°.

НС стоит ΡƒΠΏΡƒΡΠΊΠ°Ρ‚ΡŒ ΠΈΠ· Π²ΠΈΠ΄Ρƒ, Ρ‡Ρ‚ΠΎ спСциалисты ΠΈΠ· списка Π²Ρ€Π°Ρ‡Π΅ΠΉ Π² Π²ΠΎΠ΅Π½ΠΊΠΎΠΌΠ°Ρ‚Π΅, которая Π±ΡƒΠ΄Π΅Ρ‚ Π·Π°Π½ΠΈΠΌΠ°Ρ‚ΡŒΡΡ ΠΏΠ΅Ρ€Π΅ΠΎΡΠ²ΠΈΠ΄Π΅Ρ‚Π΅Π»ΡŒΡΡ‚Π²ΠΎΠ²Π°Π½ΠΈΠ΅ΠΌ Π² Π²ΠΎΠ΅Π½ΠΊΠΎΠΌΠ°Ρ‚Π΅, ΠΏΠΎ Ρ€Π΅Π·ΡƒΠ»ΡŒΡ‚Π°Ρ‚Π°ΠΌ своСго обслСдования ΠΈΠΌΠ΅ΡŽΡ‚ ΠΏΠΎΠ»Π½ΠΎΠ΅ ΠΏΡ€Π°Π²ΠΎ ΠΏΠΎΡ‚Ρ€Π΅Π±ΠΎΠ²Π°Ρ‚ΡŒ ΠΏΡ€ΠΎΡ…ΠΎΠΆΠ΄Π΅Π½ΠΈΠ΅ Π΄ΠΎΠΏΠΎΠ»Π½ΠΈΡ‚Π΅Π»ΡŒΠ½ΠΎΠ³ΠΎ мСдицинского осмотра. ПослС осущСствлСния Ρ‚Π°ΠΊΠΈΡ… исслСдований ΠΊΠ°Ρ€Ρ‚ΠΈΠ½Π° со Π·Π΄ΠΎΡ€ΠΎΠ²ΡŒΠ΅ΠΌ ΠΏΠΎΡ‚Π΅Π½Ρ†ΠΈΠ°Π»ΡŒΠ½ΠΎΠ³ΠΎ Π½ΠΎΠ²ΠΎΠ±Ρ€Π°Π½Ρ†Π° станСт Π±ΠΎΠ»Π΅Π΅ ΠΏΠΎΠ»Π½ΠΎΠΉ, поэтому Π²Ρ€Π°Ρ‡ΠΈ выносят ΠΎΠΊΠΎΠ½Ρ‡Π°Ρ‚Π΅Π»ΡŒΠ½Ρ‹ΠΉ Π²Π΅Ρ€Π΄ΠΈΠΊΡ‚ – ΡΠ»ΡƒΠΆΠΈΡ‚ΡŒ ΠΈΠ»ΠΈ Π½Π΅ ΡΠ»ΡƒΠΆΠΈΡ‚ΡŒ. Если Π½Π°Π»ΠΈΡ†ΠΎ Ρ€Π΅Π°Π»ΡŒΠ½ΠΎΠ΅ ΡƒΠ»ΡƒΡ‡ΡˆΠ΅Π½ΠΈΠ΅ Π·Π΄ΠΎΡ€ΠΎΠ²ΡŒΡ, Ρ‚ΠΎ ΠΌΠΎΠ»ΠΎΠ΄ΠΎΠΉ Ρ‡Π΅Π»ΠΎΠ²Π΅ΠΊ ΠΌΠΎΠΆΠ΅Ρ‚ Π΄Π°ΠΆΠ΅ Ρ€Π°ΡΡΡ‡ΠΈΡ‚Ρ‹Π²Π°Ρ‚ΡŒ Π½Π° присвоСниС ΠΊΠ°Ρ‚Π΅Π³ΠΎΡ€ΠΈΠΈ А. И, Π½Π°ΠΏΡ€ΠΎΡ‚ΠΈΠ², Ссли Π΄ΠΎΠΏΠΎΠ»Π½ΠΈΡ‚Π΅Π»ΡŒΠ½ΠΎΠ΅ обслСдованиС Π²ΠΎΠ΅Π½ΠΊΠΎΠΌΠ°Ρ‚Π° выявило, Ρ‡Ρ‚ΠΎ Π·Π΄ΠΎΡ€ΠΎΠ²ΡŒΠ΅ Π½Π° самом Π΄Π΅Π»Π΅ ΠΏΠΎΡˆΠ°Ρ‚Π½ΡƒΠ»ΠΎΡΡŒ Π΅Ρ‰Π΅ сильнСС, катСгория ΠΌΠΎΠΆΠ΅Ρ‚ Π±Ρ‹Ρ‚ΡŒ ΠΈΠ·ΠΌΠ΅Π½Π΅Π½Π° с А Π½Π° Π‘ ΠΈ Ρ‚. Π΄.

Π’ нашСй странС, ΡƒΠ²Ρ‹, сплошь ΠΈ рядом ΠΌΠΎΠ»ΠΎΠ΄Ρ‹Π΅ люди ΡΡ‚Π°Ρ€Π°ΡŽΡ‚ΡΡ приобрСсти Ρ‚. Π½. Β«Π±Π΅Π»Ρ‹ΠΉ Π±ΠΈΠ»Π΅Ρ‚Β» Π·Π° дСньги Ρƒ сотрудников Π²ΠΎΠ΅Π½ΠΊΠΎΠΌΠ°Ρ‚Π° ΠΈΠ»ΠΈ Π²Ρ€Π°Ρ‡Π΅ΠΉ. Π‘ΠΎ своСй стороны, Π½Π°ΡΡ‚ΠΎΡΡ‚Π΅Π»ΡŒΠ½ΠΎ Π½Π΅ Ρ€Π΅ΠΊΠΎΠΌΠ΅Π½Π΄ΡƒΠ΅ΠΌ ΠΏΡ€ΠΈΠ±Π΅Π³Π°Ρ‚ΡŒ ΠΊ ΠΏΠΎΠ΄ΠΎΠ±Π½Ρ‹ΠΌ срСдствам уклонСния ΠΎΡ‚ слуТбы, Ρ‚. ΠΊ. ΠΎΠ½ΠΈ ΠΏΠΎΠΏΠ°Π΄Π°ΡŽΡ‚ Π·Π°Ρ‡Π°ΡΡ‚ΡƒΡŽ Π½Π΅ просто Π½Π΅ ΠΏΠΎΠ΄ административныС ΠΏΡ€Π°Π²ΠΎΠ½Π°Ρ€ΡƒΡˆΠ΅Π½ΠΈΡ, Π½ΠΎ ΠΈ ΠΏΠΎΠ΄ ΡƒΠ³ΠΎΠ»ΠΎΠ²Π½Ρ‹Π΅ ΡΡ‚Π°Ρ‚ΡŒΠΈ со Π²ΠΏΠΎΠ»Π½Π΅ ΡΠ΅Ρ€ΡŒΠ΅Π·Π½Ρ‹ΠΌ Π½Π°ΠΊΠ°Π·Π°Π½ΠΈΠ΅ΠΌ.

Π₯отя ΠΏΠ΅Ρ€Π΅ΠΎΡΠ²ΠΈΠ΄Π΅Ρ‚Π΅Π»ΡŒΡΡ‚Π²ΠΎΠ²Π°Π½ΠΈΠ΅ Π² Ρ†Π΅Π»ΠΎΠΌ ΠΏΠΎΠ΄Ρ€Π°Π·ΡƒΠΌΠ΅Π²Π°Π΅Ρ‚ Π½Π΅ΠΊΠΎΡ‚ΠΎΡ€Ρ‹Π΅ слоТности, ΠΎΠ΄Π½Π°ΠΊΠΎ, Ссли Π³Ρ€Π°ΠΆΠ΄Π°Π½ΠΈΠ½ заинтСрСсован Π² ΠΎΠ±ΡŠΠ΅ΠΊΡ‚ΠΈΠ²Π½ΠΎΠΌ мСдицинском обслСдовании, ΠΏΡ€ΠΎΠΉΡ‚ΠΈ ΠΏΠ΅Ρ€Π΅ΠΎΡΠ²ΠΈΠ΄Π΅Ρ‚Π΅Π»ΡŒΡΡ‚Π²ΠΎΠ²Π°Π½ΠΈΠ΅, Π² ΠΏΡ€ΠΈΠ½Ρ†ΠΈΠΏΠ΅, особого Ρ‚Ρ€ΡƒΠ΄Π° Π½Π΅ составит.

МСдкомиссию ΠΎΡ‚ΠΌΠ΅Π½ΠΈΠ»ΠΈ ΠΈΠ·-Π·Π° коронавируса: ΠΌΠΎΠΆΠ½ΠΎ Π½Π΅ ΡΠ»ΡƒΠΆΠΈΡ‚ΡŒ?

ΠœΠΈΡ…Π΅Π΅Π²Π° Π•ΠΊΠ°Ρ‚Π΅Ρ€ΠΈΠ½Π°

Π ΡƒΠΊΠΎΠ²ΠΎΠ΄ΠΈΡ‚Π΅Π»ΡŒ ΡŽΡ€ΠΈΠ΄ΠΈΡ‡Π΅ΡΠΊΠΎΠ³ΠΎ ΠΎΡ‚Π΄Π΅Π»Π° Π‘Π»ΡƒΠΆΠ±Ρ‹ ΠŸΠΎΠΌΠΎΡ‰ΠΈ ΠŸΡ€ΠΈΠ·Ρ‹Π²Π½ΠΈΠΊΠ°ΠΌ Π² Π‘Π°Π½ΠΊΡ‚-ΠŸΠ΅Ρ‚Π΅Ρ€Π±ΡƒΡ€Π³Π΅

3Β 414 просмотров

ОбновлСно: 19.05.2020

ΠœΠΈΠ½ΠΈΡΡ‚Π΅Ρ€ΡΡ‚Π²ΠΎ ΠΎΠ±ΠΎΡ€ΠΎΠ½Ρ‹ ΠΎΡ‚ΠΊΠ°Π·Π°Π»ΠΎΡΡŒ ΠΏΠ΅Ρ€Π΅Π½ΠΎΡΠΈΡ‚ΡŒ ΠΏΡ€ΠΈΠ·Ρ‹Π² Π² Π°Ρ€ΠΌΠΈΡŽ, Π½ΠΎ пандСмия коронавируса всС Ρ€Π°Π²Π½ΠΎ повлияла Π½Π° ΠΏΡ€ΠΈΠ·Ρ‹Π²Π½Ρ‹Π΅ мСроприятия. Π‘Π½Π°Ρ‡Π°Π»Π° Π²ΠΎΠ΅Π½ΠΊΠΎΠΌΠ°Ρ‚Ρ‹ ΠΏΠ΅Ρ€Π΅ΡˆΠ»ΠΈ Π½Π° ΡƒΠ΄Π°Π»Π΅Π½Π½ΡƒΡŽ Ρ€Π°Π±ΠΎΡ‚Ρƒ, Π° Ρ‚Π΅ΠΏΠ΅Ρ€ΡŒ стало извСстно, Ρ‡Ρ‚ΠΎ мСдицинскиС комиссии пСрСносятся Π½Π° Π½Π°Ρ‡Π°Π»ΠΎ мая.

Π²ΠΎΠ΅Π½ΠΊΠΎΠΌΠ°Ρ‚ пСрСнСс ΠΏΡ€ΠΈΠ·Ρ‹Π² Π² Π°Ρ€ΠΌΠΈΡŽ

Π’ΠΎΠ΅Π½ΠΊΠΎΠΌΠ°Ρ‚ пСрСнСс мСдкомиссию: Π·Π½Π°Ρ‡ΠΈΡ‚, мСня Π½Π΅ ΠΏΡ€ΠΈΠ·ΠΎΠ²ΡƒΡ‚?

Π’ΠΎΠ΅Π½ΠΊΠΎΠΌΠ°Ρ‚Ρ‹, Π΄Π΅ΠΉΡΡ‚Π²ΠΈΡ‚Π΅Π»ΡŒΠ½ΠΎ, Π½Π°Ρ‡Π°Π»ΠΈ ΠΎΡ‚ΠΌΠ΅Π½ΡΡ‚ΡŒ ΠΈ ΠΏΠ΅Ρ€Π΅Π½ΠΎΡΠΈΡ‚ΡŒ Π΄Π°Ρ‚Ρ‹ ΠΏΡ€ΠΈΠ·Ρ‹Π²Π½Ρ‹Ρ… мСроприятий. НапримСр Π² МосквС, Π‘Π°Π½ΠΊΡ‚-ΠŸΠ΅Ρ‚Π΅Ρ€Π±ΡƒΡ€Π³Π΅, НовосибирскС ΠΈ РостовС мСдицинскиС комиссии ΠΏΠ»Π°Π½ΠΈΡ€ΡƒΡŽΡ‚ ΠΏΡ€ΠΎΠ²ΠΎΠ΄ΠΈΡ‚ΡŒ послС 6 мая. БСйчас мСдицинскиС ΠΎΡΠ²ΠΈΠ΄Π΅Ρ‚Π΅Π»ΡŒΡΡ‚Π²ΠΎΠ²Π°Π½ΠΈΡ проводятся Π² Π΅Π΄ΠΈΠ½ΠΈΡ‡Π½Ρ‹Ρ… комиссариатах ΠΈ Ρ‚ΠΎΠ»ΡŒΠΊΠΎ для Ρ‚Π΅Ρ…, Ρƒ ΠΊΠΎΠ³ΠΎ Π½Π΅Ρ‚ ΠΏΡ€ΠΎΠ±Π»Π΅ΠΌ со Π·Π΄ΠΎΡ€ΠΎΠ²ΡŒΠ΅ΠΌ.

МСдкомиссии ΠΏΠ΅Ρ€Π΅Π½Π΅ΡΠ»ΠΈΡΡŒ, Π½ΠΎ ΠΏΡ€ΠΈΠ·Ρ‹Π² Π² Π°Ρ€ΠΌΠΈΡŽ всё Ρ€Π°Π²Π½ΠΎ состоится. ВсС ΠΏΡ€ΠΈΠ·Ρ‹Π²Π½ΠΈΠΊΠΈ, Ρƒ ΠΊΠΎΡ‚ΠΎΡ€Ρ‹Ρ… Π΅ΡΡ‚ΡŒ подписанная повСстка Π½Π° Π°ΠΏΡ€Π΅Π»ΡŒ, обязаны ΠΏΡ€ΠΎΠΉΡ‚ΠΈ ΠΏΡ€ΠΈΠ·Ρ‹Π²Π½Ρ‹Π΅ мСроприятия сразу ΠΆΠ΅, ΠΊΠ°ΠΊ Ρ‚ΠΎΠ»ΡŒΠΊΠΎ комиссариаты Π½Π°Ρ‡Π½ΡƒΡ‚ ΠΏΡ€ΠΈΡ‘ΠΌ Π³Ρ€Π°ΠΆΠ΄Π°Π½. Если этого Π½Π΅ ΡΠ΄Π΅Π»Π°Ρ‚ΡŒ, Π½Π° Π½ΠΈΡ… ΠΌΠΎΠ³ΡƒΡ‚ завСсти ΡƒΠ³ΠΎΠ»ΠΎΠ²Π½ΠΎΠ΅ Π΄Π΅Π»ΠΎ Π·Π° ΡƒΠΊΠ»ΠΎΠ½Π΅Π½ΠΈΠ΅ ΠΎΡ‚ Π°Ρ€ΠΌΠΈΠΈ.Β 

Π²ΠΎΠ΅Π½ΠΊΠΎΠΌΠ°Ρ‚ пСрСнСс ΠΏΡ€ΠΈΠ·Ρ‹Π² Π² Π°Ρ€ΠΌΠΈΡŽ

Наши ΠΊΠ»ΠΈΠ΅Π½Ρ‚Ρ‹ Π½Π΅ слуТат Π² Π°Ρ€ΠΌΠΈΠΈ

ΠŸΠΎΠ»ΡƒΡ‡ΠΈΡ‚Π΅ Π±Π΅ΡΠΏΠ»Π°Ρ‚Π½ΡƒΡŽ ΠΊΠΎΠ½ΡΡƒΠ»ΡŒΡ‚Π°Ρ†ΠΈΡŽ ΠΏΠΎ Ρ‚Π΅Π»Π΅Ρ„ΠΎΠ½Ρƒ:

ΠΈΠ»ΠΈ Π·Π°ΠΊΠ°ΠΆΠΈΡ‚Π΅ ΠΎΠ±Ρ€Π°Ρ‚Π½Ρ‹ΠΉ Π·Π²ΠΎΠ½ΠΎΠΊ

Если Π²Ρ‹Π·Ρ‹Π²Π°ΡŽΡ‚ Π² Π²ΠΎΠ΅Π½ΠΊΠΎΠΌΠ°Ρ‚ ΠΏΠΎ Ρ‚Π΅Π»Π΅Ρ„ΠΎΠ½Ρƒ ΠΈΠ»ΠΈ элСктронной ΠΏΠΎΡ‡Ρ‚Π΅, ΠΌΠΎΠΆΠ½ΠΎ Π»ΠΈ ΠΏΡ€ΠΎΠΈΠ³Π½ΠΎΡ€ΠΈΡ€ΠΎΠ²Π°Ρ‚ΡŒ?

НСт, нСльзя. Если Ρƒ вас Π΅ΡΡ‚ΡŒ подписанная повСстка Π½Π° Π°ΠΏΡ€Π΅Π»ΡŒ, Ρ‚ΠΎ явка Π±ΡƒΠ΄Π΅Ρ‚ ΠΎΠ±ΡΠ·Π°Ρ‚Π΅Π»ΡŒΠ½ΠΎΠΉ. ΠŸΡ€ΠΈΠ΅Ρ…Π°Ρ‚ΡŒ Π² Π²ΠΎΠ΅Π½ΠΊΠΎΠΌΠ°Ρ‚ придСтся, Ссли Π²Π°ΠΌ позвонят, Π½Π°ΠΏΠΈΡˆΡƒΡ‚ Π² соцсСтях ΠΈΠ»ΠΈ отправят письмо ΠΏΠΎ элСктронной ΠΏΠΎΡ‡Ρ‚Π΅.

МСдкомиссии пСрСнСсли Π½Π° мСсяц: это Ρ…ΠΎΡ€ΠΎΡˆΠΎ ΠΈΠ»ΠΈ ΠΏΠ»ΠΎΡ…ΠΎ?

Для Ρ‚Π΅Ρ…, ΠΊΡ‚ΠΎ Π½Π΅ Ρ…ΠΎΡ‡Π΅Ρ‚ Π² Π°Ρ€ΠΌΠΈΡŽ, такая отсрочка ΠΌΠΎΠΆΠ΅Ρ‚ ΠΏΠΎΠΊΠ°Π·Π°Ρ‚ΡŒΡΡ настоящим Ρ‡ΡƒΠ΄ΠΎΠΌ. Но Π² Π΄Π΅ΠΉΡΡ‚Π²ΠΈΡ‚Π΅Π»ΡŒΠ½ΠΎΡΡ‚ΠΈ Ρ‚Π°ΠΊΠΎΠΉ ΠΏΠΎΠ²ΠΎΡ€ΠΎΡ‚ событий ΠΌΠΎΠΆΠ΅Ρ‚ привСсти ΠΊ появлСнию ΡΠ΅Ρ€ΡŒΠ΅Π·Π½Ρ‹Ρ… ΠΏΡ€ΠΎΠ±Π»Π΅ΠΌ.

Π’ΠΎ-ΠΏΠ΅Ρ€Π²Ρ‹Ρ…, ΠΈΠ· ΠΏΡ€ΠΈΠ·Ρ‹Π²Π½ΠΎΠΉ ΠΊΠ°ΠΌΠΏΠ°Π½ΠΈΠΈ Π²Ρ‹ΠΏΠ°Π» Ρ†Π΅Π»Ρ‹ΠΉ мСсяц. Π§Ρ‚ΠΎΠ±Ρ‹ ΡƒΡΠΏΠ΅Ρ‚ΡŒ Π²Ρ‹ΠΏΠΎΠ»Π½ΠΈΡ‚ΡŒ ΠΏΠ»Π°Π½Ρ‹ ΠΏΠΎ ΠΏΡ€ΠΈΠ·Ρ‹Π²Ρƒ, Π²ΠΎΠ΅Π½ΠΊΠΎΠΌΠ°Ρ‚Π°ΠΌ придСтся ΠΏΡ€ΠΎΠ²ΠΎΠ΄ΠΈΡ‚ΡŒ мСдицинскиС ΠΈ ΠΏΡ€ΠΈΠ·Ρ‹Π²Π½Ρ‹Π΅ комиссии Π² ускорСнном Ρ‚Π΅ΠΌΠΏΠ΅. По нашим ΠΏΡ€ΠΎΠ³Π½ΠΎΠ·Π°ΠΌ, это ΠΌΠΎΠΆΠ΅Ρ‚ привСсти ΠΊ Π±ΠΎΠ»ΡŒΡˆΠΎΠΌΡƒ количСству Π½Π°Ρ€ΡƒΡˆΠ΅Π½ΠΈΠΉ со стороны долТностных Π»ΠΈΡ†.

Π’ΠΎ-Π²Ρ‚ΠΎΡ€Ρ‹Ρ…, ΠΈΠ·-Π·Π° ситуации с коронавирусом Ρ‡Π°ΡΡ‚ΡŒ мСдицинских ΡƒΡ‡Ρ€Π΅ΠΆΠ΄Π΅Π½ΠΈΠΉ ΠΏΡ€ΠΈΠ½ΠΈΠΌΠ°Π΅Ρ‚ Ρ‚ΠΎΠ»ΡŒΠΊΠΎ экстрСнных Π±ΠΎΠ»ΡŒΠ½Ρ‹Ρ…. Π’ Π½Π΅ΠΊΠΎΡ‚ΠΎΡ€Ρ‹Ρ… Ρ€Π΅Π³ΠΈΠΎΠ½Π°Ρ… ΠΏΡ€ΠΈΠ·Ρ‹Π²Π½ΠΈΠΊΠΈ просто Π½Π΅ ΠΌΠΎΠ³ΡƒΡ‚ ΠΏΡ€ΠΎΠΉΡ‚ΠΈ ΠΏΠ»Π°Π½ΠΎΠ²ΠΎΠ΅ обслСдованиС ΠΈ ΠΏΠΎΠ»ΡƒΡ‡ΠΈΡ‚ΡŒ мСдицинскиС Π΄ΠΎΠΊΡƒΠΌΠ΅Π½Ρ‚Ρ‹ β€” Ρ‚.Π΅. Π΄ΠΎΠΊΡƒΠΌΠ΅Π½Ρ‚Ρ‹, Π±Π΅Π· ΠΊΠΎΡ‚ΠΎΡ€Ρ‹Ρ… ΠΏΡ€ΠΎΠ±Π»Π΅ΠΌΠ°Ρ‚ΠΈΡ‡Π½ΠΎ ΠΎΡΠ²ΠΎΠ±ΠΎΠ΄ΠΈΡ‚ΡŒΡΡ ΠΎΡ‚ ΠΏΡ€ΠΈΠ·Ρ‹Π²Π°. По этой ΠΆΠ΅ ΠΏΡ€ΠΈΡ‡ΠΈΠ½Π΅ ΠΏΡ€ΠΈΠ·Ρ‹Π²Π½ΠΈΠΊΠ°ΠΌ ΠΌΠΎΠ³ΡƒΡ‚ ΠΎΡ‚ΠΊΠ°Π·Π°Ρ‚ΡŒ Π² Π΄ΠΎΠΏΠΎΠ»Π½ΠΈΡ‚Π΅Π»ΡŒΠ½ΠΎΠΌ обслСдовании, Π²Ρ‹ΡΡ‚Π°Π²ΠΈΡ‚ΡŒ ΠΊΠ°Ρ‚Π΅Π³ΠΎΡ€ΠΈΡŽ годности Π½Π° основании старых Π΄ΠΎΠΊΡƒΠΌΠ΅Π½Ρ‚ΠΎΠ² ΠΈ ΠΏΡ€ΠΈΠ·Π²Π°Ρ‚ΡŒ Π² войска.

ΠœΠΎΠ³ΡƒΡ‚ Π»ΠΈ ΠΏΡ€ΠΎΠ΄Π»ΠΈΡ‚ΡŒ сроки ΠΏΡ€ΠΈΠ·Ρ‹Π²Π°?

Пока ΠœΠΈΠ½ΠΈΡΡ‚Π΅Ρ€ΡΡ‚Π²ΠΎ ΠΎΠ±ΠΎΡ€ΠΎΠ½Ρ‹ Π½Π΅ сообщало ΠΎ ΠΏΡ€ΠΎΠ΄Π»Π΅Π½ΠΈΠΈ сроков ΠΏΡ€ΠΈΠ·Ρ‹Π²Π°. Однако, Ссли ΠΊΠ°Ρ€Π°Π½Ρ‚ΠΈΠ½ затянСтся ΠΈ Π²ΠΎΠ΅Π½ΠΊΠΎΠΌΠ°Ρ‚Ρ‹ Π½Π΅ ΡƒΡΠΏΠ΅ΡŽΡ‚ Π²Ρ‹ΠΏΠΎΠ»Π½ΠΈΡ‚ΡŒ ΠΏΠ»Π°Π½ ΠΏΠΎ Π½ΠΎΠ²ΠΎΠ±Ρ€Π°Π½Ρ†Π°ΠΌ, Ρ‚Π°ΠΊΠΎΠΉ исход Π²ΠΏΠΎΠ»Π½Π΅ Π²ΠΎΠ·ΠΌΠΎΠΆΠ΅Π½ ΠΈ ΠΎΠΏΡ€Π°Π²Π΄Π°Π½.

Β 


Π‘ ΡƒΠ²Π°ΠΆΠ΅Π½ΠΈΠ΅ΠΌ ΠΊ Π²Π°ΠΌ, ΠœΠΈΡ…Π΅Π΅Π²Π° Π•ΠΊΠ°Ρ‚Π΅Ρ€ΠΈΠ½Π°, Ρ€ΡƒΠΊΠΎΠ²ΠΎΠ΄ΠΈΡ‚Π΅Π»ΡŒ ΡŽΡ€ΠΈΠ΄ΠΈΡ‡Π΅ΡΠΊΠΎΠ³ΠΎ ΠΎΡ‚Π΄Π΅Π»Π° Π‘Π»ΡƒΠΆΠ±Ρ‹ ΠŸΠΎΠΌΠΎΡ‰ΠΈ ΠŸΡ€ΠΈΠ·Ρ‹Π²Π½ΠΈΠΊΠ°ΠΌ.
ПомогаСм ΠΏΡ€ΠΈΠ·Ρ‹Π²Π½ΠΈΠΊΠ°ΠΌ ΠΏΠΎΠ»ΡƒΡ‡Π°Ρ‚ΡŒ Π²ΠΎΠ΅Π½Π½Ρ‹ΠΉ Π±ΠΈΠ»Π΅Ρ‚ Π½Π° Π·Π°ΠΊΠΎΠ½Π½Ρ‹Ρ… основаниях: 8 (800) 333-53-63

ΠŸΠΎΠ²Ρ‚ΠΎΡ€Π½Π°Ρ мСдкомиссия Π²ΠΎΠ΅Π½Π½Ρ‹ΠΉ Π±ΠΈΠ»Π΅Ρ‚ — 59 совСтов Π°Π΄Π²ΠΎΠΊΠ°Ρ‚ΠΎΠ² ΠΈ ΡŽΡ€ΠΈΡΡ‚ΠΎΠ²

1. ΠŸΠΎΠ»ΡƒΡ‡ΠΈΠ» Π²ΠΎΠ΅Π½Π½Ρ‹ΠΉ Π±ΠΈΠ»Π΅Ρ‚ ΠΏΠΎ ΠΊΠ°Ρ‚Π΅Π³ΠΎΡ€ΠΈΠΈ Π’ Π² Ρ€Π΅Π³ΠΈΠΎΠ½Π΅ ΠΏΠΎ Π²Ρ€Π΅ΠΌΠ΅Π½Π½ΠΎΠΉ рСгистрации, Π²ΠΎΠ΅Π½ΠΊΠΎΠΌΠ°Ρ‚ ΠΏΠΎ постоянной Π² Π΄Ρ€ΡƒΠ³ΠΎΠΌ Ρ€Π΅Π³ΠΈΠΎΠ½Π΅ нСсколько Π»Π΅Ρ‚ ΠΏΡ€ΠΈΠ³Π»Π°ΡˆΠ°Π΅Ρ‚ ΠΏΡ€ΠΈΠΉΡ‚ΠΈ ΠΊ Π½ΠΈΠΌ ΠΈ, Π²ΠΈΠ΄ΠΈΠΌΠΎ, Π²ΡΡ‚Π°Ρ‚ΡŒ Π½Π° ΡƒΡ‡Ρ‘Ρ‚ Ρƒ Π½ΠΈΡ…. ΠœΠΎΠ³Ρƒ Π»ΠΈ я ΠΏΡ€ΠΈΠΉΡ‚ΠΈ ΠΈ ΠΏΠΎΠΊΠ°Π·Π°Ρ‚ΡŒ Π²ΠΎΠ΅Π½Π½Ρ‹ΠΉ Π±ΠΈΠ»Π΅Ρ‚ ΠΈ ΡƒΠΉΡ‚ΠΈ? ΠœΠΎΠ³ΡƒΡ‚ Π»ΠΈ ΠΎΠ½ΠΈ Π·Π°ΠΏΡ€ΠΎΡΠΈΡ‚ΡŒ ΠΏΠΎΠ²Ρ‚ΠΎΡ€Π½ΡƒΡŽ мСдкомиссию? ΠžΠ±ΡΠ·Π°Ρ‚Π΅Π»ΡŒΠ½ΠΎ Π»ΠΈ Π²ΡΡ‚Π°Π²Π°Ρ‚ΡŒ Ρƒ Π½ΠΈΡ… Π½Π° ΡƒΡ‡Ρ‘Ρ‚?

1.1. Бтранная ситуация. Π›ΠΈΡ‡Π½ΠΎΠ΅ Π΄Π΅Π»ΠΎ Ρ‚ΠΎΠ»ΡŒΠΊΠΎ ΠΎΠ΄Π½ΠΎ ΠΈ Π΄ΠΎΠ»ΠΆΠ½ΠΎ Π½Π°Ρ…ΠΎΠ΄ΠΈΡ‚ΡŒΡΡ ΠΏΠΎ мСсту вашСго воинского ΡƒΡ‡Π΅Ρ‚Π°.
Π’Ρ‹ Π΄ΠΎΠ»ΠΆΠ½Ρ‹ ΡΡ‚ΠΎΡΡ‚ΡŒ Π½Π° воинском ΡƒΡ‡Π΅Ρ‚Π΅ ΠΏΠΎ мСсту фактичСского ΠΆΠΈΡ‚Π΅Π»ΡŒΡΡ‚Π²Π°. ΠŸΡ€ΠΈ этом Π½Π°Π»ΠΈΡ‡ΠΈΠ΅/отсутствиС рСгистрации Ρ€ΠΎΠ»ΠΈ Π½Π΅ ΠΈΠ³Ρ€Π°Π΅Ρ‚ с 2019 Π³.
ΠœΠΎΠΆΠ΅Ρ‚Π΅ Π·Π°ΠΏΡ€ΠΎΡΠΈΡ‚ΡŒ Ρƒ Π²ΠΎΠ΅Π½ΠΊΠΎΠΌΠ°Ρ‚Π°, Π³Π΄Π΅ стоитС Π½Π° Π’Π£, копию Ρ€Π΅ΡˆΠ΅Π½ΠΈΡ ΠΏΡ€ΠΈΠ·Ρ‹Π²Π½ΠΎΠΉ комиссии ΠΎ зачислСнии вас Π² запас ΠΈ Π²Ρ‹ΡΠ»Π°Ρ‚ΡŒ Π² Π²ΠΎΠ΅Π½ΠΊΠΎΠΌΠ°Ρ‚ ΠΏΠΎ мСсту постоянной рСгистрации.

2. Π― ΠΏΡ€ΠΎΡˆΠ΅Π» мСдкомиссию Π² Π²ΠΎΠ΅Π½ΠΊΠΎΠΌΠ°Ρ‚Π΅ Π·Π°Π±Ρ€Π°Π» Π²ΠΎΠ΅Π½Π½Ρ‹ΠΉ Π±ΠΈΠ»Π΅Ρ‚ Ρ‚Π°ΠΌ Написали ΠΌΠ½Π΅ Ρ‡Ρ‚ΠΎ я ΠΎΠ³Ρ€Π°Π½ΠΈΡ‡Π΅Π½ Π“ΠΎΠ΄Π΅Π½ я ΠΌΠΎΠ³Ρƒ ΠΏΠΎΠ²Ρ‚ΠΎΡ€Π½ΡƒΡŽ мСдкомиссию ΠΏΡ€ΠΎΠΉΡ‚ΠΈ Ρ‚.

2.1. АртСм, Π½Π°ΠΏΠΈΡˆΠΈΡ‚Π΅ Π² Ρ‡Π΅ΠΌ ваш вопрос.
Какая катСгория записана Π² вашСм Π±ΠΈΠ»Π΅Ρ‚Π΅?

3. ЗдрастС, Ρƒ мСня Π²ΠΎΡ‚ Ρ‚Π°ΠΊΠΎΠΉ вопрос, ΠΌΠΎΠ³Ρƒ Π»ΠΈ я Π²Ρ‚ΠΎΡ€ΠΎΠΉ Ρ€Π°Π· ΠΏΡ€ΠΎΠΉΡ‚ΠΈ мСдкомиссию?, Ссли я 5 Π»Π΅Ρ‚ Π½Π°Π·Π°Π΄ откосил ΠΎΡ‚ Π°Ρ€ΠΌΠΈΠΈ ΠΈ ΠΌΠ½Π΅ Π΄Π°Π»ΠΈ Π²ΠΎΠ΅Π½Π½Ρ‹ΠΉ Π±ΠΈΠ»Π΅Ρ‚ ΠΈ приписали Π΄ΠΈΠ°Π³Π½ΠΎΠ· психичСскоС расстройство. Π’Π΅ΠΏΠ΅Ρ€ΡŒ я Π½Π΅ ΠΌΠΎΠ³Ρƒ ΠΏΡ€Π°Π²Π° ΠΏΠΎΠ»ΡƒΡ‡ΠΈΡ‚ΡŒ Π½Π° ΠΌΠ°ΡˆΠΈΠ½Ρƒ Ρ‚ΠΎΡ‡Π½Π΅ΠΉ справку Π² Π±ΠΎΠ»ΡŒΠ½ΠΈΡ†Π΅, ΠΌΠ½Π΅ говорят Ρ‡Ρ‚ΠΎ ΠΌΠ½Π΅ Π½ΡƒΠΆΠ½ΠΎ ΠΏΠΎΠ΄Π°Π²Π°Ρ‚ΡŒ заявлСниС Π² суд Π½Π° рассмотрСниС ΠΏΠΎΠ²Ρ‚ΠΎΡ€Π½ΡƒΡŽ комиссию, Ссли я Π΄Π°ΠΆΠ΅ ΠΏΡ€ΠΎΠΉΠ΄Ρƒ мСдкомиссию Π²ΠΎΠ΅Π½ΠΊΠΎΠΌΠ°Ρ‚Π΅ ΠΈ мСня ΠΏΡ€ΠΈΠ·Π½Π°ΡŽΡ‚ Π°Π΄Π΅ΠΊΠ²Π°Ρ‚Π½Ρ‹ΠΌ, мСня ΠΌΠΎΠ³ΡƒΡ‚ Π² Π°Ρ€ΠΌΠΈΡŽ Π·Π°Π±Ρ€Π°Ρ‚ΡŒ? ΠŸΠΎΠ΄ΡΠΊΠ°ΠΆΠΈΡ‚Π΅ поТалуйста.

3.1. ЗдравствуйтС.

Π’Π΅ΠΏΠ΅Ρ€ΡŒ ΡƒΠΆΠ΅ Π½Π΅ ΠΌΠΎΠΆΠ΅Ρ‚Π΅.

3.2. Всё Π²Π΅Ρ€Π½ΠΎ. Если с Вас снимут Π΄ΠΈΠ°Π³Π½ΠΎΠ· Π’Π°ΠΌ придётся ΠΎΡ‚ΡΠ»ΡƒΠΆΠΈΡ‚ΡŒ. Но Π·Π°Ρ‚ΠΎ Π’Ρ‹ Π±ΡƒΠ΄Π΅Ρ‚Π΅ с ΠΏΡ€Π°Π²Π°ΠΌΠΈ.

4. МнС 34 Π³ΠΎΠ΄Π° я Π±Ρ‹Π» Π½Π΅ Π³ΠΎΠ΄Π΅Π½ ΠΊ Π²ΠΎΠ΅Π½Π½ΠΎΠΉ слуТбС Ρƒ мСня Π±Ρ‹Π» выявлСн хроничСский Π³Π΅ΠΏΠ°Ρ‚ΠΈΡ‚ Π‘, Π½Π΅Π΄Π°Π²Π½ΠΎ Π±Ρ‹Π» Π² Π²ΠΎΠ΅Π½ΠΊΠΎΠΌΠ°Ρ‚Π΅ Ρ‡Ρ‚ΠΎΠ±Ρ‹ ΠΏΠΎΠ»ΡƒΡ‡ΠΈΡ‚ΡŒ Π²ΠΎΠ΅Π½Π½Ρ‹ΠΉ Π±ΠΈΠ»Π΅Ρ‚ ΠΏΡ€ΠΎΡˆΡ‘Π» ΠΏΠΎΠ²Ρ‚ΠΎΡ€Π½ΠΎ мСдкомиссию сдал Π°Π½Π°Π»ΠΈΠ· ΠΊΡ€ΠΎΠ²ΠΈ Π° Π³Π΅ΠΏΠ°Ρ‚ΠΈΡ‚Π° Π½Π΅Ρ‚, я Π΄ΠΎ этого ΠΏΡ€ΠΎΠΏΠΈΠ²Π°Π» лСкарства Π΄ΠΎΡ€ΠΎΠ³ΠΈΠ΅ ΠΎΡ‚ лСчСния Π³Π΅ΠΏΠ°Ρ‚ΠΈΡ‚Π° ΠΏΠΎΡ‚Ρ€Π΅Π±ΠΎΠ²Π°Π»ΠΈ ΠΏΠΎΠ΄Ρ‚Π²Π΅Ρ€ΠΆΠ΄Π΅Π½ΠΈΠ΅ ΠΎΡ‚ Π±ΠΎΠ»ΡŒΠ½ΠΈΡ†Ρ‹ Π³Π΄Π΅ сдавал Π°Π½Π°Π»ΠΈΠ· ΠΊΡ€ΠΎΠ²ΠΈ.

4.1. ЗдравствуйтС! Π’Ρ‹ вопрос сформулируйтС. Π§Π΅Π³ΠΎ Π’Ρ‹ Ρ…ΠΎΡ‚ΠΈΡ‚Π΅ Π΄ΠΎΠ±ΠΈΡ‚ΡŒΡΡ?

5. Π’ 2014 Π³ΠΎΠ΄Ρƒ ΠΏΡ€ΠΎΡ…ΠΎΠ΄ΠΈΠ» мСдкомиссию Π½Π° ΠΏΠΎΠ»ΡƒΡ‡Π΅Π½ΠΈΠ΅ Π²ΠΎΠ΅Π½Π½ΠΎΠ³ΠΎ Π±ΠΈΠ»Π΅Ρ‚Π° ΠΈ Π²ΠΎΠ΄ΠΈΡ‚Π΅Π»ΡŒΡΠΊΠΈΡ… ΠΏΡ€Π°Π² ΠΌΠ½Π΅ Π²Ρ‹Π΄Π°Π»ΠΈ всС справки Π½Π°Ρ€ΠΊΠΎΠ»ΠΎΠ³ ΠΈ психиатр сСйчас Ρ‡Π΅Ρ€Π΅Π· Ρ‚Ρ€ΠΈ Π³ΠΎΠ΄Π° ΠΏΠΎΠ²Ρ‚ΠΎΡ€Π½ΠΎ Π±Ρ€Π°Π» ΠΌΠ΅Π΄ справку Π½Π° ΠΏΡ€Π°Π²Π° ΠΈ ΠΌΠ½Π΅ сказали Ρ‡Ρ‚ΠΎ я ΡΡ‚Π°ΡŽ Ρƒ Π½Π°Ρ€ΠΊΠΎΠ»ΠΎΠ³Π° Π½Π° ΡƒΡ‡Π΅Ρ‚Π΅ с 2010 Π³ΠΎΠ΄Π° ΠΏΠΎΠ΅Ρ…Π°Π» ΡƒΠ·Π½Π°Π²Π°Ρ‚ΡŒ ΡΠΎΠ²ΠΏΠ°Π΄Π°ΡŽΡ‚ ΠΈΠ½ΠΈΡ†ΠΈΠ°Π»Ρ‹ Π° Π΄Π°Ρ‚Π° роТдСния ΠΈ адрСс проТивания Π΄Ρ€ΡƒΠ³ΠΈΠ΅ всС Ρ€Π°Π²Π½ΠΎ справок Π½Π΅ Π΄Π°Π»ΠΈ ΠΏΠΎΡ‚Ρ€Π΅Π±ΠΎΠ²Π°Π»ΠΈ характСристика участкового Π±ΡƒΠ΄Π΅Ρ‚ Ρ€Π΅ΡˆΠ°Ρ‚ΡŒ комиссия Ρ‡Ρ‚ΠΎ ΠΌΠΎΠΆΠ½ΠΎ ΡΠ΄Π΅Π»Π°Ρ‚ΡŒ.

5.1. Π”ΠΎΠ±Ρ€Ρ‹ΠΉ дСнь, ΡƒΠ²Π°ΠΆΠ°Π΅ΠΌΡ‹ΠΉ АндрСй
Надо Ρ‚Ρ€Π΅Π±ΠΎΠ²Π°Ρ‚ΡŒ ΠΏΡ€ΠΈΠ·Π½Π°Ρ‚ΡŒ Ρ‡Ρ‚ΠΎ Ρ‡Π΅Π»ΠΎΠ²Π΅ΠΊ находящийся Π½Π° ΡƒΡ‡Π΅Ρ‚Π΅ Π½Π΅ Π²Ρ‹ Π° ваш ΠΎΠ΄Π½ΠΎΡ„Π°ΠΌΠΈΠ»Π΅Ρ†

Π£Π΄Π°Ρ‡ΠΈ Π²Π°ΠΌ ΠΈ вашим Π±Π»ΠΈΠ·ΠΊΠΈΠΌ.

5.2. Π’Π°ΠΌ Π½ΡƒΠΆΠ½ΠΎ ΠΎΠ±Ρ€Π°Ρ‚ΠΈΡ‚ΡŒΡΡ Π² ΠΏΡ€ΠΎΠΊΡƒΡ€Π°Ρ‚ΡƒΡ€Ρƒ Π·Π° Π·Π°Ρ‰ΠΈΡ‚ΠΎΠΉ своих ΠΏΡ€Π°Π². ΠŸΠΎΡ‡Π΅ΠΌΡƒ Π²Ρ‹ Π΄ΠΎΠ»ΠΆΠ½Ρ‹ ΠΎΡ‚Π²Π΅Ρ‡Π°Ρ‚ΡŒ Π·Π° Π΄Π°Π½Π½Ρ‹Π΅ Π°Π±ΡΠΎΠ»ΡŽΡ‚Π½ΠΎ Π΄Ρ€ΡƒΠ³ΠΎΠ³ΠΎ Ρ‡Π΅Π»ΠΎΠ²Π΅ΠΊΠ°. ВсСго Π΄ΠΎΠ±Ρ€ΠΎΠ³ΠΎ.

6. На Ρ€Π°Π±ΠΎΡ‚Π΅ Π·Π°ΡΡ‚Π°Π²Π»ΡΡŽΡ‚ ΠΏΡ€ΠΎΡ…ΠΎΠ΄ΠΈΡ‚ΡŒ (ΠΏΠΎ ΠΏΡ€ΠΈΠΊΠ°Π·Ρƒ) психиатричСскоС ΠΎΡΠ²ΠΈΠ΄Π΅Ρ‚Π΅Π»ΡŒΡΡ‚Π²ΠΎΠ²Π°Π½ΠΈΠ΅. Π₯отя мСдкомиссию я ΠΏΡ€ΠΎΡ…ΠΎΠ΄ΠΈΠ» мСньшС Π³ΠΎΠ΄Π° Π½Π°Π·Π°Π΄ ΠΈ СстСствСнно психиатра ΠΈ Π½Π°Ρ€ΠΊΠΎΠ»ΠΎΠ³Π° ΠΏΡ€ΠΎΡ…ΠΎΠ΄ΠΈΠ» Ρ‚ΠΎΠΆΠ΅. Π’Π΅ΠΏΠ΅Ρ€ΡŒ мСня ΠΎΠ±ΡΠ·Ρ‹Π²Π°ΡŽΡ‚ Π²Π·ΡΡ‚ΡŒ справку с мСста ΠΆΠΈΡ‚Π΅Π»ΡŒΡΡ‚Π²Π° ΠΎΡ‚ психиатра, Π΄ΠΎΠΊΡƒΠΌΠ΅Π½Ρ‚Ρ‹ (паспорт, Π²ΠΎΠ΅Π½Π½Ρ‹ΠΉ Π±ΠΈΠ»Π΅Ρ‚ ΠΈ Ρ‚.Π΄.) ΠΈ ΠΏΠΎ мСсту нахоТдСния работодатСля ΠΏΡ€ΠΎΠΉΡ‚ΠΈ всС это ΠΏΠΎΠ²Ρ‚ΠΎΡ€Π½ΠΎ. Π—Π°Ρ‚Ρ€Π°Ρ‚Ρ‹ Π½Π° справку ΠΎΠΏΠ»Π°Ρ‡ΠΈΠ²Π°ΡŽΡ‚, врСмя для прохоТдСния Π²Ρ‹Π΄Π΅Π»ΡΡŽΡ‚, ΡΠΎΠ±ΠΈΡ€Π°ΡŽΡ‚ΡΡ Π·Π°Π½ΠΎΡΠΈΡ‚ΡŒ Ρ€Π΅Π·ΡƒΠ»ΡŒΡ‚Π°Ρ‚Ρ‹ Π² Π±Π°Π·Ρƒ 1 Π‘-БУП. Насколько всС это ΠΏΡ€Π°Π²ΠΎΠΌΠ΅Ρ€Π½ΠΎ ΠΈ Π·Π°ΠΊΠΎΠ½Π½ΠΎ?

6.1. ЗдравствуйтС. НС имСя ΠΈΠ½Ρ„ΠΎΡ€ΠΌΠ°Ρ†ΠΈΠΈ ΠΎ мСстС Ρ€Π°Π±ΠΎΡ‚Ρ‹, долТности, спСцификС Ρ€Π°Π±ΠΎΡ‚, ΠΈ Ρ‚.ΠΏ. — ΡΡƒΠ΄ΠΈΡ‚ΡŒ ΠΎ законности Π½Π΅Π²ΠΎΠ·ΠΌΠΎΠΆΠ½ΠΎ. ΠžΠ±Ρ€Π°Ρ‚ΠΈΡ‚Π΅ΡΡŒ Π² ΠΏΡ€ΠΎΠΊΡƒΡ€Π°Ρ‚ΡƒΡ€Ρƒ ΠΈ Ρ‚Ρ€ΡƒΠ΄ΠΎΠ²ΡƒΡŽ ΠΈΠ½ΡΠΏΠ΅ΠΊΡ†ΠΈΡŽ — ΠΎΠ½ΠΈ запросят основания для направлСния Π½Π° ΠΎΡΠ²ΠΈΠ΄Π΅Ρ‚Π΅Π»ΡŒΡΡ‚Π²ΠΎΠ²Π°Π½ΠΈΠ΅.
ВсСго Π΄ΠΎΠ±Ρ€ΠΎΠ³ΠΎ. Бпасибо, Ρ‡Ρ‚ΠΎ Π²Ρ‹Π±Ρ€Π°Π»ΠΈ наш сайт.

7. На Ρ€Π°Π±ΠΎΡ‚Π΅ Π·Π°ΡΡ‚Π°Π²Π»ΡΡŽΡ‚ ΠΏΡ€ΠΎΡ…ΠΎΠ΄ΠΈΡ‚ΡŒ (ΠΏΠΎ ΠΏΡ€ΠΈΠΊΠ°Π·Ρƒ) психиатра. Π₯отя мСдкомиссию я ΠΏΡ€ΠΎΡ…ΠΎΠ΄ΠΈΠ» мСньшС Π³ΠΎΠ΄Π° Π½Π°Π·Π°Π΄ ΠΈ СстСствСнно психиатра ΠΈ Π½Π°Ρ€ΠΊΠΎΠ»ΠΎΠ³Π° ΠΏΡ€ΠΎΡ…ΠΎΠ΄ΠΈΠ» Ρ‚ΠΎΠΆΠ΅. Π’Π΅ΠΏΠ΅Ρ€ΡŒ мСня ΠΎΠ±ΡΠ·Ρ‹Π²Π°ΡŽΡ‚ Π²Π·ΡΡ‚ΡŒ справку с мСста ΠΆΠΈΡ‚Π΅Π»ΡŒΡΡ‚Π²Π° ΠΎΡ‚ психиатра, Π΄ΠΎΠΊΡƒΠΌΠ΅Π½Ρ‚Ρ‹ (паспорт, Π²ΠΎΠ΅Π½Π½Ρ‹ΠΉ Π±ΠΈΠ»Π΅Ρ‚ ΠΈ Ρ‚.Π΄.) ΠΈ ΠΏΠΎ мСсту нахоТдСния работодатСля ΠΏΡ€ΠΎΠΉΡ‚ΠΈ всС это ΠΏΠΎΠ²Ρ‚ΠΎΡ€Π½ΠΎ. Π—Π°Ρ‚Ρ€Π°Ρ‚Ρ‹ Π½Π° справку ΠΎΠΏΠ»Π°Ρ‡ΠΈΠ²Π°ΡŽΡ‚, врСмя для прохоТдСния Π²Ρ‹Π΄Π΅Π»ΡΡŽΡ‚, ΡΠΎΠ±ΠΈΡ€Π°ΡŽΡ‚ΡΡ Π·Π°Π½ΠΎΡΠΈΡ‚ΡŒ Ρ€Π΅Π·ΡƒΠ»ΡŒΡ‚Π°Ρ‚Ρ‹ Π² Π±Π°Π·Ρƒ 1 Π‘-БУП. Насколько всС это ΠΏΡ€Π°Π²ΠΎΠΌΠ΅Ρ€Π½ΠΎ ΠΈ Π·Π°ΠΊΠΎΠ½Π½ΠΎ?

7.1. Π”ΠΎΠ±Ρ€ΠΎΠ³ΠΎ Π’Ρ€Π΅ΠΌΠ΅Π½ΠΈ суток!

ВрСбования работодатСля ΠΏΠΎ ΠΏΡ€ΠΎΡ…ΠΎΠΆΠ΄Π΅Π½ΠΈΡŽ мСдкомиссии Π²ΠΏΠΎΠ»Π½Π΅ Π·Π°ΠΊΠΎΠ½Π½Ρ‹
Π£Π΄Π°Ρ‡ΠΈ Π’ΠΠœ! ВсСгда Ρ€Π°Π΄Ρ‹ ΠΏΠΎΠΌΠΎΡ‡ΡŒ.

8. Π’ приписном стоит катСгория Π’, ΠΈΠ·-Π·Π° атопичСского Π΄Π΅Ρ€ΠΌΠ°Ρ‚ΠΈΡ‚Π° ΠΈ Π½ΠΈΠ·ΠΊΠΎΠ³ΠΎ ИМ. ΠŸΠΎΠ»ΡƒΡ‡ΠΈΠ» отсрочку для поступлСния Π² институт, Π½ΠΎ Π²ΠΎΠ΅Π½Π½Ρ‹ΠΉ Π±ΠΈΠ»Π΅Ρ‚ Π½Π΅ Π²Ρ‹Π΄Π°Π»ΠΈ. ПослС окончания Π±Π°ΠΊΠ°Π»Π°Π²Ρ€ΠΈΠ°Ρ‚Π° Π²ΠΎΠ΅Π½ΠΊΠΎΠΌΠ°Ρ‚ Π½Π΅ бСспокоил, поступил Π² магистратуру ΠΈ сСйчас Π΅Ρ‘ Π·Π°ΠΊΠ°Π½Ρ‡ΠΈΠ²Π°ΡŽ. Как ΠΌΠ½Π΅ ΠΏΠΎΠ»ΡƒΡ‡ΠΈΡ‚ΡŒ Π²ΠΎΠ΅Π½Π½Ρ‹ΠΉ Π±ΠΈΠ»Π΅Ρ‚? ΠŸΡ€ΠΈΠ΄Ρ‘Ρ‚ΡΡ Π»ΠΈ ΠΏΡ€ΠΎΡ…ΠΎΠ΄ΠΈΡ‚ΡŒ мСдкомиссию ΠΏΠΎΠ²Ρ‚ΠΎΡ€Π½ΠΎ?

8.1. ЗдравствуйтС! Π’ Π΄Π°Π½Π½ΠΎΠΌ случаС Π²Π°ΠΌ придСтся ΠΏΡ€ΠΎΠΉΡ‚ΠΈ ΠΏΠΎΠ²Ρ‚ΠΎΡ€Π½ΠΎ ΠΌΠ΅Π΄ комиссию. Π’Π°ΠΊΠΆΠ΅ Π²Π°ΠΌ слСдуСт ΡΠΎΠ±Ρ€Π°Ρ‚ΡŒ всС Π΄ΠΎΠΊΡƒΠΌΠ΅Π½Ρ‚Ρ‹ ΠΏΠΎ ΠΏΠΎΠ²ΠΎΠ΄Ρƒ Π΄Π°Π½Π½ΠΎΠΉ Π±ΠΎΠ»Π΅Π·Π½ΠΈ.

9. Π£ сына Π΅ΡΡ‚ΡŒ Π²ΠΎΠ΅Π½Π½Ρ‹ΠΉ Π±ΠΈΠ»Π΅Ρ‚ с ΠΎΡ‚ΠΌΠ΅Ρ‚ΠΊΠΎΠΉ ΠΎΠ³Ρ€Π°Π½ΠΈΡ‡Π΅Π½Π½ΠΎ Π³ΠΎΠ΄Π΅Π½. Π‘Ρ‹Π½ судим, послС освобоТдСния ΠΏΠΎΠ΅Ρ…Π°Π» Π² Π²ΠΎΠ΅Π½ΠΊΠΎΠΌΠ°Ρ‚, Π³Π΄Π΅ Π΅ΠΌΡƒ сказали Ρ‡Ρ‚ΠΎ Π΅Π³ΠΎ ΠΈΠ· Π·Π° судимости сняли с ΡƒΡ‡Π΅Ρ‚Π°, ΠΈ Π΄Π°Π»ΠΈ Π½Π°ΠΏΡ€Π°Π²Π»Π΅Π½ΠΈΠ΅ Π½Π° мСдкомиссию для ΠΏΠΎΠ²Ρ‚ΠΎΡ€Π½ΠΎΠ³ΠΎ становлСния Π½Π° ΡƒΡ‡Π΅Ρ‚. ΠœΠΎΠ³ΡƒΡ‚ Π»ΠΈ Π΅Π³ΠΎ ΠΏΡ€ΠΈΠ·Π²Π°Ρ‚ΡŒ Π² Π°Ρ€ΠΌΠΈΡŽ?

9.1. Если ΠΏΡ€ΠΈΠ·Π½Π°ΡŽΡ‚, Ρ‡Ρ‚ΠΎ Π³ΠΎΠ΄Π΅Π½, Π° возраст ΠΌΠ΅Π½Π΅Π΅ 27 Π»Π΅Ρ‚ — ΠΌΠΎΠ³ΡƒΡ‚.


10. Π£ сына Π΅ΡΡ‚ΡŒ Π²ΠΎΠ΅Π½Π½Ρ‹ΠΉ Π±ΠΈΠ»Π΅Ρ‚ с ΠΎΡ‚ΠΌΠ΅Ρ‚ΠΊΠΎΠΉ ΠΎΠ³Ρ€Π°Π½ΠΈΡ‡Π΅Π½Π½ΠΎ Π³ΠΎΠ΄Π΅Π½. Π‘Ρ‹Π½ судим, послС освобоТдСния ΠΏΠΎΠ΅Ρ…Π°Π» Π² Π²ΠΎΠ΅Π½ΠΊΠΎΠΌΠ°Ρ‚, Π³Π΄Π΅ Π΅ΠΌΡƒ сказали Ρ‡Ρ‚ΠΎ Π΅Π³ΠΎ ΠΈΠ· Π·Π° судимости сняли с ΡƒΡ‡Π΅Ρ‚Π°, ΠΈ Π΄Π°Π»ΠΈ Π½Π°ΠΏΡ€Π°Π²Π»Π΅Π½ΠΈΠ΅ Π½Π° мСдкомиссию для ΠΏΠΎΠ²Ρ‚ΠΎΡ€Π½ΠΎΠ³ΠΎ становлСния Π½Π° ΡƒΡ‡Π΅Ρ‚. ΠœΠΎΠ³ΡƒΡ‚ Π»ΠΈ Π΅Π³ΠΎ ΠΏΡ€ΠΈΠ·Π²Π°Ρ‚ΡŒ Π² Π°Ρ€ΠΌΠΈΡŽ?

10.1. ЗдравствуйтС. Π”Π° ΠΌΠΎΠ³ΡƒΡ‚.

11. НуТно Π»ΠΈ ΠΏΡ€ΠΎΡ…ΠΎΠ΄ΠΈΡ‚ΡŒ мСдкомиссию для ΠΏΠΎΠ²Ρ‚ΠΎΡ€Π½ΠΎΠ³ΠΎ получСния Π²ΠΎΠ΅Π½Π½ΠΎΠ³ΠΎ Π±ΠΈΠ»Π΅Ρ‚Π° Ссли Π΅ΡΡ‚ΡŒ справка ΠΎ Π·Π°Π±ΠΎΠ»Π΅Π²Π°Π½ΠΈΠΈ Ρ‚ΡƒΠ±Π΅Ρ€ΠΊΡƒΠ»Ρ‘Π·Π° Π»Ρ‘Π³ΠΊΠΈΡ… 2 Π³Ρ€.

11.1. ΠŸΠΎΠ²Ρ‚ΠΎΡ€Π½ΠΎ ΠΏΡ€ΠΎΡ…ΠΎΠ΄ΠΈΡ‚ΡŒ Π½Π΅ Π½Π°Π΄ΠΎ, Π² Π»ΠΈΡ‡Π½ΠΎΠΌ Π΄Π΅Π»Π΅ ΡƒΠΆΠ΅ Π΅ΡΡ‚ΡŒ Π·Π°ΠΊΠ»ΡŽΡ‡Π΅Π½ΠΈΠ΅.

12. Π£ мСня Π΅ΡΡ‚ΡŒ Π²ΠΎΠ΅Π½Π½Ρ‹ΠΉ Π±ΠΈΠ»Π΅Ρ‚ Π½Π° Ρ€ΡƒΠΊΠ°Ρ… ΠΈ справка с ΠΏΡ€ΠΎΡˆΠ»ΠΎΠ³ΠΎ Π³ΠΎΡ€ΠΎΠ΄Π°, Ρ‡Ρ‚ΠΎ Π½Π΅ состоял Π½Π° ΡƒΡ‡Π΅Ρ‚Π΅. Но ΠΏΡ€ΠΈ ΠΏΡ€ΠΎΡ…ΠΎΠΆΠ΄Π΅Π½ΠΈΠΈ ΠΏΠΎΠ²Ρ‚ΠΎΡ€Π½ΠΎΠΉ мСдкомиссии Π½Π° ΠΏΡ€Π°Π²Π° психиатр отказался ΠΏΡ€ΠΈΠ½ΠΈΠΌΠ°Ρ‚ΡŒ, Π³ΠΎΠ²ΠΎΡ€ΠΈΡ‚ принСси Ρ€Π°ΡΡˆΠΈΡ„Ρ€ΠΎΠ²ΠΊΡƒ ΠΏΠΎ ΠΊΠ°ΠΊΠΎΠΉ ΡΡ‚Π°Ρ‚ΡŒΠ΅ Π½Π΅ слуТил. ΠŸΡ€Π°Π²ΠΎΠΌΠ΅Ρ€Π½ΠΎ Π»ΠΈ это?

12.1. Π’Ρ€Π°Ρ‡ Π΄ΠΎΠ»ΠΆΠ΅Π½ ΠΏΡ€ΠΎΠ²Π΅Ρ€ΠΈΡ‚ΡŒ, Π½Π΅ состояли Π»ΠΈ Π²Ρ‹ Ρ€Π°Π½Π΅Π΅ Π½Π° ΡƒΡ‡Ρ‘Ρ‚Π΅ Ρƒ психиатра.

13. Π£ мСня Ρ‚Π°ΠΊΠΎΠΉ вопрос, Π² 18 Π»Π΅Ρ‚ ΠΌΠ½Π΅ Π² Π²ΠΎΠ΅Π½ΠΊΠΎΠΌΠ°Ρ‚Π΅ поставили ΠΊΠ°Ρ‚Π΅Π³ΠΎΡ€ΠΈΡŽ Π’ 1, ΠΎΡ‚Π΄Π°Π»ΠΈ Π²ΠΎΠ΅Π½Π½Ρ‹ΠΉ Π±ΠΈΠ»Π΅Ρ‚ ΠΈ всС гуляй. А сСйчас ΠΌΠ½Π΅ 22 Π³ΠΎΠ΄Π° ΠΈ я Ρ…ΠΎΡ‡Ρƒ ΠΏΠΎΠΉΡ‚ΠΈ ΡΠ»ΡƒΠΆΠΈΡ‚ΡŒ, Π²ΠΎΠ·ΠΌΠΎΠΆΠ½ΠΎ Π»ΠΈ ΠΏΡ€ΠΎΠΉΡ‚ΠΈ ΠΏΠΎΠ²Ρ‚ΠΎΡ€Π½ΡƒΡŽ мСдкомиссию, ΠΏΠΎΠΌΠ΅Π½ΡΡ‚ΡŒ ΠΊΠ°Ρ‚Π΅Π³ΠΎΡ€ΠΈΡŽ ΠΈ ΠΏΠΎΠΉΡ‚ΠΈ ΡΠ»ΡƒΠΆΠΈΡ‚ΡŒ?

13.1. ЗдравствуйтС! Π”Π°, ΠΌΠΎΠΆΠ΅Ρ‚Π΅ ΠΏΡ€ΠΎΠΉΡ‚ΠΈ ΠΏΠΎΠ²Ρ‚ΠΎΡ€Π½ΡƒΡŽ мСдкомиссию.

14. Π’Π°ΠΊΠΎΠ²: я Π½Π΅ ΠΏΡ€ΠΎΡˆΠ΅Π» мСдкомиссию Π² Π°Ρ€ΠΌΠΈΡŽ, ΠΈ ΠΏΠΎΠ»ΡƒΡ‡ΠΈΠ» Π²ΠΎΠ΅Π½Π½Ρ‹ΠΉ Π±ΠΈΠ»Π΅Ρ‚, Ρ‚Π°ΠΌ написано ΠΎΠ³Ρ€Π°Π½ΠΈΡ‡Π΅Π½Π½ΠΎ Π³ΠΎΠ΄Π΅Π½, ΠΌΠΎΠ³Ρƒ Π»ΠΈ я Π½Π°ΡΡ‚ΠΎΡΡ‚ΡŒ Π½Π° прохоТдСния ΠΏΠΎΠ²Ρ‚ΠΎΡ€Π½ΠΎΠΉ мСдкомиссии? ΠŸΡ€ΠΎΡˆΠ΅Π» ΡƒΠΆΠ΅ Π³ΠΎΠ΄ ΠΊΠ°ΠΊ я ΠΏΠΎΠ»ΡƒΡ‡ΠΈΠ» Π²ΠΎΠ΅Π½Π½Ρ‹ΠΉ Π±ΠΈΠ»Π΅Ρ‚.

14.1. Π”Π° ΠΌΠΎΠΆΠ΅Ρ‚Π΅ ΠΊΠΎΠ½Π΅Ρ‡Π½ΠΎ, ΠΎΠ±Ρ€Π°Ρ‰Π°ΠΉΡ‚Π΅ΡΡŒ Π² Π²ΠΎΠ΅Π½ΠΊΠΎΠΌΠ°Ρ‚.

15. ΠœΠΎΠ΅ΠΌΡƒ ΠΌΡƒΠΆΡƒ Π½Π° днях ΠΏΡ€ΠΈΡˆΠ»Π° повСстка для явки Π² Π²ΠΎΠ΅Π½ΠΊΠΎΠΌΠ°Ρ‚, Ρ‡Ρ‚ΠΎΠ± ΠΏΡ€ΠΎΠ²Π΅Ρ€ΠΈΡ‚ΡŒ свои Π΄Π°Π½Π½Ρ‹Π΅ ΠΈ ΠΏΡ€ΠΎΠΉΡ‚ΠΈ ΠΏΠ΅Ρ€Π΅ΠΊΠΎΠΌΠΈΡΡΠΈΡŽ. Но Ρƒ Π½Π΅Π³ΠΎ Π² Π²ΠΎΠ΅Π½Π½ΠΎΠΌ Π±ΠΈΠ»Π΅Ρ‚Π΅ написано: НС ΠΏΠΎΠ΄Π»Π΅ΠΆΠΈΡ‚ ΠΏΠΎΠ²Ρ‚ΠΎΡ€Π½ΠΎΠΉ мСдкомиссии ΠΈ Π½Π΅ Π³ΠΎΠ΄Π΅Π½ Π² ΠΌΠΈΡ€Π½ΠΎΠ΅ врСмя, ΠΎΠ³Ρ€Π°Π½ΠΈΡ‡Π΅Π½Π½ΠΎ Π³ΠΎΠ΄Π΅Π½ Π² Π²ΠΎΠ΅Π½Π½ΠΎΠ΅ врСмя. Π§Ρ‚ΠΎ Π΄Π΅Π»Π°Ρ‚ΡŒ. ΠŸΡ€Π°Π²ΠΎΠΌΠ΅Ρ€Π½Π° Π»ΠΈ эта повСстка?

15.1. Π”ΠΎΠ±Ρ€Ρ‹ΠΉ дСнь ЕвгСния. Π’ связи с ΠΏΡ€ΠΎΠ²Π΅Π΄Π΅Π½ΠΈΠ΅ΠΌ ΠΌΠΎΠ±ΠΈΠ»ΠΈΠ·Π°Ρ†ΠΈΠΈ Π² особливий ΠΏΠ΅Ρ€ΠΈΠΎΠ΄ Ρ‚ΠΎ ΠΌΠΎΠΆΠ΅Ρ‚ Π±Ρ‹Ρ‚ΡŒ ΠΏΡ€ΠΈΠ·Π²Π°Π½ ΠΏΠΎ ΠΌΠΎΠ±ΠΈΠ»ΠΈΠ·Π°Ρ†ΠΈΠΈ. Пока ΠΌΠΎΠ±ΠΈΠ»ΠΈΠ·Π°Ρ†ΠΈΠΈ Π½Π΅Ρ‚.

16. Π₯ΠΎΡ‡Ρƒ ΠΏΠΎΡΡ‚ΡƒΠΏΠΈΡ‚ΡŒ Π½Π° слуТбу Π² ΠΌΠΈΠ»ΠΈΡ†ΠΈΡŽ, имСю Π±Π΅Π»Ρ‹ΠΉ Π±ΠΈΠ»Π΅Ρ‚ ΠΈ снят с ΡƒΡ‡Π΅Ρ‚Π° Π² Π²ΠΎΠ΅Π½ΠΊΠΎΠΌΠ°Ρ‚Π΅, ΠΏΡ€ΠΈΡ…ΠΎΠ΄ΠΈΠ» Π² Π²ΠΎΠ΅Π½ΠΊΠΎΠΌΠ°Ρ‚ писал заявлСниС Π½Π° ΠΏΠΎΠ²Ρ‚ΠΎΡ€Π½ΡƒΡŽ мСдкомиссию, ΠΊΠ°ΠΊ ΠΎΠ± стСну Π³ΠΎΡ€ΠΎΡ…ΠΎΠΌ… имСю Π²Ρ‹ΡΡˆΠ΅Π΅ спортивноС ΠΎΠ±Ρ€Π°Π·ΠΎΠ²Π°Π½ΠΈΠ΅ 25 Π»Π΅Ρ‚. ΠΌΠΎΠ³Ρƒ Π»ΠΈ я ΡƒΡΡ‚Ρ€ΠΎΠΈΡ‚ΡŒΡΡ Π½Π° слуТбу Π±Π΅Π· Π²ΠΎΠ΅Π½Π½ΠΎΠ³ΠΎ Π±ΠΈΠ»Π΅Ρ‚Π°? Π•ΡΡ‚ΡŒ Π»ΠΈ для этого Π·Π°ΠΊΠΎΠ½Π½Ρ‹Π΅ ΠΏΡ€Π°Π²Π°, Ссли ΠΏΡ€ΠΎΠΉΠ΄Ρƒ мСдкомиссию Π² ΠΏΠΎΠ»ΠΈΠΊΠ»ΠΈΠ½ΠΈΠΊΠ΅ ΠœΠ’Π‘?

16.1. ΠŸΡ€Π΅ΠΆΠ΄Π΅ всСго, ΡƒΠΊΠ°ΠΆΠΈΡ‚Π΅, ΠΏΠΎ ΠΊΠ°ΠΊΠΈΠΌ основаниям Π±Ρ‹Π»ΠΈ освобоТдСны ΠΎΡ‚ слуТбы?

17. Π§Π΅Ρ€Π΅Π· сотрудника Π²ΠΎΠ΅Π½ΠΊΠΎΠΌΠ°Ρ‚Π° комиссия ΠΏΡ€ΠΈΠ·Π½Π°Π»Π° мСня Π½Π΅Π³ΠΎΠ΄Π½Ρ‹ΠΌ, сСйчас Π΅Π³ΠΎ ΡƒΠ²ΠΎΠ»ΠΈΠ»ΠΈ Π±Ρ‹Π»Π° прокурорская ΠΏΡ€ΠΎΠ²Π΅Ρ€ΠΊΠ°, мою комиссию ΠΈ списаниС поставили ΠΏΠΎΠ΄ сомнСниС, Π²Ρ€ΠΎΠ΄Π΅ Π½Π°ΠΏΡ€Π°Π²Π»ΡΡŽΡ‚ Π½Π° ΠΏΠΎΠ²Ρ‚ΠΎΡ€Π½ΡƒΡŽ, ΠΌΠΎΠ³Ρƒ Π»ΠΈ я Π±Π΅Π· прохоТдСния ΠΏΠΎΠ²Ρ‚ΠΎΡ€Π½ΠΎΠΉ мСдкомиссии Π·Π°Π±Ρ€Π°Ρ‚ΡŒ Π²ΠΎΠ΅Π½Π½Ρ‹ΠΉ Π±ΠΈΠ»Π΅Ρ‚, вСдь пСрвая мСня ΠΏΡ€ΠΈΠ·Π½Π°Π»Π° Π½Π΅Π³ΠΎΠ΄Π½ΠΎΠΉ.

17.1. ΠœΠΎΠ³Ρƒ Π»ΠΈ я Π±Π΅Π· прохоТдСния ΠΏΠΎΠ²Ρ‚ΠΎΡ€Π½ΠΎΠΉ мСдкомиссии Π·Π°Π±Ρ€Π°Ρ‚ΡŒ Π²ΠΎΠ΅Π½Π½Ρ‹ΠΉ Π±ΠΈΠ»Π΅Ρ‚, вСдь пСрвая мСня ΠΏΡ€ΠΈΠ·Π½Π°Π»Π° Π½Π΅Π³ΠΎΠ΄Π½ΠΎΠΉ
—ΠΏΠΎΠΏΡ€ΠΎΠ±ΡƒΠΉΡ‚Π΅ Π·Π°Π±Π΅Ρ€ΠΈΡ‚Π΅. А для Π½Π°Ρ‡Π°Π»Π°. ΠŸΡ€ΠΎΠΉΠ΄Ρ‘Ρ‚Π΅ ΠΏΠΎΠ²Ρ‚ΠΎΡ€Π½ΡƒΡŽ Π’ΠšΠš.

18. Как ΡΠ½ΡΡ‚ΡŒ ΡΡ‚Π°Ρ‚ΡŒΡŽ Π² Π²ΠΎΠ΅Π½Π½ΠΎΠΌ Π±ΠΈΠ»Π΅Ρ‚Π΅ 62 Π± для Ρ‚ΠΎΠ³ΠΎ Ρ‡Ρ‚ΠΎΠ±Ρ‹ ΠΏΠΎΠΏΠ°ΡΡ‚ΡŒ Π½Π° ΠΏΠΎΠ²Ρ‚ΠΎΡ€Π½ΡƒΡŽ мСдкомиссию Π² Π²ΠΎΠ΅Π½ΠΊΠΎΠΌΠ°Ρ‚Π΅ для прохоТдСния слуТбы Π² Π°Ρ€ΠΌΠΈΠΈ? Π•ΡΡ‚ΡŒ справка ΠΈΠ· ΠΊΠΎΠΆΠ½ΠΎΠ³ΠΎ диспансСра ΠΎ ΠΏΠΎΠ»Π½ΠΎΠΌ Π²Ρ‹Π·Π΄ΠΎΡ€ΠΎΠ²Π»Π΅Π½ΠΈΠΈ с ΠΈΡ… ΠΏΠ΅Ρ‡Π°Ρ‚ΡŒΡŽ. Возраст 23 Π³ΠΎΠ΄Π°.

18.1. ΠŸΠ΅Ρ€Π΅ΠΎΡΠ²ΠΈΠ΄Π΅Ρ‚Π΅Π»ΡŒΡΡ‚Π²ΠΎΠ²Π°Π½ΠΈΠ΅ ΠΏΡ€ΠΎΠΉΡ‚ΠΈ ΠΌΠΎΠΆΠ΅Ρ‚Π΅, Π° Π²ΠΎΡ‚ Π² Π°Ρ€ΠΌΠΈΡŽ Вас Π½Π΅ ΠΏΡ€ΠΈΠ·ΠΎΠ²ΡƒΡ‚, Ρ‚Π°ΠΊ ΠΊΠ°ΠΊ Π²Ρ‹ Π² запасС.

19. Π― ΠΏΠΎΠ»ΡƒΡ‡ΠΈΠ» Π²ΠΎΠ΅Π½Π½Ρ‹ΠΉ Π±ΠΈΠ»Π΅Ρ‚, Π½ΠΎ Π½Π° мСдкомиссии скрыл, Ρ‡Ρ‚ΠΎ имСю ΠΏΡ€ΠΎΠ±Π»Π΅ΠΌΡ‹ с шССй, Ρ‚Π°ΠΊ ΠΊΠ°ΠΊ собирался ΠΈΠ΄Ρ‚ΠΈ Π½Π° ΠΊΠΎΠ½Ρ‚Ρ€Π°ΠΊΡ‚. БСйчас ΠΏΠ΅Ρ€Π΅Π΄ΡƒΠΌΠ°Π». Как ΠΌΠ½Π΅ ΠΏΠΎΠ²Ρ‚ΠΎΡ€Π½ΠΎ ΠΏΡ€ΠΎΠΉΡ‚ΠΈ комиссию, Ρ‡Ρ‚ΠΎΠ± ΠΏΠΎΠ»ΡƒΡ‡ΠΈΡ‚ΡŒ Π±Π΅Π»Ρ‹ΠΉ Π±ΠΈΠ»Π΅Ρ‚? РСально Π»ΠΈ это Ρ‚Π΅ΠΏΠ΅Ρ€ΡŒ, имСя Π½Π° Ρ€ΡƒΠΊΠ°Ρ… Π²ΠΎΠ΅Π½Π½Ρ‹Ρ…?

19.1. Π”Π°, ΠΎΠ±Ρ€Π°Ρ‚ΠΈΡ‚Π΅ΡΡŒ с заявлСниСм Π² Π²ΠΎΠ΅Π½ΠΊΠΎΠΌΠ°Ρ‚.

20. МнС 25 Π»Π΅Ρ‚, я ΠΏΠΎΠ»ΡƒΡ‡ΠΈΠ» Π²ΠΎΠ΅Π½Π½Ρ‹ΠΉ Π±ΠΈΠ»Π΅Ρ‚ Π² 18 (Π½Π΅ Π³ΠΎΠ΄Π΅Π½ ΠΏΠΎ ΡΠΎΡΡ‚ΠΎΡΠ½ΠΈΡŽ Π·Π΄ΠΎΡ€ΠΎΠ²ΡŒΡ) БСйчас Π― мСняю ΠŸΠœΠ– ΠΈ МнС Π²Ρ‹Π΄Π°Π»ΠΈ Π±ΡƒΠΌΠ°Π³Ρƒ с ΠΊΠΎΡ‚ΠΎΡ€ΠΎΠΉ я Π΄ΠΎΠ»ΠΆΠ΅Π½ ΡΠ²ΠΈΡ‚ΡŒΡΡ Π² Π’ΠΎΠ΅Π½ΠΊΠΎΠΌΠ°Ρ‚ ΠΏΠΎ Π½ΠΎΠ²ΠΎΠΌΡƒ мСсту прописки.
Вопрос: Надо Π»ΠΈ ΠΏΡ€ΠΎΡ…ΠΎΠ΄ΠΈΡ‚ΡŒ Π΅Ρ‰Ρ‘ Ρ€Π°Π· мСдкомиссию? Обязан Π»ΠΈ Π― ΠΏΡ€ΠΎΡ…ΠΎΠ΄ΠΈΡ‚ΡŒ Π΅Ρ‘ ΠΏΠΎΠ²Ρ‚ΠΎΡ€Π½ΠΎ? Или это обычная Ρ„ΠΎΡ€ΠΌΠ°Π»ΡŒΠ½ΠΎΡΡ‚ΡŒ. ΠŸΡ€ΠΎΡΡ‚ΠΎ внСсут ΠΏΠΎΠΌΠ΅Ρ‚ΠΊΡƒ ΠΎ смСнС ΠŸΠœΠ– ΠΈ всС?
КакиС ΠΌΠΎΠ³ΡƒΡ‚ Π±Ρ‹Ρ‚ΡŒ послСдствия Ссли Π― Π½Π΅ явлюсь Π² Π²ΠΎΠ΅Π½ΠΊΠΎΠΌΠ°Ρ‚ для увСдомлСния.

20.1. Π‘Π»Π΅Π΄ΡƒΠ΅Ρ‚ ΡΠ²ΠΈΡ‚ΡŒΡΡ Π² Π²ΠΎΠ΅Π½ΠΊΠΎΠΌΠ°Ρ‚ ΠΏΠΎ Π½ΠΎΠ²ΠΎΠΌΡƒ мСсту ΠΆΠΈΡ‚Π΅Π»ΡŒΡΡ‚Π²Π°, ΠΏΡ€Π΅Π΄ΡŠΡΠ²ΠΈΡ‚ΡŒ Ρ‚Π°ΠΌ Π²ΠΎΠ΅Π½Π½Ρ‹ΠΉ Π±ΠΈΠ»Π΅Ρ‚ ΠΈ приписноС ΡΠ²ΠΈΠ΄Π΅Ρ‚Π΅Π»ΡŒΡΡ‚Π²ΠΎ.
МСдкомиссию ΠΏΡ€ΠΎΡ…ΠΎΠ΄ΠΈΡ‚ΡŒ Π½Π΅ придСтся.

21. Π‘Ρ‹Π½ ΠΏΡ€ΠΎΡ…ΠΎΠ΄ΠΈΠ» ΠΏΠΎΠ²Ρ‚ΠΎΡ€Π½ΡƒΡŽ мСдкомиссию Π² Π²ΠΎΠ΅Π½ΠΊΠΎΠΌΠ°Ρ‚Π΅, Ρ€Π΅ΡˆΠ΅Π½ΠΈΠ΅-Π½Π΅ Π³ΠΎΠ΄Π΅Π½. Π—Π°Π±Ρ€Π°Π»ΠΈ приписноС. Π’ ΠΊΠ°ΠΊΠΈΠ΅ сроки Π΄ΠΎΠ»ΠΆΠ½Ρ‹ Π²Ρ‹Π΄Π°Ρ‚ΡŒ Π²ΠΎΠ΅Π½Π½Ρ‹ΠΉ Π±ΠΈΠ»Π΅Ρ‚?

21.1. ΠœΠΎΠ³ΡƒΡ‚ — Π²ΠΎΠΎΠ±Ρ‰Π΅ Π½Π΅ Π΄Π°Ρ‚ΡŒ. А Ρ‚ΠΎΠ»ΡŒΠΊΠΎ справку.

22. Если я ΠΏΡ€ΠΈΠ·Π½Π°Π½ ΠΎΠ³Ρ€Π°Π½ΠΈΡ‡Π΅Π½Π½ΠΎ Π³ΠΎΠ΄Π½Ρ‹ΠΌ ΠΊ Π²ΠΎΠ΅Π½Π½ΠΎΠΉ слуТбС, ΠΈ Ρƒ мСня Π΅ΡΡ‚ΡŒ Π²ΠΎΠ΅Π½Π½Ρ‹ΠΉ Π±ΠΈΠ»Π΅Ρ‚, ΠΌΠΎΠ³ΡƒΡ‚ Π»ΠΈ мСня ΠΏΡ€ΠΈΠ½ΡƒΠ΄ΠΈΡ‚Π΅Π»ΡŒΠ½ΠΎ Π½Π°ΠΏΡ€Π°Π²ΠΈΡ‚ΡŒ Π½Π° ΠΏΠΎΠ²Ρ‚ΠΎΡ€Π½ΡƒΡŽ мСдкомиссию? Бпасибо.

22.1. —Π½Π΅ ΠΈΡΠΊΠ»ΡŽΡ‡Π΅Π½ΠΎ Ρ‡Ρ‚ΠΎ ΠΌΠΎΠ³ΡƒΡ‚. Если ваш Π±ΠΈΠ»Π΅Ρ‚ Π²Ρ‹Π·Ρ‹Π²Π°Π΅Ρ‚ сомнСния.

23. МнС 21 Π³ΠΎΠ΄. ΠŸΡ€ΠΈ ΠΏΡ€ΠΎΡ…ΠΎΠΆΠ΄Π΅Π½ΠΈΠΈ мСдкомиссии 3 Π³ΠΎΠ΄Π° Π½Π°Π·Π°Π΄ ΠΏΠΎΠ»ΡƒΡ‡ΠΈΠ» Π²ΠΎΠ΅Π½Π½Ρ‹ΠΉ Π±ΠΈΠ»Π΅Ρ‚ ΠΊΠ°Ρ‚Π΅Π³ΠΎΡ€ΠΈΠΈ Π’. БСйчас Ρ…ΠΎΡ‡Ρƒ ΡΠ»ΡƒΠΆΠΈΡ‚ΡŒ Π² Π°Ρ€ΠΌΠΈΠΈ, ΡΡ‡ΠΈΡ‚Π°ΡŽ, Ρ‡Ρ‚ΠΎ Π²ΠΏΠΎΠ»Π½Π΅ Π·Π΄ΠΎΡ€ΠΎΠ². Π’ Π²ΠΎΠ΅Π½ΠΊΠΎΠΌΠ°Ρ‚Π΅ сказали, Ρ‡Ρ‚ΠΎ послС ΠΏΠ΅Ρ€Π΅ΠΊΠ²Π°Π»ΠΈΡ„ΠΈΠΊΠ°Ρ†ΠΈΠΈ ΠΊΠ°Ρ‚Π΅Π³ΠΎΡ€ΠΈΠΈ Π½Π° Π³ΠΎΠ΄Π΅Π½ (Π² случаС ΠΏΠΎΠ²Ρ‚ΠΎΡ€Π½ΠΎΠ³ΠΎ прохоТдСния мСдкомиссии) Π½Π° ΡΡ€ΠΎΡ‡Π½ΡƒΡŽ слуТбу мСня Π½Π΅ Π²ΠΎΠ·ΡŒΠΌΡƒΡ‚ всС Ρ€Π°Π²Π½ΠΎ. Π’ΠΎΠ·ΠΌΠΎΠΆΠ½ΠΎ Π»ΠΈ Π² ΠΌΠΎΠ΅ΠΌ случаС ΠΏΠΎΠΉΡ‚ΠΈ Π½Π° слуТбу ΠΏΠΎ ΠΊΠΎΠ½Ρ‚Ρ€Π°ΠΊΡ‚Ρƒ? АлСксСй.

23.1. Π”Π° Π²ΠΎΠ·ΠΌΠΎΠΆΠ½ΠΎ Ρ‚Π°ΠΊΠΎΠ΅. ΠžΠ±Ρ€Π°Ρ‚ΠΈΡ‚Π΅ΡΡŒ Π² Π²ΠΎΠ΅Π½ΠΊΠΎΠΌΠ°Ρ‚ с этим вопросом.

24. Π’ 20 Π»Π΅Ρ‚ ΠΏΠΎΠ»ΡƒΡ‡ΠΈΠ» Π²ΠΎΠ΅Π½Π½Ρ‹ΠΉ Π±ΠΈΠ»Π΅Ρ‚ послС обслСдования Π² психиатричСской ΠΊΠ»ΠΈΠ½ΠΈΠΊΠ΅. ΠŸΡ€ΠΈΠ·Π½Π°Π»ΠΈ Π½Π΅Π³ΠΎΠ΄Π½Ρ‹ΠΌ. Бпустя 4 Π³ΠΎΠ΄Π° ΠΏΡ€ΠΎΡˆΠ΅Π» мСдкомиссию Π½Π° Π²ΠΎΠΆΠ΄Π΅Π½ΠΈΠ΅, Π΄ΠΎΠΊΠ°Π·Π°Π² Ρ‡Ρ‚ΠΎ я Π·Π΄ΠΎΡ€ΠΎΠ² психичСски. ΠœΠΎΠ³ΡƒΡ‚ Π»ΠΈ мСня ΠΏΠΎΠ²Ρ‚ΠΎΡ€Π½ΠΎ ΠΏΡ€ΠΈΠ·Π²Π°Ρ‚ΡŒ ΠΊΠ°ΠΊ Π²Ρ‹Π·Π΄ΠΎΡ€ΠΎΠ²Π΅Π²ΡˆΠ΅Π³ΠΎ?

24.1. Π’ вашСй ситуации ΠΌΠΎΠ³ΡƒΡ‚ Ρ‚Π°ΠΊ сСйчас Π½Π΅Ρ‚ ΠΏΡ€ΠΎΡ‚ΠΈΠ²ΠΎΠΏΠΎΠΊΠ°Π·Π°Π½ΠΈΠΉ для ΠΏΡ€ΠΈΠ·Ρ‹Π²Π° Π² Π°Ρ€ΠΌΠΈΡŽ ΠΏΠΎ ΡΠΎΡΡ‚ΠΎΡΠ½ΠΈΡŽ Π·Π΄ΠΎΡ€ΠΎΠ²ΡŒΡ.

25. ΠœΠΎΠ³ΡƒΡ‚ Π»ΠΈ мСня Π²Π·ΡΡ‚ΡŒ Π½Π° ΠΏΠ΅Ρ€Π΅ΠΏΠΎΠ΄Π³ΠΎΡ‚ΠΎΠ²ΠΊΡƒ, Ссли Ρƒ мСня Π½Π΅Ρ‚ Π²ΠΎΠ΅Π½Π½ΠΎΠ³ΠΎ Π±ΠΈΠ»Π΅Ρ‚Π° ΠΈ Π½ΠΈΠΊΠΎΠ³Π΄Π° Π΅Π³ΠΎ Π½Π΅ Π±Ρ‹Π»ΠΎ. Π’ 1997 Π³ΠΎΠ΄Ρƒ я ΠΏΡ€ΠΎΡˆΡ‘Π» мСдкомиссию, Π±Ρ‹Π» ΠΏΡ€ΠΈΠ·Π½Π°Π½ Π½Π΅Π³ΠΎΠ΄Π΅Π½, Π² Π²ΠΎΠ΅Π½ΠΊΠΎΠΌΠ°Ρ‚Π΅ сказали, Ρ‡Ρ‚ΠΎ ΠΎΠ½ΠΈ мСня ΠΏΠΎΡ‚ΠΎΠΌ Π²Ρ‹Π·ΠΎΠ²ΡƒΡ‚ Π΅Ρ‰Ρ‘ для ΠΏΠΎΠ²Ρ‚ΠΎΡ€Π½ΠΎΠΉ комиссии ΠΈ… Π²ΠΎΡ‚ 2014 Π³ΠΎΠ΄ Π΄ΠΎ сих ΠΏΠΎΡ€ Π½ΠΈ ΠΊΡ‚ΠΎ Π½ΠΈ ΠΊΡƒΠ΄Π° Π½Π΅ Π²Ρ‹Π·Ρ‹Π²Π°Π». А я Π΄ΠΎΠΆΠΈΠ» ΡƒΠΆΠ΅ Π΄ΠΎ 42 Π»Π΅Ρ‚ Π±Π΅Π· Π²ΠΎΠ΅Π½Π½ΠΎΠ³ΠΎ Π±ΠΈΠ»Π΅Ρ‚Π°.

25.1. НСт, Π½Π΅ Π²ΠΎΠ·ΡŒΠΌΡƒΡ‚.

25.2. Василий, здравствуйтС. ΠšΠΎΠ½Π΅Ρ‡Π½ΠΎ, Π½Π΅Ρ‚.

26. Π£ мСня катСгория годности Π’ Π² Π²ΠΎΠ΅Π½Π½ΠΎΠΌ Π±ΠΈΠ»Π΅Ρ‚Π΅ ΠΌΠΎΠ³Ρƒ Π»ΠΈ ΠΏΡ€ΠΎΠΉΡ‚ΠΈ ΠΏΠΎΠ²Ρ‚ΠΎΡ€Π½ΠΎ мСдкомиссию, ΠΏΡ€ΠΎΡˆΠ»ΠΎ ΡƒΠΆΠ΅ 5 Π»Π΅Ρ‚ послС Ρ‚ΠΎΠ³ΠΎ ΠΊΠ°ΠΊ ΠΌΠ½Π΅ присвоили эту ΠΊΠ°Ρ‚Π΅Π³ΠΎΡ€ΠΈΡŽ, Π½Π΅Π΄Π°Π²Π½ΠΎ отучился Π² Ρ‚Π΅Ρ…Π½ΠΈΠΊΡƒΠΌΠ΅, ΠΏΠΎΠ»ΡƒΡ‡ΠΈΠ» Π΄ΠΈΠΏΠ»ΠΎΠΌ, Π½Π° Ρ€Π°Π±ΠΎΡ‚Π΅ сказали Ρ‡Ρ‚ΠΎ Π½ΡƒΠΆΠ½ΠΎ ΠΈΠ΄Ρ‚ΠΈ Π² Π²ΠΎΠ΅Π½ΠΊΠΎΠΌΠ°Ρ‚ для Ρ‚ΠΎΠ³ΠΎ Ρ‡Ρ‚ΠΎΠ±Ρ‹, ΠΌΠ½Π΅ Ρ‚Π°ΠΌ поставили ΠΎΡ‚ΠΌΠ΅Ρ‚ΠΊΡƒ ΠΎΠ± ΠΎΠ±Ρ€Π°Π·ΠΎΠ²Π°Π½ΠΈΠΈ.

26.1. Π£Π²Π°ΠΆΠ°Π΅ΠΌΡ‹ΠΉ Π”ΠΌΠΈΡ‚Ρ€ΠΈΠΉ.
МоТно ΠΏΡ€ΠΎΠΉΡ‚ΠΈ мСдкомиссию ΠΏΠΎ своСму ТСланию. Для этого Π½ΡƒΠΆΠ½ΠΎ ΠΏΡ€ΠΈΠΉΡ‚ΠΈ Π² Π²ΠΎΠ΅Π½ΠΊΠΎΠΌΠ°Ρ‚ ΠΈ ΠΏΠΎΠΏΡ€ΠΎΡΠΈΡ‚ΡŒ, Ρ‡Ρ‚ΠΎΠ±Ρ‹ Π’Π°ΠΌ Π²Ρ‹Π΄Π°Π»ΠΈ повСстку Π½Π° ΠΏΡ€ΠΎΡ…ΠΎΠΆΠ΄Π΅Π½ΠΈΠ΅ мСдкомиссии. Волько нСпонятно, Π’Ρ‹ ΠΏΠΈΡˆΠΈΡ‚Π΅, Ρ‡Ρ‚ΠΎ Π½Π° Ρ€Π°Π±ΠΎΡ‚Π΅ ΠΏΠΎΡ‚Ρ€Π΅Π±ΠΎΠ²Π°Π»ΠΈ ΡΡ…ΠΎΠ΄ΠΈΡ‚ΡŒ Π² Π²ΠΎΠ΅Π½ΠΊΠΎΠΌΠ°Ρ‚ ΠΈ ΠΏΠΎΡΡ‚Π°Π²ΠΈΡ‚ΡŒ Ρ‚Π°ΠΌ ΠΎΡ‚ΠΌΠ΅Ρ‚ΠΊΡƒ ΠžΠ‘ ΠžΠ‘Π ΠΠ—ΠžΠ’ΠΠΠ˜Π˜? Π’ΠΎΠ΅Π½ΠΊΠΎΠΌΠ°Ρ‚ Ρ‚Π°ΠΊΠΈΠΌΠΈ Π²Π΅Ρ‰Π°ΠΌΠΈ Π½Π΅ занимаСтся.

27. Π£ мСня катСгория годности Π’ Π² Π²ΠΎΠ΅Π½Π½ΠΎΠΌ Π±ΠΈΠ»Π΅Ρ‚Π΅, ΠΌΠΎΠ³Ρƒ Π»ΠΈ ΠΏΡ€ΠΎΠΉΡ‚ΠΈ ΠΏΠΎΠ²Ρ‚ΠΎΡ€Π½ΠΎ мСдкомиссию, ΠΏΡ€ΠΎΡˆΠ»ΠΎ ΡƒΠΆΠ΅ 5 Π»Π΅Ρ‚ послС Ρ‚ΠΎΠ³ΠΎ ΠΊΠ°ΠΊ ΠΌΠ½Π΅ присвоили эту ΠΊΠ°Ρ‚Π΅Π³ΠΎΡ€ΠΈΡŽ, ΠΈ ΠΏΠΎΠΉΡ‚ΠΈ ΠΏΠΎ ΠΊΠΎΠ½Ρ‚Ρ€Π°ΠΊΡ‚Ρƒ, Π² Π°Ρ€ΠΌΠΈΠΈ Π½Π΅ слуТил, Π·Π²ΠΎΠ½ΠΈΠ» Π² Π²ΠΎΠ΅Π½ΠΊΠΎΠΌΠ°Ρ‚ сказали Ρ‡Ρ‚ΠΎ нСльзя.

27.1. НС Π²ΠΎΠ·ΠΌΠΎΠΆΠ½ΠΎ Π΄ΠΎΠ±ΠΈΡ‚ΡŒΡΡ Π² Π²ΠΎΠ΅Π½ΠΊΠΎΠΌΠ°Ρ‚Π΅, чСстно говоря.

28. Π£ ΠΌΠΎΠ΅Π³ΠΎ ΠΌΡƒΠΆΠ° стоит катСгория Π’ ΠΈ Π² Π²ΠΎΠ΅Π½Π½ΠΎΠΌ Π±ΠΈΠ»Π΅Ρ‚Π΅ стоит НС ΠΏΠΎΠ΄Π»Π΅ΠΆΠΈΡ‚ ΠΏΠΎΠ²Ρ‚ΠΎΡ€Π½ΠΎΠΉ мСдкомиссии. Π’ΠΎΠ·ΠΌΠΎΠΆΠ½ΠΎ Π»ΠΈ это ΠΈΡΠΏΡ€Π°Π²ΠΈΡ‚ΡŒ? Он ΠΎΡ‡Π΅Π½ΡŒ Ρ…ΠΎΡ‡Π΅Ρ‚ ΡΠ»ΡƒΠΆΠΈΡ‚ΡŒ.

28.1. НС ΠΏΠΎΠ΄Π»Π΅ΠΆΠΈΡ‚ ΠΏΠΎΠ²Ρ‚ΠΎΡ€Π½ΠΎΠΉ мСдкомиссии. Π’ΠΎΠ·ΠΌΠΎΠΆΠ½ΠΎ Π»ΠΈ это ΠΈΡΠΏΡ€Π°Π²ΠΈΡ‚ΡŒ? Он ΠΎΡ‡Π΅Π½ΡŒ Ρ…ΠΎΡ‡Π΅Ρ‚ ΡΠ»ΡƒΠΆΠΈΡ‚ΡŒ.
ΠΠ°Ρ‚Π°Π»ΡŒΡ Π˜Π»ΡŒΠΈΠ½ΠΈΡ‡Π½Π°, Π½ΠΈΡ‡Π΅Π³ΠΎ ΡƒΠΆΠ΅ ΠΈΡΠΏΡ€Π°Π²ΠΈΡ‚ΡŒ Π½Π΅ Π²ΠΎΠ·ΠΌΠΎΠΆΠ½ΠΎ.

28.2. ΠŸΡ€Π΅Π΄Ρ‹Π΄ΡƒΡ‰ΠΈΠΉ ΠΎΡ‚Π²Π΅Ρ‚ Π½Π΅ΠΏΡ€Π°Π²ΠΈΠ»ΡŒΠ½Ρ‹ΠΉ!
ПослС измСнСния состояния Π·Π΄ΠΎΡ€ΠΎΠ²ΡŒΡ, Π²Ρ‹ ΠΈΠΌΠ΅Π΅Ρ‚Π΅ ΠΏΡ€Π°Π²ΠΎ Π½Π° ΠΏΠ΅Ρ€Π΅ΠΎΡΠ²ΠΈΠ΄Π΅Ρ‚Π΅Π»ΡŒΡΡ‚Π²ΠΎΠ²Π°Π½ΠΈΠ΅ ΠΈ ΠΈΠ·ΠΌΠ΅Π½Π΅Π½ΠΈΠ΅ ΠΊΠ°Ρ‚Π΅Π³ΠΎΡ€ΠΈΠΈ годности, Ссли Π² состоянии Π΅Π³ΠΎ Π·Π΄ΠΎΡ€ΠΎΠ²ΡŒΡ ΠΏΡ€ΠΎΠΈΠ·ΠΎΡˆΠ»ΠΈ измСнСния, Π΄Π°ΡŽΡ‰ΠΈΠ΅ основания для пСрСсмотра Π·Π°ΠΊΠ»ΡŽΡ‡Π΅Π½ΠΈΡ Π²ΠΎΠ΅Π½Π½ΠΎ-Π²Ρ€Π°Ρ‡Π΅Π±Π½ΠΎΠΉ комиссии

Π’Π°ΠΌ Π½Π΅ΠΎΠ±Ρ…ΠΎΠ΄ΠΈΠΌΠΎ ΠΏΠΎΠ΄Π°Ρ‚ΡŒ заявлСниС Π½Π° имя Π½Π°Ρ‡Π°Π»ΡŒΠ½ΠΈΠΊΠ° ΠΎΡ‚Π΄Π΅Π»Π° Π’Πš ΠΏΠΎ мСсту воинского ΡƒΡ‡Π΅Ρ‚Π° ΠΎ ΠΏΡ€ΠΎΠ²Π΅Π΄Π΅Π½ΠΈΠΈ ΠΏΠΎΠ²Ρ‚ΠΎΡ€Π½ΠΎΠ³ΠΎ ΠΌΠ΅Π΄. ΠΎΡΠ²ΠΈΠ΄Π΅Ρ‚Π΅Π»ΡŒΡΡ‚Π²ΠΎΠ²Π°Π½ΠΈΡ с ΠΏΡ€ΠΈΠ»ΠΎΠΆΠ΅Π½ΠΈΠ΅ΠΌ ΠΊΠΎΠΏΠΈΠΉ ΠΌΠ΅Π΄. Π΄ΠΎΠΊΡƒΠΌΠ΅Π½Ρ‚ΠΎΠ² ΠΏΠΎΠ΄Ρ‚Π²Π΅Ρ€ΠΆΠ΄Π°ΡŽΡ‰ΠΈΡ… измСнСния Π² состоянии Π·Π΄ΠΎΡ€ΠΎΠ²ΡŒΡ.
Π Π°ΡΡΠΌΠΎΡ‚Ρ€Π΅Ρ‚ΡŒ Π’Π°ΡˆΠ΅ заявлСниС обязаны Π² Ρ‚Π΅Ρ‡Π΅Π½ΠΈΠΈ 30 Π΄Π½Π΅ΠΉ.
ΠžΡ‚ΠΊΠ°Π· ΠΌΠΎΠΆΠ΅Ρ‚ Π±Ρ‹Ρ‚ΡŒ оспорСн Π² судС.


29. На Ρ€ΡƒΠΊΠ°Ρ… Ρƒ сына Π²ΠΎΠ΅Π½Π½Ρ‹ΠΉ Π±ΠΈΠ»Π΅Ρ‚, Π²Ρ‹Π΄Π°Π½Π½Ρ‹ΠΉ ΠΏΠΎ ΡΠΎΡΡ‚ΠΎΡΠ½ΠΈΡŽ Π·Π΄ΠΎΡ€ΠΎΠ²ΡŒΡ Π² 2013 Π³ΠΎΠ΄Ρƒ (плоскостопиС 2 ΠΈ 3 стСпСни), Π²ΠΎΠ΅Π½ΠΊΠΎΠΌΠ°Ρ‚ снова выписал повСстку Π½Π° ΠΏΠΎΠ²Ρ‚ΠΎΡ€Π½ΠΎΠ΅ ΠΏΡ€ΠΎΡ…ΠΎΠΆΠ΄Π΅Π½ΠΈΠ΅ мСдкомиссии (Π½Π° основании Π½ΠΎΠ²ΠΎΠ³ΠΎ Π·Π°ΠΊΠΎΠ½Π° ΠΏΡƒΡ‚ΠΈΠ½Π°), ΠΌΠΎΠ³ΡƒΡ‚ Π»ΠΈ Π΅Π³ΠΎ Π·Π°Π±Ρ€Π°Ρ‚ΡŒ Π² Π°Ρ€ΠΌΠΈΡŽ ΠΈ Π·Π°ΠΊΠΎΠ½Π½ΠΎ Π»ΠΈ это?

29.1. Π’Ρ‹ ΠΈΠΌΠ΅Π΅Ρ‚Π΅ ΠΏΡ€Π°Π²ΠΎ Π²Ρ‹Ρ€Π°Π·ΠΈΡ‚ΡŒ нСсогласиС с Π·Π°ΠΊΠ»ΡŽΡ‡Π΅Π½ΠΈΠ΅ΠΌ мСдкомиссии ΠΈ Ρ‚Ρ€Π΅Π±ΠΎΠ²Π°Ρ‚ΡŒ провСдСния ΠΠ•Π—ΠΠ’Π˜Π‘Π˜ΠœΠžΠ™ мСдицинской комиссии (Π½Π° основании. Π‘Ρ‚.ст. 25,51,53 Основ Π·Π°ΠΊΠΎΠ½ΠΎΠ΄Π°Ρ‚Π΅Π»ΡŒΡΡ‚Π²Π° Π Π€ ΠΎΠ± ΠΎΡ…Ρ€Π°Π½Π΅ Π·Π΄ΠΎΡ€ΠΎΠ²ΡŒΡ Π³Ρ€Π°ΠΆΠ΄Π°Π½).

30. Π£ мСня Π²ΠΎΡ‚ такая ΠΏΡ€ΠΎΠ±Π»Π΅ΠΌΠ°, Ρƒ мСня Π² Π²ΠΎΠ΅Π½Π½ΠΎΠΌ Π±ΠΈΠ»Π΅Ρ‚Π΅ стоит Π³ΠΎΠ΄Π΅Π½ с ограничСниями Π±-3, ΠΏΠΎ давлСнию, я Π±Ρ‹ Ρ…ΠΎΡ‚Π΅Π» ΡΠ½ΡΡ‚ΡŒ это ΠΎΠ³Ρ€Π°Π½ΠΈΡ‡Π΅Π½ΠΈΠ΅ Π½Π° Π±ΡƒΠΊΠ²Ρƒ Π°-1,Π½ΠΎ Π² Π²ΠΎΠ΅Π½ΠΊΠΎΠΌΠ°Ρ‚Π΅ ΠΎΡ‚ΠΊΠ°Π·Ρ‹Π²Π°ΡŽΡ‚ Π² ΠΏΠΎΠ²Ρ‚ΠΎΡ€Π½ΠΎΠΉ мСдкомиссии говорят, Ρ‡Ρ‚ΠΎ я списан Π² запас Ρ‚Π°ΠΊ ΠΊΠ°ΠΊ я ΠΏΡ€ΠΎΡˆΡ‘Π» Π°Ρ€ΠΌΠΈΡŽ, ΠΊΠ°ΠΊ ΠΌΠ½Π΅ Π±Ρ‹Ρ‚ΡŒ Π² этой ситуации? Они ΡΡΡ‹Π»Π°ΡŽΡ‚ΡΡ Π½Π° ΡƒΠΊΠ°Π· ΠΊΠΎΡ‚ΠΎΡ€Ρ‹ΠΉ Π·Π°ΠΏΡ€Π΅Ρ‰Π°Π΅Ρ‚.

30.1. ЗдравствуйтС, ПавСл.
ΠŸΠΎΡ‚Ρ€Π΅Π±ΡƒΠΉΡ‚Π΅ ΠΏΠΈΡΡŒΠΌΠ΅Π½Π½Ρ‹ΠΉ ΠΎΡ‚ΠΊΠ°Π·.

30.2. ЗдравствуйтС!
Π’Π°ΠΌ Π½Π΅ΠΎΠ±Ρ…ΠΎΠ΄ΠΈΠΌΠΎ ΠΏΠΎΠ΄Π°Ρ‚ΡŒ заявлСниС Π½Π° имя Π½Π°Ρ‡Π°Π»ΡŒΠ½ΠΈΠΊΠ° ΠΎΡ‚Π΄Π΅Π»Π° Π’Πš ΠΏΠΎ мСсту воинского ΡƒΡ‡Π΅Ρ‚Π° ΠΎ ΠΏΡ€ΠΎΠ²Π΅Π΄Π΅Π½ΠΈΠΈ ΠΏΠΎΠ²Ρ‚ΠΎΡ€Π½ΠΎΠ³ΠΎ ΠΌΠ΅Π΄. ΠΎΡΠ²ΠΈΠ΄Π΅Ρ‚Π΅Π»ΡŒΡΡ‚Π²ΠΎΠ²Π°Π½ΠΈΡ с ΠΏΡ€ΠΈΠ»ΠΎΠΆΠ΅Π½ΠΈΠ΅ΠΌ ΠΊΠΎΠΏΠΈΠΉ ΠΌΠ΅Π΄. Π΄ΠΎΠΊΡƒΠΌΠ΅Π½Ρ‚ΠΎΠ² ΠΏΠΎΠ΄Ρ‚Π²Π΅Ρ€ΠΆΠ΄Π°ΡŽΡ‰ΠΈΡ… измСнСния Π² состоянии Π·Π΄ΠΎΡ€ΠΎΠ²ΡŒΡ.
Π Π°ΡΡΠΌΠΎΡ‚Ρ€Π΅Ρ‚ΡŒ Π’Π°ΡˆΠ΅ заявлСниС обязаны Π² Ρ‚Π΅Ρ‡Π΅Π½ΠΈΠΈ 30 Π΄Π½Π΅ΠΉ.
ΠžΡ‚ΠΊΠ°Π· ΠΌΠΎΠΆΠ΅Ρ‚ Π±Ρ‹Ρ‚ΡŒ оспорСн Π² судС.


ΠŸΠΎΠ²Ρ‚ΠΎΡ€Π½Π°Ρ мСдкомиссия Π² Π²ΠΎΠ΅Π½ΠΊΠΎΠΌΠ°Ρ‚Π΅ — 130 совСтов Π°Π΄Π²ΠΎΠΊΠ°Ρ‚ΠΎΠ² ΠΈ ΡŽΡ€ΠΈΡΡ‚ΠΎΠ²

1. Π― Π·Π°ΠΊΠΎΠ½Ρ‡ΠΈΠ» Π±Π°ΠΊΠ°Π»Π°Π²Ρ€ΠΈΠ°Ρ‚ ΠΈ Π±ΡƒΠ΄Ρƒ ΠΏΠΎΡΡ‚ΡƒΠΏΠ°Ρ‚ΡŒ Π² магистратуру, ΠΏΡ€ΠΈΡˆΡ‘Π» ΠΏΠΎ Π·Π²ΠΎΠ½ΠΊΡƒ ΠΈΠ· Π²ΠΎΠ΅Π½ΠΊΠΎΠΌΠ°Ρ‚Π° для уточнСния Π΄Π°Π½Π½Ρ‹Ρ…. МСня Π² этот ΠΆΠ΅ дСнь ΠΎΡ‚ΠΏΡ€Π°Π²ΠΈΠ»ΠΈ Π½Π° мСдкомиссию ΠΈ Π½Π° ΠΏΡ€ΠΈΠ·Ρ‹Π²Π½ΡƒΡŽ комиссию. Π’Ρ€ΡƒΡ‡ΠΈΠ»ΠΈ повСстку ΠΎΠ± ΠΎΡ‚ΠΏΡ€Π°Π²ΠΊΠ΅ Π² Π°Ρ€ΠΌΠΈΡŽ Ρ‡Π΅Ρ€Π΅Π· нСдСлю. Π’ Π»ΠΈΡ‡Π½ΠΎΠΌ Π΄Π΅Π»Π΅ находится справка ΠΈΠ· Π’ΡƒΠ·Π°, Ρ‡Ρ‚ΠΎ я являюсь студСнтом Π΄ΠΎ 31 августа. ΠŸΠΎΠ²Ρ‚ΠΎΡ€Π½ΠΎ принСсСнная Π»ΠΈΡ‡Π½ΠΎ такая ΠΆΠ΅ справка принята Π½Π΅ Π±Ρ‹Π»Π°. Π§Ρ‚ΠΎ Π±ΡƒΠ΄Π΅Ρ‚, Ссли я Π½Π΅ явлюсь ΠΏΠΎ повСсткС? Π Π°Π·Π²Π΅ Π½Π΅ сущСствуСт Π·Π°ΠΊΠΎΠ½Π° ΠΎ нСпрСрывности образования?

1.1. ЗдравствуйтС, Иван.
Π’Ρ‹Π½ΡƒΠΆΠ΄Π΅Π½ вас ΠΎΠ³ΠΎΡ€Ρ‡ΠΈΡ‚ΡŒ. Какого-Π»ΠΈΠ±ΠΎ Π·Π°ΠΊΠΎΠ½Π°, Π΄Π°ΡŽΡ‰Π΅Π³ΠΎ ΠΏΡ€Π°Π²ΠΎ Π½Π° отсрочку Π±Π°ΠΊΠ°Π»Π°Π²Ρ€Ρƒ для поступлСния Π² магистратуру, Π½Π΅ сущСствуСт. И Π΄Π°ΠΆΠ΅ ΠΏΠΎΠΏΡ‹Ρ‚ΠΊΠΈ Π²ΠΎΡΠΏΠΎΠ»ΡŒΠ·ΠΎΠ²Π°Ρ‚ΡŒΡΡ ΠΏΡ€Π°Π²ΠΎΠΌ Π½Π° послСдипломныС ΠΊΠ°Π½ΠΈΠΊΡƒΠ»Ρ‹ Π½Π΅ приводят ΠΊ ΠΆΠ΅Π»Π°Π΅ΠΌΠΎΠΌΡƒ Ρ€Π΅Π·ΡƒΠ»ΡŒΡ‚Π°Ρ‚Ρƒ. Π‘ΡƒΠ΄Ρ‹, Π² ΠΊΠΎΡ‚ΠΎΡ€Ρ‹Π΅ ΠΎΠ±Ρ€Π°Ρ‰Π°ΡŽΡ‚ΡΡ ΠΏΡ€ΠΈΠ·Ρ‹Π²Π½ΠΈΠΊΠΈ, Π·Π°ΠΊΠΎΠ½Ρ‡ΠΈΠ²ΡˆΠΈΠ΅ Π±Π°ΠΊΠ°Π»Π°Π²Ρ€ΠΈΠ°Ρ‚, воспользовавшиСся ΠΏΡ€Π°Π²ΠΎΠΌ Π½Π° ΠΊΠ°Π½ΠΈΠΊΡƒΠ»Ρ‹ ΠΈ ΡƒΡ‚Π²Π΅Ρ€ΠΆΠ΄Π°ΡŽΡ‰ΠΈΠ΅, Ρ‡Ρ‚ΠΎ ΠΈΡ… ΠΏΡ€ΠΈΠ·Ρ‹Π² Π½Π΅ Π·Π°ΠΊΠΎΠ½Π΅Π½, ΠΏΡ€ΠΈΠ½ΠΈΠΌΠ°ΡŽΡ‚ сторону Π²ΠΎΠ΅Π½ΠΊΠΎΠΌΠ°Ρ‚Π°, ΠΊΠΎΡ‚ΠΎΡ€Ρ‹ΠΉ настаиваСт Π½Π° Ρ‚ΠΎΠΌ, Ρ‡Ρ‚ΠΎ отсрочка ΠΎΡ‚ ΠΏΡ€ΠΈΠ·Ρ‹Π²Π° прСдоставляСтся для обучСния, поэтому заканчиваСтся сразу послС сдачи госэкзамСнов ΠΈΠ»ΠΈ Π·Π°Ρ‰ΠΈΡ‚Ρ‹ Π΄ΠΈΠΏΠ»ΠΎΠΌΠΎΠ² ΠΈ Π½Π΅ распространяСтся Π½Π° послСдипломныС ΠΊΠ°Π½ΠΈΠΊΡƒΠ»Ρ‹.
Π― ΠΌΠΎΠ³Ρƒ Π²Π°ΠΌ ΠΏΡ€ΠΈΡΠ»Π°Ρ‚ΡŒ ссылку Π½Π° ΠΎΠ΄Π½ΠΎ ΠΈΠ· Ρ‚Π°ΠΊΠΈΡ… судСбных Ρ€Π΅ΡˆΠ΅Π½ΠΈΠΉ.
Π§Ρ‚ΠΎ сСйчас Π΄Π΅Π»Π°Ρ‚ΡŒ для Ρ‚ΠΎΠ³ΠΎ, Ρ‡Ρ‚ΠΎΠ±Ρ‹ Π½Π΅ ΠΎΡ‚ΠΏΡ€Π°Π²ΠΈΡ‚ΡŒΡΡ Π² Π°Ρ€ΠΌΠΈΡŽ ΠΈ ΠΏΡ€ΠΎΠ΄ΠΎΠ»ΠΆΠΈΡ‚ΡŒ ΠΎΠ±ΡƒΡ‡Π΅Π½ΠΈΠ΅?
НС ΡΠ²Π»ΡΡ‚ΡŒΡΡ Π² Π²ΠΎΠ΅Π½ΠΊΠΎΠΌΠ°Ρ‚ ΠΏΠΎ повСсткС для ΠΎΡ‚ΠΏΡ€Π°Π²ΠΊΠΈ — Π½Π΅ Π»ΡƒΡ‡ΡˆΠΈΠΉ Π²Π°Ρ€ΠΈΠ°Π½Ρ‚, ΠΏΠΎΡΠΊΠΎΠ»ΡŒΠΊΡƒ Ρ‡Ρ€Π΅Π²Π°Ρ‚ Π²ΠΎΠ·Π±ΡƒΠΆΠ΄Π΅Π½ΠΈΠ΅ΠΌ ΡƒΠ³ΠΎΠ»ΠΎΠ²Π½ΠΎΠ³ΠΎ Π΄Π΅Π»Π°.
Π—Π°ΠΊΠΎΠ½Π½Ρ‹ΠΉ способ Ρ‚ΠΎΠ»ΡŒΠΊΠΎ ΠΎΠ΄ΠΈΠ½ — ΠΎΠ±ΠΆΠ°Π»ΠΎΠ²Π°Ρ‚ΡŒ Ρ€Π΅ΡˆΠ΅Π½ΠΈΠ΅ ΠΏΡ€ΠΈΠ·Ρ‹Π²Π½ΠΎΠΉ комиссии ΠΎ ΠΏΡ€ΠΈΠ·Ρ‹Π²Π΅, послС Ρ‡Π΅Π³ΠΎ ΠΎΡ‚ΠΏΡ€Π°Π²ΠΊΠ° Π² Π°Ρ€ΠΌΠΈΡŽ Π΄ΠΎΠ»ΠΆΠ½Π° Π±Ρ‹Ρ‚ΡŒ приостановлСна.
ΠœΠΎΠΆΠ΅Ρ‚Π΅ Π½Π°ΠΏΠΈΡΠ°Ρ‚ΡŒ ΠΌΠ½Π΅ Π’ΠšΠΎΠ½Ρ‚Π°ΠΊΡ‚Π΅ https://vk.com/id403317017 ΠΈ Ссли позволяСт врСмя Π΄ΠΎ ΠΎΡ‚ΠΏΡ€Π°Π²ΠΊΠΈ, я смогу Π²Π°ΠΌ ΠΏΠΎΠΌΠΎΡ‡ΡŒ.

2. ΠŸΠΎΠ»ΡƒΡ‡ΠΈΠ» Π²ΠΎΠ΅Π½Π½Ρ‹ΠΉ Π±ΠΈΠ»Π΅Ρ‚ ΠΏΠΎ ΠΊΠ°Ρ‚Π΅Π³ΠΎΡ€ΠΈΠΈ Π’ Π² Ρ€Π΅Π³ΠΈΠΎΠ½Π΅ ΠΏΠΎ Π²Ρ€Π΅ΠΌΠ΅Π½Π½ΠΎΠΉ рСгистрации, Π²ΠΎΠ΅Π½ΠΊΠΎΠΌΠ°Ρ‚ ΠΏΠΎ постоянной Π² Π΄Ρ€ΡƒΠ³ΠΎΠΌ Ρ€Π΅Π³ΠΈΠΎΠ½Π΅ нСсколько Π»Π΅Ρ‚ ΠΏΡ€ΠΈΠ³Π»Π°ΡˆΠ°Π΅Ρ‚ ΠΏΡ€ΠΈΠΉΡ‚ΠΈ ΠΊ Π½ΠΈΠΌ ΠΈ, Π²ΠΈΠ΄ΠΈΠΌΠΎ, Π²ΡΡ‚Π°Ρ‚ΡŒ Π½Π° ΡƒΡ‡Ρ‘Ρ‚ Ρƒ Π½ΠΈΡ…. ΠœΠΎΠ³Ρƒ Π»ΠΈ я ΠΏΡ€ΠΈΠΉΡ‚ΠΈ ΠΈ ΠΏΠΎΠΊΠ°Π·Π°Ρ‚ΡŒ Π²ΠΎΠ΅Π½Π½Ρ‹ΠΉ Π±ΠΈΠ»Π΅Ρ‚ ΠΈ ΡƒΠΉΡ‚ΠΈ? ΠœΠΎΠ³ΡƒΡ‚ Π»ΠΈ ΠΎΠ½ΠΈ Π·Π°ΠΏΡ€ΠΎΡΠΈΡ‚ΡŒ ΠΏΠΎΠ²Ρ‚ΠΎΡ€Π½ΡƒΡŽ мСдкомиссию? ΠžΠ±ΡΠ·Π°Ρ‚Π΅Π»ΡŒΠ½ΠΎ Π»ΠΈ Π²ΡΡ‚Π°Π²Π°Ρ‚ΡŒ Ρƒ Π½ΠΈΡ… Π½Π° ΡƒΡ‡Ρ‘Ρ‚?

2.1. Бтранная ситуация. Π›ΠΈΡ‡Π½ΠΎΠ΅ Π΄Π΅Π»ΠΎ Ρ‚ΠΎΠ»ΡŒΠΊΠΎ ΠΎΠ΄Π½ΠΎ ΠΈ Π΄ΠΎΠ»ΠΆΠ½ΠΎ Π½Π°Ρ…ΠΎΠ΄ΠΈΡ‚ΡŒΡΡ ΠΏΠΎ мСсту вашСго воинского ΡƒΡ‡Π΅Ρ‚Π°.
Π’Ρ‹ Π΄ΠΎΠ»ΠΆΠ½Ρ‹ ΡΡ‚ΠΎΡΡ‚ΡŒ Π½Π° воинском ΡƒΡ‡Π΅Ρ‚Π΅ ΠΏΠΎ мСсту фактичСского ΠΆΠΈΡ‚Π΅Π»ΡŒΡΡ‚Π²Π°. ΠŸΡ€ΠΈ этом Π½Π°Π»ΠΈΡ‡ΠΈΠ΅/отсутствиС рСгистрации Ρ€ΠΎΠ»ΠΈ Π½Π΅ ΠΈΠ³Ρ€Π°Π΅Ρ‚ с 2019 Π³.
ΠœΠΎΠΆΠ΅Ρ‚Π΅ Π·Π°ΠΏΡ€ΠΎΡΠΈΡ‚ΡŒ Ρƒ Π²ΠΎΠ΅Π½ΠΊΠΎΠΌΠ°Ρ‚Π°, Π³Π΄Π΅ стоитС Π½Π° Π’Π£, копию Ρ€Π΅ΡˆΠ΅Π½ΠΈΡ ΠΏΡ€ΠΈΠ·Ρ‹Π²Π½ΠΎΠΉ комиссии ΠΎ зачислСнии вас Π² запас ΠΈ Π²Ρ‹ΡΠ»Π°Ρ‚ΡŒ Π² Π²ΠΎΠ΅Π½ΠΊΠΎΠΌΠ°Ρ‚ ΠΏΠΎ мСсту постоянной рСгистрации.

3. Π― ΠΏΡ€ΠΎΡˆΠ΅Π» мСдкомиссию Π² Π²ΠΎΠ΅Π½ΠΊΠΎΠΌΠ°Ρ‚Π΅ Π·Π°Π±Ρ€Π°Π» Π²ΠΎΠ΅Π½Π½Ρ‹ΠΉ Π±ΠΈΠ»Π΅Ρ‚ Ρ‚Π°ΠΌ Написали ΠΌΠ½Π΅ Ρ‡Ρ‚ΠΎ я ΠΎΠ³Ρ€Π°Π½ΠΈΡ‡Π΅Π½ Π“ΠΎΠ΄Π΅Π½ я ΠΌΠΎΠ³Ρƒ ΠΏΠΎΠ²Ρ‚ΠΎΡ€Π½ΡƒΡŽ мСдкомиссию ΠΏΡ€ΠΎΠΉΡ‚ΠΈ Ρ‚.

3.1. АртСм, Π½Π°ΠΏΠΈΡˆΠΈΡ‚Π΅ Π² Ρ‡Π΅ΠΌ ваш вопрос.
Какая катСгория записана Π² вашСм Π±ΠΈΠ»Π΅Ρ‚Π΅?

4. Π‘Π°ΠΌ ΠΏΡ€ΠΈΡˆΠ΅Π» Π² Π²ΠΎΠ΅Π½ΠΊΠΎΠΌΠ°Ρ‚ Π²Ρ‹Π΄Π°Π»ΠΈ повСстку Π½Π° 19 число 2019 Π³. Π²Ρ‡Π΅Ρ€Π° 16 числа я Π·Π°Π±ΠΎΠ»Π΅Π». ПошСл ΠΊ Π²Ρ€Π°Ρ‡Ρƒ, ΠΌΠ½Π΅ выписали Π°Π½Π°Π»ΠΈΠ·Ρ‹, Ρ€Π΅Π½Ρ‚Π³Π΅Π½. НазначСна повторная запись Π½Π° 22 число. Π’ΠΎΠ΅Π½ΠΊΠΎΠΌΠ° я ΠΏΡ€Π΅Π΄ΡƒΠΏΡ€Π΅Π΄ΠΈΠ» смс. ΠΎΠ½ΠΈ ΠΌΠ½Π΅ написали, Ρ‡Ρ‚ΠΎ Ссли я Π½Π΅ явлюсь, Ρ‚ΠΎ Π±ΡƒΠ΄Π΅Ρ‚ ΡƒΠ³ΠΎΠ»ΠΎΠ²Π½ΠΎ Π½Π°ΠΊΠ°Π·ΡƒΠ΅ΠΌΠΎ. Π§Ρ‚ΠΎ ΠΌΠ½Π΅ ΡΠ΄Π΅Π»Π°Ρ‚ΡŒ? НС ΠΈΠ΄Ρ‚ΠΈ ΠΆΠ΅ Π±ΠΎΠ»ΡŒΠ½Ρ‹ΠΌ? И ΠΌΠ½ΠΎΠ³ΠΈΠ΅ Π°Π½Π°Π»ΠΈΠ·Ρ‹ Π΅Ρ‰Π΅ Π½Π΅ сданы для мСдкомиссии.

4.1. ЗдравствуйтС!

ΠŸΡ€Π΅Π΄ΠΎΡΡ‚Π°Π²ΡŒΡ‚Π΅ Π² Π²ΠΎΠ΅Π½ΠΊΠΎΠΌΠ°Ρ‚ Π±ΠΎΠ»ΡŒΠ½ΠΈΡ‡Π½Ρ‹ΠΉ лист ΠΈ ΠΏΠΎΠ΄Ρ‚Π²Π΅Ρ€Π΄ΠΈΡ‚Π΅ свою Π²Ρ€Π΅ΠΌΠ΅Π½Π½ΡƒΡŽ Π½Π΅Ρ‚Ρ€ΡƒΠ΄ΠΎΡΠΏΠΎΡΠΎΠ±Π½ΠΎΡΡ‚ΡŒ, Π»ΠΈΠ±ΠΎ Π΄ΠΎΠΊΡƒΠΌΠ΅Π½Ρ‚Π°Π»ΡŒΠ½ΠΎΠ΅ ΠΏΠΎΠ΄Ρ‚Π²Π΅Ρ€ΠΆΠ΄Π΅Π½ΠΈΠ΅, Ρ‡Ρ‚ΠΎ Π±ΠΎΠ»ΡŒΠ½ΠΈΡ‡Π½Ρ‹ΠΉ лист ΠΎΡ‚ΠΊΡ€Ρ‹Ρ‚. Π’ Ρ‚Π°ΠΊΠΎΠΌ случаС Π½ΠΈ ΠΎ ΠΊΠ°ΠΊΠΎΠΉ ΡƒΠ³ΠΎΠ»ΠΎΠ²Π½ΠΎΠΉ отвСтствСнности Ρ€Π΅Ρ‡ΠΈ ΠΈΠ΄Ρ‚ΠΈ Π½Π΅ ΠΌΠΎΠΆΠ΅Ρ‚.

Π‘ ΡƒΠ²Π°ΠΆΠ΅Π½ΠΈΠ΅ΠΌ, ΡŽΡ€ΠΈΡΡ‚ Бмольянинова Н. Π’.

5. Π‘Π°ΠΌ ΠΏΡ€ΠΈΡˆΠ΅Π» Π² Π²ΠΎΠ΅Π½ΠΊΠΎΠΌΠ°Ρ‚ Π²Ρ‹Π΄Π°Π»ΠΈ повСстку Π½Π° 19 число 2019 Π³. Π²Ρ‡Π΅Ρ€Π° 16 числа я Π·Π°Π±ΠΎΠ»Π΅Π». ПошСл ΠΊ Π²Ρ€Π°Ρ‡Ρƒ, ΠΌΠ½Π΅ выписали Π°Π½Π°Π»ΠΈΠ·Ρ‹, Ρ€Π΅Π½Ρ‚Π³Π΅Π½. НазначСна повторная запись Π½Π° 22 число. Π’ΠΎΠ΅Π½ΠΊΠΎΠΌΠ° я ΠΏΡ€Π΅Π΄ΡƒΠΏΡ€Π΅Π΄ΠΈΠ» смс. ΠΎΠ½ΠΈ ΠΌΠ½Π΅ написали, Ρ‡Ρ‚ΠΎ Ссли я Π½Π΅ явлюсь, Ρ‚ΠΎ Π±ΡƒΠ΄Π΅Ρ‚ ΡƒΠ³ΠΎΠ»ΠΎΠ²Π½ΠΎ Π½Π°ΠΊΠ°Π·ΡƒΠ΅ΠΌΠΎ. Π§Ρ‚ΠΎ ΠΌΠ½Π΅ ΡΠ΄Π΅Π»Π°Ρ‚ΡŒ? НС ΠΈΠ΄Ρ‚ΠΈ ΠΆΠ΅ Π±ΠΎΠ»ΡŒΠ½Ρ‹ΠΌ? И ΠΌΠ½ΠΎΠ³ΠΈΠ΅ Π°Π½Π°Π»ΠΈΠ·Ρ‹ Π΅Ρ‰Π΅ Π½Π΅ сданы для мСдкомиссии.

5.1. Π’ΠΎΠ΅Π½ΠΊΠΎΠΌΠ°Ρ‚ Π½Π΅ΠΎΠ±Ρ…ΠΎΠ΄ΠΈΠΌΠΎ ΡƒΠ²Π΅Π΄ΠΎΠΌΠΈΡ‚ΡŒ письмСнно. Π—Π°Π±ΠΎΠ»Π΅Π²Π°Π½ΠΈΠ΅ связанноС с ΡƒΡ‚Ρ€Π°Ρ‚ΠΎΠΉ трудоспособности являСтся ΡƒΠ²Π°ΠΆΠΈΡ‚Π΅Π»ΡŒΠ½ΠΎΠΉ ΠΏΡ€ΠΈΡ‡ΠΈΠ½ΠΎΠΉ согласно ΠΏ 2. Π‘Ρ‚.7 Π€Π— «О воинской обязанности ΠΈ Π²ΠΎΠ΅Π½Π½ΠΎΠΉ слуТбС» . Но, ΠΏΠΎ истСчСнии ΡƒΠ²Π°ΠΆΠΈΡ‚Π΅Π»ΡŒΠ½ΠΎΠΉ ΠΏΡ€ΠΈΡ‡ΠΈΠ½Ρ‹ Π½Π΅ΠΎΠ±Ρ…ΠΎΠ΄ΠΈΠΌΠΎ ΡΠ²ΠΈΡ‚ΡŒΡΡ Π² Π²ΠΎΠ΅Π½Π½Ρ‹ΠΉ комиссариат Π½Π΅ΠΌΠ΅Π΄Π»Π΅Π½Π½ΠΎ, Π±Π΅Π· Π΄ΠΎΠΏΠΎΠ»Π½ΠΈΡ‚Π΅Π»ΡŒΠ½ΠΎΠ³ΠΎ Π²Ρ‹Π·ΠΎΠ²Π°. НСобходимо Π±ΡƒΠ΄Π΅Ρ‚ ΠΎΡ„ΠΎΡ€ΠΌΠΈΡ‚ΡŒ Π±ΠΎΠ»ΡŒΠ½ΠΈΡ‡Π½Ρ‹ΠΉ лист Ρ‡Ρ‚ΠΎΠ± Π΄ΠΎΠΊΠ°Π·Π°Ρ‚ΡŒ Ρ‡Ρ‚ΠΎ Π²Ρ‹ Π½Π΅ просто гуляли.
(я студСнт, Π΅Ρ‰Ρ‘ ΡƒΡ‡ΡƒΡΡŒ, Π½ΠΎ надСюсь Π’Π°ΠΌ ΠΏΠΎΠΌΠΎΠ³)

6. Если ΡƒΠΆΠ΅ Π΅ΡΡ‚ΡŒ Ρ€Π΅ΡˆΠ΅Π½ΠΈΠ΅ мСдкомиссии, стоит катСгория годности А 1, Π° сСйчас Ρƒ мСня Π΅ΡΡ‚ΡŒ Π·Π°ΠΊΠ»ΡŽΡ‡Π΅Π½ΠΈΠ΅ Π²Ρ€Π°Ρ‡Π°, Π² ΠΊΠΎΡ‚ΠΎΡ€ΠΎΠΌ ΡƒΠΊΠ°Π·Π°Π½Ρ‹ Π½Π°Ρ€ΡƒΡˆΠ΅Π½ΠΈΡ Ρ„ΡƒΠ½ΠΊΡ†ΠΈΠΈ ΠΏΠΎΡ‡Π΅ΠΊ, ΠΌΠ½Π΅ стоит ΠΎΠ±ΠΆΠ°Π»ΠΎΠ²Π°Ρ‚ΡŒ Ρ€Π΅ΡˆΠ΅Π½ΠΈΠ΅ Π² судС ΠΈΠ»ΠΈ ΠΏΡ€ΠΈΠ΄Ρ‚ΠΈ Π² Π²ΠΎΠ΅Π½ΠΊΠΎΠΌΠ°Ρ‚ Π² дСнь повСстки ΠΈ ΠΏΡ€ΠΎΠΉΡ‚ΠΈ ΠΏΠΎΠ²Ρ‚ΠΎΡ€Π½ΠΎ мСдкомиссию Ρ‚Π°ΠΌ?

6.1. Π’Ρ‹ Π²ΠΏΡ€Π°Π²Π΅ ΠΎΠ±ΠΆΠ°Π»ΠΎΠ²Π°Ρ‚ΡŒ Ρ€Π΅ΡˆΠ΅Π½ΠΈΠ΅ Π²Π²ΠΊ Π² Π²Ρ‹ΡˆΠ΅ΡΡ‚ΠΎΡΡ‰ΡƒΡŽ Π²Π²ΠΊ Π° ΠΏΠΎΡ‚ΠΎΠΌ Π² суд. Ссли Ρ€Π΅Π°Π»ΡŒΠ½ΠΎ ΠΏΠΎΡ‡ΠΊΠΈ. Но Ссли Π° 1 Π·Π΄ΠΎΡ€ΠΎΠ²ΡŒΠ΅ ΠΎΡ‚ΠΌΠ΅Π½Π½ΠΎΠ΅ ΠΈΠ΄Ρ‚ΠΈ Π² спСцназ.

7. НуТно Π»ΠΈ ΠΏΡ€ΠΎΡ…ΠΎΠ΄ΠΈΡ‚ΡŒ мСдкомиссию Π² Π’ΠΎΠ΅Π½Π½ΠΎ-мСдицинской Π°ΠΊΠ°Π΄Π΅ΠΌΠΈΠΈ ΠΈΠΌ ΠšΠΈΡ€ΠΎΠ²Π° ΠΏΠΎΠ²Ρ‚ΠΎΡ€Π½ΠΎ ΠΏΡ€ΠΈ поступлСнии, Ссли я Π΅Π΅ ΡƒΠΆΠ΅ ΠΏΡ€ΠΎΡˆΠ΅Π» ΠΎΡ‚ Π²ΠΎΠ΅Π½ΠΊΠΎΠΌΠ°Ρ‚Π° Π² Π³.Π•ΠΊΠ°Ρ‚Π΅Ρ€ΠΈΠ½Π±ΡƒΡ€Π³Π΅ Π·Π° 3 мСсяца Π΄ΠΎ поступлСния.

7.1. Π’ΠΎΠ΅Π½ΠΊΠΎΠΌΠ°Ρ‚-Π²ΠΎΠ΅Π½ΠΊΠΎΠΌΠ°Ρ‚ΠΎΠΌ, Π° комиссия Π² Π² самой Π°ΠΊΠ°Π΄Π΅ΠΌΠΈΠΈ ΠΎΠ±ΡΠ·Π°Ρ‚Π΅Π»ΡŒΠ½Π°.

7.2. Π’ ΠΎΠ±ΡΠ·Π°Ρ‚Π΅Π»ΡŒΠ½ΠΎΠΌ порядкС Π²Π°ΠΌ Π½Π΅ΠΎΠ±Ρ…ΠΎΠ΄ΠΈΠΌΠΎ ΠΏΡ€ΠΎΠΉΡ‚ΠΈ Π’Π’Πš ΠΏΡ€ΠΈ поступлСнии Π² Π’ΠΎΠ΅Π½Π½ΠΎ-ΠΌΠ΅Π΄ΠΈΡ†ΠΈΠ½ΡΠΊΡƒΡŽ акадСмию ΠΈΠΌ ΠšΠΈΡ€ΠΎΠ²Π°.

8. ЗдрастС, Ρƒ мСня Π²ΠΎΡ‚ Ρ‚Π°ΠΊΠΎΠΉ вопрос, ΠΌΠΎΠ³Ρƒ Π»ΠΈ я Π²Ρ‚ΠΎΡ€ΠΎΠΉ Ρ€Π°Π· ΠΏΡ€ΠΎΠΉΡ‚ΠΈ мСдкомиссию?, Ссли я 5 Π»Π΅Ρ‚ Π½Π°Π·Π°Π΄ откосил ΠΎΡ‚ Π°Ρ€ΠΌΠΈΠΈ ΠΈ ΠΌΠ½Π΅ Π΄Π°Π»ΠΈ Π²ΠΎΠ΅Π½Π½Ρ‹ΠΉ Π±ΠΈΠ»Π΅Ρ‚ ΠΈ приписали Π΄ΠΈΠ°Π³Π½ΠΎΠ· психичСскоС расстройство. Π’Π΅ΠΏΠ΅Ρ€ΡŒ я Π½Π΅ ΠΌΠΎΠ³Ρƒ ΠΏΡ€Π°Π²Π° ΠΏΠΎΠ»ΡƒΡ‡ΠΈΡ‚ΡŒ Π½Π° ΠΌΠ°ΡˆΠΈΠ½Ρƒ Ρ‚ΠΎΡ‡Π½Π΅ΠΉ справку Π² Π±ΠΎΠ»ΡŒΠ½ΠΈΡ†Π΅, ΠΌΠ½Π΅ говорят Ρ‡Ρ‚ΠΎ ΠΌΠ½Π΅ Π½ΡƒΠΆΠ½ΠΎ ΠΏΠΎΠ΄Π°Π²Π°Ρ‚ΡŒ заявлСниС Π² суд Π½Π° рассмотрСниС ΠΏΠΎΠ²Ρ‚ΠΎΡ€Π½ΡƒΡŽ комиссию, Ссли я Π΄Π°ΠΆΠ΅ ΠΏΡ€ΠΎΠΉΠ΄Ρƒ мСдкомиссию Π²ΠΎΠ΅Π½ΠΊΠΎΠΌΠ°Ρ‚Π΅ ΠΈ мСня ΠΏΡ€ΠΈΠ·Π½Π°ΡŽΡ‚ Π°Π΄Π΅ΠΊΠ²Π°Ρ‚Π½Ρ‹ΠΌ, мСня ΠΌΠΎΠ³ΡƒΡ‚ Π² Π°Ρ€ΠΌΠΈΡŽ Π·Π°Π±Ρ€Π°Ρ‚ΡŒ? ΠŸΠΎΠ΄ΡΠΊΠ°ΠΆΠΈΡ‚Π΅ поТалуйста.

8.1. ЗдравствуйтС.

Π’Π΅ΠΏΠ΅Ρ€ΡŒ ΡƒΠΆΠ΅ Π½Π΅ ΠΌΠΎΠΆΠ΅Ρ‚Π΅.

8.2. Всё Π²Π΅Ρ€Π½ΠΎ. Если с Вас снимут Π΄ΠΈΠ°Π³Π½ΠΎΠ· Π’Π°ΠΌ придётся ΠΎΡ‚ΡΠ»ΡƒΠΆΠΈΡ‚ΡŒ. Но Π·Π°Ρ‚ΠΎ Π’Ρ‹ Π±ΡƒΠ΄Π΅Ρ‚Π΅ с ΠΏΡ€Π°Π²Π°ΠΌΠΈ.

9. МнС 34 Π³ΠΎΠ΄Π° я Π±Ρ‹Π» Π½Π΅ Π³ΠΎΠ΄Π΅Π½ ΠΊ Π²ΠΎΠ΅Π½Π½ΠΎΠΉ слуТбС Ρƒ мСня Π±Ρ‹Π» выявлСн хроничСский Π³Π΅ΠΏΠ°Ρ‚ΠΈΡ‚ Π‘, Π½Π΅Π΄Π°Π²Π½ΠΎ Π±Ρ‹Π» Π² Π²ΠΎΠ΅Π½ΠΊΠΎΠΌΠ°Ρ‚Π΅ Ρ‡Ρ‚ΠΎΠ±Ρ‹ ΠΏΠΎΠ»ΡƒΡ‡ΠΈΡ‚ΡŒ Π²ΠΎΠ΅Π½Π½Ρ‹ΠΉ Π±ΠΈΠ»Π΅Ρ‚ ΠΏΡ€ΠΎΡˆΡ‘Π» ΠΏΠΎΠ²Ρ‚ΠΎΡ€Π½ΠΎ мСдкомиссию сдал Π°Π½Π°Π»ΠΈΠ· ΠΊΡ€ΠΎΠ²ΠΈ Π° Π³Π΅ΠΏΠ°Ρ‚ΠΈΡ‚Π° Π½Π΅Ρ‚, я Π΄ΠΎ этого ΠΏΡ€ΠΎΠΏΠΈΠ²Π°Π» лСкарства Π΄ΠΎΡ€ΠΎΠ³ΠΈΠ΅ ΠΎΡ‚ лСчСния Π³Π΅ΠΏΠ°Ρ‚ΠΈΡ‚Π° ΠΏΠΎΡ‚Ρ€Π΅Π±ΠΎΠ²Π°Π»ΠΈ ΠΏΠΎΠ΄Ρ‚Π²Π΅Ρ€ΠΆΠ΄Π΅Π½ΠΈΠ΅ ΠΎΡ‚ Π±ΠΎΠ»ΡŒΠ½ΠΈΡ†Ρ‹ Π³Π΄Π΅ сдавал Π°Π½Π°Π»ΠΈΠ· ΠΊΡ€ΠΎΠ²ΠΈ.

9.1. ЗдравствуйтС! Π’Ρ‹ вопрос сформулируйтС. Π§Π΅Π³ΠΎ Π’Ρ‹ Ρ…ΠΎΡ‚ΠΈΡ‚Π΅ Π΄ΠΎΠ±ΠΈΡ‚ΡŒΡΡ?


10. Π”Π²Π° Π³ΠΎΠ΄Π° Π½Π°Π·Π°Π΄ я ΠΏΡ€ΠΎΡ…ΠΎΠ΄ΠΈΠ» мСдкомиссию Π² Π²ΠΎΠ΅Π½ΠΊΠΎΠΌΠ°Ρ‚Π΅ ΠΈ Π±Ρ‹Π» Π³ΠΎΠ΄Π΅Π½, ΠΏΠΎΡ‚ΠΎΠΌ ΠΏΠΎΠ»ΡƒΡ‡ΠΈΠ» отсрочку, Ρ‚.ΠΊ. поступил Π² Π²Ρ‹ΡΡˆΠ΅Π΅ ΡƒΡ‡Π΅Π±Π½ΠΎΠ΅ Π·Π°Π²Π΅Π΄Π΅Π½ΠΈΠ΅, Π½ΠΎ сСйчас Ρƒ мСня ΠΎΠ±Π½Π°Ρ€ΡƒΠΆΠΈΠ»Π°ΡΡŒ дискинСзия ТСлчСвыводящих ΠΏΡƒΡ‚Π΅ΠΉ. ΠœΠΎΠ³Ρƒ Π»ΠΈ я сСйчас ΠΏΡ€ΠΎΠΉΡ‚ΠΈ ΠΏΠΎΠ²Ρ‚ΠΎΡ€Π½ΡƒΡŽ мСдкомиссию ΠΈ ΠΊΠ°ΠΊΡƒΡŽ ΠΊΠ°Ρ‚Π΅Π³ΠΎΡ€ΠΈΡŽ годности я ΠΌΠΎΠ³Ρƒ ΠΏΠΎΠ»ΡƒΡ‡ΠΈΡ‚ΡŒ?

10.1. ЗдравствуйтС! Π‘ дискинСзиСй ТСлчСвыводящих ΠΏΡƒΡ‚Π΅ΠΉ Π’Ρ‹ ΠΏΠΎΠ»ΡƒΡ‡ΠΈΡ‚Π΅ ΠΏΡ€ΠΈΠ·Π½Π°Π½Ρ‹ Π³ΠΎΠ΄Π½Ρ‹ΠΌ ΠΊ ΠΏΡ€ΠΎΡ…ΠΎΠΆΠ΄Π΅Π½ΠΈΡŽ слуТбы.
Π’Π°ΠΌ Π½Π°Π·Π½Π°Ρ‡Π°Ρ‚ ΠΆΠ΅Π»Ρ‡Π΅Π³ΠΎΠ½Π½Ρ‹Π΅, Π΄ΠΈΠ΅Ρ‚Ρƒ, спазмолитики ΠΏΡ€ΠΎΠΉΠ΄Π΅Ρ‚Π΅ Π½Π΅ большой курс лСчСния ΠΈ Π’Π°ΡˆΠ° дискСнизия ΠΏΡ€ΠΎΠΉΠ΄Π΅Ρ‚.

11. ΠŸΠΎΠΆΠ°Π»ΡƒΠΉΡΡ‚Π° расписался Π½Π΅ своСй Ρ€ΠΎΡΠΏΠΈΡΡŒΡŽ ΠΈ прСдставился Π±Ρ€Π°Ρ‚ΠΎΠΌ Π² повСсткС Π½Π° мСдкомиссию Π² Π²ΠΎΠ΅Π½ΠΊΠΎΠΌΠ°Ρ‚. БСгодня прислали письмо Ρ‡Ρ‚ΠΎ Ссли Π½Π΅ ΠΏΡ€ΠΈΠ΄Ρƒ ΠΏΠΎΠ²Ρ‚ΠΎΡ€Π½ΠΎ Π±ΡƒΠ΄Π΅Ρ‚ выписан ΡˆΡ‚Ρ€Π°Ρ„ Π² Ρ€Π°Π·ΠΌΠ΅Ρ€Π΅ 500 Ρ€ ΠΈ Π΄Π΅Π»ΠΎ ΠΎΡ‚ΠΏΡ€Π°Π²Π»Π΅Π½ΠΎ Π² ΠΏΠΎΠ»ΠΈΡ†ΠΈΡŽ Ρ‡Ρ‚ΠΎ я Π½Π°Ρ…ΠΎΠΆΡƒΡΡŒ Π² розыскС.

11.1. ЗдравствуйтС. ΠžΡ‚Π²Π΅Ρ‚ΡΡ‚Π²Π΅Π½Π½ΠΎΡΡ‚ΡŒ ΠΌΠΎΠΆΠ΅Ρ‚ Π½Π°ΡΡ‚ΡƒΠΏΠΈΡ‚ΡŒ Ρ‚ΠΎΠ»ΡŒΠΊΠΎ Π² случаС Π»ΠΈΡ‡Π½ΠΎΠ³ΠΎ вручСния повСстки ΠŸΠ Π˜Π—Π«Π’ΠΠ˜ΠšΠ£ Π‘ΠžΠ’Π Π£Π”ΠΠ˜ΠšΠΠœΠ˜ Π’ΠžΠ•ΠΠšΠžΠœΠΠ’Π. ΠŸΠ΅Ρ€Π΅Π΄Π°Ρ‡Π° Ρ‡Π΅Ρ€Π΅Π· родствСнников Π½Π΅ Π²Π»Π΅Ρ‡Π΅Ρ‚ оснований для привлСчСния ΠΊ отвСтствСнности.
ВсСго Π’Π°ΠΌ самого Π΄ΠΎΠ±Ρ€ΠΎΠ³ΠΎ ΠΈ ΡƒΠ΄Π°Ρ‡Π½ΠΎΠ³ΠΎ Ρ€Π°Π·Ρ€Π΅ΡˆΠ΅Π½ΠΈΡ ΠΏΡ€ΠΎΠ±Π»Π΅ΠΌΡ‹. Бпасибо Π·Π° Π²Ρ‹Π±ΠΎΡ€ нашСго сайта.

12. Π’ приписном стоит катСгория Π’, ΠΈΠ·-Π·Π° атопичСского Π΄Π΅Ρ€ΠΌΠ°Ρ‚ΠΈΡ‚Π° ΠΈ Π½ΠΈΠ·ΠΊΠΎΠ³ΠΎ ИМ. ΠŸΠΎΠ»ΡƒΡ‡ΠΈΠ» отсрочку для поступлСния Π² институт, Π½ΠΎ Π²ΠΎΠ΅Π½Π½Ρ‹ΠΉ Π±ΠΈΠ»Π΅Ρ‚ Π½Π΅ Π²Ρ‹Π΄Π°Π»ΠΈ. ПослС окончания Π±Π°ΠΊΠ°Π»Π°Π²Ρ€ΠΈΠ°Ρ‚Π° Π²ΠΎΠ΅Π½ΠΊΠΎΠΌΠ°Ρ‚ Π½Π΅ бСспокоил, поступил Π² магистратуру ΠΈ сСйчас Π΅Ρ‘ Π·Π°ΠΊΠ°Π½Ρ‡ΠΈΠ²Π°ΡŽ. Как ΠΌΠ½Π΅ ΠΏΠΎΠ»ΡƒΡ‡ΠΈΡ‚ΡŒ Π²ΠΎΠ΅Π½Π½Ρ‹ΠΉ Π±ΠΈΠ»Π΅Ρ‚? ΠŸΡ€ΠΈΠ΄Ρ‘Ρ‚ΡΡ Π»ΠΈ ΠΏΡ€ΠΎΡ…ΠΎΠ΄ΠΈΡ‚ΡŒ мСдкомиссию ΠΏΠΎΠ²Ρ‚ΠΎΡ€Π½ΠΎ?

12.1. ЗдравствуйтС! Π’ Π΄Π°Π½Π½ΠΎΠΌ случаС Π²Π°ΠΌ придСтся ΠΏΡ€ΠΎΠΉΡ‚ΠΈ ΠΏΠΎΠ²Ρ‚ΠΎΡ€Π½ΠΎ ΠΌΠ΅Π΄ комиссию. Π’Π°ΠΊΠΆΠ΅ Π²Π°ΠΌ слСдуСт ΡΠΎΠ±Ρ€Π°Ρ‚ΡŒ всС Π΄ΠΎΠΊΡƒΠΌΠ΅Π½Ρ‚Ρ‹ ΠΏΠΎ ΠΏΠΎΠ²ΠΎΠ΄Ρƒ Π΄Π°Π½Π½ΠΎΠΉ Π±ΠΎΠ»Π΅Π·Π½ΠΈ.

13. ΠŸΠ΅Ρ€Π²ΡƒΡŽ мСдкомиссии Π² Π²ΠΎΠ΅Π½ΠΊΠΎΠΌΠ°Ρ‚Π΅ ΠΏΡ€ΠΎΡ…ΠΎΠ΄ΠΈΠ» 5 Π»Π΅Ρ‚ Π½Π°Π·Π°Π΄. Π‘ Π΄ΠΈΠ°Π³Π½ΠΎΠ·ΠΎΠΌ Π±Ρ€ΠΎΠ½Ρ…ΠΈΠ°Π»ΡŒΠ½Π°Ρ астма присвоили ΠΊΠ°Ρ‚Π΅Π³ΠΎΡ€ΠΈΡŽ «Π”» ΠΏΠΎ Π³Ρ€ 1 ст. 52″Π°». МоТно Π»ΠΈ ΡΠ½ΡΡ‚ΡŒ Π΄ΠΈΠ°Π³Π½ΠΎΠ·, ΠΏΡ€ΠΎΠΉΡ‚ΠΈ ΠΏΠΎΠ²Ρ‚ΠΎΡ€Π½ΠΎ комиссию ΠΈ ΠΏΠΎΠΉΡ‚ΠΈ ΠΏΠΎ ΠΊΠΎΠ½Ρ‚Ρ€Π°ΠΊΡ‚Ρƒ? Π’Ρ‹ΡΡˆΠ΅Π΅ ΠΎΠ±Ρ€Π°Π·ΠΎΠ²Π°Π½ΠΈΠ΅ Π΅ΡΡ‚ΡŒ.

13.1. ЗдравствуйтС. Насколько ΠΌΠ½Π΅ извСстно, Π²ΠΎΠ΅Π½ΠΊΠΎΠΌΠ°Ρ‚Ρ‹ сСйчас заинтСрСсованы Π² Π½Π°Π±ΠΎΡ€Π΅»ΠΊΠΎΠ½Ρ‚Ρ€Π°ΠΊΡ‚Π½ΠΈΠΊΠΎΠ²» с Π²Ρ‹ΡΡˆΠΈΠΌ ΠΎΠ±Ρ€Π°Π·ΠΎΠ²Π°Π½ΠΈΠ΅ΠΌ. Но Ρƒ Вас катСгория «Π”,» поэтому вопрос спорный. ΠžΠ±Ρ€Π°Ρ‚ΠΈΡ‚Π΅ΡΡŒ Π² Π²ΠΎΠ΅Π½ΠΊΠΎΠΌΠ°Ρ‚ Π² Π½Π°Ρ‡Π°Π»Π΅ ΡΠ»Π΅Π΄ΡƒΡŽΡ‰Π΅Π³ΠΎ ΠΏΡ€ΠΈΠ·Ρ‹Π²Π° (этот ΡƒΠΆΠ΅ оканчиваСтся). Π§Π΅Π³ΠΎ Π³Π°Π΄Π°Ρ‚ΡŒ Ρ‚ΠΎ? Всё ΠΈΠ½Π΄ΠΈΠ²ΠΈΠ΄ΡƒΠ°Π»ΡŒΠ½ΠΎ Ρ€Π΅ΡˆΠ°Π΅Ρ‚ΡΡ. И ΠΏΠΎΠΌΠ½ΠΈΡ‚Π΅, Ρ‡Ρ‚ΠΎ Ρƒ Вас Π΅ΡΡ‚ΡŒ ΠΏΡ€Π°Π²ΠΎ прохоТдСния нСзависимого Π’Πš ΠΏΠΎ собствСнной ΠΈΠ½ΠΈΡ†ΠΈΠ°Ρ‚ΠΈΠ²Π΅ Π² ΠΌΠ΅Π΄. ΡƒΡ‡Ρ€Π΅ΠΆΠ΄Π΅Π½ΠΈΠΈ (ΠΏΠΎ Π²Ρ‹Π±ΠΎΡ€Ρƒ), ΠΈΠΌΠ΅ΡŽΡ‰Π΅ΠΌ ΡΠΎΠΎΡ‚Π²Π΅Ρ‚ΡΡ‚Π²ΡƒΡŽΡ‰ΡƒΡŽ Π»ΠΈΡ†Π΅Π½Π·ΠΈΡŽ.

14. Π’ 18 Π»Π΅Ρ‚ ΠΏΡ€ΠΎΡ…ΠΎΠ΄ΠΈΠ» мСдкомиссию Π² Π²ΠΎΠ΅Π½ΠΊΠΎΠΌΠ°Ρ‚Π΅ Π½Π΅ ΠΏΡ€ΠΎΡˆΠ΅Π» ΠΏΠΎ Π΄ΠΈΠ°Π³Π½ΠΎΠ·Ρƒ Π­Π½ΡƒΡ€Π΅Π·. МнС сСйчас 23 этой Π±ΠΎΠ»Π΅Π·Π½ΠΈ Π½Π΅Ρ‚Ρƒ Π΄Π°Π²Π½ΠΎ ΠΈ Ρ…ΠΎΡ‚Π΅Π» ΡΠΏΡ€ΠΎΡΠΈΡ‚ΡŒ Ссли я ΠΏΡ€ΠΎΠΉΠ΄Ρƒ ΠΏΠΎΠ²Ρ‚ΠΎΡ€Π½ΠΎ ΠΌΠ΅Π΄ обслСдованиС мСня ΠΌΠΎΠ³ΡƒΡ‚ Π²Π·ΡΡ‚ΡŒ Π² Π°Ρ€ΠΌΠΈΡŽ?

14.1. ЗдравствуйтС, ΠšΠΎΠ½ΡΡ‚Π°Π½Ρ‚ΠΈΠ½
Если ΠΏΡ€ΠΎΠΉΠ΄Π΅Ρ‚Π΅ мСдкомиссию ΠΈ всСх Π²Ρ€Π°Ρ‡Π΅ΠΉ, Ρ‚ΠΎ ΠΊΠΎΠ½Π΅Ρ‡Π½ΠΎ ΠΌΠΎΠ³ΡƒΡ‚ Π²Π·ΡΡ‚ΡŒ Π² Π°Ρ€ΠΌΠΈΡŽ

Π–Π΅Π»Π°ΡŽ Π’Π°ΠΌ ΡƒΠ΄Π°Ρ‡ΠΈ ΠΈ всСх Π±Π»Π°Π³!

15. ΠŸΡ€ΠΈΡΠ»Π°Π»ΠΈ повСстку Π½Π° мСдкомиссию ΠΌΠ½Π΅ 18 Π»Π΅Ρ‚ ΡƒΡ‡ΡƒΡΡŒ Π² ΠΊΠΎΠ»Π»Π΅Π΄ΠΆΠ΅ послС 11 классов мСдкомиссию ΠΏΡ€ΠΎΡˆΡ‘Π» выявили Π΄Π²Π΅ Π±ΠΎΠ»Π΅Π·Π½ΠΈ сказали Ρ‡Ρ‚ΠΎΠ±Ρ‹ Ρ‡Π΅Ρ€Π΅Π· Π΄Π²Π΅ Π½Π΅Π΄Π΅Π»ΠΈ явился с Ρ€Π΅Π·ΡƒΠ»ΡŒΡ‚Π°Ρ‚Π°ΠΌΠΈ обслСдования Π½Π° ΠΏΠΎΠ²Ρ‚ΠΎΡ€Π½ΡƒΡŽ мСдкомиссию ΠΌΠΎΠ³Ρƒ Π»ΠΈ я ΡΡ‚Π°Ρ‚ΡŒ Π½Π° ΡƒΡ‡Ρ‘Ρ‚ Π² Π΄Ρ€ΡƒΠ³ΠΎΠΉ Π²ΠΎΠ΅Π½ΠΊΠΎΠΌΠ°Ρ‚ ΠΈ ΠΏΡ€ΠΎΠΉΡ‚ΠΈ ΠΏΠΎΠ²Ρ‚ΠΎΡ€Π½ΡƒΡŽ мСдкомиссию Ρ‚Π°ΠΌ?

15.1. ЗдравствуйтС! Π’Ρ‹ Π΄ΠΎΠ»ΠΆΠ½Ρ‹ ΡΡ‚ΠΎΡΡ‚ΡŒ Π½Π° ΡƒΡ‡Π΅Ρ‚Π΅ Π² Π²ΠΎΠ΅Π½ΠΊΠΎΠΌΠ°Ρ‚Π΅ ΠΏΠΎ мСсту Π’Π°ΡˆΠ΅Π³ΠΎ прСбывания. Если Π²Ρ‹ состоитС сСйчас Π² Π΄Ρ€ΡƒΠ³ΠΎΠΌ Π²ΠΎΠ΅Π½ΠΊΠΎΠΌΠ°Ρ‚Π΅ — Ρ‚ΠΎ ΠΌΠΎΠΆΠ΅Ρ‚Π΅ Π²ΡΡ‚Π°Ρ‚ΡŒ Π½Π° ΡƒΡ‡Π΅Ρ‚ Π² Π΄Ρ€ΡƒΠ³ΠΎΠΌ. Но, ΠΏΡ€ΠΈ этом, вас снова ΠΏΠΎΡˆΠ»ΡŽΡ‚ Π½Π° обслСдованиС ΠΈ ΠΏΠΎΡ‚ΠΎΠΌ снова Π½Π°Π΄ΠΎ Π±ΡƒΠ΄Π΅ ΠΏΡ€ΠΎΡ…ΠΎΠ΄ΠΈΡ‚ΡŒ ΠΏΠΎΠ²Ρ‚ΠΎΡ€Π½ΠΎΠ΅ ΠΎΡΠ²ΠΈΠ΄Π΅Ρ‚Π΅Π»ΡŒΡΡ‚Π²ΠΎΠ²Π°Π½ΠΈΠ΅.

16. Π― ΠΏΡ€ΠΎΡˆΡ‘Π» ΠΊΠΎΠ½Ρ‚Ρ€ΠΎΠ»ΡŒΠ½ΡƒΡŽ мСдкомиссию Π² Π²ΠΎΠ΅Π½ΠΊΠΎΠΌΠ°Ρ‚Π΅, Ρ€Π°Π½ΡŒΡˆΠ΅ стояла Π’, Π² ΠΈΡ‚ΠΎΠ³Π΅ поставили Π‘ 3 Π² Π°ΠΏΡ€Π΅Π»Π΅ ΠΏΠΎ ΠΈΠ΄Π΅Π΅ Π±ΡƒΠ΄Π΅Ρ‚ повСстка, ΠΌΠΎΠΆΠ½ΠΎ Π»ΠΈ ΠΏΠΎΠ²Ρ‚ΠΎΡ€Π½ΠΎ ΠΏΡ€ΠΎΠΉΡ‚ΠΈ мСдкомиссию, послС Ρ‚ΠΎΠ³ΠΎ ΠΊΠ°ΠΊ ΠΏΡ€ΠΎΡˆΡ‘Π» ΠΊΠΎΠ½Ρ‚Ρ€ΠΎΠ»ΡŒΠ½ΡƒΡŽ комиссию?

16.1. Π”ΠΎΠ±Ρ€Ρ‹ΠΉ дСнь, Ссли Ρƒ вас для этого появились ΠΊΠ°ΠΊΠΈΠ΅ Ρ‚ΠΎ Π½ΠΎΠ²Ρ‹Π΅ основания, Ρ‚ΠΎ ΠΌΠΎΠΆΠ½ΠΎ Π² этом случаС Π²Π°ΠΌ ΠΏΠΎΠ²Ρ‚ΠΎΡ€Π½ΠΎ ΠΏΡ€ΠΎΠΉΡ‚ΠΈ мСдкомиссию.

16.2. На Π΄Π°Π½Π½Ρ‹ΠΉ ΠΌΠΎΠΌΠ΅Π½Ρ‚ Ρƒ вас установлСна опрСдСлСнная Π³Ρ€ΡƒΠΏΠΏΠ° годности ΠΊ ΠΏΡ€ΠΎΡ…ΠΎΠΆΠ΄Π΅Π½ΠΈΡŽ Π²ΠΎΠ΅Π½Π½ΠΎΠΉ слуТбы. ΠžΠΊΠΎΠ½Ρ‡Π°Ρ‚Π΅Π»ΡŒΠ½ΠΎΠ΅ Ρ€Π΅ΡˆΠ΅Π½ΠΈΠ΅ ΠΎ ΠΏΡ€ΠΈΠ·Ρ‹Π²Π΅ ΠΏΡ€ΠΈΠ½ΠΈΠΌΠ°Π΅Ρ‚ призывная комиссия ΠΈ Π΄ΠΎ апрСля всС ΠΌΠΎΠΆΠ΅Ρ‚ измСнится, Π½Π°ΠΏΡ€ΠΈΠΌΠ΅Ρ€ Π²Ρ‹ ΠΌΠΎΠΆΠ΅Ρ‚Π΅ ΠΏΡ€ΠΎΠΉΡ‚ΠΈ Π΄ΠΎΠΏΠΎΠ»Π½ΠΈΡ‚Π΅Π»ΡŒΠ½ΠΎΠ΅ обслСдованиС ΠΊΠΎΡ‚ΠΎΡ€ΠΎΠ΅ ΠΌΠΎΠΆΠ΅Ρ‚ ΠΏΠΎΠ²Π»ΠΈΡΡ‚ΡŒ Π½Π° ΠΈΡ‚ΠΎΠ³ΠΎΠ²Ρ‹ΠΉ Π²Ρ‹Π²ΠΎΠ΄. Π­Ρ‚ΠΎ ΠΈ Π±ΡƒΠ΄Π΅Ρ‚ ΠΎΠ΄Π½ΠΎ ΠΈΠ· оснований ΠΏΠΎΠ²Ρ‚ΠΎΡ€Π½ΠΎΠ³ΠΎ направлСния вас Π½Π° Π’ΠšΠš.

16.3. ЗдравствуйтС Иван!
, ΠΌΠΎΠΆΠ½ΠΎ Π»ΠΈ ΠΏΠΎΠ²Ρ‚ΠΎΡ€Π½ΠΎ ΠΏΡ€ΠΎΠΉΡ‚ΠΈ мСдкомиссию, послС Ρ‚ΠΎΠ³ΠΎ ΠΊΠ°ΠΊ ΠΏΡ€ΠΎΡˆΡ‘Π» ΠΊΠΎΠ½Ρ‚Ρ€ΠΎΠ»ΡŒΠ½ΡƒΡŽ комиссию?
К соТалСнию Ссли Π²Ρ‹ Π±ΡƒΠ΄Π΅Ρ‚Π΅ Ρ‚ΠΎΠ»ΡŒΠΊΠΎ Π½Π°ΡΡ‚Π°ΠΈΠ²Π°Ρ‚ΡŒ Π½Π° ΠΏΠΎΠ²Ρ‚ΠΎΡ€Π½ΠΎΠΌ обслСдовании. Если Π½Π΅ Ρ…ΠΎΡ‚ΠΈΡ‚Π΅ Π² Π°Ρ€ΠΌΠΈΡŽ собСритС Π½Π΅ΠΎΠ±Ρ…ΠΎΠ΄ΠΈΠΌΡ‹Π΅ Π΄ΠΎΠΊΡƒΠΌΠ΅Π½Ρ‚Ρ‹ ΠΏΠΎ вашим болСзням.

16.4. ЗдравствуйтС!
ΠŸΠΎΠ²Ρ‚ΠΎΡ€Π½ΠΎ ΠΌΠΎΠΆΠ½ΠΎ ΠΏΡ€ΠΎΠΉΡ‚ΠΈ Ρ‚ΠΎΠ»ΡŒΠΊΠΎ Ссли состояниС Π·Π΄ΠΎΡ€ΠΎΠ²ΡŒΡ Ρƒ Вас измСнится. ΠŸΡ€ΠΎΡΡ‚ΠΎ Ρ‚Π°ΠΊ Π½Π° комиссию Вас Π½Π΅ отправят.

17. Π£ сына Π΅ΡΡ‚ΡŒ Π²ΠΎΠ΅Π½Π½Ρ‹ΠΉ Π±ΠΈΠ»Π΅Ρ‚ с ΠΎΡ‚ΠΌΠ΅Ρ‚ΠΊΠΎΠΉ ΠΎΠ³Ρ€Π°Π½ΠΈΡ‡Π΅Π½Π½ΠΎ Π³ΠΎΠ΄Π΅Π½. Π‘Ρ‹Π½ судим, послС освобоТдСния ΠΏΠΎΠ΅Ρ…Π°Π» Π² Π²ΠΎΠ΅Π½ΠΊΠΎΠΌΠ°Ρ‚, Π³Π΄Π΅ Π΅ΠΌΡƒ сказали Ρ‡Ρ‚ΠΎ Π΅Π³ΠΎ ΠΈΠ· Π·Π° судимости сняли с ΡƒΡ‡Π΅Ρ‚Π°, ΠΈ Π΄Π°Π»ΠΈ Π½Π°ΠΏΡ€Π°Π²Π»Π΅Π½ΠΈΠ΅ Π½Π° мСдкомиссию для ΠΏΠΎΠ²Ρ‚ΠΎΡ€Π½ΠΎΠ³ΠΎ становлСния Π½Π° ΡƒΡ‡Π΅Ρ‚. ΠœΠΎΠ³ΡƒΡ‚ Π»ΠΈ Π΅Π³ΠΎ ΠΏΡ€ΠΈΠ·Π²Π°Ρ‚ΡŒ Π² Π°Ρ€ΠΌΠΈΡŽ?

17.1. Если ΠΏΡ€ΠΈΠ·Π½Π°ΡŽΡ‚, Ρ‡Ρ‚ΠΎ Π³ΠΎΠ΄Π΅Π½, Π° возраст ΠΌΠ΅Π½Π΅Π΅ 27 Π»Π΅Ρ‚ — ΠΌΠΎΠ³ΡƒΡ‚.

18. Π£ сына Π΅ΡΡ‚ΡŒ Π²ΠΎΠ΅Π½Π½Ρ‹ΠΉ Π±ΠΈΠ»Π΅Ρ‚ с ΠΎΡ‚ΠΌΠ΅Ρ‚ΠΊΠΎΠΉ ΠΎΠ³Ρ€Π°Π½ΠΈΡ‡Π΅Π½Π½ΠΎ Π³ΠΎΠ΄Π΅Π½. Π‘Ρ‹Π½ судим, послС освобоТдСния ΠΏΠΎΠ΅Ρ…Π°Π» Π² Π²ΠΎΠ΅Π½ΠΊΠΎΠΌΠ°Ρ‚, Π³Π΄Π΅ Π΅ΠΌΡƒ сказали Ρ‡Ρ‚ΠΎ Π΅Π³ΠΎ ΠΈΠ· Π·Π° судимости сняли с ΡƒΡ‡Π΅Ρ‚Π°, ΠΈ Π΄Π°Π»ΠΈ Π½Π°ΠΏΡ€Π°Π²Π»Π΅Π½ΠΈΠ΅ Π½Π° мСдкомиссию для ΠΏΠΎΠ²Ρ‚ΠΎΡ€Π½ΠΎΠ³ΠΎ становлСния Π½Π° ΡƒΡ‡Π΅Ρ‚. ΠœΠΎΠ³ΡƒΡ‚ Π»ΠΈ Π΅Π³ΠΎ ΠΏΡ€ΠΈΠ·Π²Π°Ρ‚ΡŒ Π² Π°Ρ€ΠΌΠΈΡŽ?

18.1. ЗдравствуйтС. Π”Π° ΠΌΠΎΠ³ΡƒΡ‚.

19. ΠŸΡ€ΠΎΡˆΠ΅Π» мСдкомиссию Π² Π²ΠΎΠ΅Π½ΠΊΠΎΠΌΠ°Ρ‚Π΅, ΠΏΠΎΠ΅Ρ…Π°Π» ΠΏΠΎ повСсткС Π² ΠΏΡƒΠ½ΠΊΡ‚ сбора. ПослС обслСдования послали Π΄ΠΎΠΌΠΎΠΉ сославшись Π½Π° Ρ‚ΠΎ Ρ‡Ρ‚ΠΎ мСня Π½Π΅ Π·Π°Ρ…ΠΎΡ‚Π΅Π»ΠΈ Π±Ρ€Π°Ρ‚ΡŒ ΠΈΠ· Π·Π° Ρ‚Π°Ρ‚ΡƒΠΈΡ€ΠΎΠ²ΠΊΠΈ ΠΈ Π½Π° Ρ€ΡƒΠΊΠ΅, Π΄Π°Π»ΠΈ ΠΏΠΎΠ²Ρ‚ΠΎΡ€Π½ΡƒΡŽ повСстку Π½Π° 20 число. МоТно Π»ΠΈ ΠΊΠ°ΠΊ Ρ‚ΠΎ Π½Π΅ ΠΏΠΎΠΉΡ‚ΠΈ Π² Π°Ρ€ΠΌΠΈΡŽ.

19.1. ЗдравствуйтС! Π’Π°Ρ‚ΡƒΠΈΡ€ΠΎΠ²ΠΊΠ° Π½Π΅ являСтся прСпятствиСм для ΠΏΡ€ΠΈΠ·Ρ‹Π²Π° Π² Π°Ρ€ΠΌΠΈΡŽ, Π² Π³Ρ€Π°Ρ„Π΅ «особыС ΠΏΡ€ΠΈΠΌΠ΅Ρ‚Ρ‹Β» ΡΠ΄Π΅Π»Π°ΡŽΡ‚ ΡΠΎΠΎΡ‚Π²Π΅Ρ‚ΡΡ‚Π²ΡƒΡŽΡ‰ΡƒΡŽ ΠΎΡ‚ΠΌΠ΅Ρ‚ΠΊΡƒ. Вас ΠΌΠΎΠ³ΡƒΡ‚ Π·Π°ΡΡ‚Π°Π²ΠΈΡ‚ΡŒ ΠΏΡ€ΠΎΠΉΡ‚ΠΈ ΡΠΎΠΎΡ‚Π²Π΅Ρ‚ΡΡ‚Π²ΡƒΡŽΡ‰Π΅Π΅ обслСдованиС, Π½Π° Π΅Π³ΠΎ основании Π±ΡƒΠ΄Π΅Ρ‚ принято Ρ€Π΅ΡˆΠ΅Π½ΠΈΠ΅ ΠΎ ΠΏΡ€ΠΈΠ·Ρ‹Π²Π΅.

19.2. Николай, Π΄ΠΎΠ±Ρ€Ρ‹ΠΉ дСнь! МоТно ΠΏΠΎΠΏΡ€ΠΎΠ±ΠΎΠ²Π°Ρ‚ΡŒ ΡΠΎΠ±Ρ€Π°Ρ‚ΡŒ ΠΏΠ°ΠΊΠ΅Ρ‚ Π΄ΠΎΠΊΡƒΠΌΠ΅Π½Ρ‚ΠΎΠ² для Π°Π»ΡŒΡ‚Π΅Ρ€Π½Π°Ρ‚ΠΈΠ²Π½ΠΎΠΉ слуТбы. Π― сам слуТил, Ρ‚Π°ΠΌ Π½ΠΈΡ‡Π΅Π³ΠΎ Ρ…ΠΎΡ€ΠΎΡˆΠ΅Π³ΠΎ Π½Π΅Ρ‚. Π›ΡƒΡ‡ΡˆΠ΅ Π°Π»ΡŒΡ‚Π΅Ρ€Π½Π°Ρ‚ΠΈΠ²Π°. Π‘Π²ΡΠ·Π°Ρ‚ΡŒΡΡ со ΠΌΠ½ΠΎΠΉ ΠΌΠΎΠΆΠ΅Ρ‚Π΅ ΠΏΠΎ ΠΏΠΎΡ‡Ρ‚Π΅: [email protected]

Π‘ ΡƒΠ²Π°ΠΆΠ΅Π½ΠΈΠ΅ΠΌ, ΡŽΡ€ΠΈΡΡ‚ Π“Π°Π±Π΄Ρ€Π°Ρ…ΠΌΠ°Π½ΠΎΠ² ДСнис Π“Π°Π±Π΄ΡƒΠ»Π»ΠΎΠ²ΠΈΡ‡!

20. Π’ 17 Π»Π΅Ρ‚ я ΠΏΡ€ΠΎΡ…ΠΎΠ΄ΠΈΠ» мСдкомиссию ΠΈ ΠΌΠ½Π΅ поставили ΡˆΡ‚Π°ΠΌΠΏ ΠžΠ³Ρ€Π°Π½ΠΈΡ‡Π΅Π½Π½ΠΎ Π³ΠΎΠ΄Π΅Π½ Π½ΡƒΠΆΠ½ΠΎ Π»ΠΈ ΠΌΠ½Π΅ ΠΏΠΎ Π΄ΠΎΡΡ‚ΠΈΠΆΠ΅Π½ΠΈΡŽ ΡΠΎΠ²Π΅Ρ€ΡˆΠ΅Π½Π½ΠΎΠ»Π΅Ρ‚ΠΈΡ ΡΠ²ΠΈΡ‚ΡŒΡΡ Π² Π²ΠΎΠ΅Π½ΠΊΠΎΠΌΠ°Ρ‚ для ΠΏΠΎΠ²Ρ‚ΠΎΡ€Π½ΠΎΠΉ мСдкомиссии ΠΈ ΠΎΠΊΠΎΠ½Ρ‡Π°Ρ‚Π΅Π»ΡŒΠ½ΠΎΠ³ΠΎ Π·Π°ΠΊΠ»ΡŽΡ‡Π΅Π½ΠΈΡ?

20.1. ЗдравствуйтС! Π”Π°,ΠΎΠ±ΡΠ·Π°Ρ‚Π΅Π»ΡŒΠ½ΠΎ.

21. Π£ мСня такая ситуация… Π’ ΠΏΡ€ΠΎΡˆΠ»ΠΎΠΌ Π³ΠΎΠ΄Ρƒ я ΠΏΡ€ΠΎΡ…ΠΎΠ΄ΠΈΠ» мСдкомиссию Π² Π²ΠΎΠ΅Π½ΠΊΠΎΠΌΠ°Ρ‚Π΅ ΠΈ ΠΌΠ½Π΅ поставили ΠΊΠ°Ρ‚Π΅Π³ΠΎΡ€ΠΈΡŽ годности Π’. И Π² Ρ‚ΠΎΠΌ ΠΆΠ΅ Π³ΠΎΠ΄Ρƒ ΠΌΡ‹ с ΠΆΠ΅Π½ΠΎΠΉ ΠΏΠ΅Ρ€Π΅Π΅Ρ…Π°Π»ΠΈ ΠΆΠΈΡ‚ΡŒ Π·Π°Π³Ρ€Π°Π½ΠΈΡ†Ρƒ. Π― снялся с прописки ΠΈ с Π²ΠΎΠ΅Π½Π½ΠΎΠ³ΠΎ ΡƒΡ‡Π΅Ρ‚Π°. Однако Π² этом Π³ΠΎΠ΄Ρƒ ΠΌΡ‹ Ρ€Π΅ΡˆΠΈΠ»ΠΈ Π²Π΅Ρ€Π½ΡƒΡ‚ΡŒΡΡ ΠΎΠ±Ρ€Π°Ρ‚Π½ΠΎ. Π’ Ρ‚ΠΎΡ‚ ΠΆΠ΅ Π³ΠΎΡ€ΠΎΠ΄. Вопрос: ΠΏΡ€ΠΈ ΠΏΠΎΠ²Ρ‚ΠΎΡ€Π½ΠΎΠΉ постановкС Π½Π° ΡƒΡ‡Π΅Ρ‚, с ΠΊΠ°Ρ‚Π΅Π³ΠΎΡ€ΠΈΠ΅ΠΉ годности Π’, Π½Π°Π΄ΠΎ Π»ΠΈ ΠΌΠ½Π΅ Π±ΡƒΠ΄Π΅Ρ‚ вновь ΠΏΡ€ΠΎΡ…ΠΎΠ΄ΠΈΡ‚ мСдкомиссию?

21.1. Π”Π°. Π½ΡƒΠΆΠ½ΠΎ Π±ΡƒΠ΄Π΅Ρ‚ ΠΏΡ€ΠΎΡ…ΠΎΠ΄ΠΈΡ‚ΡŒ ΠΌΠ΅Π΄. комиссию.

22. Π£ мСня Ρ‚Π°ΠΊΠΎΠΉ вопрос, Π² 18 Π»Π΅Ρ‚ ΠΌΠ½Π΅ Π² Π²ΠΎΠ΅Π½ΠΊΠΎΠΌΠ°Ρ‚Π΅ поставили ΠΊΠ°Ρ‚Π΅Π³ΠΎΡ€ΠΈΡŽ Π’ 1, ΠΎΡ‚Π΄Π°Π»ΠΈ Π²ΠΎΠ΅Π½Π½Ρ‹ΠΉ Π±ΠΈΠ»Π΅Ρ‚ ΠΈ всС гуляй. А сСйчас ΠΌΠ½Π΅ 22 Π³ΠΎΠ΄Π° ΠΈ я Ρ…ΠΎΡ‡Ρƒ ΠΏΠΎΠΉΡ‚ΠΈ ΡΠ»ΡƒΠΆΠΈΡ‚ΡŒ, Π²ΠΎΠ·ΠΌΠΎΠΆΠ½ΠΎ Π»ΠΈ ΠΏΡ€ΠΎΠΉΡ‚ΠΈ ΠΏΠΎΠ²Ρ‚ΠΎΡ€Π½ΡƒΡŽ мСдкомиссию, ΠΏΠΎΠΌΠ΅Π½ΡΡ‚ΡŒ ΠΊΠ°Ρ‚Π΅Π³ΠΎΡ€ΠΈΡŽ ΠΈ ΠΏΠΎΠΉΡ‚ΠΈ ΡΠ»ΡƒΠΆΠΈΡ‚ΡŒ?

22.1. ЗдравствуйтС! Π”Π°, ΠΌΠΎΠΆΠ΅Ρ‚Π΅ ΠΏΡ€ΠΎΠΉΡ‚ΠΈ ΠΏΠΎΠ²Ρ‚ΠΎΡ€Π½ΡƒΡŽ мСдкомиссию.

23. На протяТСнии Π΄Π»ΠΈΡ‚Π΅Π»ΡŒΠ½ΠΎΠ³ΠΎ Π²Ρ€Π΅ΠΌΠ΅Π½ΠΈ ΠΏΡ€ΠΎΡ…ΠΎΠ΄ΠΈΠ» мСдкомиссию Π² Π²ΠΎΠ΅Π½ΠΊΠΎΠΌΠ°Ρ‚Π΅ ΠΈ ΠΏΠΎ Π·Ρ€Π΅Π½ΠΈΡŽ Π±Ρ‹Π» ΠΏΡ€ΠΈΠ·Π½Π°Π½ ΠΎΠ³Ρ€Π°Π½ΠΈΡ‡Π΅Π½Π½ΠΎ Π³ΠΎΠ΄Π½Ρ‹ΠΌ. ΠšΠ°Ρ‚Π΅Π³ΠΎΡ€ΠΈΡ Π’.ПослС Π»Π°Π·Π΅Ρ€Π½ΠΎΠΉ ΠΊΠΎΡ€Ρ€Π΅ΠΊΡ†ΠΈΠΈ Π·Ρ€Π΅Π½ΠΈΠ΅ ΡƒΠ»ΡƒΡ‡ΡˆΠΈΠ»ΠΎΡΡŒ Π΄ΠΎ-0.5 (Π±Ρ‹Π»ΠΎ минус 7) На ΠΌΠΎΠΉ вопрос ΠΎ ΠΏΡ€ΠΎΡ…ΠΎΠΆΠ΄Π΅Π½ΠΈΠΈ ΠΏΠΎΠ²Ρ‚ΠΎΡ€Π½ΠΎΠΉ мСдкомиссии Π² Π²ΠΎΠ΅Π½ΠΊΠΎΠΌΠ°Ρ‚Π΅ ΠΈ Π·Π°ΠΌΠ΅Π½Π΅ ΠΊΠ°Ρ‚Π΅Π³ΠΎΡ€ΠΈΠΈ с ΠΎΠ³Ρ€Π°Π½ΠΈΡ‡Π΅Π½ΠΈΠ΅ΠΌ ΠΊ слуТбС, ΠΏΠΎΠ»ΡƒΡ‡ΠΈΠ» ΠΎΡ‚ΠΊΠ°Π·. Π’Ρ€Π°Ρ‡ΠΈ объясняли Ρ‡Ρ‚ΠΎ Π·Π°ΠΌΠ΅Π½ΠΈΡ‚ΡŒ ΠΊΠ°Ρ‚Π΅Π³ΠΎΡ€ΠΈΡŽ ΠΎΠ½ΠΈ Π½Π΅ ΠΌΠΎΠ³ΡƒΡ‚. Но ΠΌΠΎΠΆΠ΅Ρ‚ это Π²ΠΎΠ·ΠΌΠΎΠΆΠ½ΠΎ. ΠŸΠΎΠ΄ΡΠΊΠ°ΠΆΠΈΡ‚Π΅ поТалуйста?

23.1. Π’Π°Ρˆ вопрос сугубо мСдицинский Π§Ρ‚ΠΎ касаСтся ΡŽΡ€ΠΈΠ΄ΠΈΡ‡Π΅ΡΠΊΠΎΠΉ стороны Ρ‚ΠΎ Π²Π°ΠΌ Π½ΡƒΠΆΠ½ΠΎ Π½Π°ΠΏΠΈΡΠ°Ρ‚ΡŒ заявлСниС Π² Π’Πš ΠΎ Π·Π°ΠΌΠ΅Π½Π΅ ΠΊΠ°Ρ‚Π΅Π³ΠΎΡ€ΠΈΠΈ Если Π±ΡƒΠ΄Π΅Ρ‚ ΠΎΡ‚ΠΊΠ°Π·, Ρ‚ΠΎ придСтся ΠΎΠ±Ρ€Π°Ρ‰Π°Ρ‚ΡŒΡΡ Π² суд ΠΈ Π΄ΠΎΠΊΠ°Π·Ρ‹Π²Π°Ρ‚ΡŒ Ρ‡Ρ‚ΠΎ с вашим Π·Ρ€Π΅Π½ΠΈΠ΅ΠΌ Π²ΠΎΠ·ΠΌΠΎΠΆΠ½Π° Π·Π°ΠΌΠ΅Π½Π° ΠΊΠ°Ρ‚Π΅Π³ΠΎΡ€ΠΈΠΈ Π― Π»ΠΈΡ‡Π½ΠΎ Π½Π΅ знаю смоТСтС Π»ΠΈ Π’Ρ‹ это Π΄ΠΎΠΊΠ°Π·Π°Ρ‚ΡŒ.

24. МнС 26 Π»Π΅Ρ‚. Π’ 18 Π»Π΅Ρ‚ ΠΏΡ€ΠΎΡˆΠ΅Π» мСдкомиссию Π² Π²ΠΎΠ΅Π½ΠΊΠΎΠΌΠ°Ρ‚Π΅ ΠΈ ΠΏΡ€ΠΈΠ·Π½Π°Π»ΠΈ ΠΎΠ³Ρ€Π°Π½ΠΈΡ‡Π΅Π½Π½ΠΎ Π³ΠΎΠ΄Π½Ρ‹ΠΌ (катСгория Π’). ΠœΠΎΠ³Ρƒ Π»ΠΈ я ΠΏΡ€ΠΎΠΉΡ‚ΠΈ мСдкомиссию ΠΏΠΎΠ²Ρ‚ΠΎΡ€Π½ΠΎ Ссли я Ρ…ΠΎΡ‡Ρƒ ΡΠ»ΡƒΠΆΠΈΡ‚ΡŒ?

24.1. ΠœΠΎΠΆΠ΅Ρ‚Π΅ ΠΏΡ€ΠΎΠΉΡ‚ΠΈ ΠΏΠΎΠ²Ρ‚ΠΎΡ€Π½ΠΎ-ΠΎΠ±Ρ€Π°Ρ‚ΠΈΡ‚Π΅ΡΡŒ с заявлСниСм Π² Π²ΠΎΠ΅Π½ΠΊΠΎΠΌΠ°Ρ‚.

25. НуТно Π»ΠΈ ΠΌΠ½Π΅ Π±ΡƒΠ΄Π΅Ρ‚ ΠΏΠΎΠ²Ρ‚ΠΎΡ€Π½ΠΎ ΠΏΡ€ΠΎΡ…ΠΎΠ΄ΠΈΡ‚ΡŒ мСдкомиссию Π² Π²ΠΎΠ΅Π½ΠΊΠΎΠΌΠ°Ρ‚Π΅ Π² 18 Π»Π΅Ρ‚? И ΠΌΠΎΠΆΠ΅Ρ‚ Π»ΠΈ Ρ‚Π°ΠΊ ΠΏΠΎΠ»ΡƒΡ‡ΠΈΡ‚ΡŒΡΡ, Ρ‡Ρ‚ΠΎ ΠΊΠ°Ρ‚Π΅Π³ΠΎΡ€ΠΈΡŽ годности Π’, Π΄Π°Π½Π½ΡƒΡŽ ΠΌΠ½Π΅ Π² 17 Π»Π΅Ρ‚, смСнят Π½Π° Π΄Ρ€ΡƒΠ³ΡƒΡŽ?

25.1. ЗдравствуйтС. ΠŸΡ€ΠΈΠ΄Π΅Ρ‚ΡΡ ΠΈ Π΅Ρ‰Ρ‘ Π½Π΅ ΠΎΠ΄Π½Ρƒ. ΠšΠ°Ρ‚Π΅Π³ΠΎΡ€ΠΈΡ ΠΌΠΎΠΆΠ΅Ρ‚ ΠΈΠ·ΠΌΠ΅Π½ΠΈΡ‚ΡŒΡΡ.

26. Π’ Π²ΠΎΠ΅Π½ΠΊΠΎΠΌΠ°Ρ‚Π΅ поставили ΠΊΠ°Ρ‚Π΅Π³ΠΎΡ€ΠΈΡŽ Π‘-3. ΠœΠΎΠ³Ρƒ Π»ΠΈ я ΠΏΠΎΠ²Ρ‚ΠΎΡ€Π½ΠΎ ΠΏΡ€ΠΎΠΉΡ‚ΠΈ мСдкомиссию Ссли Π½Π΅ согласСн с выставлСнной ΠΊΠ°Ρ‚Π΅Π³ΠΎΡ€ΠΈΠ΅ΠΉ?

26.1. НЕВ, придСтся ΠΎΠ±ΠΆΠ°Π»ΠΎΠ²Π°Ρ‚ΡŒ Ρ€Π΅Π·ΡƒΠ»ΡŒΡ‚Π°Ρ‚Ρ‹ комиссии Π² судС.

Π£Π”ΠΠ§Π˜ Π’ΠΠœ.

27. Π‘Ρ‹Π½Ρƒ 16 Π»Π΅Ρ‚. ΠŸΡ€ΠΈΠ³Π»Π°ΡˆΠ°ΡŽΡ‚ Π² Π²ΠΎΠ΅Π½ΠΊΠΎΠΌΠ°Ρ‚ Π½Π° постановку Π½Π° ΡƒΡ‡Π΅Ρ‚ ΠΈ ΠΏΡ€ΠΎΠ²Π΅Π΄Π΅Π½ΠΈΠ΅ мСдкомиссии. Π’ ΠΏΠΎΠ»ΠΈΠΊΠ»ΠΈΠ½ΠΈΠΊΠ΅ Ρ€Π°Π½Π΅Π΅ поставили Π΄ΠΈΠ°Π³Π½ΠΎΠ· ΠΏΡ€ΠΎΠ΄ΠΎΠ»ΡŒΠ½ΠΎΠ΅ плоскостопиС 2-ΠΉ стСпСни. НСдавно сдСлали ΠΏΠΎΠ²Ρ‚ΠΎΡ€Π½Ρ‹ΠΉ Ρ€Π΅Π½Ρ‚Π³Π΅Π½ — Π·Π°ΠΊΠ»ΡŽΡ‡Π΅Π½ΠΈΠ΅ Ρ€Π΅Π½Ρ‚Π³Π΅Π½ΠΎΠ»ΠΎΠ³Π° — ΠΎΠ΄Π½Π° Π½ΠΎΠ³Π° ΠΏΡ€ΠΎΠ΄ΠΎΠ»ΡŒΠ½ΠΎΠ΅ плоскостопиС 2-ΠΉ стСпСни, вторая — 3-ΠΉ стСпСни. Π”ΠΎΠ»ΠΆΠ΅Π½ Π»ΠΈ Π²ΠΎΠ΅Π½ΠΊΠΎΠΌΠ°Ρ‚ ΠΏΠΎΡΡ‚Π°Π²ΠΈΡ‚ΡŒ Π³Ρ€ΡƒΠΏΠΏΡƒ Π’? ΠΈ ΠΊΠ°ΠΊΠΈΠ΅ Π΄ΠΎΠΊΡƒΠΌΠ΅Π½Ρ‚Ρ‹ Π½Π°Π΄ΠΎ принСсти Π² Π²ΠΎΠ΅Π½ΠΊΠΎΠΌΠ°Ρ‚?
Π—Π°Ρ€Π°Π½Π΅Π΅ спасибо.
Π”ΠΌΠΈΡ‚Ρ€ΠΈΠΉ.

27.1. ЗдравствуйтС! ΠœΠΎΠ³ΡƒΡ‚ Π½Π° основании Π’Πš.Π‘ΠΎ всСх мСдицинских Π΄ΠΎΠΊΡƒΠΌΠ΅Π½Ρ‚ΠΎΠ² снимитС ксСрокопии, Π² письмСнном Π²ΠΈΠ΄Π΅ ΠΏΠΈΡˆΠΈΡ‚Π΅ заявлСниС ΠΈ ΠΏΡ€ΠΈΠ»Π°Π³Π°Π΅Ρ‚Π΅ ΠΊΠΎΠΏΠΈΠΈ Π΄ΠΎΠΊΡƒΠΌΠ΅Π½Ρ‚ΠΎΠ², сдаСтС ΡΠ΅ΠΊΡ€Π΅Ρ‚Π°Ρ€ΡŽ ΠΈΠ»ΠΈ Π΄Ρ€ΡƒΠ³ΠΎΠΌΡƒ долТностному Π»ΠΈΡ†Ρƒ Π²ΠΎΠ΅Π½ΠΊΠΎΠΌΠ°Ρ‚Π° ΠΏΠΎΠ΄ подпись Π½Π° вашСм (Π²Ρ‚ΠΎΡ€ΠΎΠΌ) экзСмплярС заявлСния с ΡƒΠΊΠ°Π·Π°Π½ΠΈΠ΅ΠΌ Π΄Π°Ρ‚Ρ‹ ΠΏΡ€ΠΈΠ΅ΠΌΠ°. ΠŸΡ€ΠΎΡΠ»Π΅Π΄ΠΈΡ‚Π΅, Ρ‡Ρ‚ΠΎΠ±Ρ‹ мСдицинская ΠΊΠ°Ρ€Ρ‚Π° ΠΏΡ€ΠΈΠ·Ρ‹Π²Π½ΠΈΠΊΠ° Π±Ρ‹Π»Π° Π² Π²ΠΎΠ΅Π½ΠΊΠΎΠΌΠ°Ρ‚Π΅ ΠΏΡ€ΠΈ ΠΎΡΠ²ΠΈΠ΄Π΅Ρ‚Π΅Π»ΡŒΡΡ‚Π²ΠΎΠ²Π°Π½ΠΈΠΈ.

28. Π’ 2009 поступал Π² Π’Π£Π— с государствСнной Π°ΠΊΠΊΡ€Π΅Π΄ΠΈΡ‚Π°Ρ†ΠΈΠ΅ΠΉ, Π½ΠΎ Π±Ρ‹Π» отчислСн Π·Π° Π°ΠΊΠ°Π΄Π΅ΠΌΠΈΡ‡Π΅ΡΠΊΡƒΡŽ Π½Π΅ ΡƒΡΠΏΠ΅Π²Π°Π΅ΠΌΠΎΡΡ‚ΡŒ. Π’ 2010 поступал Π² Ρ‚ΠΎΡ‚ ΠΆΠ΅ Π’Π£Π—, Π½ΠΎ Π½Π° Π΄Ρ€ΡƒΠ³ΠΎΠΉ Ρ„Π°ΠΊΡƒΠ»ΡŒΡ‚Π΅Ρ‚, ΠΊΠΎΡ‚ΠΎΡ€Ρ‹ΠΉ ΡƒΡΠΏΠ΅ΡˆΠ½ΠΎ ΠΎΠΊΠΎΠ½Ρ‡ΠΈΠ», ΠΈ ΠΏΠΎΠ»ΡƒΡ‡ΠΈΠ» Π΄ΠΈΠΏΠ»ΠΎΠΌ. БСйчас ΠΏΡ€ΠΎΡ…ΠΎΠΆΡƒ мСдкомиссию ΠΎΡ‚ Π²ΠΎΠ΅Π½ΠΊΠΎΠΌΠ°Ρ‚Π°, Π² процСссС ΠΊΠΎΡ‚ΠΎΡ€ΠΎΠΉ выявлСн Π½ΠΈΠ·ΠΊΠΈΠΉ индСкс массы Ρ‚Π΅Π»Π° (15,5).ΠœΠΎΠ³ΡƒΡ‚ Π»ΠΈ ΠΌΠ½Π΅ ΠΏΠΎΠ²Ρ‚ΠΎΡ€Π½ΠΎ Π²Ρ‹Π΄Π°Ρ‚ΡŒ отсрочку ΠΏΠΎ ΡΠΎΡΡ‚ΠΎΡΠ½ΠΈΡŽ Π·Π΄ΠΎΡ€ΠΎΠ²ΡŒΡ (ст.13 ΠΏ.Π³ расписания Π±ΠΎΠ»Π΅Π·Π½Π΅ΠΉ)?

28.1. ΠœΠΎΠ³ΡƒΡ‚, Π½ΠΎ Ссли Π½Π΅ прСдоставят Π²Ρ‹ Π²ΠΏΡ€Π°Π²Π΅ Ρ€Π΅ΡˆΠ΅Π½ΠΈΠ΅ комиссии ΠΎΠ±ΠΆΠ°Π»ΠΎΠ²Π°Ρ‚ΡŒ.


29. МСдкомиссия ΠΎΡ‚ Π²ΠΎΠ΅Π½ΠΊΠΎΠΌΠ°Ρ‚Π° Ρƒ ΠΌΠΎΠ΅Π³ΠΎ сына. НазначСно Π₯ΠΎΠ»Ρ‚Π΅Ρ€-обслСдованиС ΠΏΠ»Π°Ρ‚Π½ΠΎ. Π—Π°ΠΊΠΎΠ½Π½ΠΎ это ΠΈΠ»ΠΈ Π½Π΅Ρ‚. Π’Ρ…ΠΎΠ΄ΠΈΡ‚ Π»ΠΈ это обслСдованиС Π² ΠΏΠ΅Ρ€Π΅Ρ‡Π΅Π½ΡŒ бСсплатных услуг ΠΏΠΎ ΠΌΠ΅Π΄. полису. Π’ июнС этого Π³ΠΎΠ΄Π° Π΄Π°Π½Π½ΠΎΠ΅ обслСдованиС Π½Π°Π·Π½Π°Ρ‡Π°Π»ΠΈ Π΅ΠΌΡƒ бСсплатно, Π° ΠΏΠΎΠ²Ρ‚ΠΎΡ€Π½ΠΎΠ΅ ΠΏΠ»Π°Ρ‚Π½ΠΎ.

29.1. Π­Ρ‚ΠΎ Π½Π΅Π·Π°ΠΊΠΎΠ½Π½ΠΎ-Π½Π°ΠΏΠΈΡˆΠΈΡ‚Π΅ ΠΆΠ°Π»ΠΎΠ±Ρƒ Π² ΠΏΡ€ΠΎΠΊΡƒΡ€Π°Ρ‚ΡƒΡ€Ρƒ Π½Π° основании ст 10 Π€Π— О ΠΏΡ€ΠΎΠΊΡƒΡ€Π°Ρ‚ΡƒΡ€Π΅ Π Π€.

30. Π― ΡƒΡ‡ΡƒΡΡŒ Π² магистратурС (дСйствуСт отсрочка), сразу послС Π±Π°ΠΊΠ°Π»Π°Π²Ρ€ΠΈΠ°Ρ‚Π° поступил. ΠŸΠ°Ρ€Ρƒ Π΄Π½Π΅ΠΉ Π½Π°Π·Π°Π΄ ΠΏΡ€ΠΈΡˆΠ»Π° повСстка (мСня Π½Π΅ Π±Ρ‹Π»ΠΎ Π΄ΠΎΠΌΠ°)… По нСпонятным ΠΏΡ€ΠΈΡ‡ΠΈΠ½Π°ΠΌ эту повСстку оставили Π² Π–Π­Π£ ΠΈ Ρ‚Π΅ΠΏΠ΅Ρ€ΡŒ Π–Π­Π£ Π·Π²ΠΎΠ½ΠΈΡ‚ ΠΏΠΎ адрСсу прописки ΠΈ Ρ‚Ρ€Π΅Π±ΡƒΠ΅Ρ‚ Π·Π°Π±Ρ€Π°Ρ‚ΡŒ повСстку. Π˜ΡΡ…ΠΎΠ΄Ρ ΠΈΠ· Π΄Π°Π½Π½ΠΎΠΉ ситуации ΡΠ»Π΅Π΄ΡƒΡŽΡ‰ΠΈΠ΅ вопросы: 1) УмСстно Π»ΠΈ Π–Π­Π£ Π² Π΄Π°Π½Π½ΠΎΠΉ ситуации 2) НуТно Π»ΠΈ ΠΌΠ½Π΅ ΠΏΡ€ΠΈΡ…ΠΎΠ΄ΠΈΡ‚ΡŒ Π² Π²ΠΎΠ΅Π½ΠΊΠΎΠΌΠ°Ρ‚ (Π΄Π°ΠΆΠ΅ Ссли Π½ΡƒΠΆΠ½ΠΎ Π±ΡƒΠ΄Π΅Ρ‚ ΠΏΠΎΠ²Ρ‚ΠΎΡ€Π½ΠΎ ΠΏΡ€ΠΎΠΉΡ‚ΠΈ мСдкомиссию, ΠΊΠ°ΠΊ ΠΎΠ½ΠΈ ΠΌΠΎΠ³ΡƒΡ‚ ΡΠΊΠ°Π·Π°Ρ‚ΡŒ)

30.1. ЗдравствуйтС! Π”Π°, всС ΠΏΡ€Π°Π²ΠΎΠΌΠ΅Ρ€Π½ΠΎ.


Π“ΠΎΠ΄Π΅Π½? Или ΠΊΠ°ΠΊ я сходил Π² Π²ΠΎΠ΅Π½ΠΊΠΎΠΌΠ°Ρ‚ Π½Π° мСдкомиссию

БСгодня сходил Π² Π²ΠΎΠ΅Π½ΠΊΠΎΠΌΠ°Ρ‚ Π½Π° ΠΌΠ΅Π΄ΠΈΡ†ΠΈΠ½ΡΠΊΡƒΡŽ ΠΊΠΎΠΌΠΈΡΡΠΈΡŽβ€¦

Всё Π½Π°Ρ‡Π°Π»ΠΎΡΡŒ с ΠΎΠ±Ρ‰Π΅Π³ΠΎ собрания (Π½Π΅ знаю ΠΊΠ°ΠΊ Ρ‚ΠΎΡ‡Π½ΠΎ Π½Π°Π·Π²Π°Ρ‚ΡŒ). ΠšΡ‚ΠΎ ΠΈΠ΄Π΅Ρ‚ Π² Π°Ρ€ΠΌΠΈΡŽ Π² блиТайшСС врСмя ΠΈ Ρ‚Π΅ Ρƒ ΠΊΠΎΠ³ΠΎ заканчиваСтся отсрочка (Π² Ρ‚ΠΎΠΌ числС я) писали тСст Π½Π° ΠΏΡ€ΠΎΡ„ΠΏΡ€ΠΈΠ³ΠΎΠ΄Π½ΠΎΡΡ‚ΡŒ. Π₯отя я помню Ρ€Π°Π½Π΅Π΅, Π½Π° ΠΏΠ΅Ρ€Π²ΠΎΠΉ мСдкомиссии, писал ΠΏΠΎΠ΄ΠΎΠ±Π½Ρ‹Π΅ тСсты. Ну Π΄Π° Π»Π°Π΄Π½ΠΎ. ΠŸΠΎΡ‚ΠΎΠΌ ΠΌΡ‹ Β«ΠΏΠΎΠΎΠ±Ρ‰Π°Π»ΠΈΡΡŒΒ» с Ρ€Π°Π±ΠΎΡ‚Π½ΠΈΠΊΠ°ΠΌΠΈ Π²ΠΎΠ΅Π½ΠΊΠΎΠΌΠ°Ρ‚Π°, ΡƒΡ‚ΠΎΡ‡Π½ΠΈΠ»ΠΈ Π΄Π°Π½Π½Ρ‹Π΅ Π² Π»ΠΈΡ‡Π½ΠΎΠΌ Π΄Π΅Π»Π΅ ΠΈ пошли Π½Π° мСдосмотр. ΠšΡΡ‚Π°Ρ‚ΠΈ, ΠΈΠ΄ΠΈΠΎΡ‚ΠΎΠ² Π½Π΅ ΠΏΠΎΠ½ΠΈΠΌΠ°ΡŽΡ‰ΠΈΡ… вопросы тСста ΠΈ Π²Π΅Π΄ΡƒΡ‰ΠΈΡ… сСбя Π½Π΅ ΠΏΠΎΠ΄ΠΎΠ±Π°ΡŽΡ‰ΠΈΠΌ ΠΎΠ±Ρ€Π°Π·ΠΎΠΌ ΠΊΠ°ΠΊ всСгда Ρ…Π²Π°Ρ‚Π°Π»ΠΎ.

ΠœΠ΅Π΄ΠΈΡ†ΠΈΠ½ΡΠΊΠΈΠΉ осмотр начался с осмотра стоматолога. ΠŸΡ€ΠΈΡˆΠ»ΠΎΡΡŒ ΠΏΠΎΠ΄ΠΎΠΆΠ΄Π°Ρ‚ΡŒ Π½Π°Π²Π΅Ρ€Π½ΠΎΠ΅ ΠΌΠΈΠ½ΡƒΡ‚ 30-60 ΠΏΡ€Π΅ΠΆΠ΄Π΅ Ρ‡Π΅ΠΌ Π²Ρ‹Π·Π²Π°Π»ΠΈ. Π₯отя Π½Π°Ρ€ΠΎΠ΄Π° Π½Π΅ Ρ‚Π°ΠΊ ΡƒΠΆ ΠΈ ΠΌΠ½ΠΎΠ³ΠΎ Π±Ρ‹Π»ΠΎ. Π‘Ρ‚ΠΎΠΌΠ°Ρ‚ΠΎΠ»ΠΎΠ³ посмотрСла Π±Ρ‹ΡΡ‚Ρ€Π΅Π½ΡŒΠΊΠΎ Π·ΡƒΠ±Ρ‹ ΠΈ вынСсла Π²Π΅Ρ€Π΄ΠΈΠΊΡ‚: «ВсС Π·ΡƒΠ±Ρ‹ Π·Π΄ΠΎΡ€ΠΎΠ²Ρ‹Β». Ну Π΄Π°, ΠΊΠ°ΠΊ ΠΆΠ΅. Π—Π½Π°ΡŽ, Ρ‡Ρ‚ΠΎ Π½Π°Π΄ΠΎ ΠΎΠ΄ΠΈΠ½ Π·ΡƒΠ± ΠΏΠΎΠ΄Π»Π΅Ρ‡ΠΈΡ‚ΡŒ. Всё Π½ΠΈΠΊΠ°ΠΊ Π½Π΅ ΠΌΠΎΠ³Ρƒ ΡΡ…ΠΎΠ΄ΠΈΡ‚ΡŒ Π΄ΠΎ Π·ΡƒΠ±Π½ΠΎΠΉ. Но ΠΈ Ρ…ΠΎΡ€ΠΎΡˆΠΎ, Ρ‡Ρ‚ΠΎ Π² Π²ΠΎΠ΅Π½ΠΊΠΎΠΌΠ°Ρ‚Π΅ Π½Π΅ стали Π»Π΅Ρ‡ΠΈΡ‚ΡŒ. НС Π΄ΠΎΠ²Π΅Ρ€ΡΡŽ я ΠΈΠΌ, Ρ‚ΠΎΡ‡Π½Π΅Π΅ ΡΠΊΠ°Π·Π°Ρ‚ΡŒ спСциалисты Π½Π°Π²Π΅Ρ€Π½ΠΎΠ΅ Ρ…ΠΎΡ€ΠΎΡˆΠΈΠ΅, Π½ΠΎ Π²ΠΎΡ‚ ΠΊΠ°ΠΊΠΈΠ΅ ΠΌΠ°Ρ‚Π΅Ρ€ΠΈΠ°Π»Ρ‹ ΠΎΠ½ΠΈ ΠΈΡΠΏΠΎΠ»ΡŒΠ·ΡƒΡŽΡ‚β€¦ ΠŸΡ€ΠΎΡΡ‚ΠΎ Π² ΠΏΡ€ΠΎΡˆΠ»Ρ‹ΠΉ Ρ€Π°Π·, Π½Π° ΠΏΠ΅Ρ€Π²ΠΎΠΌ мСдосмотрС, рСбятам Π»Π΅Ρ‡ΠΈΠ»ΠΈ Π·ΡƒΠ±Ρ‹ Π² Π²ΠΎΠ΅Π½ΠΊΠΎΠΌΠ°Ρ‚Π΅ ΠΈ Ρƒ Π½ΠΈΡ… ΠΏΠ»ΠΎΠΌΠ±Ρ‹ Π²Ρ‹ΠΏΠ°Π΄Π°Π»ΠΈ Π½Π° ΡΠ»Π΅Π΄ΡƒΡŽΡ‰ΠΈΠ΅ Π΄Π½ΠΈ. ))

Π”Π°Π»Π΅Π΅ я отправился Π½Π° осмотр ΠΊ психиатру ΠΈ Π½Π°Ρ€ΠΊΠΎΠ»ΠΎΠ³Ρƒ. ΠŸΡΠΈΡ…ΠΈΠ°Ρ‚Ρ€ Π·Π°Π΄Π°Π» нСсколько вопрос. НапримСр, Π΅ΡΡ‚ΡŒ Π»ΠΈ ΠΆΠ°Π»ΠΎΠ±Ρ‹ ΠΈ Ρ‚.ΠΏ. ΠŸΠΎΠΈΠ½Ρ‚Π΅Ρ€Π΅ΡΠΎΠ²Π°Π»ΡΡ Π³Π΄Π΅ я ΡƒΡ‡ΡƒΡΡŒ ΠΈ ΠΊΡƒΠ΄Π° Ρ…ΠΎΡ‡Ρƒ ΠΈΠ΄Ρ‚ΠΈ Ρ€Π°Π±ΠΎΡ‚Π°Ρ‚ΡŒ. Π‘ΠΊΠ°Π·Π°Π», Ρ‡Ρ‚ΠΎ я ΡƒΡ‡ΠΈΡ‚Π΅Π»ΡŒ Ρ„ΠΈΠ·ΠΈΠΊΠΈ-ΠΈΠ½Ρ„ΠΎΡ€ΠΌΠ°Ρ‚ΠΈΠΊΠΈ. ПосмСялись с Π½ΠΈΠΌ Π½Π° счСт Π·Π°Ρ€ΠΏΠ»Π°Ρ‚Ρ‹. Π― сказал, Ρ‡Ρ‚ΠΎ Π½Π΅ ΠΏΠΎΠΉΠ΄Ρƒ Ρ€Π°Π±ΠΎΡ‚Π°Ρ‚ΡŒ Π² ΡˆΠΊΠΎΠ»Ρƒ, ΠΏΠΎΠΊΠ° Π½Π΅ ΠΏΠΎΠ΄Π½ΠΈΠΌΡƒΡ‚ Π·Π°Ρ€ΠΏΠ»Π°Ρ‚Ρ‹. Π•ΠΌΡƒ стало смСшно. Π”Π΅ΠΉΡΡ‚Π²ΠΈΡ‚Π΅Π»ΡŒΠ½ΠΎ слова «учитСлям ΠΏΠΎΠ΄Π½ΠΈΠΌΡƒΡ‚ Π·Π°Ρ€ΠΏΠ»Π°Ρ‚ΡƒΒ» Π²Ρ‹Π·Ρ‹Π²Π°Π΅Ρ‚ смСх. ) А Π½Π°Ρ€ΠΊΠΎΠ»ΠΎΠ³ Π·Π°Π΄Π°Π»Π° нСсколько вопросов, Π° ΠΈΠΌΠ΅Π½Π½ΠΎ: употрСблял Π»ΠΈ я наркотичСскиС срСдства, ΠΊΡƒΡ€ΡŽ Π»ΠΈ я, ΠΏΡ€ΠΎΠ±ΠΎΠ²Π°Π» Π»ΠΈ ΠΊΡƒΡ€ΠΈΡ‚Π΅Π»ΡŒΠ½Ρ‹Π΅ смСси. ΠšΡΡ‚Π°Ρ‚ΠΈ, Π·Π°Ρ‡Π΅ΠΌ ΡΠΏΡ€Π°ΡˆΠΈΠ²Π°Ρ‚ΡŒ ΠΎΡ‚Π΄Π΅Π»ΡŒΠ½ΠΎ ΠΏΡ€ΠΎ ΠΊΡƒΡ€ΠΈΡ‚Π΅Π»ΡŒΠ½Ρ‹Π΅ смСси, Ссли ΠΎΠ½ΠΈ ΠΏΡ€ΠΈΡ€Π°Π²Π½ΠΈΠ²Π°ΡŽΡ‚ΡΡ ΠΊ наркотичСским срСдствам. Π’ΠΈΠ΄ΠΈΠΌΠΎ пояснСниС для ΠΈΠ΄ΠΈΠΎΡ‚ΠΎΠ². )) А ΠΏΡ€ΠΎ ΡƒΠΏΠΎΡ‚Ρ€Π΅Π±Π»Π΅Π½ΠΈΠ΅ алкоголя Π²ΠΎΠΎΠ±Ρ‰Π΅ Π½ΠΈΡ‡Π΅Π³ΠΎ Π½Π΅ спросила. Π§Ρ‚ΠΎ Π·Π½Π°Ρ‡ΠΈΡ‚ ΡƒΠΏΠΎΡ‚Ρ€Π΅Π±Π»Π΅Π½ΠΈΠ΅ алкоголя являСтся Π½ΠΎΡ€ΠΌΠΎΠΉ? И всС ΠΌΠΎΠΈ ΠΎΡ‚Π²Π΅Ρ‚Ρ‹ «НСт» ΠΎΠ½Π° (Π²Ρ€Π°Ρ‡-Π½Π°Ρ€ΠΊΠΎΠ»ΠΎΠ³)Β  ΠΊΠΎΠΌΠΌΠ΅Π½Ρ‚ΠΈΡ€ΠΎΠ²Π°Π»Π°: Β«Π£Π³Ρƒ..Β», «Ну ладно…». Π’ΠΈΠΏΠ° Π½Ρƒ Π»Π°Π΄Π½ΠΎ я Ρ‚Π΅Π±Π΅ Π²Π΅Ρ€ΡŽ. ΠœΠΎΠΆΠ΅Ρ‚ я Π·Π°ΠΊΠΈΠ΄Ρ‹Π²Π°ΡŽΡΡŒ Π²Ρ€Π΅ΠΌΠ΅Π½Π°ΠΌΠΈ, Ρ‚Ρ€Π°Π²Ρƒ ΠΊΡƒΡ€ΡŽ ΠΈΠ»ΠΈ Π΅Ρ‰Π΅ Ρ‡Ρ‚ΠΎ. Никак Π½Π΅ ΠΏΡ€ΠΎΠ²Π΅Ρ€ΡΡŽΡ‚. Волько Π½Π° слово вСрят.

ΠŸΡ€ΠΈΡˆΠ»ΠΎΡΡŒ Π΄ΠΎΠ»Π³ΠΎ ΡΠΈΠ΄Π΅Ρ‚ΡŒ ΠΈ ΠΆΠ΄Π°Ρ‚ΡŒ ΠΏΡ€ΠΈΠ΅ΠΌΠ° Ρƒ психотСрапСвта. Π’Π°ΠΌ ΠΎΠ΄ΠΈΠ½ Β«ΠΎΠ΄Π°Ρ€Π΅Π½Π½Ρ‹ΠΉΒ» Ρ‚ΠΎΡ€Ρ‡Π°Π» Π½Π°Π²Π΅Ρ€Π½ΠΎΠ΅ час ΠΈ ΠΌΡ‹ Π΅Π³ΠΎ ΠΆΠ΄Π°Π»ΠΈ. ΠŸΠΎΡ‚ΠΎΠΌ ΠΎΠ½ Π²Ρ‹ΡˆΠ΅Π» ΠΈ Ρ…ΠΎΠ΄ΠΈΠ» матСрился. ругался. Π½Π΅ зря Π΅Π³ΠΎ Ρ‚Π°ΠΌ Π΄Π΅Ρ€ΠΆΠ°Π»ΠΈ. ))) Он Ρ‚Π°ΠΊΠΎΠΉ Π½Π΅ сдСрТанный Π½Π΅ ΠΎΠ΄ΠΈΠ½ Π±Ρ‹Π».

Π”Π°Π»Π΅Π΅ отправился ΠΊ ΡΠ»Π΅Π΄ΡƒΡŽΡ‰Π΅ΠΌΡƒ спСциалисту β€” ΠΎΡ‚ΠΎΠ»Π°Ρ€ΠΈΠ½Π³ΠΎΠ»ΠΎΠ³Ρƒ (Π»ΠΎΡ€, ухогорлонос)). ΠžΠΏΡΡ‚ΡŒ стоял ΠΈ ΡΠ»ΡƒΡˆΠ°Π», Ρ‡Ρ‚ΠΎ ΠΌΠ½Π΅ Ρ‚Π°ΠΌ ΡˆΠ΅ΠΏΡ‡ΡƒΡ‚. Π‘ΠΎ слухом всё Ρ…ΠΎΡ€ΠΎΡˆΠΎ. Π—Π°Π΄Π°Π²Π°Π»ΠΈ вопросы: Π±ΠΎΠ»ΠΈΡ‚ Π»ΠΈ Π³ΠΎΡ€Π»ΠΎ, Π±Ρ‹Π»Π° Π»ΠΈ гнойная Π°Π½Π³ΠΈΠ½Π°. ΠŸΡ€ΠΎ нос Π½ΠΈΡ‡Π΅Π³ΠΎ Π½Π΅ спросила ΠΈ я Π½Π΅ сказал, Π±Π»ΠΈΠ½. Π£ мСня ΠΆΠ΅ ΠΏΠ΅Ρ€Π΅Π³ΠΎΡ€ΠΎΠ΄ΠΊΠ° искривлСна, ΠΎΠ΄Π½Π° ноздря Π΄Ρ‹ΡˆΠΈΡ‚ Ρ…ΡƒΠΆΠ΅. Π§Ρ‚ΠΎ ΠΆΠ΅ Π΄Π΅Π»Π°Ρ‚ΡŒ?

Π‘Π»Π΅Π΄ΡƒΡŽΡ‰ΠΈΠΉ спСциалист β€” окулист. ) Π£ мСня Π² ΠΊΠ°Ρ€Ρ‚Π΅ Π±Ρ‹Π»ΠΎ ΡƒΠΊΠ°Π·Π°Π½ΠΎ, Ρ‡Ρ‚ΠΎ Π·Ρ€Π΅Π½ΠΈΠ΅ ΠΏΠ»ΠΎΡ…ΠΎΠ΅. ΠŸΠΎΡΡ‚ΠΎΠΌΡƒ Π½Π° мСня сразу Π½Π°Ρ†Π΅ΠΏΠΈΠ»ΠΈ Β«ΠΎΡ‡ΠΊΠΈΒ» ΠΊΡƒΠ΄Π° вставляли Ρ€Π°Π·Π½Ρ‹Π΅ Π»ΠΈΠ½Π·Ρ‹ ΠΈ смотрСли с ΠΊΠ°ΠΊΠΈΠΌΠΈ я Π²ΠΈΠΆΡƒ Π»ΡƒΡ‡ΡˆΠ΅ ΠΈ ΠΊΠ°ΠΊΠΎΠΉ ряд Π±ΡƒΠΊΠ² ΠΌΠΎΠ³Ρƒ Ρ€Π°Π·ΠΎΠ±Ρ€Π°Ρ‚ΡŒ. Π’ ΠΎΠ±Ρ‰Π΅ΠΌ такая Π½Π΅ точная опСрация ΠΏΠΎ ΠΏΠΎΠ΄Π±ΠΎΡ€Ρƒ ΠΎΡ‡ΠΊΠΎΠ². Π― ΠΈ с Π»ΠΈΠ½Π·Π°ΠΌΠΈ Ρ‚Π°ΠΊ ΠΈ Π½Π΅ смог Ρ€Π°Π·ΠΎΠ±Ρ€Π°Ρ‚ΡŒ Π½ΠΈΠΆΠ½ΠΈΠ΅ строчки. По ΠΌΠΎΠ΅ΠΌΡƒ Π·Ρ€Π΅Π½ΠΈΠ΅ стало Π΅Ρ‰Π΅ Ρ…ΡƒΠΆΠ΅. Π₯отя особых нСудобств я Π½Π΅ ΠΈΡΠΏΡ‹Ρ‚Ρ‹Π²Π°ΡŽ. Π‘Ρ‹Π»ΠΈ ΠΏΡ€ΠΎΠ±Π»Π΅ΠΌΡ‹ со Π·Ρ€Π΅Π½ΠΈΠ΅ΠΌ, Π΄ΡƒΠΌΠ°Π» ΠΎΡ‡ΠΊΠΈ ΠΊΡƒΠΏΠΈΡ‚ΡŒ. Π‘Ρ…ΠΎΠ΄ΠΈΠ» Π² ΠΌΠ°Π³Π°Π·ΠΈΠ½ ΠΎΠΏΡ‚ΠΈΠΊΠΈ, Ρ‚Π°ΠΌ Π²Ρ€Π°Ρ‡ окулист ΠΏΡ€ΠΎΠ²Π΅Π»Π° диагностику ΠΈ сказала, Ρ‡Ρ‚ΠΎ ΠΏΠΎΠΊΠ° ΠΎΡ‡ΠΊΠΈ Π½Π΅ Π½Π°Π΄ΠΎ. Π‘Ρ‚Ρ€Π°Π½Π½ΠΎ, Π΄Π°? По ΠΈΠ΄Π΅Π΅ Π΄ΠΎΠ»ΠΆΠ½Π° Π±Ρ‹Π»Π° Π²Ρ‹ΠΏΠΈΡΠ°Ρ‚ΡŒ ΠΎΡ‡ΠΊΠΈ, Ρ‡Ρ‚ΠΎΠ±Ρ‹ я ΠΊΡƒΠΏΠΈΠ» ΠΈΡ… Ρƒ Π½ΠΈΡ… Π² ΠΌΠ°Π³Π°Π·ΠΈΠ½Π΅. Ну Π»Π°Π΄Π½ΠΎ, это я Ρ‚Π°ΠΊ ΡˆΡƒΡ‡Ρƒ. )) Π’ΠΈΠ΄ΠΈΠΌΠΎ это Π·Π½Π°Ρ‡ΠΈΡ‚, Ρ‡Ρ‚ΠΎ Π²Ρ€Π°Ρ‡ Π±Ρ‹Π»Π° просто адСкватная.

Ну Π° дальшС Π½Π°Ρ‡Π°Π»ΠΎΡΡŒ Β«Ρ…ΠΎΠΆΠ΄Π΅Π½ΠΈΠ΅ ΠΏΠΎ Π»Π°Π±ΠΈΡ€ΠΈΠ½Ρ‚Ρƒ Π² трусах» ΠΎΡ‚ спСциалиста ΠΊ спСциалисту. Для Π½Π°Ρ‡Π°Π»Π° мСня взвСсили ΠΈ ΠΈΠ·ΠΌΠ΅Ρ€ΠΈΠ»ΠΈ рост. Они составили соотвСтствСнно 62.3 ΠΊΠ³ (странно, сколько Π½Π΅ взвСшивался Π½Π° Β Ρ€Π°Π·Π½Ρ‹Ρ… Π±Ρ‹Ρ‚ΠΎΠ²Ρ‹Ρ… вСсах, ΠΏΠΎΠΊΠ°Π·Ρ‹Π²Π°Π»ΠΎ 65 ΠΊΠ³) ΠΈ 173 см. ΠŸΠ΅Ρ€Π²Ρ‹ΠΌ Π½Π° ΠΎΡ‡Π΅Ρ€Π΅Π΄ΠΈ спСциалистом Β«Π»Π°Π±ΠΈΡ€ΠΈΠ½Ρ‚Π°Β» Π±Ρ‹Π» Π½Π΅Π²Ρ€ΠΎΠΏΠ°Ρ‚ΠΎΠ»ΠΎΠ³. Бпросил Π΅ΡΡ‚ΡŒ Π»ΠΈ ΠΆΠ°Π»ΠΎΠ±Ρ‹. Π― ΠΎΡ‚Π²Π΅Ρ‚ΠΈΠ» Ρ‡Ρ‚ΠΎ спина Π±ΠΎΠ»ΠΈΡ‚. Π”Π°Π»Π΅Π΅ ΠΎΠ½ постучал своим ΠΌΠΎΠ»ΠΎΡ‚ΠΎΡ‡ΠΊΠΎΠΌ, ΠΏΡ€ΠΎΠ²Π΅Π» Π΄Ρ€ΡƒΠ³ΠΈΠ΅ ΠΎΠ±Ρ‹Ρ‡Π½Ρ‹Π΅ дСйствия. А Π½Π° ΠΌΠΎΠΉ вопрос: «А Ρ‡Ρ‚ΠΎ со спиной? К Ρ…ΠΈΡ€ΡƒΡ€Π³Ρƒ ΠΎΠ±Ρ€Π°Ρ‰Π°Ρ‚ΡŒΡΡ?Β» ΠžΡ‚Π²Π΅Ρ‚ΠΈΠ»: Β«Π”Π°, ΠΊ Ρ…ΠΈΡ€ΡƒΡ€Π³ΡƒΒ». НСмного Π²ΠΏΠ΅Ρ€Π΅Π΄ Π·Π°Π±Π΅Π³Ρƒ. МнС Ρ…ΠΈΡ€ΡƒΡ€Π³, послС обращСния ΠΊ Π½Π΅ΠΌΡƒ с ΠΆΠ°Π»ΠΎΠ±ΠΎΠΉ Π½Π° спину, сказал: Β» А Ρ‡Π΅Π³ΠΎ Ρ‚Ρ‹ ΠΊΠΎ ΠΌΠ½Π΅ идСшь, это Π΄ΠΎΠ»ΠΆΠ΅Π½ ΡΠΌΠΎΡ‚Ρ€Π΅Ρ‚ΡŒ Π½Π΅Π²Ρ€ΠΎΠΏΠ°Ρ‚ΠΎΠ»ΠΎΠ³Β». Π’ΠΎΡ‚ Ρ‚Π°ΠΊΠΈΠ΅ Π΄Π΅Π»Π°.

Π‘Π»Π΅Π΄ΡƒΡŽΡ‰ΠΈΠΌ Π±Ρ‹Π» Ρ‚Π΅Ρ€Π°ΠΏΠ΅Π²Ρ‚. Π’Π°ΠΊ ΠΆΠ΅ спросила Π΅ΡΡ‚ΡŒ Π»ΠΈ ΠΆΠ°Π»ΠΎΠ±Ρ‹, Π±ΠΎΠ»ΠΈΡ‚ ΠΈ ΠΆΠΈΠ²ΠΎΡ‚ ΠΈ Ρ‚.ΠΏ. Π― ΠΎΡ‚Π²Π΅Ρ‚ΠΈΠ» Ρ‡Ρ‚ΠΎ Π΄Π°, Π±Ρ‹Π²Π°Π΅Ρ‚ Π±ΠΎΠ»ΠΈΡ‚ ΠΆΠ΅Π»ΡƒΠ΄ΠΎΠΊ ΠΈ ΠΈΠ·ΠΆΠΎΠ³Π°. На это ΠΌΠ½Π΅ ΠΎΡ‚Π²Π΅Ρ‚ΠΈΠ»Π° Ρ‚Π°ΠΊ: Β» ΠœΡ‹ тСбя провСряли Π² ΠΏΡ€ΠΎΡˆΠ»Ρ‹ΠΉ Ρ€Π°Π· (ΡƒΡ‚ΠΎΡ‡Π½Π΅Π½ΠΈΠ΅: я Ρ…ΠΎΠ΄ΠΈΠ» Β«Π³Π»ΠΎΡ‚Π°Π» Π»Π°ΠΌΠΏΠΎΡ‡ΠΊΡƒΒ»). Π’Ρ‹ Π·Π΄ΠΎΡ€ΠΎΠ²!Β» Ну Π½Π΅ ΠΊΠ°ΠΏΠ΅Ρ† Π»ΠΈ? ΠŸΡ€ΠΎΡˆΠ»Ρ‹ΠΉ Ρ€Π°Π· Π±Ρ‹Π» 6 Π»Π΅Ρ‚ Π½Π°Π·Π°Π΄ практичСски. ΠŸΡ€ΠΈΡˆΠ»ΠΎΡΡŒ Π΅Ρ‰Π΅ Π²Π·ΡΡ‚ΡŒ направлСния Π½Π° сдачу Π°Π½Π°Π»ΠΈΠ·Π° ΠΊΡ€ΠΎΠ²ΠΈ ΠΈ прохоТдСния Π­ΠšΠ“.

Π”Π°Π»Π΅Π΅ Π½Π° ΠΎΡ‡Π΅Ρ€Π΅Π΄ΠΈ Π±Ρ‹Π» Π²Ρ€Π°Ρ‡ Π΄Π΅Ρ€ΠΌΠ°Ρ‚ΠΎΠ»ΠΎΠ³ со своими ΠΏΡ€ΠΎΡΡŒΠ±Π°ΠΌΠΈ ΠΎΠ³ΠΎΠ»ΠΈΡ‚ΡŒ Π³ΠΎΠ»ΠΎΠ²ΠΊΡƒ ΠΈ Ρ‚.ΠΏ. ))) ΠŸΡ€ΠΎΠ±Π»Π΅ΠΌ, связанных с ΠΊΠΎΠΆΠ½ΠΎ-вСнСричСскими заболСваниями Ρƒ мСня Π½Π΅Ρ‚. Ну ΠΈ Ρ…ΠΎΡ€ΠΎΡˆΠΎ. ) Рядом сидСл Ρ…ΠΈΡ€ΡƒΡ€Π³, ΠΏΡ€ΠΎΠ΄ΠΎΠ»ΠΆΠΈΠ» осмотр. ПоТаловался Π΅ΠΌΡƒ Π½Π° Π±ΠΎΠ»ΠΈ Π² спинС ΠΈ ΠΊΠΎΠ»Π΅Π½Π΅. По Π΅Π³ΠΎ мнСнию я Π·Π΄ΠΎΡ€ΠΎΠ². Π‘ΠΏΡ€Π°ΡˆΠΈΠ²Π°Π» ΠΏΠΎΡ‡Π΅ΠΌΡƒ я Ρ€Π°Π½Π΅Π΅ Π½Π΅ обращался. А ΠΏΠΎ ΠΈΡ… мнСнию, Ссли Π½Π΅ обращался, Π·Π½Π°Ρ‡ΠΈΡ‚ Π·Π΄ΠΎΡ€ΠΎΠ². ΠžΡ‚Π»ΠΈΡ‡Π½Π°Ρ Π»ΠΎΠ³ΠΈΠΊΠ° Π½Π΅ Ρ‚Π°ΠΊ Π»ΠΈ? Взял Ρƒ Π½Π΅Π³ΠΎ направлСния Π½Π° снимки Π³Ρ€ΡƒΠ΄Π½ΠΎΠΉ ΠΊΠ»Π΅Ρ‚ΠΊΠΈ ΠΈ ΠΊΠΎΠ»Π΅Π½Π°. Π•Ρ‰Π΅ ΠΎΠ½ ΠΌΠ½Π΅ сказал, Ρ‡Ρ‚ΠΎ искривлСния ΠΏΠΎΠ·Π²ΠΎΠ½ΠΎΡ‡Π½ΠΈΠΊΠ° Π½Π΅Ρ‚. Π― Ρ‚ΠΎΠ³Π΄Π° Π½Π΅ понимаю ΠΏΠΎΡ‡Π΅ΠΌΡƒ Ρƒ мСня ΠΎΠ΄Π½ΠΎ ΠΏΠ»Π΅Ρ‡ΠΎ Π²Ρ‹ΡˆΠ΅ другого…

Из Π²ΠΎΠ΅Π½ΠΊΠΎΠΌΠ°Ρ‚Π° я ΡƒΡˆΠ΅Π» ΡƒΠΆΠ΅ послС Π΄Π²Π΅Π½Π°Π΄Ρ†Π°Ρ‚ΠΈ часов. ΠŸΡ€ΠΈΠΌΠ΅Ρ€Π½ΠΎ Π² 12:30. Π‘Ρ€Π°Π·Ρƒ сходил сдСлал снимки. ΠšΡΡ‚Π°Ρ‚ΠΈ, Π² понСдСльник (3 дня Π½Π°Π·Π°Π΄) Ρ…ΠΎΠ΄ΠΈΠ» Π΄Π΅Π»Π°Π» Ρ„Π»ΡŽΠΎΡ€ΠΎΠ³Ρ€Π°Ρ„ΠΈΡŽ. Β Π’Π°ΠΊ Ρ‡Ρ‚ΠΎ Π² суммС, Π½Π°Π²Π΅Ρ€Π½ΠΎΠ΅, Π½Π΅ ΠΌΠ°Π»Π΅Π½ΡŒΠΊΡƒΡŽ Π΄ΠΎΠ·Ρƒ облучСния ΠΏΠΎΠ»ΡƒΡ‡ΠΈΠ» Π·Π° эти Π΄Π½ΠΈ. )) Π’ Ρ‚Ρ€ΠΈ часа сходил сдал ΠΊΡ€ΠΎΠ²ΡŒ ΠΈ сдСлал Π­ΠšΠ“.

Π—Π°Π²Ρ‚Ρ€Π° снова Π² Π²ΠΎΠ΅Π½ΠΊΠΎΠΌΠ°Ρ‚ ΠΊ 9 часам. Π‘ΡƒΠ΄ΡƒΡ‚ Ρ€Π΅ΡˆΠ°Ρ‚ΡŒ со ΠΌΠ½ΠΎΠΉ вопрос ΠΎ Ρ‚ΠΎΠΌ, Π³ΠΎΠ΄Π΅Π½ Π»ΠΈ я. ΠŸΠΎΡ‡Π΅ΠΌΡƒ Ρ‚ΠΎ ΠΌΠ½Π΅ каТСтся, Π§Ρ‚ΠΎ ΠΎΠ½ΠΈ вынСсут Π²Π΅Ρ€Π΄ΠΈΠΊΡ‚ β€” Β«Π“ΠΎΠ΄Π΅Π½Β». Π½ΠΎ я Π½Π΅ согласСн с ΠΈΡ… Π»ΠΎΠ³ΠΈΠΊΠΎΠΉ, Ρ‡Ρ‚ΠΎ Ссли Π½Π΅ обращался, Π·Π½Π°Ρ‡ΠΈΡ‚ Π·Π΄ΠΎΡ€ΠΎΠ². Π‘Ρ…ΠΎΠΆΡƒ Π½Π° ΠΏΡ€ΠΈΠ·Ρ‹Π²Π½ΡƒΡŽ комиссию, Π±ΡƒΠ΄Ρƒ ΠΏΡ€ΠΎΡΠΈΡ‚ΡŒΡΡ Π½Π° обслСдованиС Ρƒ Π΄Ρ€ΡƒΠ³ΠΈΡ… спСциалистов. Π’Π΅ΠΌ Π±ΠΎΠ»Π΅Π΅ Ρ‚Π°ΠΊΠΎΠ΅ ΠΏΡ€Π°Π²ΠΎ Π΅ΡΡ‚ΡŒ. НСмного ΠΏΠΎΡ‡ΠΈΡ‚Π°Π» Π½Π° эту Ρ‚Π΅ΠΌΡƒ. ΠœΠΎΠ³Ρƒ привСсти нСсколько интСрСсных Π²Ρ‹Π΄Π΅Ρ€ΠΆΠ΅ΠΊ ΠΈΠ· Π·Π°ΠΊΠΎΠ½ΠΎΠ² ΠΈ толкования ΠΈΡ… Π½Π° Π±ΠΎΠ»Π΅Π΅ простом языкС Π² Π²ΠΈΠ΄Π΅ вопрос-ΠΎΡ‚Π²Π΅Ρ‚. Если Π’Π°ΠΌ это Π½Π΅ интСрСсно, Ρ‚ΠΎ ΠΌΠΎΠΆΠ΅Ρ‚Π΅ ΠΏΡ€ΠΎΠΏΡƒΡΡ‚ΠΈΡ‚ΡŒ, ΠΏΡ€ΠΎΠΊΡ€ΡƒΡ‚ΠΈΡ‚ΡŒ Π²Π½ΠΈΠ·. ))

Π’ΠΎΡ‚ Ρ‚Π°ΠΊΠΎΠ΅, Π½Π°ΠΏΡ€ΠΈΠΌΠ΅Ρ€.

Как принимаСтся Ρ€Π΅ΡˆΠ΅Π½ΠΈΠ΅ ΠΎ ΠΏΡ€ΠΈΠ·Ρ‹Π²Π΅?

РСшСниС ΠΎ ΠΏΡ€ΠΈΠ·Ρ‹Π²Π΅ Π½Π° Π²ΠΎΠ΅Π½Π½ΡƒΡŽ слуТбу, прСдоставлСнии отсрочки ΠΈΠ»ΠΈ освобоТдСнии ΠΎΡ‚ Π½Π΅Π΅, принимаСтся ΠΏΡ€ΠΈΠ·Ρ‹Π²Π½ΠΎΠΉ комиссиСй.
Π‘Π½Π°Ρ‡Π°Π»Π° Π²Π°ΠΌ ΠΏΡ€Π΅Π΄Π»ΠΎΠΆΠ°Ρ‚ ΠΏΡ€ΠΎΠΉΡ‚ΠΈ мСдицинскоС ΠΎΡΠ²ΠΈΠ΄Π΅Ρ‚Π΅Π»ΡŒΡΡ‚Π²ΠΎΠ²Π°Π½ΠΈΠ΅ Π² Π²ΠΎΠ΅Π½ΠΊΠΎΠΌΠ°Ρ‚Π΅ для Ρ‚ΠΎΠ³ΠΎ, Ρ‡Ρ‚ΠΎΠ±Ρ‹ ΠΎΠΏΡ€Π΅Π΄Π΅Π»ΠΈΡ‚ΡŒ ΡΡ‚Π΅ΠΏΠ΅Π½ΡŒ вашСй годности ΠΊ Π²ΠΎΠ΅Π½Π½ΠΎΠΉ слуТбС. Π’ соотвСтствии с ПолоТСниСм ΠΎ Π²ΠΎΠ΅Π½Π½ΠΎ-Π²Ρ€Π°Ρ‡Π΅Π±Π½ΠΎΠΉ экспСртизС Π΄ΠΎ ΠΎΡΠ²ΠΈΠ΄Π΅Ρ‚Π΅Π»ΡŒΡΡ‚Π²ΠΎΠ²Π°Π½ΠΈΡ Π²Ρ‹ Ρ‚Π°ΠΊΠΆΠ΅ Π΄ΠΎΠ»ΠΆΠ½Ρ‹ Π² ΠΎΠ±ΡΠ·Π°Ρ‚Π΅Π»ΡŒΠ½ΠΎΠΌ порядкС (ΠΏΠΎ Π½Π°ΠΏΡ€Π°Π²Π»Π΅Π½ΠΈΡŽ Π²ΠΎΠ΅Π½ΠΊΠΎΠΌΠ°Ρ‚Π°) ΠΏΡ€ΠΎΠΉΡ‚ΠΈ диагностичСскиС исслСдования: Ρ„Π»ΡŽΠΎΡ€ΠΎΠ³Ρ€Π°Ρ„ΠΈΡŽ (Ρ€Π΅Π½Ρ‚Π³Π΅Π½) Π³Ρ€ΡƒΠ΄Π½ΠΎΠΉ ΠΊΠ»Π΅Ρ‚ΠΊΠΈ Π² Π΄Π²ΡƒΡ… проСкциях, ΠΎΠ±Ρ‰ΠΈΠΉ Π°Π½Π°Π»ΠΈΠ· ΠΊΡ€ΠΎΠ²ΠΈ, ΠΎΠ±Ρ‰ΠΈΠΉ Π°Π½Π°Π»ΠΈΠ· ΠΌΠΎΡ‡ΠΈ ΠΈ элСктрокардиограмму. По мСдицинским показаниям ΠΌΠΎΠ³ΡƒΡ‚ ΠΏΡ€ΠΎΠ²ΠΎΠ΄ΠΈΡ‚ΡŒΡΡ Π΄ΠΎΠΏΠΎΠ»Π½ΠΈΡ‚Π΅Π»ΡŒΠ½Ρ‹Π΅ исслСдования. Π’Ρ‹ Ρ‚Π°ΠΊΠΆΠ΅ ΠΈΠΌΠ΅Π΅Ρ‚Π΅ ΠΏΡ€Π°Π²ΠΎ ΠΏΡ€ΠΎΠΉΡ‚ΠΈ Π΄ΠΎΠΏΠΎΠ»Π½ΠΈΡ‚Π΅Π»ΡŒΠ½ΠΎ нСзависимоС мСдицинскоС обслСдованиС Π² любом мСдицинском ΡƒΡ‡Ρ€Π΅ΠΆΠ΄Π΅Π½ΠΈΠΈ, ΠΈΠΌΠ΅ΡŽΡ‰Π΅ΠΌ Π»ΠΈΡ†Π΅Π½Π·ΠΈΡŽ.
Π—Π°Ρ‚Π΅ΠΌ Π²Ρ‹ Π΄ΠΎΠ»ΠΆΠ½Ρ‹ ΠΏΡ€ΠΎΠΉΡ‚ΠΈ ΠΏΡ€ΠΈΠ·Ρ‹Π²Π½ΡƒΡŽ комиссию, ΠΊΠΎΡ‚ΠΎΡ€ΡƒΡŽ возглавляСт ΠΏΡ€Π΅Π΄ΡΡ‚Π°Π²ΠΈΡ‚Π΅Π»ΡŒ мСстной администрации. Если Ρƒ вас Π΅ΡΡ‚ΡŒ основания для отсрочки ΠΈΠ»ΠΈ освобоТдСния, Π²Π°ΠΌ слСдуСт Π·Π°ΡΠ²ΠΈΡ‚ΡŒ ΠΎΠ± этом Π½Π° ΠΏΡ€ΠΈΠ·Ρ‹Π²Π½ΠΎΠΉ комиссии. Π›ΡƒΡ‡ΡˆΠ΅ всСго ΡΠ΄Π΅Π»Π°Ρ‚ΡŒ это Π·Π°Ρ€Π°Π½Π΅Π΅ ΠΈ Π² письмСнной Ρ„ΠΎΡ€ΠΌΠ΅. Π£Ρ‡Ρ‚ΠΈΡ‚Π΅, Ρ‡Ρ‚ΠΎ просто ΠΏΠΎΠΆΠ°Π»ΠΎΠ²Π°Ρ‚ΡŒΡΡ Π½Π° состояниС Π·Π΄ΠΎΡ€ΠΎΠ²ΡŒΡ ΠΈΠ»ΠΈ болСзнь β€” ΠΌΠ°Π»ΠΎ. НСобходимо ΠΈΠΌΠ΅Ρ‚ΡŒ мСдицинскиС Π΄ΠΎΠΊΡƒΠΌΠ΅Π½Ρ‚Ρ‹ ΠΏΠΎΠ΄Ρ‚Π²Π΅Ρ€ΠΆΠ΄Π°ΡŽΡ‰ΠΈΠ΅ вашС Π·Π°Π±ΠΎΠ»Π΅Π²Π°Π½ΠΈΠ΅, ΠΈ ΠΏΡ€Π΅Π΄ΡΡ‚Π°Π²ΠΈΡ‚ΡŒ ΠΈΡ… ΠΊΠΎΠΏΠΈΠΈ для приобщСния Π² вашС Π»ΠΈΡ‡Π½ΠΎΠ΅ Π΄Π΅Π»ΠΎ ΠΏΡ€ΠΈΠ·Ρ‹Π²Π½ΠΈΠΊΠ°.
Π—Π°Π΄Π°Ρ‡Π° ΠΏΡ€ΠΈΠ·Ρ‹Π²Π½ΠΎΠΉ комиссии β€” ΡƒΡ‡Π΅ΡΡ‚ΡŒ всС ΠΎΠ±ΡΡ‚ΠΎΡΡ‚Π΅Π»ΡŒΡΡ‚Π²Π° ΠΈ ΠΏΡ€ΠΈΠ½ΡΡ‚ΡŒ Π² ΠΎΡ‚Π½ΠΎΡˆΠ΅Π½ΠΈΠΈ вас ΠΎΠ΄Π½ΠΎ ΠΈΠ· ΡΠ»Π΅Π΄ΡƒΡŽΡ‰ΠΈΡ… Ρ€Π΅ΡˆΠ΅Π½ΠΈΠΉ:
β€’ ΠΎ ΠΏΡ€ΠΈΠ·Ρ‹Π²Π΅ вас Π½Π° Π²ΠΎΠ΅Π½Π½ΡƒΡŽ слуТбу;
β€’ ΠΎ Π½Π°ΠΏΡ€Π°Π²Π»Π΅Π½ΠΈΠΈ вас Π½Π° Π°Π»ΡŒΡ‚Π΅Ρ€Π½Π°Ρ‚ΠΈΠ²Π½ΡƒΡŽ Π³Ρ€Π°ΠΆΠ΄Π°Π½ΡΠΊΡƒΡŽ слуТбу;
β€’ ΠΎ прСдоставлСнии Π²Π°ΠΌ отсрочки ΠΎΡ‚ ΠΏΡ€ΠΈΠ·Ρ‹Π²Π° Π½Π° Π²ΠΎΠ΅Π½Π½ΡƒΡŽ слуТбу;
β€’ ΠΎΠ± освобоТдСнии вас ΠΎΡ‚ ΠΏΡ€ΠΈΠ·Ρ‹Π²Π° Π½Π° Π²ΠΎΠ΅Π½Π½ΡƒΡŽ слуТбу;
β€’ ΠΎ зачислСнии вас Π² запас;
β€’ ΠΎΠ± освобоТдСнии вас ΠΎΡ‚ исполнСния воинской обязанности.
РСшСниС ΠΏΡ€ΠΈΠ·Ρ‹Π²Π½ΠΎΠΉ комиссии Π² ΠΎΡ‚Π½ΠΎΡˆΠ΅Π½ΠΈΠΈ ΠΊΠ°ΠΆΠ΄ΠΎΠ³ΠΎ ΠΏΡ€ΠΈΠ·Ρ‹Π²Π½ΠΈΠΊΠ° Π² Ρ‚ΠΎΡ‚ ΠΆΠ΅ дСнь заносится Π² ΠΏΡ€ΠΎΡ‚ΠΎΠΊΠΎΠ» засСдания ΠΏΡ€ΠΈΠ·Ρ‹Π²Π½ΠΎΠΉ комиссии. ΠŸΡ€ΠΎΡ‚ΠΎΠΊΠΎΠ» ΠΏΠΎΠ΄ΠΏΠΈΡΡ‹Π²Π°ΡŽΡ‚ ΠΏΡ€Π΅Π΄ΡΠ΅Π΄Π°Ρ‚Π΅Π»ΡŒ ΠΏΡ€ΠΈΠ·Ρ‹Π²Π½ΠΎΠΉ комиссии ΠΈ Π΅Π΅ Ρ‡Π»Π΅Π½Ρ‹. Π­Ρ‚ΠΎ Ρ€Π΅ΡˆΠ΅Π½ΠΈΠ΅ заносится Π² вашС удостовСрСниС Π³Ρ€Π°ΠΆΠ΄Π°Π½ΠΈΠ½Π°, ΠΏΠΎΠ΄Π»Π΅ΠΆΠ°Ρ‰Π΅Π³ΠΎ ΠΏΡ€ΠΈΠ·Ρ‹Π²Ρƒ Π½Π° Π²ΠΎΠ΅Π½Π½ΡƒΡŽ слуТбу (приписноС ΡΠ²ΠΈΠ΄Π΅Ρ‚Π΅Π»ΡŒΡΡ‚Π²ΠΎ) ΠΈ Π² Π²Π°ΡˆΡƒ ΡƒΡ‡Π΅Ρ‚Π½ΡƒΡŽ ΠΊΠ°Ρ€Ρ‚Ρƒ ΠΏΡ€ΠΈΠ·Ρ‹Π²Π½ΠΈΠΊΠ°.

Или Ρ‚Π°ΠΊΠΎΠ΅.

ΠœΠΎΠ³Ρƒ Π»ΠΈ я Π½Π΅ ΡΠΎΠ³Π»Π°ΡΠΈΡ‚ΡŒΡΡ с Ρ€Π΅ΡˆΠ΅Π½ΠΈΠ΅ΠΌ Ρ€Π°ΠΉΠΎΠ½Π½ΠΎΠΉ ΠΏΡ€ΠΈΠ·Ρ‹Π²Π½ΠΎΠΉ комиссии ΠΎ ΠΌΠΎΠ΅ΠΌ ΠΏΡ€ΠΈΠ·Ρ‹Π²Π΅?

Π”Π°, Ρ‚Π°ΠΊΠΎΠ΅ ΠΏΡ€Π°Π²ΠΎ Ρƒ вас Π΅ΡΡ‚ΡŒ. Π”Π°ΠΆΠ΅ Ссли Π²Ρ‹ ΠΏΠΎΠ»ΡƒΡ‡ΠΈΠ»ΠΈ повСстку для ΠΎΡ‚ΠΏΡ€Π°Π²ΠΊΠΈ Π² войска, согласно ΠΏ.7 ст. 28 Π€Π΅Π΄Π΅Ρ€Π°Π»ΡŒΠ½ΠΎΠ³ΠΎ Π·Π°ΠΊΠΎΠ½Π° «О воинской обязанности ΠΈ Π²ΠΎΠ΅Π½Π½ΠΎΠΉ слуТбС», Π²Ρ‹ ΠΌΠΎΠΆΠ΅Ρ‚Π΅ ΠΎΠ±ΠΆΠ°Π»ΠΎΠ²Π°Ρ‚ΡŒ это Ρ€Π΅ΡˆΠ΅Π½ΠΈΠ΅ Π² судСбном порядкС. ΠŸΠΎΠ΄Π°Ρ‡Π° заявлСния Π² суд приостанавливаСт всС дСйствия ΠΏΠΎ ΠΏΡ€ΠΈΠ·Ρ‹Π²Ρƒ Π΄ΠΎ вступлСния Π² Π·Π°ΠΊΠΎΠ½Π½ΡƒΡŽ силу Ρ€Π΅ΡˆΠ΅Π½ΠΈΡ суда. НС пропуститС ΠΎΡ‚Π²Π΅Π΄Π΅Π½Π½Ρ‹ΠΉ для обТалования срок β€” 3 мСсяца.

И Π΅Ρ‰Π΅.

Как ΠΏΡ€ΠΎΡ…ΠΎΠ΄ΠΈΡ‚ мСдицинскоС ΠΎΡΠ²ΠΈΠ΄Π΅Ρ‚Π΅Π»ΡŒΡΡ‚Π²ΠΎΠ²Π°Π½ΠΈΠ΅?

Π­Ρ‚ΠΎ ΠΎΠ±ΡΠ·Π°Ρ‚Π΅Π»ΡŒΠ½ΠΎΠ΅ мСроприятиС ΠΏΡ€ΠΈ ΠΏΡ€ΠΈΠ·Ρ‹Π²Π΅ Π½Π° Π²ΠΎΠ΅Π½Π½ΡƒΡŽ слуТбу. Оно проводится Π²Ρ€Π°Ρ‡Π°ΠΌΠΈ-спСциалистами ΠΏΡ€ΠΈ ΠΏΡ€ΠΈΠ·Ρ‹Π²Π½ΠΎΠΉ комиссии. Π”ΠΎ Π½Π°Ρ‡Π°Π»Π° ΠΎΡΠ²ΠΈΠ΄Π΅Ρ‚Π΅Π»ΡŒΡΡ‚Π²ΠΎΠ²Π°Π½ΠΈΡ Π²Π°ΠΌ Π΄ΠΎΠ»ΠΆΠ½Ρ‹ Π΄Π°Ρ‚ΡŒ направлСния Π² Π²ΠΎΠ΅Π½ΠΊΠΎΠΌΠ°Ρ‚Π΅ для прохоТдСния диагностичСских исслСдований ΠΏΠΎ Π½Π°ΠΏΡ€Π°Π²Π»Π΅Π½ΠΈΡŽ Π²ΠΎΠ΅Π½ΠΊΠΎΠΌΠ°Ρ‚Π°. Они проводятся Π½Π΅ Ρ€Π°Π½ΡŒΡˆΠ΅, Ρ‡Π΅ΠΌ Π·Π° 30 Π΄Π½Π΅ΠΉ Π΄ΠΎ ΠΎΡΠ²ΠΈΠ΄Π΅Ρ‚Π΅Π»ΡŒΡΡ‚Π²ΠΎΠ²Π°Π½ΠΈΡ (Π° Π½Π° сказали Π½Π΅ Ρ€Π°Π½Π΅Π΅ 3-4 Π΄Π½Π΅ΠΉ) ΠΈ Π²ΠΊΠ»ΡŽΡ‡Π°ΡŽΡ‚ Ρ„Π»ΡŽΠΎΡ€ΠΎΠ³Ρ€Π°Ρ„ΠΈΡŽ (Ρ€Π΅Π½Ρ‚Π³Π΅Π½) Π³Ρ€ΡƒΠ΄Π½ΠΎΠΉ ΠΊΠ»Π΅Ρ‚ΠΊΠΈ ΠΈ Π°Π½Π°Π»ΠΈΠ· ΠΊΡ€ΠΎΠ²ΠΈ. Π—Π°Ρ‚Π΅ΠΌ с Ρ€Π΅Π·ΡƒΠ»ΡŒΡ‚Π°Ρ‚Π°ΠΌΠΈ исслСдования (Ρ‚.Π½. Π°ΠΊΡ‚ мСдицинского обслСдования) Π²Ρ‹ ΠΏΡ€ΠΎΠΉΠ΄Π΅Ρ‚Π΅ обслСдованиС Ρƒ Π½Π΅ΡΠΊΠΎΠ»ΡŒΠΊΠΈΡ… Π²Ρ€Π°Ρ‡Π΅ΠΉ: Ρ…ΠΈΡ€ΡƒΡ€Π³Π°, Ρ‚Π΅Ρ€Π°ΠΏΠ΅Π²Ρ‚Π°, Π½Π΅Π²Ρ€ΠΎΠΏΠ°Ρ‚ΠΎΠ»ΠΎΠ³Π°, психиатра, окулиста, ΠΎΡ‚ΠΎΠ»Π°Ρ€ΠΈΠ½Π³ΠΎΠ»ΠΎΠ³Π° ΠΈ стоматолога. ΠšΠ°ΠΆΠ΄Ρ‹ΠΉ ΠΈΠ· Π½ΠΈΡ… Π΄ΠΎΠ»ΠΆΠ΅Π½ ΠΎΠΏΡ€Π΅Π΄Π΅Π»ΠΈΡ‚ΡŒ ΡΡ‚Π΅ΠΏΠ΅Π½ΡŒ вашСй годности ΠΊ Π²ΠΎΠ΅Π½Π½ΠΎΠΉ слуТбС. ΠžΠ±Ρ€Π°Ρ‰Π°Π΅ΠΌ вашС Π²Π½ΠΈΠΌΠ°Π½ΠΈΠ΅, Ρ‡Ρ‚ΠΎ Π²Ρ€Π°Ρ‡ΠΈ Π½Π΅ ΠΌΠΎΠ³ΡƒΡ‚ ΡΠ°ΠΌΠΎΡΡ‚ΠΎΡΡ‚Π΅Π»ΡŒΠ½ΠΎ ΡΡ‚Π°Π²ΠΈΡ‚ΡŒ Π΄ΠΈΠ°Π³Π½ΠΎΠ·. Π˜Ρ… компСтСнция ограничиваСтся сопоставлСниСм Ρ„ΠΎΡ€ΠΌΡƒΠ»ΠΈΡ€ΠΎΠ²ΠΊΠΈ Π΄ΠΈΠ°Π³Π½ΠΎΠ·Π°, ΠΏΠΎΠ»ΡƒΡ‡Π΅Π½Π½ΠΎΠ³ΠΎ Π² Ρ€Π΅Π·ΡƒΠ»ΡŒΡ‚Π°Ρ‚Π΅ обслСдования Π² мСдицинском ΡƒΡ‡Ρ€Π΅ΠΆΠ΄Π΅Π½ΠΈΠΈ ΠΏΠΎ Π½Π°ΠΏΡ€Π°Π²Π»Π΅Π½ΠΈΡŽ Π²ΠΎΠ΅Π½ΠΊΠΎΠΌΠ°Ρ‚Π°, с Ρ„ΠΎΡ€ΠΌΡƒΠ»ΠΈΡ€ΠΎΠ²ΠΊΠΎΠΉ Π΄ΠΈΠ°Π³Π½ΠΎΠ·Π°, ΠΏΡ€ΠΈΡΡƒΡ‚ΡΡ‚Π²ΡƒΡŽΡ‰Π΅ΠΉ Π² расписании Π±ΠΎΠ»Π΅Π·Π½Π΅ΠΉ. Как ΠΏΡ€Π°Π²ΠΈΠ»ΠΎ, мСдицинскоС ΠΎΡΠ²ΠΈΠ΄Π΅Ρ‚Π΅Π»ΡŒΡΡ‚Π²ΠΎΠ²Π°Π½ΠΈΠ΅ проходят Π·Π° ΠΎΠ΄ΠΈΠ½ дСнь.
Π’Ρ€Π°Ρ‡ΠΈ, ΠΊΠΎΡ‚ΠΎΡ€Ρ‹Π΅ проводят мСдицинскоС ΠΎΡΠ²ΠΈΠ΄Π΅Ρ‚Π΅Π»ΡŒΡΡ‚Π²ΠΎΠ²Π°Π½ΠΈΠ΅, Π½Π΅ ΠΏΡ€ΠΈΠ½ΠΈΠΌΠ°ΡŽΡ‚ Ρ€Π΅ΡˆΠ΅Π½ΠΈΠ΅ ΠΎ ΠΏΡ€ΠΈΠ·Ρ‹Π²Π΅ (ΠΎΠ± отсрочкС, ΠΎΠ± освобоТдСнии). Π­Ρ‚ΠΈΠΌ занимаСтся призывная комиссия. А Π²Ρ€Π°Ρ‡ΠΈ Π½Π° мСдицинском ΠΎΡΠ²ΠΈΠ΄Π΅Ρ‚Π΅Π»ΡŒΡΡ‚Π²ΠΎΠ²Π°Π½ΠΈΠΈ выносят Π·Π°ΠΊΠ»ΡŽΡ‡Π΅Π½ΠΈΠ΅ ΠΎ ΠΊΠ°Ρ‚Π΅Π³ΠΎΡ€ΠΈΠΈ годности.
ΠžΠ±Ρ€Π°Ρ‚ΠΈΡ‚Π΅ Π²Π½ΠΈΠΌΠ°Π½ΠΈΠ΅, Ρ‡Ρ‚ΠΎ ΠΏΡ€ΠΎΡ…ΠΎΠ΄ΠΈΡ‚ΡŒ мСдицинскоС ΠΎΡΠ²ΠΈΠ΄Π΅Ρ‚Π΅Π»ΡŒΡΡ‚Π²ΠΎΠ²Π°Π½ΠΈΠ΅ Π²Ρ‹ ΠΌΠΎΠΆΠ΅Ρ‚Π΅ Ρ‚ΠΎΠ»ΡŒΠΊΠΎ Π² сроки ΠΏΡ€ΠΈΠ·Ρ‹Π²Π° ΠΈ Ссли Ρƒ вас Π½Π΅Ρ‚ Π΄Π΅ΠΉΡΡ‚Π²ΡƒΡŽΡ‰Π΅ΠΉ отсрочки ΠΎΡ‚ Π²ΠΎΠ΅Π½Π½ΠΎΠΉ слуТбы. Если Π²Π°ΠΌ прСдлагаСтся ΠΏΡ€ΠΎΠΉΡ‚ΠΈ мСдицинскоС ΠΎΡΠ²ΠΈΠ΄Π΅Ρ‚Π΅Π»ΡŒΡΡ‚Π²ΠΎΠ²Π°Π½ΠΈΠ΅ Π²Π½Π΅ этих сроков, Π²Ρ‹ ΠΌΠΎΠΆΠ΅Ρ‚Π΅ ΠΎΡ‚ΠΊΠ°Π·Π°Ρ‚ΡŒΡΡ, сославшись Π½Π° ΠΏ.1 ст.26 Π—Π’ΠžΠ’Π‘. По Ρ€Π΅Π·ΡƒΠ»ΡŒΡ‚Π°Ρ‚Π°ΠΌ мСдицинского обслСдования опрСдСляСтся ΡΡ‚Π΅ΠΏΠ΅Π½ΡŒ годности ΠΊ Π²ΠΎΠ΅Π½Π½ΠΎΠΉ слуТбС.

И Π΅Ρ‰Π΅.

Π§Ρ‚ΠΎ ΠΌΠΎΠ³ΡƒΡ‚ Π²Ρ€Π°Ρ‡ΠΈ-спСциалисты Π² Π²ΠΎΠ΅Π½ΠΊΠΎΠΌΠ°Ρ‚Π΅, ΠΊΠ°ΠΊΠΈΠ΅ Ρƒ Π½ΠΈΡ… полномочия?

Волько ΡΠΎΠΏΠΎΡΡ‚Π°Π²ΠΈΡ‚ΡŒ Ρ„ΠΎΡ€ΠΌΡƒΠ»ΠΈΡ€ΠΎΠ²ΠΊΡƒ Π΄ΠΈΠ°Π³Π½ΠΎΠ·Π°, ΠΏΠΎΠ»ΡƒΡ‡Π΅Π½Π½ΠΎΠ³ΠΎ Π² Ρ€Π΅Π·ΡƒΠ»ΡŒΡ‚Π°Ρ‚Π΅ обслСдования Π² мСдицинском ΡƒΡ‡Ρ€Π΅ΠΆΠ΄Π΅Π½ΠΈΠΈ ΠΏΠΎ Π½Π°ΠΏΡ€Π°Π²Π»Π΅Π½ΠΈΡŽ ΠΏΡ€ΠΈΠ·Ρ‹Π²Π½ΠΎΠΉ комиссии, с Ρ„ΠΎΡ€ΠΌΡƒΠ»ΠΈΡ€ΠΎΠ²ΠΊΠΎΠΉ Π΄ΠΈΠ°Π³Π½ΠΎΠ·Π° ΠΈΠ· Расписания Π±ΠΎΠ»Π΅Π·Π½Π΅ΠΉ. И ΠΏΠΎΡ‚ΠΎΠΌ ΠΎΠΏΡ€Π΅Π΄Π΅Π»ΠΈΡ‚ΡŒ ΠΊΠ°Ρ‚Π΅Π³ΠΎΡ€ΠΈΡŽ годности ΠΊ Π²ΠΎΠ΅Π½Π½ΠΎΠΉ слуТбС. Π‘Π°ΠΌΠΎΡΡ‚ΠΎΡΡ‚Π΅Π»ΡŒΠ½ΠΎ ΡΡ‚Π°Π²ΠΈΡ‚ΡŒ Π΄ΠΈΠ°Π³Π½ΠΎΠ· ΠΎΠ½ΠΈ Π½Π΅ Π²ΠΏΡ€Π°Π²Π΅.

Π•Ρ‰Π΅.

ИмССм Π»ΠΈ ΠΌΡ‹ ΠΏΡ€Π°Π²ΠΎ Π½Π° Π½Π΅Π·Π°Π²ΠΈΡΠΈΠΌΡƒΡŽ ΠΌΠ΅Π΄ΠΈΡ†ΠΈΠ½ΡΠΊΡƒΡŽ экспСртизу Π² случаС нСсогласия с Ρ€Π΅Π·ΡƒΠ»ΡŒΡ‚Π°Ρ‚ΠΎΠΌ мСдицинского ΠΎΡΠ²ΠΈΠ΄Π΅Ρ‚Π΅Π»ΡŒΡΡ‚Π²ΠΎΠ²Π°Π½ΠΈΡ, ΠΏΡ€ΠΎΠ²Π΅Π΄Π΅Π½Π½ΠΎΠ³ΠΎ Π² Ρ€Π°ΠΌΠΊΠ°Ρ… Ρ€Π°Π±ΠΎΡ‚Ρ‹ ΠΏΡ€ΠΈΠ·Ρ‹Π²Π½ΠΎΠΉ комиссии ΠΈΠ»ΠΈ комиссии ΠΏΠΎ постановкС Π³Ρ€Π°ΠΆΠ΄Π°Π½ Π½Π° воинский ΡƒΡ‡Π΅Ρ‚?

ΠšΠΎΠ½Π΅Ρ‡Π½ΠΎ, ΠΈΠΌΠ΅Π΅Ρ‚Π΅. Об этом говорится Π² ст. 53 Основ Π·Π°ΠΊΠΎΠ½ΠΎΠ΄Π°Ρ‚Π΅Π»ΡŒΡΡ‚Π²Π° Π Π€ ΠΎΠ± ΠΎΡ…Ρ€Π°Π½Π΅ Π·Π΄ΠΎΡ€ΠΎΠ²ΡŒΡ Π³Ρ€Π°ΠΆΠ΄Π°Π½. ΠšΡ€ΠΎΠΌΠ΅ Ρ‚ΠΎΠ³ΠΎ, 20 июля 2008 Π³ΠΎΠ΄Π° Π²Ρ‹ΡˆΠ»ΠΎ ΠŸΠΎΡΡ‚Π°Π½ΠΎΠ²Π»Π΅Π½ΠΈΠ΅ ΠŸΡ€Π°Π²ΠΈΡ‚Π΅Π»ΡŒΡΡ‚Π²Π° β„– 574, ΠΊΠΎΡ‚ΠΎΡ€Ρ‹ΠΌ ΡƒΡ‚Π²Π΅Ρ€ΠΆΠ΄Π΅Π½ΠΎ «ПолоТСниС ΠΎ нСзависимой Π²ΠΎΠ΅Π½Π½ΠΎ-Π²Ρ€Π°Ρ‡Π΅Π±Π½ΠΎΠΉ экспСртизС».

И Π²ΠΎΡ‚ Π΅Ρ‰Π΅ ΠΊΠΎΠ΅-Ρ‡Ρ‚ΠΎ. Π’Π°ΠΆΠ½ΠΎΠ΅ ΠΈ для мСня.

Π’Ρ…ΠΎΠ΄ΠΈΡ‚ Π»ΠΈ Π² ΠΊΠΎΠΌΠΏΠ΅Ρ‚Π΅Π½Ρ†ΠΈΡŽ Π²Ρ€Π°Ρ‡Π°-спСциалиста ΠΈΠ»ΠΈ Π²Ρ€Π°Ρ‡Π°, руководящСго Ρ€Π°Π±ΠΎΡ‚ΠΎΠΉ ΠΏΠΎ мСдицинскому ΠΎΡΠ²ΠΈΠ΄Π΅Ρ‚Π΅Π»ΡŒΡΡ‚Π²ΠΎΠ²Π°Π½ΠΈΡŽ Π³Ρ€Π°ΠΆΠ΄Π°Π½, ΠΏΠΎΠ΄Π»Π΅ΠΆΠ°Ρ‰ΠΈΡ… ΠΏΡ€ΠΈΠ·Ρ‹Π²Ρƒ Π½Π° Π²ΠΎΠ΅Π½Π½ΡƒΡŽ слуТбу, постановка Π½ΠΎΠ²ΠΎΠ³ΠΎ Π΄ΠΈΠ°Π³Π½ΠΎΠ·Π°?

НСт, Π½Π΅ Π²Ρ…ΠΎΠ΄ΠΈΡ‚. Π’Ρ€Π°Ρ‡-спСциалист Π² Π²ΠΎΠ΅Π½ΠΊΠΎΠΌΠ°Ρ‚Π΅ ΠΏΡ€ΠΎΠ²ΠΎΠ΄ΠΈΡ‚ Π½Π΅ обслСдованиС Π³Ρ€Π°ΠΆΠ΄Π°Π½, Π° Ρ‚ΠΎΠ»ΡŒΠΊΠΎ ΠΈΡ… ΠΎΡΠ²ΠΈΠ΄Π΅Ρ‚Π΅Π»ΡŒΡΡ‚Π²ΠΎΠ²Π°Π½ΠΈΠ΅. Он Ρ€Π°Π±ΠΎΡ‚Π°Π΅Ρ‚ с Π΄ΠΎΠΊΡƒΠΌΠ΅Π½Ρ‚Π°ΠΌΠΈ β€” Ρ€Π΅Π·ΡƒΠ»ΡŒΡ‚Π°Ρ‚Π°ΠΌΠΈ Π°Π½Π°Π»ΠΈΠ·ΠΎΠ², Ρ‚Π΅ΠΌΠΈ мСдицинскими Π΄ΠΎΠΊΡƒΠΌΠ΅Π½Ρ‚Π°ΠΌΠΈ, ΠΊΠΎΡ‚ΠΎΡ€Ρ‹Π΅ прСдставит Π΅ΠΌΡƒ ΠΏΡ€ΠΈΠ·Ρ‹Π²Π½ΠΈΠΊ. Π’ случаС нСсогласия с Π΄ΠΈΠ°Π³Π½ΠΎΠ·ΠΎΠΌ, выставлСнном Π² этих Π΄ΠΎΠΊΡƒΠΌΠ΅Π½Ρ‚Π°Ρ…, Π²Ρ€Π°Ρ‡ Π² Π²ΠΎΠ΅Π½ΠΊΠΎΠΌΠ°Ρ‚Π΅ обязан Π½Π΅ Π²Ρ‹ΡΡ‚Π°Π²Π»ΡΡ‚ΡŒ свой Π΄ΠΈΠ°Π³Π½ΠΎΠ·, Π° Π½Π°ΠΏΡ€Π°Π²ΠΈΡ‚ΡŒ ΠΏΡ€ΠΈΠ·Ρ‹Π²Π½ΠΈΠΊΠ° Π½Π° Π΄ΠΎΠΏΠΎΠ»Π½ΠΈΡ‚Π΅Π»ΡŒΠ½ΠΎΠ΅ обслСдованиС. Π’Π°ΠΊ ΠΆΠ΅ ΠΎΠ½ Π΄ΠΎΠ»ΠΆΠ΅Π½ ΠΏΠΎΡΡ‚ΡƒΠΏΠΈΡ‚ΡŒ Π² случаС ΠΆΠ°Π»ΠΎΠ±Ρ‹ ΠΏΡ€ΠΈΠ·Ρ‹Π²Π½ΠΈΠΊΠ° Π½Π° ΠΊΠ°ΠΊΠΎΠ΅-Π»ΠΈΠ±ΠΎ Π·Π°Π±ΠΎΠ»Π΅Π²Π°Π½ΠΈΠ΅, Π΄Π°ΠΆΠ΅ Ссли ΠΎΠ½ΠΎ Π½Π΅ ΠΏΠΎΠ΄Ρ‚Π²Π΅Ρ€ΠΆΠ΄Π΅Π½ΠΎ Π΄ΠΎΠΊΡƒΠΌΠ΅Π½Ρ‚Π°Π»ΡŒΠ½ΠΎ.

Π― Π²ΠΎΡ‚ ΠΏΡ€Π°Π²Π΄Π° Π½Π΅ разобрался ΠΊΠ°ΠΊ ΠΌΠ½Π΅ ΠΏΡ€Π°Π²ΠΈΠ»ΡŒΠ½ΠΎ ΡΠΎΡΡ‚Π°Π²ΠΈΡ‚ΡŒ заявлСниС ΠΎ Ρ‚ΠΎΠΌ, Ρ‡Ρ‚ΠΎ я Π½Π΅ согласСн с Ρ€Π΅Π·ΡƒΠ»ΡŒΡ‚Π°Ρ‚Π°ΠΌΠΈ. НадСюсь Ρ‚Π°ΠΌ подскаТут.

Π‘Π»ΡƒΠΆΠΈΡ‚ΡŒ, ΠΊΠΎΠ½Π΅Ρ‡Π½ΠΎ, ΠΌΠ½Π΅ Π½Π΅ хочСтся. Π•ΡΡ‚ΡŒ ΠΎΠΏΡ€Π΅Π΄Π΅Π»Π΅Π½Π½Ρ‹Π΅ ΠΎΠ±ΡΡ‚ΠΎΡΡ‚Π΅Π»ΡŒΡΡ‚Π²Π°, Π° Π½Π΅ просто какая Ρ‚ΠΎ боязнь. Π₯ΠΎΡ‡Ρƒ Ρƒ Π΄Ρ€ΡƒΠ³ΠΈΡ… спСциалистов ΠΏΡ€ΠΎΠΉΡ‚ΠΈ обслСдования, ΡƒΡ‚ΠΎΡ‡Π½ΠΈΡ‚ΡŒ Π΄ΠΈΠ°Π³Π½ΠΎΠ·Ρ‹. Π’Π΅Π΄ΡŒ ΠΏΠΎΡ‡Π΅ΠΌΡƒ Ρ‚ΠΎ Ρƒ мСня спина ΠΈ ΠΊΠΎΠ»Π΅Π½ΠΎ болят, Π° Π²Ρ€Π°Ρ‡ΠΈ Π² Π²ΠΎΠ΅Π½ΠΊΠΎΠΌΠ°Ρ‚Π΅ говорят, Ρ‡Ρ‚ΠΎ я Π·Π΄ΠΎΡ€ΠΎΠ². ΠœΠΎΠΆΠ΅Ρ‚ я ΠΈ Π·Π΄ΠΎΡ€ΠΎΠ² ΠΈ ΠΎΠ³Ρ€Π°Π½ΠΈΡ‡Π΅Π½Π½ΠΎ Π³ΠΎΠ΄Π΅Π½, Π½ΠΎ Ρ…ΠΎΡ‡Ρƒ это ΡƒΡΠ»Ρ‹ΡˆΠ°Ρ‚ΡŒ ΠΎΡ‚ Π΄Ρ€ΡƒΠ³ΠΈΡ… спСциалистов. Π― ΠΏΡ€Π°Π²Π΄Π° Π½Π΅ ΠΏΡ€Π΅Π΄ΡΡ‚Π°Π²Π»ΡΡŽ ΠΊΠ°ΠΊ ΠΌΠΎΠΆΠ½ΠΎ ΡΠ»ΡƒΠΆΠΈΡ‚ΡŒ Π² Π°Ρ€ΠΌΠΈΠΈ с больно спиной, Ссли сСйчас ΠΎΠ½Π° Π±ΠΎΠ»ΠΈΡ‚ Π±Π΅Π· Π½Π°Π³Ρ€ΡƒΠ·ΠΎΠΊ, Π° Ρ‚Π°ΠΌ придСтся Π±Π΅Π³Π°Ρ‚ΡŒ Π² ΠΏΠΎΠ»Π½ΠΎΠΌ ΠΎΠ±ΠΌΡƒΠ½Π΄ΠΈΡ€ΠΎΠ²Π°Π½ΠΈΠΈ ΠΈ Ρ‚.ΠΏ. Π’ΠΎΡ‚ Ρ‚Π°ΠΊΠΈΠ΅ Π΄Π΅Π»Π°.

P.S. ΠœΠΎΠΆΠ΅Ρ‚ я Ρ‡Π΅Π³ΠΎ Ρ‚ΠΎ Π½Π΅ понимаю ΠΈΠ»ΠΈ Π½Π΅ знаю. ΠŸΠΎΠ΄ΡΠΊΠ°ΠΆΠΈΡ‚Π΅, ΠΏΡ€ΠΎΠΊΠΎΠΌΠΌΠ΅Π½Ρ‚ΠΈΡ€ΡƒΠΉΡ‚Π΅ запись.

UPD 1. Π‘Ρ…ΠΎΠ΄ΠΈΠ» Π² Π²ΠΎΠ΅Π½ΠΊΠΎΠΌΠ°Ρ‚. Π‘Π½ΠΈΠΌΠΊΠΈ Π³Ρ€ΡƒΠ΄Π½ΠΎΠΉ ΠΊΠ»Π΅Ρ‚ΠΊΠΈ ΠΈ ΠΊΠΎΠ»Π΅Π½Π° Π½Π΅ выявили Π½ΠΈΠΊΠ°ΠΊΠΈΡ… Ρ„ΡƒΠ½ΠΊΡ†ΠΈΠΎΠ½Π°Π»ΡŒΠ½Ρ‹Ρ… ΠΈΠ·ΠΌΠ΅Π½Π΅Π½ΠΈΠΉ. А это Π·Π½Π°Ρ‡ΠΈΡ‚, Ρ‡Ρ‚ΠΎ я Π·Π΄ΠΎΡ€ΠΎΠ² ΠΈ Π³ΠΎΠ΄Π΅Π½ ΠΊ ΠΏΡ€ΠΎΡ…ΠΎΠΆΠ΄Π΅Π½ΠΈΡŽ Π’Π‘. БСгодня ΠΏΡ€Π°Π²Π΄Π° Π½Π°ΠΏΡ€Π°Π²ΠΈΠ»ΠΈ Π΅Ρ‰Π΅ Π½Π° снимок Π³ΠΎΠ»Π΅Π½ΠΈ, Ρ‚.ΠΊ. Π² дСтствС Π±Ρ‹Π»ΠΎ ΡΠ΅Ρ€ΡŒΠ΅Π·Π½ΠΎΠ΅ Π·Π°Π±ΠΎΠ»Π΅Π²Π°Π½ΠΈΠ΅ β€” остСомиСлит. Бтоял Π΄Π°ΠΆΠ΅ Π½Π° Π³Ρ€ΡƒΠΏΠΏΠ΅ ΠΏΠΎ Π΄Π°Π½Π½ΠΎΠΌΡƒ заболСванию, Π½ΠΎ Π³Ρ€ΡƒΠΏΠΏΡƒ сняли. Π₯ΠΈΡ€ΡƒΡ€Π³ Π½Π°ΠΏΡ€Π°Π²ΠΈΠ» Π½Π° снимок, Ρ‡Ρ‚ΠΎΠ±Ρ‹ ΡƒΡ‚ΠΎΡ‡Π½ΠΈΡ‚ΡŒ. Π”Π° ΠΈ, Π½Π°Π²Π΅Ρ€Π½ΠΎΠ΅, Ρ‡Ρ‚ΠΎΠ±Ρ‹ ΠΏΡ€Π΅Ρ‚Π΅Π½Π·ΠΈΠΉ ΠΊ Π½Π΅ΠΌΡƒ Π½Π΅ Π±Ρ‹Π»ΠΎ.

Π’ ΠΎΠ±Ρ‰Π΅ΠΌ сходил сдСлал, снова облучился β€” Ρ‚Ρ€Π΅Ρ‚ΠΈΠΉ Ρ€Π°Π· Π·Π° нСдСлю. Надо Π±Ρ‹ ΠΏΠΎΠ΄ΡΡ‡ΠΈΡ‚Π°Ρ‚ΡŒ Π΄ΠΎΠ·Ρƒ облучСния: Ρ„Π»ΡŽΠΎΡ€ΠΎΠ³Ρ€Π°Ρ„ΠΈΡ, снимки Π³Ρ€ΡƒΠ΄Π½ΠΎΠΉ ΠΊΠ»Π΅Ρ‚ΠΊΠΈ ΠΈ ΠΊΠΎΠ»Π΅Π½Π° Π² Π΄Π²ΡƒΡ… плоскостях ΠΈ сСгодняшний снимок Π³ΠΎΠ»Π΅Π½ΠΈ Ρ‚Π°ΠΊΠΆΠ΅Β Π² Π΄Π²ΡƒΡ… плоскостях.

Π’ понСдСльник снова ΠΈΠ΄Ρƒ ΠΊ 9 часам Π² Π²ΠΎΠ΅Π½ΠΊΠΎΠΌΠ°Ρ‚ ΠΈ Ρ‚Π°ΠΌ ΡƒΠΆΠ΅ ΠΎΠΊΠΎΠ½Ρ‡Π°Ρ‚Π΅Π»ΡŒΠ½ΠΎ всё Ρ€Π΅ΡˆΠ°Ρ‚ (ΠžΠΏΡΡ‚ΡŒ ΠΏΡ€ΠΎΠΏΡƒΡ‰Ρƒ ΠΏΠ°Ρ€Ρ‹ Ρƒ Π‘Π°ΡƒΡ€ΠΎΠ²Π°, ΠΏΠΎΡ‚ΠΎΠΌ Π±ΡƒΠ΄Ρƒ Π±Π΅Π³Π°Ρ‚ΡŒ, Ρ‡Ρ‚ΠΎΠ±Ρ‹ Π·Π°Ρ‡Π΅Ρ‚ поставил. Π­Ρ…. Π˜Ρ‚Π°ΠΊ ΡƒΠΆΠ΅ сколько ΠΏΠ°Ρ€ пропустил…). РСбятам Π΄Π°Π²Π°Π»ΠΈ повСстки, Π²Ρ€ΠΎΠ΄Π΅, Π½Π° 23 апрСля, Ρ‡Ρ‚ΠΎΠ±Ρ‹ Ρ‚Π΅ явились Π½Π° Π”Π΅Π½ΡŒ ΠΏΡ€ΠΈΠ·Ρ‹Π²Π½ΠΈΠΊΠ°. Π’Π°ΠΌ Π΄ΠΎΠ»ΠΆΠ½ΠΎ Ρ‚ΠΎΡ‡Π½ΠΎ ΡΠΊΠ°Π·Π°Ρ‚ΡŒ Π² ΠΊΠ°ΠΊΠΈΠ΅ войска отправят. МСня, Π½Π°Π²Π΅Ρ€Π½ΠΎΠ΅, с ΠΏΠ»ΠΎΡ…ΠΈΠΌ Π·Ρ€Π΅Π½ΠΈΠ΅ΠΌ опрСдСлят Π² связисты ΠΈΠ»ΠΈ Ρ‡Ρ‚ΠΎ-Π½ΠΈΠ±ΡƒΠ΄ΡŒ ΠΏΠΎΠ΄ΠΎΠ±Π½ΠΎΠ΅. Π“Π»Π°Π²Π½ΠΎΠ΅, Ρ‡Ρ‚ΠΎΠ±Ρ‹ ΡƒΡ‡ΠΈΠ»ΠΈ Π²ΠΎΠ΅Π½Π½ΠΎΠΌΡƒ Π΄Π΅Π»Ρƒ. НСохота ΠΌΠ½Π΅ ΡΠ»ΠΎΠ½ΡΡ‚ΡŒΡΡ Π±Π΅Π· Π΄Π΅Π»Π°, ΡΡ‚Ρ€Π°Π΄Π°Ρ‚ΡŒ Ρ…Π΅Ρ€Π½Π΅ΠΉ, Π±Ρ‹Ρ‚ΡŒ подсобным Ρ€Π°Π±ΠΎΡ‡ΠΈΠΌ β€” бСсплатной Ρ€Π°Π±.силой.

Π£ΠΏΠΎΠΌΠΈΠ½Π°Π»ΠΈ ΠΏΡ€ΠΎΡ…ΠΎΠΆΠ΄Π΅Π½ΠΈΠ΅ Π°Π»ΡŒΡ‚Π΅Ρ€Π½Π°Ρ‚ΠΈΠ²Π½ΠΎΠΉ граТданской слуТбы. Но я ΠΎ Π½Π΅ΠΉ Π½Π΅ Π΄ΡƒΠΌΠ°Π» ΠΈ Π½Π΅ настроСн Π½Π° Π½Π΅Ρ‘.

UPD 2. Π’ ΠΎΡ‡Π΅Ρ€Π΅Π΄Π½ΠΎΠΉ Ρ€Π°Π· сходил Π² Π²ΠΎΠ΅Π½ΠΊΠΎΠΌΠ°Ρ‚ (11.04.2011). Π‘Π½ΠΈΠΌΠΎΠΊ Π³ΠΎΠ»Π΅Π½ΠΈ Ρ‚Π°ΠΊ ΠΆΠ΅ Π½Π΅ выявил Π½ΠΈΠΊΠ°ΠΊΠΈΡ… ΠΈΠ·ΠΌΠ΅Π½Π΅Π½ΠΈΠΉ. Π‘Ρ‚Ρ€Π°Π½Π½ΠΎ, я Π΄ΡƒΠΌΠ°Π» Ρ‡Ρ‚ΠΎ Π½ΠΈΠ±ΡƒΠ΄ΡŒ Π±ΡƒΠ΄Π΅Ρ‚, вСдь Π±Ρ‹Π»ΠΎ ΡΠ΅Ρ€ΡŒΠ΅Π·Π½ΠΎΠ΅ заболСвания (ΠΏΡƒΡΡ‚ΡŒ ΠΈ Π² дСтствС). Β«ΠšΠΎΡΡ‚ΡŒ дСвствСнно Π·Π΄ΠΎΡ€ΠΎΠ²Π°Β» β€” Ρ‚Π°ΠΊ сказал Ρ…ΠΈΡ€ΡƒΡ€Π³ ΠΎ костях Π³ΠΎΠ»Π΅Π½ΠΈ. Π‘ΠΊΠ°Π·Π°Π», Ρ‡Ρ‚ΠΎ хроничСским остСомиСлитом ΠΈ Π½Π΅ ΠΏΠ°Ρ…Π½Π΅Ρ‚. Ну ΠΈ Ρ…ΠΎΡ€ΠΎΡˆΠΎ, Π² ΠΏΡ€ΠΈΠ½Ρ†ΠΈΠΏΠ΅ ΠΊΠ°ΠΊΠΈΡ… Ρ‚ΠΎ ΠΆΠ°Π»ΠΎΠ± Π½Π° Π½ΠΎΠ³Ρƒ ΠΈ Π½Π΅ Π±Ρ‹Π»ΠΎ. ΠŸΡ€ΠΈΠ·Π½Π°Π»ΠΈ Π³ΠΎΠ΄Π½Ρ‹ΠΌ Π±Π΅Π· ΠΎΠ³Ρ€Π°Π½ΠΈΡ‡Π΅Π½ΠΈΠΉ, ΠΏΡ€Π°Π²Π΄Π° я Π΄ΡƒΠΌΠ°Π», Ρ‡Ρ‚ΠΎ ΠΏΠΎ Π·Ρ€Π΅Π½ΠΈΡŽ снизят.

23 апрСля ΠΈΠ΄Ρƒ Π½Π° Π”Π΅Π½ΡŒ ΠΏΡ€ΠΈΠ·Ρ‹Π²Π½ΠΈΠΊΠ°. Π’Π°ΠΌ Π±ΡƒΠ΄ΡƒΡ‚ ΡΠΎΠΎΠ±Ρ‰Π°Ρ‚ΡŒ ΠΊΡƒΠ΄Π° ΠΈ Π² ΠΊΠ°ΠΊΠΈΠ΅ войска Π±ΡƒΠ΄ΡƒΡ‚ ΠΎΡ‚ΠΏΡ€Π°Π²Π»ΡΡ‚ΡŒ. БСгодня ΠΆΠ΅ всС Π΄ΠΎΠΊΡƒΠΌΠ΅Π½Ρ‚Ρ‹ ΠΎΡ„ΠΎΡ€ΠΌΠΈΠ», ΠΏΠΎΠ»ΡƒΡ‡ΠΈΠ» повСстку Π½Π° Π”Π΅Π½ΡŒ ΠΏΡ€ΠΈΠ·Ρ‹Π²Π½ΠΈΠΊΠ°, Π·Π°ΠΏΠΎΠ»Π½ΠΈΠ» Π΄ΠΎΠΊΡƒΠΌΠ΅Π½Ρ‚ для доступа ΠΊ гос. Ρ‚Π°ΠΉΠ½Π΅. )) На сСгодняшнюю ΠΏΡ€ΠΈΠ·Ρ‹Π²Π½ΡƒΡŽ комиссию ΡƒΠΆΠ΅ ΠΈΠ΄Ρ‚ΠΈ Π½Π΅ Π½Π°Π΄ΠΎ.

Π’Π΅ΠΏΠ΅Ρ€ΡŒ Π½Π°Π΄ΠΎ ΠΊΠ°ΠΊ Ρ‚ΠΎ Π·Π°ΠΊΠΎΠ½Ρ‡ΠΈΡ‚ΡŒ ΡƒΠ½ΠΈΠ²Π΅Ρ€ ΠΈ прямиком Π² Π°Ρ€ΠΌΠΈΡŽ. ΠžΡ‚ΡΡ€ΠΎΡ‡ΠΊΠ° Ρƒ мСня Π΄ΠΎ 30 июня, Π° Π·Π°ΠΊΠ°Π½Ρ‡ΠΈΠ²Π°Π΅ΠΌ ΠΌΡ‹ послС 3-Π³ΠΎ июля. Π”ΠΈΠΏΠ»ΠΎΠΌ сначала хочСтся ΠΏΠΎΠ»ΡƒΡ‡ΠΈΡ‚ΡŒ, Π° ΠΏΠΎΡ‚ΠΎΠΌ ΠΎΡ‚ΠΏΡ€Π°Π²Π»ΡΡ‚ΡŒ ΡƒΠΆΠ΅ ΡΠ»ΡƒΠΆΠΈΡ‚ΡŒ. Надо Π±ΡƒΠ΄Π΅Ρ‚ ΠΊΠ°ΠΊ Ρ‚ΠΎ Ρ€Π΅ΡˆΠΈΡ‚ΡŒ это вопрос.

Ρ‡Ρ‚ΠΎ Π±ΡƒΠ΄Π΅Ρ‚ Π·Π° Π½Π° нСявку, ΠΊΠ°ΠΊ ΠΏΡ€ΠΎΠΉΡ‚ΠΈ, ΠΊΠΎΠ³Π΄Π° Π·Π°Π±Π΅Ρ€ΡƒΡ‚ Π² Π°Ρ€ΠΌΠΈΡŽ

Π”ΠΎΠΏΠΎΠ»Π½ΠΈΡ‚Π΅Π»ΡŒΠ½ΠΎΠ΅ обслСдованиС

Если юноша ΠΏΠΎΠΏΠ°Π΄Π°Π΅Ρ‚ ΠΏΠΎΠ΄ ΠΊΠ°Ρ‚Π΅Π³ΠΎΡ€ΠΈΡŽ ΠΎΠ³Ρ€Π°Π½ΠΈΡ‡Π΅Π½Π½ΠΎΠΉ годности ΠΈΠ»ΠΈ ΠΏΠΎΠ»Π½ΠΎΠΉ нСгодности ΠΊ Π²ΠΎΠ΅Π½Π½ΠΎΠΉ слуТбС ΠΏΠΎ ΡΠΎΡΡ‚ΠΎΡΠ½ΠΈΡŽ Π·Π΄ΠΎΡ€ΠΎΠ²ΡŒΡ, Ρ‚ΠΎ слСдуСт ΠΎΠ±Ρ€Π°Ρ‚ΠΈΡ‚ΡŒΡΡ Π² ΠΏΠΎΠ»ΠΈΠΊΠ»ΠΈΠ½ΠΈΠΊΡƒ ΠΏΠΎ мСсту прописки ΠΈ ΠΏΠΎΠΏΡ€ΠΎΡΠΈΡ‚ΡŒ участкового Π²Ρ€Π°Ρ‡Π° ΡΠ΄Π΅Π»Π°Ρ‚ΡŒ выписку ΠΎΠ± ΠΈΠΌΠ΅ΡŽΡ‰Π΅ΠΉΡΡ Π±ΠΎΠ»Π΅Π·Π½ΠΈ. Выписка выдаСтся бСсплатно, Π΅Π΅ Π½ΡƒΠΆΠ½ΠΎ ΠΎΠ±ΡΠ·Π°Ρ‚Π΅Π»ΡŒΠ½ΠΎ Π·Π°Π²Π΅Ρ€ΠΈΡ‚ΡŒ подписью Π²Ρ€Π°Ρ‡Π° ΠΈ ΠΏΠ΅Ρ‡Π°Ρ‚ΡŒΡŽ мСдицинского учрСТдСния.

НСобходимо ΡΠ΄Π΅Π»Π°Ρ‚ΡŒ нСсколько ксСрокопий этого Π΄ΠΎΠΊΡƒΠΌΠ΅Π½Ρ‚Π°. Когда ΠΏΡ€ΠΎΡ…ΠΎΠ΄ΠΈΡ‚ мСдкомиссия Π² Π²ΠΎΠ΅Π½ΠΊΠΎΠΌΠ°Ρ‚Π΅, Π²Ρ€Π°Ρ‡, ΠΏΠΎ ΠΏΡ€ΠΎΡ„ΠΈΠ»ΡŽ ΠΊΠΎΡ‚ΠΎΡ€ΠΎΠ³ΠΎ имССтся Π·Π°Π±ΠΎΠ»Π΅Π²Π°Π½ΠΈΠ΅, юноша Π΄ΠΎΠ»ΠΆΠ΅Π½ сразу Ρ€Π°ΡΡΠΊΠ°Π·Π°Ρ‚ΡŒ ΠΎ своСй Π±ΠΎΠ»Π΅Π·Π½ΠΈ, ΠΏΡ€Π΅Π΄ΠΎΡΡ‚Π°Π²ΠΈΡ‚ΡŒ выписку.

Π’Π°ΠΆΠ½ΠΎ! Π’Ρ€Π°Ρ‡ послС изучСния Π΄ΠΎΠΊΡƒΠΌΠ΅Π½Ρ‚Π° ΠΏΡ€ΠΈΠΎΠ±Ρ‰Π°Π΅Ρ‚ Π΅Π³ΠΎ копию ΠΊ Π»ΠΈΡ‡Π½ΠΎΠΌΡƒ Π΄Π΅Π»Ρƒ. Π—Π°Ρ‚Π΅ΠΌ спСциалист направляСт ΡŽΠ½ΠΎΡˆΡƒ Π½Π° Π΄ΠΎΠΏΠΎΠ»Π½ΠΈΡ‚Π΅Π»ΡŒΠ½ΠΎΠ΅ обслСдованиС Π² Π³ΠΎΡ€ΠΎΠ΄ΡΠΊΡƒΡŽ ΠΈΠ»ΠΈ ΠΎΠ±Π»Π°ΡΡ‚Π½ΡƒΡŽ ΠΏΠΎΠ»ΠΈΠΊΠ»ΠΈΠ½ΠΈΠΊΡƒ. Π’ канцСлярии Π²Ρ‹Π΄Π°Π΄ΡƒΡ‚ повСстку Π½Π° ΡΠ»Π΅Π΄ΡƒΡŽΡ‰Π΅Π΅ ΠΎΡΠ²ΠΈΠ΄Π΅Ρ‚Π΅Π»ΡŒΡΡ‚Π²ΠΎΠ²Π°Π½ΠΈΠ΅.

ПослС Π΅Π³ΠΎ прохоТдСния Π² ΠΏΠΎΠ»ΠΈΠΊΠ»ΠΈΠ½ΠΈΠΊΠ΅ Π΄Π°Π΄ΡƒΡ‚ Π·Π°ΠΏΠΎΠ»Π½Π΅Π½Π½Ρ‹Π΅ Π°ΠΊΡ‚Ρ‹ ΠΎ состоянии Π·Π΄ΠΎΡ€ΠΎΠ²ΡŒΡ, ΠΏΠΎΠ΄ΠΏΠΈΡˆΡƒΡ‚, поставят ΠΏΠ΅Ρ‡Π°Ρ‚ΠΈ ΠΈ с этими Π΄ΠΎΠΊΡƒΠΌΠ΅Π½Ρ‚Π°ΠΌΠΈ Π½ΡƒΠΆΠ½ΠΎ снова Π²Π΅Ρ€Π½ΡƒΡ‚ΡŒΡΡ Π² Π²ΠΎΠ΅Π½ΠΊΠΎΠΌΠ°Ρ‚ Π² ΡƒΠΊΠ°Π·Π°Π½Π½ΠΎΠ΅ Π² повСсткС врСмя.

Π’Π°ΠΆΠ½ΠΎ! Π‘Ρ€Π°Π·Ρƒ Π½ΡƒΠΆΠ½ΠΎ ΠΎΠ±Ρ€Π°Ρ‚ΠΈΡ‚ΡŒΡΡ ΠΊ ΠΏΡ€ΠΎΡ„ΠΈΠ»ΡŒΠ½ΠΎΠΌΡƒ Π²Ρ€Π°Ρ‡Ρƒ, ΠΊΠΎΡ‚ΠΎΡ€Ρ‹ΠΉ Π²Ρ‹Π΄Π°Π» Π½Π°ΠΏΡ€Π°Π²Π»Π΅Π½ΠΈΠ΅ Π½Π° Π΄ΠΎΠΏΠΎΠ»Π½ΠΈΡ‚Π΅Π»ΡŒΠ½ΠΎΠ΅ обслСдованиС. Он ΠΈΠ·ΡƒΡ‡ΠΈΡ‚ Π°ΠΊΡ‚Ρ‹ ΠΈ ΠΎΠΏΡ€Π΅Π΄Π΅Π»ΠΈΡ‚ ΠΊΠ°Ρ‚Π΅Π³ΠΎΡ€ΠΈΡŽ годности.

ΠŸΡ€ΠΈΠ·Π½Π°Π½ΠΈΠ΅ Π³Ρ€Π°ΠΆΠ΄Π°Π½ΠΈΠ½Π° Π½Π΅Π³ΠΎΠ΄Π½Ρ‹ΠΌ ΠΊ воинской слуТбС призывная комиссия ΠΏΡ€ΠΈΠ½ΠΈΠΌΠ°Π΅Ρ‚ Π½Π° основании ΡΠ»Π΅Π΄ΡƒΡŽΡ‰Π΅ΠΉ Π΄ΠΎΠΊΡƒΠΌΠ΅Π½Ρ‚Π°Ρ†ΠΈΠΈ:

  • листок мСдицинского ΠΎΡΠ²ΠΈΠ΄Π΅Ρ‚Π΅Π»ΡŒΡΡ‚Π²ΠΎΠ²Π°Π½ΠΈΡ с Π·Π°ΠΊΠ»ΡŽΡ‡Π΅Π½ΠΈΠ΅ΠΌ всСх Π²Ρ€Π°Ρ‡Π΅ΠΉ;
  • Π°ΠΊΡ‚ исслСдования состояния Π·Π΄ΠΎΡ€ΠΎΠ²ΡŒΡ ΠΌΠΎΠ»ΠΎΠ΄ΠΎΠ³ΠΎ Ρ‡Π΅Π»ΠΎΠ²Π΅ΠΊΠ° с ΡƒΠΊΠ°Π·Π°Π½ΠΈΠ΅ΠΌ всСх ΠΏΡ€ΠΎΠ²Π΅Π΄Π΅Π½Π½Ρ‹Ρ… Π°Π½Π°Π»ΠΈΠ·ΠΎΠ², Π΄ΠΈΠ°Π³Π½ΠΎΠ·ΠΎΠΌ, выпиской ΠΈΠ· истории Π±ΠΎΠ»Π΅Π·Π½ΠΈ;
  • ΠΊΠ°Ρ€Ρ‚Π° ΠΌΠ΅Π΄ΠΎΡΠ²ΠΈΠ΄Π΅Ρ‚Π΅Π»ΡŒΡΡ‚Π²ΠΎΠ²Π°Π½ΠΈΡ.

ПослС этого Π³Ρ€Π°ΠΆΠ΄Π°Π½ΠΈΠ½ ΠΏΠΎΠ»ΡƒΡ‡Π°Π΅Ρ‚ Π²ΠΎΠ΅Π½Π½Ρ‹ΠΉ Π±ΠΈΠ»Π΅Ρ‚, Π² ΠΊΠΎΡ‚ΠΎΡ€ΠΎΠΌ ставится катСгория Β«Π”Β». Π‘ Π²ΠΎΠ΅Π½Π½ΠΎ-Π²Ρ€Π°Ρ‡Π΅Π±Π½ΠΎΠΉ комиссиСй Ρƒ ΠΏΡ€ΠΈΠ·Ρ‹Π²Π½ΠΈΠΊΠ° ΠΌΠΎΠ³ΡƒΡ‚ Π²ΠΎΠ·Π½ΠΈΠΊΠ½ΡƒΡ‚ΡŒ спорныС вопросы.

Π’Ρ€Π°Ρ‡ΠΈ Π½Π° мСдкомиссии Π² Π²ΠΎΠ΅Π½ΠΊΠΎΠΌΠ°Ρ‚Π΅ Π½Π΅ заинтСрСсованы Π² освобоТдСнии юношСй ΠΎΡ‚ воинской обязанности, поэтому осмотр проводят повСрхностно, часто Π½Π΅ ΠΎΠ±Ρ€Π°Ρ‰Π°ΡŽΡ‚ внимания Π½Π° ΠΆΠ°Π»ΠΎΠ±Ρ‹ ΠΌΠΎΠ»ΠΎΠ΄Ρ‹Ρ… людСй ΠΈ ΠΏΡ€ΠΈΠ·Π½Π°ΡŽΡ‚ ΠΈΡ… Π³ΠΎΠ΄Π½Ρ‹ΠΌΠΈ.

Если ΠΏΡ€ΠΈΠ·Ρ‹Π²Π½ΠΈΠΊ Π½Π΅ согласСн с Ρ€Π΅Π·ΡƒΠ»ΡŒΡ‚Π°Ρ‚Π°ΠΌΠΈ ΠΎΡΠ²ΠΈΠ΄Π΅Ρ‚Π΅Π»ΡŒΡΡ‚Π²ΠΎΠ²Π°Π½ΠΈΡ, ΠΎΠ½ ΠΌΠΎΠΆΠ΅Ρ‚ ΠΏΠΎΠ΄Π°Ρ‚ΡŒ ΠΆΠ°Π»ΠΎΠ±Ρƒ ΠΏΡ€Π΅Π΄ΡΠ΅Π΄Π°Ρ‚Π΅Π»ΡŽ ΠΏΡ€ΠΈΠ·Ρ‹Π²Π½ΠΎΠΉ комиссии ΠΈΠ»ΠΈ Π·Π°ΠΊΠ°Π·Π°Ρ‚ΡŒ Π½Π΅ΠΏΡ€Π΅Π΄Π²Π·ΡΡ‚ΡƒΡŽ Π²ΠΎΠ΅Π½Π½ΠΎ-Π²Ρ€Π°Ρ‡Π΅Π±Π½ΡƒΡŽ экспСртизу Π·Π° свой счСт. Если Π²Ρ€Π°Ρ‡ Π½Π΅ΠΏΡ€Π°Π²ΠΈΠ»ΡŒΠ½ΠΎ поставит ΠΊΠ°Ρ‚Π΅Π³ΠΎΡ€ΠΈΡŽ годности (даст отсрочку ΠΎΡ‚ ΠΏΡ€ΠΈΠ·Ρ‹Π²Π° вмСсто Ρ‚ΠΎΠ³ΠΎ, Ρ‡Ρ‚ΠΎΠ±Ρ‹ ΠΊΠΎΠΌΠΈΡΡΠΎΠ²Π°Ρ‚ΡŒ), ΠΏΡ€ΠΈΠ·Ρ‹Π²Π½ΠΈΠΊ Π²ΠΏΡ€Π°Π²Π΅ ΠΎΠ±ΠΆΠ°Π»ΠΎΠ²Π°Ρ‚ΡŒ это Ρ€Π΅ΡˆΠ΅Π½ΠΈΠ΅ Π² судС.

ΠšΡ‚ΠΎ Π΄ΠΎΠ»ΠΆΠ΅Π½ ΠΏΡ€ΠΎΠΉΡ‚ΠΈ ΠΈΠΌΠΌΠΈΠ³Ρ€Π°Ρ†ΠΈΠΎΠ½Π½Ρ‹ΠΉ мСдицинский осмотр

Π˜Π½ΠΎΡΡ‚Ρ€Π°Π½Π½Ρ‹Π΅ Π³Ρ€Π°ΠΆΠ΄Π°Π½Π΅, ΠΏΠΎΠ΄Π°ΡŽΡ‰ΠΈΠ΅ заявлСниС Π½Π° ΠΏΠΎΠ»ΡƒΡ‡Π΅Π½ΠΈΠ΅ Π²ΠΈΠ·Ρ‹ постоянного ТитСля ΠΈΠ»ΠΈ ΠΏΡ€Π΅Ρ‚Π΅Π½Π΄ΡƒΡŽΡ‰ΠΈΠ΅ Π½Π° ΠΏΡ€Π΅Π±Ρ‹Π²Π°Π½ΠΈΠ΅ Π² КанадС Π² качСствС постоянного ТитСля, Π΄ΠΎΠ»ΠΆΠ½Ρ‹ ΠΏΡ€ΠΎΠΉΡ‚ΠΈ мСдицинскоС обслСдованиС [R30 (1) (a) (i)].

ЕдинствСнным ΠΈΡΠΊΠ»ΡŽΡ‡Π΅Π½ΠΈΠ΅ΠΌ являСтся Π±ΠΎΠ»ΡŒΡˆΠΈΠ½ΡΡ‚Π²ΠΎ заявитСлСй-постоянных ΠΆΠΈΡ‚Π΅Π»Π΅ΠΉ, ΠΊΠΎΡ‚ΠΎΡ€Ρ‹Π΅ ΡΠ²Π»ΡΡŽΡ‚ΡΡ Ρ‡Π»Π΅Π½Π°ΠΌΠΈ класса ΠΎΠΏΠ΅ΠΊΡƒΠ½ΠΎΠ² с ΠΏΡ€ΠΎΠΆΠΈΠ²Π°Π½ΠΈΠ΅ΠΌ Π² сСмьС (LC1) Π² соотвСтствии с ΠΏΠ°Ρ€Π°Π³Ρ€Π°Ρ„ΠΎΠΌ R30 (1) (g). Однако Π²Π°ΠΆΠ½ΠΎ ΠΎΡ‚ΠΌΠ΅Ρ‚ΠΈΡ‚ΡŒ, Ρ‡Ρ‚ΠΎ Π½Π΅ всС заявитСли LC1 освобоТдСны ΠΎΡ‚ этого.

ВсС ΡΠΎΠΏΡ€ΠΎΠ²ΠΎΠΆΠ΄Π°ΡŽΡ‰ΠΈΠ΅ Ρ‡Π»Π΅Π½Ρ‹ сСмьи иностранного Π³Ρ€Π°ΠΆΠ΄Π°Π½ΠΈΠ½Π°, ΠΏΠΎΠ΄Π°ΡŽΡ‰ΠΈΠ΅ заявлСниС Π½Π° ΠΏΠΎΠ»ΡƒΡ‡Π΅Π½ΠΈΠ΅ Π²ΠΈΠ·Ρ‹ постоянного ТитСля ΠΈΠ»ΠΈ ΠΏΡ€Π΅Ρ‚Π΅Π½Π΄ΡƒΡŽΡ‰Π΅Π³ΠΎ Π½Π° ΠΏΡ€Π΅Π±Ρ‹Π²Π°Π½ΠΈΠ΅ Π² КанадС Π² качСствС постоянного ТитСля, Π΄ΠΎΠ»ΠΆΠ½Ρ‹ ΠΏΡ€ΠΎΠΉΡ‚ΠΈ мСдицинскоС обслСдованиС.

Иногда Ρƒ иностранных Π³Ρ€Π°ΠΆΠ΄Π°Π½ Π΅ΡΡ‚ΡŒ родствСнники, ΠΊΠΎΡ‚ΠΎΡ€Ρ‹Π΅ Π½Π΅ ΠΏΠΎΠ΅Π΄ΡƒΡ‚ Π² ΠšΠ°Π½Π°Π΄Ρƒ. Π—Π° ΠΎΡ‡Π΅Π½ΡŒ Ρ€Π΅Π΄ΠΊΠΈΠΌ ΠΈΡΠΊΠ»ΡŽΡ‡Π΅Π½ΠΈΠ΅ΠΌ, эти Ρ‡Π»Π΅Π½Ρ‹ сСмьи Ρ‚Π°ΠΊΠΆΠ΅ Π΄ΠΎΠ»ΠΆΠ½Ρ‹ ΠΏΡ€ΠΎΠΉΡ‚ΠΈ мСдицинскоС обслСдованиС. Π˜ΡΠΊΠ»ΡŽΡ‡Π΅Π½ΠΈΡ пСрСчислСны Π² ΠΏΠ°Ρ€Π°Π³Ρ€Π°Ρ„Π΅ R30 (1):

  • Ρ‡Π»Π΅Π½ сСмьи Π·Π°Ρ‰ΠΈΡ‰Π°Π΅ΠΌΠΎΠ³ΠΎ Π»ΠΈΡ†Π°, Ссли Ρ‡Π»Π΅Π½ сСмьи Π½Π΅ Π²ΠΊΠ»ΡŽΡ‡Π΅Π½ Π² заявлСниС Π·Π°Ρ‰ΠΈΡ‰Π°Π΅ΠΌΠΎΠ³ΠΎ Π»ΠΈΡ†Π° Π½Π° ΠΏΡ€Π΅Π±Ρ‹Π²Π°Π½ΠΈΠ΅ Π² КанадС Π² качСствС постоянного ТитСля [R30 (1) (e) ];
  • Π½Π΅ΡΠΎΠΏΡ€ΠΎΠ²ΠΎΠΆΠ΄Π°ΡŽΡ‰ΠΈΠΉ Ρ‡Π»Π΅Π½ сСмьи иностранного Π³Ρ€Π°ΠΆΠ΄Π°Π½ΠΈΠ½Π°, подавшСго заявлСниС ΠΎ прСдоставлСнии статуса Π±Π΅ΠΆΠ΅Π½Ρ†Π° [R30 (1) (f)].

ΠœΠ΅Π΄ΠΈΡ†ΠΈΠ½ΡΠΊΠΎΠ΅ ΠΎΡΠ²ΠΈΠ΄Π΅Ρ‚Π΅Π»ΡŒΡΡ‚Π²ΠΎΠ²Π°Π½ΠΈΠ΅ Π½Π΅ трСбуСтся, Ссли Π·Π°ΡΠ²ΠΈΡ‚Π΅Π»ΡŒ Π½Π΅ намСрСваСтся Ρ€Π°Π±ΠΎΡ‚Π°Ρ‚ΡŒ ΠΏΠΎ профСссии, Ρ‚Ρ€Π΅Π±ΡƒΡŽΡ‰Π΅ΠΉ Π·Π°Ρ‰ΠΈΡ‚Ρ‹ общСствСнного Π·Π΄ΠΎΡ€ΠΎΠ²ΡŒΡ. Π’ ΡΠ»Π΅Π΄ΡƒΡŽΡ‰Π΅ΠΌ спискС ΠΏΡ€ΠΈΠ²Π΅Π΄Π΅Π½Ρ‹ ΠΏΡ€ΠΈΠΌΠ΅Ρ€Ρ‹ Ρ‚Π°ΠΊΠΈΡ… занятий. Π­Ρ‚ΠΎΡ‚ список Π½Π΅ ΠΈΡΡ‡Π΅Ρ€ΠΏΡ‹Π²Π°ΡŽΡ‰ΠΈΠΉ.

  • ΠŸΡ€ΠΎΡ„Π΅ΡΡΠΈΠΈ, ΠΎΠ±Π΅ΡΠΏΠ΅Ρ‡ΠΈΠ²Π°ΡŽΡ‰ΠΈΠ΅ тСсный ΠΊΠΎΠ½Ρ‚Π°ΠΊΡ‚ заявитСля с людьми, Π° ΠΈΠΌΠ΅Π½Π½ΠΎ:
    • Ρ€Π°Π±ΠΎΡ‚Π½ΠΈΠΊΠΈ Π² области здравоохранСния;
    • клиничСских Π»Π°Π±ΠΎΡ€Π°Π½Ρ‚ΠΎΠ²;
    • санитаров Π² Π΄ΠΎΠΌΠ°Ρ… прСстарСлых ΠΈ прСстарСлых;
    • студСнтов-ΠΌΠ΅Π΄ΠΈΠΊΠΎΠ² приняты Π² ΠšΠ°Π½Π°Π΄Ρƒ для обучСния Π² унивСрситСтах;
    • Π²Ρ€Π°Ρ‡Π΅Π±Π½Ρ‹Ρ… Ρ„Π°ΠΊΡƒΠ»ΡŒΡ‚Π°Ρ‚ΠΈΠ²ΠΎΠ² ΠΈ Π²Ρ€Π°Ρ‡Π΅ΠΉ Π½Π° краткосрочной основС;
    • ΡƒΡ‡ΠΈΡ‚Π΅Π»Π΅ΠΉ Π½Π°Ρ‡Π°Π»ΡŒΠ½Ρ‹Ρ… ΠΈ срСдних школ ΠΈΠ»ΠΈ Π΄Ρ€ΡƒΠ³ΠΈΡ… ΡƒΡ‡ΠΈΡ‚Π΅Π»Π΅ΠΉ ΠΌΠ»Π°Π΄ΡˆΠΈΡ… классов
    • Π΄ΠΎΠΌΡ€Π°Π±ΠΎΡ‚Π½ΠΈΠΊΠΎΠ²;
    • Ρ€Π°Π±ΠΎΡ‚Π½ΠΈΠΊΠΎΠ², ΡƒΡ…Π°ΠΆΠΈΠ²Π°ΡŽΡ‰ΠΈΡ… Π½Π° Π΄ΠΎΠΌΡƒ Π·Π° Π΄Π΅Ρ‚ΡŒΠΌΠΈ, прСстарСлыми ΠΈ ΠΈΠ½Π²Π°Π»ΠΈΠ΄Π°ΠΌΠΈ;
    • Ρ€Π°Π±ΠΎΡ‚Π½ΠΈΠΊΠΎΠ² яслСй;
    • ΡΠ΅Π»ΡŒΡΠΊΠΎΡ…ΠΎΠ·ΡΠΉΡΡ‚Π²Π΅Π½Π½Ρ‹Ρ… Ρ€Π°Π±ΠΎΡ‡ΠΈΡ… ΠΈΠ· ΠΎΠΏΡ€Π΅Π΄Π΅Π»Π΅Π½Π½Ρ‹Ρ… стран / Ρ‚Π΅Ρ€Ρ€ΠΈΡ‚ΠΎΡ€ΠΈΠΉ.Π‘Ρ‚Ρ€Π°Π½Π° ΠΈΠ»ΠΈ тСрритория обозначаСтся, Ссли Π² столбцС «ВрСбуСтся ΠΈΠΌΠΌΠΈΠ³Ρ€Π°Ρ†ΠΈΠΎΠ½Π½Ρ‹ΠΉ мСдицинский осмотр (IME)Β» Π² спискС ΡƒΠΊΠ°Π·Π°Π½Π½Ρ‹Ρ… стран ΠΈ Ρ‚Π΅Ρ€Ρ€ΠΈΡ‚ΠΎΡ€ΠΈΠΉ стоит «ДА».

ΠŸΡ€ΠΈ расчСтС Ρ‚ΠΎΠ³ΠΎ, прСвысит Π»ΠΈ ΠΏΡ€Π΅Π±Ρ‹Π²Π°Π½ΠΈΠ΅ Π² странС ΡˆΠ΅ΡΡ‚ΡŒ мСсяцСв, фактичСскоС ΠΈΠ»ΠΈ Π·Π°ΠΏΠ»Π°Π½ΠΈΡ€ΠΎΠ²Π°Π½Π½ΠΎΠ΅ отсутствиС Π² КанадС ΠΏΡ€ΠΎΠ΄ΠΎΠ»ΠΆΠΈΡ‚Π΅Π»ΡŒΠ½ΠΎΡΡ‚ΡŒΡŽ ΠΌΠ΅Π½Π΅Π΅ 14 Π΄Π½Π΅ΠΉ Π½Π΅ влияСт Π½Π° ΠΎΠΏΡ€Π΅Π΄Π΅Π»Π΅Π½ΠΈΠ΅.

ΠœΠ΅Π΄ΠΈΡ†ΠΈΠ½ΡΠΊΠΎΠ΅ ΠΎΡΠ²ΠΈΠ΄Π΅Ρ‚Π΅Π»ΡŒΡΡ‚Π²ΠΎΠ²Π°Π½ΠΈΠ΅ трСбуСтся для:

  • ΠŸΡ€ΠΎΡ„Π΅ΡΡΠΈΠΉ, Ρ‚Ρ€Π΅Π±ΡƒΡŽΡ‰ΠΈΡ… Π·Π°Ρ‰ΠΈΡ‚Ρ‹ общСствСнного Π·Π΄ΠΎΡ€ΠΎΠ²ΡŒΡ ΠΈ ΠΎΠ±Π΅ΡΠΏΠ΅Ρ‡ΠΈΠ²Π°ΡŽΡ‰ΠΈΡ… тСсный ΠΊΠΎΠ½Ρ‚Π°ΠΊΡ‚ заявитСля с людьми, Π° ΠΈΠΌΠ΅Π½Π½ΠΎ:
    • Ρ€Π°Π±ΠΎΡ‚Π½ΠΈΠΊΠΎΠ² сфСры здравоохранСния;
    • клиничСских Π»Π°Π±ΠΎΡ€Π°Π½Ρ‚ΠΎΠ²;
    • санитаров Π² Π΄ΠΎΠΌΠ°Ρ… прСстарСлых ΠΈ прСстарСлых;
    • студСнтов-ΠΌΠ΅Π΄ΠΈΠΊΠΎΠ² приняты Π² ΠšΠ°Π½Π°Π΄Ρƒ для обучСния Π² унивСрситСтах;
    • Π²Ρ€Π°Ρ‡Π΅Π±Π½Ρ‹Ρ… Ρ„Π°ΠΊΡƒΠ»ΡŒΡ‚Π°Ρ‚ΠΈΠ²ΠΎΠ² ΠΈ Π²Ρ€Π°Ρ‡Π΅ΠΉ Π½Π° краткосрочной основС;
    • ΡƒΡ‡ΠΈΡ‚Π΅Π»Π΅ΠΉ Π½Π°Ρ‡Π°Π»ΡŒΠ½Ρ‹Ρ… ΠΈ срСдних школ ΠΈΠ»ΠΈ Π΄Ρ€ΡƒΠ³ΠΈΡ… ΡƒΡ‡ΠΈΡ‚Π΅Π»Π΅ΠΉ дошкольников;
    • Π΄ΠΎΠΌΡ€Π°Π±ΠΎΡ‚Π½ΠΈΠΊΠΎΠ²;
    • Ρ€Π°Π±ΠΎΡ‚Π½ΠΈΠΊΠΎΠ², ΡƒΡ…Π°ΠΆΠΈΠ²Π°ΡŽΡ‰ΠΈΡ… Π½Π° Π΄ΠΎΠΌΡƒ Π·Π° Π΄Π΅Ρ‚ΡŒΠΌΠΈ, прСстарСлыми ΠΈ ΠΈΠ½Π²Π°Π»ΠΈΠ΄Π°ΠΌΠΈ;
    • сотрудников яслСй.

ΠŸΡ€ΠΈΠΌΠ΅Ρ‡Π°Π½ΠΈΠ΅: Π’Ρ€Π΅ΠΌΠ΅Π½Π½Ρ‹ΠΉ Ρ€Π΅Π·ΠΈΠ΄Π΅Π½Ρ‚ Π² опасной для ΠΆΠΈΠ·Π½ΠΈ Ρ‡Ρ€Π΅Π·Π²Ρ‹Ρ‡Π°ΠΉΠ½ΠΎΠΉ ситуации, ΠΎΠ±Ρ€Π°Ρ‰Π°ΡŽΡ‰ΠΈΠΉΡΡ Π² Π±ΠΎΠ»ΡŒΠ½ΠΈΡ†Ρƒ, ΡΠΏΠ΅Ρ†ΠΈΠ°Π»ΠΈΠ·ΠΈΡ€ΡƒΡŽΡ‰ΡƒΡŽΡΡ Π½Π° Π»Π΅Ρ‡Π΅Π½ΠΈΠΈ, нСдоступном Π² Π΄Ρ€ΡƒΠ³ΠΎΠΌ мСстС, ΠΌΠΎΠΆΠ΅Ρ‚ Π·Π°ΡΠ»ΡƒΠΆΠΈΡ‚ΡŒ ускорСнноС рассмотрСниС Π΄Π΅Π»Π°. Π§Ρ‚ΠΎΠ±Ρ‹ ΠΎΡ„ΠΈΡ†Π΅Ρ€ убСдился Π² Ρ‚ΠΎΠΌ, Ρ‡Ρ‚ΠΎ ΠΎΠΏΠ»Π°Ρ‚Π° лСчСния Π³Π°Ρ€Π°Π½Ρ‚ΠΈΡ€ΠΎΠ²Π°Π½Π° ΠΈ Ρ‡Ρ‚ΠΎ ΠΎΠ½ΠΎ Π½Π΅ вытСснит ΠΊΠ°Π½Π°Π΄Ρ†Π΅Π², ΠΎΠΆΠΈΠ΄Π°ΡŽΡ‰ΠΈΡ… лСчСния, ΠΏΠ΅Ρ€Π΅Π΄ принятиСм Ρ€Π΅ΡˆΠ΅Π½ΠΈΡ слСдуСт ΠΏΡ€ΠΎΠΊΠΎΠ½ΡΡƒΠ»ΡŒΡ‚ΠΈΡ€ΠΎΠ²Π°Ρ‚ΡŒΡΡ с Π²Ρ€Π°Ρ‡ΠΎΠΌ. Авиакомпаниям, пСрСвозящим Ρ‚Π°ΠΊΠΈΡ… Π»ΠΈΡ†, Π²ΠΎΠ·ΠΌΠΎΠΆΠ½ΠΎ, потрСбуСтся ΠΏΡ€ΠΎΠΈΠ½Ρ„ΠΎΡ€ΠΌΠΈΡ€ΠΎΠ²Π°Ρ‚ΡŒ ΠΎ Ρ…Π°Ρ€Π°ΠΊΡ‚Π΅Ρ€Π΅ состояния, ΠΈ ΠΎΡ„ΠΈΡ†Π΅Ρ€Ρ‹ Π΄ΠΎΠ»ΠΆΠ½Ρ‹ ΠΏΠΎΡΠΎΠ²Π΅Ρ‚ΠΎΠ²Π°Ρ‚ΡŒ Π²Ρ€Π΅ΠΌΠ΅Π½Π½ΠΎΠΌΡƒ Ρ€Π΅Π·ΠΈΠ΄Π΅Π½Ρ‚Ρƒ ΡƒΠ²Π΅Π΄ΠΎΠΌΠΈΡ‚ΡŒ авиакомпанию ΠΏΠ΅Ρ€Π΅Π΄ ΠΏΠΎΠ΅Π·Π΄ΠΊΠΎΠΉ, Ρ‡Ρ‚ΠΎΠ±Ρ‹ ΠΈΠ·Π±Π΅ΠΆΠ°Ρ‚ΡŒ ослоТнСний Π²ΠΎ врСмя ΠΏΡƒΡ‚Π΅ΡˆΠ΅ΡΡ‚Π²ΠΈΡ.

,

ΠœΠ΅Π΄ΠΈΡ†ΠΈΠ½ΡΠΊΠΈΠ΅ осмотры посСтитСлСй, студСнтов ΠΈ Ρ€Π°Π±ΠΎΡ‡ΠΈΡ…

На этой страницС

ΠšΠΎΠΌΡƒ Π½ΡƒΠΆΠ΅Π½ мСдицинский осмотр

Если Π²Ρ‹ ΠΏΠ»Π°Π½ΠΈΡ€ΡƒΠ΅Ρ‚Π΅ ΠΎΡΡ‚Π°Ρ‚ΡŒΡΡ Π½Π° 6 мСсяцСв ΠΈΠ»ΠΈ ΠΌΠ΅Π½Π΅Π΅

ΠžΠ±Ρ‹Ρ‡Π½ΠΎ Π²Π°ΠΌ Π½Π΅ трСбуСтся мСдицинский осмотр, Ссли Π²Ρ‹ Π½Π΅ ΠΏΠ»Π°Π½ΠΈΡ€ΡƒΠ΅Ρ‚Π΅ Ρ€Π°Π±ΠΎΡ‚Π°Ρ‚ΡŒ Π½Π° ΠΎΠΏΡ€Π΅Π΄Π΅Π»Π΅Π½Π½Ρ‹Ρ… долТностях.

Π Π°Π±ΠΎΡ‚Π°, для ΠΊΠΎΡ‚ΠΎΡ€ΠΎΠΉ трСбуСтся мСдицинский осмотр

Π’Π°ΠΌ ΠΌΠΎΠΆΠ΅Ρ‚ ΠΏΠΎΡ‚Ρ€Π΅Π±ΠΎΠ²Π°Ρ‚ΡŒΡΡ мСдицинский осмотр ΠΈΠ·-Π·Π° Ρ‚ΠΎΠ³ΠΎ, Ρ‡Π΅ΠΌ Π²Ρ‹ Ρ…ΠΎΡ‚ΠΈΡ‚Π΅ Π·Π°Π½ΠΈΠΌΠ°Ρ‚ΡŒΡΡ Π² КанадС.

ΠŸΡ€ΠΈΠΌΠ΅Ρ€Ρ‹ Ρ‚Π°ΠΊΠΈΡ… вакансий:

  1. Вакансии, ΠΊΠΎΡ‚ΠΎΡ€Ρ‹Π΅ позволят Π²Π°ΠΌ Π±Π»ΠΈΠ·ΠΊΠΎ ΠΏΠΎΠ·Π½Π°ΠΊΠΎΠΌΠΈΡ‚ΡŒΡΡ с людьми, Ρ‚Π°ΠΊΠΈΠΌΠΈ ΠΊΠ°ΠΊ:
    • Ρ€Π°Π±ΠΎΡ‚Π½ΠΈΠΊΠΎΠ² сфСры здравоохранСния
    • клиничСских Π»Π°Π±ΠΎΡ€Π°Π½Ρ‚ΠΎΠ²
    • санитаров Π² Π΄ΠΎΠΌΠ°Ρ… прСстарСлых ΠΈ прСстарСлых
    • студСнтов-ΠΌΠ΅Π΄ΠΈΠΊΠΎΠ² приняты Π² ΠšΠ°Π½Π°Π΄Ρƒ для обучСния Π² унивСрситСтах
    • Π²Ρ€Π°Ρ‡Π΅Π±Π½Ρ‹Ρ… Ρ„Π°ΠΊΡƒΠ»ΡŒΡ‚Π°Ρ‚ΠΈΠ²ΠΎΠ² ΠΈ Π²Ρ€Π°Ρ‡Π΅ΠΉ Π½Π° краткосрочном ΠΏΠ΅Ρ€ΠΈΠΎΠ΄Π΅
    • ΡƒΡ‡ΠΈΡ‚Π΅Π»Π΅ΠΉ Π½Π°Ρ‡Π°Π»ΡŒΠ½Ρ‹Ρ… ΠΈΠ»ΠΈ срСдних школ ΠΈΠ»ΠΈ Π΄Ρ€ΡƒΠ³ΠΈΡ… ΡƒΡ‡ΠΈΡ‚Π΅Π»Π΅ΠΉ ΠΌΠ»Π°Π΄ΡˆΠΈΡ… классов
    • домашнСС хозяйство
    • Ρ€Π°Π±ΠΎΡ‚Π½ΠΈΠΊΠΎΠ², ΡƒΡ…Π°ΠΆΠΈΠ²Π°ΡŽΡ‰ΠΈΡ… Π½Π° Π΄ΠΎΠΌΡƒ Π·Π° Π΄Π΅Ρ‚ΡŒΠΌΠΈ, стариками ΠΈ ΠΈΠ½Π²Π°Π»ΠΈΠ΄Π°ΠΌΠΈ
    • сотрудников яслСй ΠΈ
    • Π΄Ρ€ΡƒΠ³ΠΈΠ΅ ΠΏΠΎΡ…ΠΎΠΆΠΈΠ΅ вакансии
  2. ΡΠ΅Π»ΡŒΡΠΊΠΎΡ…ΠΎΠ·ΡΠΉΡΡ‚Π²Π΅Π½Π½Ρ‹Ρ… Ρ€Π°Π±ΠΎΡ‡ΠΈΡ…, ΠΊΠΎΡ‚ΠΎΡ€Ρ‹Π΅ посСтили ΠΈΠ»ΠΈ ΠΏΡ€ΠΎΠΆΠΈΠ²Π°Π»ΠΈ Π² ΠΎΠ΄Π½ΠΎΠΉ ΠΈΠ· этих стран Π±ΠΎΠ»Π΅Π΅ 6 мСсяцСв Π² Ρ‚Π΅Ρ‡Π΅Π½ΠΈΠ΅ ΠΏΡ€ΠΎΡˆΠ»ΠΎΠ³ΠΎ Π³ΠΎΠ΄Π°.

Если Π²Ρ‹ ΠΏΠ»Π°Π½ΠΈΡ€ΡƒΠ΅Ρ‚Π΅ ΠΎΡΡ‚Π°Ρ‚ΡŒΡΡ Π±ΠΎΠ»Π΅Π΅ 6 мСсяцСв

Π’Π°ΠΌ Π½Π΅ΠΎΠ±Ρ…ΠΎΠ΄ΠΈΠΌΠΎ ΠΏΡ€ΠΎΠΉΡ‚ΠΈ мСдицинскоС обслСдованиС, Ссли Π²Ρ‹ соотвСтствуСтС Π»ΡŽΠ±ΠΎΠΌΡƒ ΠΈΠ· ΡΠ»Π΅Π΄ΡƒΡŽΡ‰ΠΈΡ… ΠΊΡ€ΠΈΡ‚Π΅Ρ€ΠΈΠ΅Π²:

Если Π²Π°ΠΌ Π½ΡƒΠΆΠ΅Π½ мСдицинский осмотр, Π²ΠΈΠ·ΠΎΠ²Ρ‹ΠΉ ΠΎΡ‚Π΄Π΅Π» сообщит Π²Π°ΠΌ, Ρ‡Ρ‚ΠΎ Π΄Π΅Π»Π°Ρ‚ΡŒ дальшС.

ΠšΡ‚ΠΎ ΠΌΠΎΠΆΠ΅Ρ‚ ΡΠ΄Π°Ρ‚ΡŒ экзамСн

Π’Ρ‹ Π΄ΠΎΠ»ΠΆΠ½Ρ‹ ΠΎΠ±Ρ€Π°Ρ‚ΠΈΡ‚ΡŒΡΡ ΠΊ Π²Ρ€Π°Ρ‡Ρƒ ΠΈΠ· списка Ρ‚Π΅Ρ€Π°ΠΏΠ΅Π²Ρ‚ΠΎΠ². Π’Π°Ρˆ собствСнный Π²Ρ€Π°Ρ‡ Π½Π΅ ΠΌΠΎΠΆΠ΅Ρ‚ ΠΏΡ€ΠΎΠΉΡ‚ΠΈ мСдицинский осмотр.

Π’Ρ€Π°Ρ‡-Ρ‚Π΅Ρ€Π°ΠΏΠ΅Π²Ρ‚ Π½Π΅ ΠΏΡ€ΠΈΠ½ΠΈΠΌΠ°Π΅Ρ‚ ΠΎΠΊΠΎΠ½Ρ‡Π°Ρ‚Π΅Π»ΡŒΠ½ΠΎΠ³ΠΎ Ρ€Π΅ΡˆΠ΅Π½ΠΈΡ ΠΏΠΎ ΠΏΠΎΠ²ΠΎΠ΄Ρƒ вашСго мСдицинского осмотра .ΠœΡ‹ ΠΏΡ€ΠΈΠ½ΠΈΠΌΠ°Π΅ΠΌ это Ρ€Π΅ΡˆΠ΅Π½ΠΈΠ΅. Если Π²ΠΎΠ·Π½ΠΈΠΊΠ½ΡƒΡ‚ ΠΏΡ€ΠΎΠ±Π»Π΅ΠΌΡ‹ с мСдицинским осмотром, ΠΌΡ‹ свяТСмся с Π²Π°ΠΌΠΈ Π² письмСнной Ρ„ΠΎΡ€ΠΌΠ΅.

НайдитС Ρ‚Π΅Ρ€Π°ΠΏΠ΅Π²Ρ‚Π° для провСдСния вашСго исслСдования.

Когда ΠΏΡ€ΠΎΡ…ΠΎΠ΄ΠΈΡ‚ΡŒ мСдицинский осмотр

Π’Ρ‹ ΠΌΠΎΠΆΠ΅Ρ‚Π΅ ΠΏΡ€ΠΎΠΉΡ‚ΠΈ мСдицинский осмотр Π΄ΠΎ ΠΈΠ»ΠΈ послС ΠΏΠΎΠ΄Π°Ρ‡ΠΈ заявлСния.

Π‘Π΄Π°Ρ‡Π° экзамСна ΠΏΠ΅Ρ€Π΅Π΄ ΠΏΠΎΠ΄Π°Ρ‡Π΅ΠΉ заявки

Π£ вас Π΅ΡΡ‚ΡŒ Π²ΠΎΠ·ΠΌΠΎΠΆΠ½ΠΎΡΡ‚ΡŒ ΡΠ΄Π°Ρ‚ΡŒ экзамСн Π΄ΠΎ ΠΏΠΎΠ΄Π°Ρ‡ΠΈ заявлСния. Π­Ρ‚ΠΎ называСтся ΠΏΡ€Π΅Π΄Π²Π°Ρ€ΠΈΡ‚Π΅Π»ΡŒΠ½Ρ‹ΠΌ мСдицинским осмотром. Π§Ρ‚ΠΎΠ±Ρ‹ ΠΏΠΎΠ»ΡƒΡ‡ΠΈΡ‚ΡŒ Π΅Π³ΠΎ, ΠΎΠ±Ρ€Π°Ρ‚ΠΈΡ‚Π΅ΡΡŒ нСпосрСдствСнно ΠΊ Π»Π΅Ρ‡Π°Ρ‰Π΅ΠΌΡƒ Π²Ρ€Π°Ρ‡Ρƒ.

Π’Ρ‹ ΠΌΠΎΠΆΠ΅Ρ‚Π΅ ΠΏΠΎΠ»ΡƒΡ‡ΠΈΡ‚ΡŒ Π΅Π³ΠΎ, ΠΎΠ±Ρ€Π°Ρ‚ΠΈΠ²ΡˆΠΈΡΡŒ ΠΏΠΎ Ρ‚Π΅Π»Π΅Ρ„ΠΎΠ½Ρƒ:

  • посСщСниС (Π²ΠΊΠ»ΡŽΡ‡Π°Ρ супСрвизу Ρ€ΠΎΠ΄ΠΈΡ‚Π΅Π»Π΅ΠΉ ΠΈ Π΄Π΅Π΄ΡƒΡˆΠ΅ΠΊ ΠΈ Π΄Π΅Π΄ΡƒΡˆΠ΅ΠΊ)
  • Ρ€Π°Π±ΠΎΡ‚Π°
  • ΠΊΠ°Π±ΠΈΠ½Π΅Ρ‚

Π‘Π΄Π°Ρ‡Π° экзамСна послС ΠΏΠΎΠ΄Π°Ρ‡ΠΈ заявки

ΠœΡ‹ ΠΏΡ€ΠΈΡˆΠ»Π΅ΠΌ Π²Π°ΠΌ инструкции ΠΎ Ρ‚ΠΎΠΌ, ΠΊΠ°ΠΊ ΠΏΡ€ΠΎΠΉΡ‚ΠΈ мСдицинский осмотр. Π’Ρ‹ Π΄ΠΎΠ»ΠΆΠ½Ρ‹ ΠΏΡ€ΠΎΠΉΡ‚ΠΈ мСдицинский осмотр Π² Ρ‚Π΅Ρ‡Π΅Π½ΠΈΠ΅ 30 Π΄Π½Π΅ΠΉ послС послС получСния этих инструкций.

Если Π²Ρ‹ Π½Π΅ Π²Ρ‹ΠΏΠΎΠ»Π½ΠΈΡ‚Π΅ эти инструкции, ΠΌΡ‹ ΠΌΠΎΠΆΠ΅ΠΌ ΠΎΡ‚ΠΊΠ»ΠΎΠ½ΠΈΡ‚ΡŒ Π²Π°ΡˆΡƒ заявку.

Π§Ρ‚ΠΎ Π²Π·ΡΡ‚ΡŒ с собой

Когда Π²Ρ‹ ΠΈΠ΄Π΅Ρ‚Π΅ Π½Π° ΠΏΡ€ΠΈΠ΅ΠΌ для мСдицинского осмотра, Π²Ρ‹ Π΄ΠΎΠ»ΠΆΠ½Ρ‹ принСсти:

  • надлСТащая идСнтификация — ΠΊΠ°ΠΊ ΠΌΠΈΠ½ΠΈΠΌΡƒΠΌ 1 Π΄ΠΎΠΊΡƒΠΌΠ΅Π½Ρ‚ государствСнного ΠΎΠ±Ρ€Π°Π·Ρ†Π° с вашСй Ρ„ΠΎΡ‚ΠΎΠ³Ρ€Π°Ρ„ΠΈΠ΅ΠΉ ΠΈ подписью, Π½Π°ΠΏΡ€ΠΈΠΌΠ΅Ρ€, паспорт ΠΈΠ»ΠΈ Π½Π°Ρ†ΠΈΠΎΠ½Π°Π»ΡŒΠ½ΠΎΠ΅ удостовСрСниС личности
    • Π’Ρ‹ Ρ‚Π°ΠΊΠΆΠ΅ ΠΌΠΎΠΆΠ΅Ρ‚Π΅ ΠΈΡΠΏΠΎΠ»ΡŒΠ·ΠΎΠ²Π°Ρ‚ΡŒ канадскиС Π²ΠΎΠ΄ΠΈΡ‚Π΅Π»ΡŒΡΠΊΠΈΠ΅ ΠΏΡ€Π°Π²Π°, Π½ΠΎ Ρ‚ΠΎΠ»ΡŒΠΊΠΎ Π² КанадС
  • ΠžΡ‡ΠΊΠΈ ΠΈΠ»ΠΈ ΠΊΠΎΠ½Ρ‚Π°ΠΊΡ‚Π½Ρ‹Π΅ Π»ΠΈΠ½Π·Ρ‹, Ссли Π²Ρ‹ ΠΈΡ… носитС
  • Π»ΡŽΠ±Ρ‹Π΅ мСдицинскиС ΠΎΡ‚Ρ‡Π΅Ρ‚Ρ‹ ΠΈΠ»ΠΈ Ρ€Π΅Π·ΡƒΠ»ΡŒΡ‚Π°Ρ‚Ρ‹ Π°Π½Π°Π»ΠΈΠ·ΠΎΠ², ΠΊΠΎΡ‚ΠΎΡ€Ρ‹Π΅ Ρƒ вас Π΅ΡΡ‚ΡŒ Π½Π° ΠΊΠ°ΠΊΠΈΠ΅-Π»ΠΈΠ±ΠΎ ΠΏΡ€Π΅Π΄Ρ‹Π΄ΡƒΡ‰ΠΈΠ΅ ΠΈΠ»ΠΈ ΡΡƒΡ‰Π΅ΡΡ‚Π²ΡƒΡŽΡ‰ΠΈΠ΅ мСдицинскиС условия
  • список Π²Π°ΡˆΠΈΡ… Ρ‚Π΅ΠΊΡƒΡ‰ΠΈΡ… лСкарств
  • Π€ΠΎΡ€ΠΌΠ° мСдицинского Π·Π°ΠΊΠ»ΡŽΡ‡Π΅Π½ΠΈΡ (IMM 1017E), Ссли Ρƒ вас Π½Π΅Ρ‚ ΠΏΡ€Π΅Π΄Π²Π°Ρ€ΠΈΡ‚Π΅Π»ΡŒΠ½ΠΎΠ³ΠΎ мСдицинского осмотра
    • ΠœΡ‹ Π²Ρ‹ΡˆΠ»Π΅ΠΌ Π²Π°ΠΌ эту Ρ„ΠΎΡ€ΠΌΡƒ

Π’Ρ‹ Ρ‚Π°ΠΊΠΆΠ΅ Π΄ΠΎΠ»ΠΆΠ½Ρ‹ принСсти 4 Π½Π΅Π΄Π°Π²Π½ΠΈΠ΅ Ρ„ΠΎΡ‚ΠΎΠ³Ρ€Π°Ρ„ΠΈΠΈ , Ссли Π²Ρ€Π°Ρ‡ Π³Ρ€ΡƒΠΏΠΏΡ‹ Π½Π΅ ΠΈΡΠΏΠΎΠ»ΡŒΠ·ΡƒΠ΅Ρ‚ eMedical .Π‘Π²ΡΠΆΠΈΡ‚Π΅ΡΡŒ с Π»Π΅Ρ‡Π°Ρ‰ΠΈΠΌ Π²Ρ€Π°Ρ‡ΠΎΠΌ ΠΏΠ΅Ρ€Π΅Π΄ Π½Π°Π·Π½Π°Ρ‡Π΅Π½ΠΈΠ΅ΠΌ, Ρ‡Ρ‚ΠΎΠ±Ρ‹ ΡƒΠ·Π½Π°Ρ‚ΡŒ.

Если вас направят Π½Π° Ρ€Π΅Π½Ρ‚Π³Π΅Π½ ΠΈΠ»ΠΈ Π΄Ρ€ΡƒΠ³ΠΈΠ΅ Π°Π½Π°Π»ΠΈΠ·Ρ‹, вас ΠΌΠΎΠ³ΡƒΡ‚ ΠΏΠΎΠΏΡ€ΠΎΡΠΈΡ‚ΡŒ снова ΠΏΡ€Π΅Π΄ΡŠΡΠ²ΠΈΡ‚ΡŒ удостовСрСниС личности , ΠΊΠΎΠ³Π΄Π° Π²Ρ‹ ΠΏΠΎΠΉΠ΄Π΅Ρ‚Π΅ Π½Π° эти Π°Π½Π°Π»ΠΈΠ·Ρ‹.

Бколько Π½ΡƒΠΆΠ½ΠΎ ΠΏΠ»Π°Ρ‚ΠΈΡ‚ΡŒ Π·Π°

Π’Ρ‹ Π΄ΠΎΠ»ΠΆΠ½Ρ‹ ΠΎΠΏΠ»Π°Ρ‚ΠΈΡ‚ΡŒ всС сборы, связанныС с мСдицинским осмотром, ΠΊΠΎΠ³Π΄Π° Π²Ρ‹ Π±ΡƒΠ΄Π΅Ρ‚Π΅ Ρ‚Π°ΠΌ, Π² Ρ‚ΠΎΠΌ числС:

  • ΠΏΠ»Π°Ρ‚Π° Π·Π° Π²Ρ€Π°Ρ‡Π° ΠΈΠ»ΠΈ Ρ€Π°Π΄ΠΈΠΎΠ»ΠΎΠ³Π°
  • Π»ΡŽΠ±Ρ‹Π΅ ΡΠΏΠ΅Ρ†ΠΈΠ°Π»ΡŒΠ½Ρ‹Π΅ тСсты, исслСдования ΠΈΠ»ΠΈ Π»Π΅Ρ‡Π΅Π½ΠΈΠ΅, Π½Π΅ΠΎΠ±Ρ…ΠΎΠ΄ΠΈΠΌΡ‹Π΅
  • Π»ΡŽΠ±Ρ‹Π΅ спСциалисты, ΠΊΠΎΡ‚ΠΎΡ€Ρ‹Π΅ Π²Π°ΠΌ Π½ΡƒΠΆΠ½Ρ‹, Ρ‡Ρ‚ΠΎΠ±Ρ‹ ΡƒΠ²ΠΈΠ΄Π΅Ρ‚ΡŒ

Если ΠΌΡ‹ ΠΎΡ‚ΠΊΠ»ΠΎΠ½ΠΈΠΌ Π²Π°ΡˆΡƒ заявку послС вашСго мСдицинского осмотра, ΠΌΡ‹ Π½Π΅ Π½Π΅ возмСстим эти сборы.

Π‘Π΅ΠΆΠ΅Π½Ρ†Ρ‹ ΠΈ проситСли ΡƒΠ±Π΅ΠΆΠΈΡ‰Π° освобоТдСны ΠΎΡ‚ ΡƒΠΏΠ»Π°Ρ‚Ρ‹ пошлин.

Π§Π΅Π³ΠΎ ΠΎΠΆΠΈΠ΄Π°Ρ‚ΡŒ Π²ΠΎ врСмя экзамСна

Волько ΡƒΡ‚Π²Π΅Ρ€ΠΆΠ΄Π΅Π½Π½Ρ‹ΠΉ Π²Ρ€Π°Ρ‡ Π³Ρ€ΡƒΠΏΠΏΡ‹ ΠΌΠΎΠΆΠ΅Ρ‚ ΠΏΡ€ΠΎΠΉΡ‚ΠΈ ΠΏΠΎΠ»Π½Ρ‹ΠΉ мСдицинский осмотр ΠΏΠΎ ΠΈΠΌΠΌΠΈΠ³Ρ€Π°Ρ†ΠΈΠΎΠ½Π½Ρ‹ΠΌ ΠΏΡ€ΠΈΡ‡ΠΈΠ½Π°ΠΌ.

Когда Π²Ρ‹ ΠΏΡ€ΠΈΠ΅Π΄Π΅Ρ‚Π΅

Π’Ρ€Π°Ρ‡ Π³Ρ€ΡƒΠΏΠΏΡ‹ ΠΈΠ»ΠΈ пСрсонал ΠΊΠ»ΠΈΠ½ΠΈΠΊΠΈ попросят вас ΠΏΡ€Π΅Π΄ΡŠΡΠ²ΠΈΡ‚ΡŒ удостовСрСниС личности, Ρ‡Ρ‚ΠΎΠ±Ρ‹ ΠΏΠΎΠ΄Ρ‚Π²Π΅Ρ€Π΄ΠΈΡ‚ΡŒ Π²Π°ΡˆΡƒ Π»ΠΈΡ‡Π½ΠΎΡΡ‚ΡŒ. Если вас направят Π½Π° Ρ€Π΅Π½Ρ‚Π³Π΅Π½ ΠΈΠ»ΠΈ Π΄Ρ€ΡƒΠ³ΠΈΠ΅ тСсты, вас ΠΌΠΎΠ³ΡƒΡ‚ ΠΏΠΎΠΏΡ€ΠΎΡΠΈΡ‚ΡŒ снова ΠΏΡ€Π΅Π΄ΡŠΡΠ²ΠΈΡ‚ΡŒ удостовСрСниС личности, ΠΊΠΎΠ³Π΄Π° Π²Ρ‹ ΠΏΠΎΠΉΠ΄Π΅Ρ‚Π΅ Π½Π° эти тСсты. Π’Π°ΡˆΠ° фотография Ρ‚Π°ΠΊΠΆΠ΅ Π±ΡƒΠ΄Π΅Ρ‚ сдСлана для Π½Π°ΡˆΠΈΡ… записСй.

АнкСта ΠΏΠΎ истории Π±ΠΎΠ»Π΅Π·Π½ΠΈ

Π’Ρ€Π°Ρ‡ Π·Π°ΠΏΠΎΠ»Π½ΠΈΡ‚ с Π²Π°ΠΌΠΈ Π°Π½ΠΊΠ΅Ρ‚Ρƒ ΠΈΠ· истории Π±ΠΎΠ»Π΅Π·Π½ΠΈ. Π­Ρ‚Π° Π°Π½ΠΊΠ΅Ρ‚Π° касаСтся Π»ΡŽΠ±Ρ‹Ρ… ΠΏΡ€Π΅Π΄Ρ‹Π΄ΡƒΡ‰ΠΈΡ… ΠΈΠ»ΠΈ ΡΡƒΡ‰Π΅ΡΡ‚Π²ΡƒΡŽΡ‰ΠΈΡ… Π·Π°Π±ΠΎΠ»Π΅Π²Π°Π½ΠΈΠΉ. Они Ρ‚Π°ΠΊΠΆΠ΅ спросят вас, ΠΊΠ°ΠΊΠΈΠ΅ лСкарства Π²Ρ‹ ΠΏΡ€ΠΈΠ½ΠΈΠΌΠ°Π΅Ρ‚Π΅.

Π’Π°ΠΆΠ½ΠΎ ΡΠΎΠΎΠ±Ρ‰ΠΈΡ‚ΡŒ Π»Π΅Ρ‡Π°Ρ‰Π΅ΠΌΡƒ Π²Ρ€Π°Ρ‡Ρƒ ΠΎ Π»ΡŽΠ±Ρ‹Ρ… ΠΏΡ€Π΅Π΄Ρ‹Π΄ΡƒΡ‰ΠΈΡ… ΠΈΠ»ΠΈ ΡΡƒΡ‰Π΅ΡΡ‚Π²ΡƒΡŽΡ‰ΠΈΡ… заболСваниях. Если Π²Ρ‹ этого Π½Π΅ сдСлаСтС, ΠΎΠ±Ρ€Π°Π±ΠΎΡ‚ΠΊΠ° вашСго мСдицинского осмотра ΠΌΠΎΠΆΠ΅Ρ‚ Π·Π°Π½ΡΡ‚ΡŒ большС Π²Ρ€Π΅ΠΌΠ΅Π½ΠΈ.

ΠœΠ΅Π΄ΠΈΡ†ΠΈΠ½ΡΠΊΠΈΠΉ осмотр

Π’Ρ‹ ΠΏΡ€ΠΎΠΉΠ΄Π΅Ρ‚Π΅ мСдицинский осмотр.

Π’Ρ€Π°Ρ‡ ΠΈΠ»ΠΈ мСдицинский пСрсонал ΠΏΠΎΠ»ΠΈΠΊΠ»ΠΈΠ½ΠΈΠΊΠΈ Π±ΡƒΠ΄Π΅Ρ‚:

  • вСс
  • ΠΈΠ·ΠΌΠ΅Ρ€ΠΈΡ‚ΡŒ свой рост
  • ΠΏΡ€ΠΎΠ²Π΅Ρ€ΡŒΡ‚Π΅ слух ΠΈ Π·Ρ€Π΅Π½ΠΈΠ΅
  • ΠΈΠ·ΠΌΠ΅Ρ€ΠΈΡ‚ΡŒ Π°Ρ€Ρ‚Π΅Ρ€ΠΈΠ°Π»ΡŒΠ½ΠΎΠ΅ Π΄Π°Π²Π»Π΅Π½ΠΈΠ΅
  • почувствуй ΠΏΡƒΠ»ΡŒΡ
  • ΡΠ»ΡƒΡˆΠ°ΠΉ своС сСрдцС ΠΈ Π»Π΅Π³ΠΊΠΈΠ΅
  • ΠΏΠΎΡ‰ΡƒΠΏΠ°ΠΉ ΠΆΠΈΠ²ΠΎΡ‚
  • ΠΏΡ€ΠΎΠ²Π΅Ρ€ΡŒΡ‚Π΅, ΠΊΠ°ΠΊ Π΄Π²ΠΈΠ³Π°ΡŽΡ‚ΡΡ ваши конСчности
  • посмотри Π½Π° свою ΠΊΠΎΠΆΡƒ

Π’Ρ€Π°Ρ‡ ΠΈΠ»ΠΈ мСдицинский пСрсонал ΠΊΠ»ΠΈΠ½ΠΈΠΊΠΈ Π½Π΅ Π±ΡƒΠ΄Π΅Ρ‚ ΠΎΠ±ΡΠ»Π΅Π΄ΠΎΠ²Π°Ρ‚ΡŒ ваши Π³Π΅Π½ΠΈΡ‚Π°Π»ΠΈΠΈ ΠΈΠ»ΠΈ Ρ€Π΅ΠΊΡ‚Π°Π»ΡŒΠ½ΡƒΡŽ ΠΎΠ±Π»Π°ΡΡ‚ΡŒ.Π­Ρ‚ΠΈ части Ρ‚Π΅Π»Π° Π½Π΅ Ρ‚Ρ€Π΅Π±ΡƒΡŽΡ‚ΡΡ для ΠΈΠΌΠΌΠΈΠ³Ρ€Π°Ρ†ΠΈΠΎΠ½Π½ΠΎΠ³ΠΎ мСдицинского осмотра.

Π’ΠΎΠ·ΠΌΠΎΠΆΠ½ΠΎ, Π²Ρ€Π°Ρ‡Ρƒ потрСбуСтся ΠΎΡΠΌΠΎΡ‚Ρ€Π΅Ρ‚ΡŒ Π²Π°ΡˆΡƒ Π³Ρ€ΡƒΠ΄ΡŒ. Если ΠΎΠ½ΠΈ это ΡΠ΄Π΅Π»Π°ΡŽΡ‚, ΠΎΠ½ΠΈ Π±ΡƒΠ΄ΡƒΡ‚:

  • ΠΎΠ±ΡŠΡΡΠ½ΡΡ‚, ΠΏΠΎΡ‡Π΅ΠΌΡƒ ΠΈ ΠΊΠ°ΠΊ проводится экзамСн

Π”Ρ€ΡƒΠ³ΠΈΠ΅ Π²ΠΎΠ·ΠΌΠΎΠΆΠ½Ρ‹Π΅ испытания

Π’ зависимости ΠΎΡ‚ вашСго возраста вас ΠΌΠΎΠ³ΡƒΡ‚ ΠΏΠΎΠΏΡ€ΠΎΡΠΈΡ‚ΡŒ ΡΠ΄Π΅Π»Π°Ρ‚ΡŒ Ρ€Π΅Π½Ρ‚Π³Π΅Π½ Π³Ρ€ΡƒΠ΄Π½ΠΎΠΉ ΠΊΠ»Π΅Ρ‚ΠΊΠΈ ΠΈ Π»Π°Π±ΠΎΡ€Π°Ρ‚ΠΎΡ€Π½Ρ‹Π΅ Π°Π½Π°Π»ΠΈΠ·Ρ‹ Π² ΠΊΠ»ΠΈΠ½ΠΈΠΊΠ΅ ΠΈΠ»ΠΈ Π»Π°Π±ΠΎΡ€Π°Ρ‚ΠΎΡ€ΠΈΠΈ. Π­Ρ‚ΠΎ ΠΎΠ±Ρ‹Ρ‡Π½Ρ‹ΠΉ осмотр, ΠΈ Π²Ρ€Π°Ρ‡ обсудит с Π²Π°ΠΌΠΈ Π»ΡŽΠ±Ρ‹Π΅ отклонСния ΠΎΡ‚ Π½ΠΎΡ€ΠΌΡ‹.

Π’ зависимости ΠΎΡ‚ Ρ€Π΅Π·ΡƒΠ»ΡŒΡ‚Π°Ρ‚ΠΎΠ² мСдицинского осмотра вас ΠΌΠΎΠ³ΡƒΡ‚ Π½Π°ΠΏΡ€Π°Π²ΠΈΡ‚ΡŒ ΠΊ спСциалисту для провСдСния Π΄ΠΎΠΏΠΎΠ»Π½ΠΈΡ‚Π΅Π»ΡŒΠ½Ρ‹Ρ… Π°Π½Π°Π»ΠΈΠ·ΠΎΠ². ΠœΡ‹ просим вас Π·Π°ΠΏΠΎΠ»Π½ΠΈΡ‚ΡŒ этот запрос ΠΊΠ°ΠΊ ΠΌΠΎΠΆΠ½ΠΎ скорСС, Ρ‡Ρ‚ΠΎΠ±Ρ‹ ΠΈΠ·Π±Π΅ΠΆΠ°Ρ‚ΡŒ Π·Π°Π΄Π΅Ρ€ΠΆΠ΅ΠΊ Π² ΠΎΠ±Ρ€Π°Π±ΠΎΡ‚ΠΊΠ΅ вашСго мСдицинского обслСдования.

ΠŸΡ€Π°Π²ΠΎ ΠΈΠΌΠ΅Ρ‚ΡŒ ΡΠΎΠΏΡ€ΠΎΠ²ΠΎΠΆΠ΄Π°ΡŽΡ‰Π΅Π³ΠΎ

Π£ вас Π΅ΡΡ‚ΡŒ ΠΏΡ€Π°Π²ΠΎ Π½Π° сопровоТдСниС Π² любоС врСмя Π²ΠΎ врСмя мСдицинского осмотра.

Π’Ρ‹ ΠΌΠΎΠΆΠ΅Ρ‚Π΅:

  • ΠΏΠΎΠΏΡ€ΠΎΡΠΈΡ‚ΡŒ Π² ΠΏΠΎΠ»ΠΈΠΊΠ»ΠΈΠ½ΠΈΠΊΠ΅ сотрудника Π² ΠΊΠ°Π±ΠΈΠ½Π΅Ρ‚Π΅
  • ΠΎΡΡ‚Π°Π½ΠΎΠ²ΠΈΡ‚ΡŒ обслСдованиС Π² любой ΠΌΠΎΠΌΠ΅Π½Ρ‚, Ρ‡Ρ‚ΠΎΠ±Ρ‹ Π·Π°Π΄Π°Ρ‚ΡŒ вопросы ΠΎ Ρ‚ΠΎΠΌ, Ρ‡Ρ‚ΠΎ Π΄Π΅Π»Π°Π΅Ρ‚ Π²Ρ€Π°Ρ‡
  • ΠΎΡΡ‚Π°Π½ΠΎΠ²ΠΈΡ‚ΡŒ экзамСн ΠΈ ΠΏΠΎΠΏΡ€ΠΎΡΠΈΡ‚ΡŒ ΡΠΎΠΏΡ€ΠΎΠ²ΠΎΠΆΠ΄Π°ΡŽΡ‰Π΅Π³ΠΎ, Π΄Π°ΠΆΠ΅ Ссли Π²Ρ‹ сначала ΠΎΡ‚ΠΊΠ°Π·Π°Π»ΠΈΡΡŒ ΠΎΡ‚ Π½Π΅Π³ΠΎ

Если Ρƒ вас Π΅ΡΡ‚ΡŒ вопросы ΠΈΠ»ΠΈ Π²Ρ‹ чувствуСтС сСбя Π½Π΅ΠΊΠΎΠΌΡ„ΠΎΡ€Ρ‚Π½ΠΎ Π²ΠΎ врСмя части экзамСна, попроситС Π²Ρ€Π°Ρ‡Π° Π³Ρ€ΡƒΠΏΠΏΡ‹ ΠΎΡΡ‚Π°Π½ΠΎΠ²ΠΈΡ‚ΡŒΡΡ ΠΈ Ρ€Π°ΡΡΠΊΠ°Π·Π°Ρ‚ΡŒ Π΅ΠΌΡƒ ΠΎ своих опасСниях.

ПослС сдачи экзамСна

ПослС Ρ‚ΠΎΠ³ΠΎ, ΠΊΠ°ΠΊ обслСдованиС Π±ΡƒΠ΄Π΅Ρ‚ ΠΏΡ€ΠΎΠ²Π΅Π΄Π΅Π½ΠΎ, Π²Ρ€Π°Ρ‡ ΠΎΡ‚ΠΏΡ€Π°Π²ΠΈΡ‚ Π½Π°ΠΌ Ρ€Π΅Π·ΡƒΠ»ΡŒΡ‚Π°Ρ‚Ρ‹. Π’Ρ€Π°Ρ‡ выдаст Π²Π°ΠΌ Π΄ΠΎΠΊΡƒΠΌΠ΅Π½Ρ‚, ΠΏΠΎΠ΄Ρ‚Π²Π΅Ρ€ΠΆΠ΄Π°ΡŽΡ‰ΠΈΠΉ, Ρ‡Ρ‚ΠΎ Π²Ρ‹ ΠΏΡ€ΠΎΡˆΠ»ΠΈ мСдицинскоС обслСдованиС.

НСзависимо ΠΎΡ‚ Ρ‚ΠΎΠ³ΠΎ, ΠΏΡ€ΠΎΡ…ΠΎΠ΄ΠΈΠ»ΠΈ Π»ΠΈ Π²Ρ‹ ΠΏΡ€Π΅Π΄Π²Π°Ρ€ΠΈΡ‚Π΅Π»ΡŒΠ½Ρ‹ΠΉ экзамСн ΠΈΠ»ΠΈ Π½Π΅Ρ‚, сохранитС копию распСчатки, Π²Ρ‹Π΄Π°Π½Π½ΠΎΠΉ Π²Π°ΠΌ Π»Π΅Ρ‡Π°Ρ‰ΠΈΠΌ Π²Ρ€Π°Ρ‡ΠΎΠΌ, Π² качСствС Π΄ΠΎΠΊΠ°Π·Π°Ρ‚Π΅Π»ΡŒΡΡ‚Π²Π° вашСго ΠΈΠΌΠΌΠΈΠ³Ρ€Π°Ρ†ΠΈΠΎΠ½Π½ΠΎΠ³ΠΎ мСдицинского осмотра.

Если Π²Ρ‹ Π½Π΅ ΡƒΠ΄ΠΎΠ²Π»Π΅Ρ‚Π²ΠΎΡ€Π΅Π½Ρ‹ Ρ‚Π΅ΠΌ, ΠΊΠ°ΠΊ Ρ‚Π΅Ρ€Π°ΠΏΠ΅Π²Ρ‚ ΠΈΠ»ΠΈ Ρ€Π΅Π½Ρ‚Π³Π΅Π½ΠΎΠ»ΠΎΠ³ ΠΏΡ€ΠΎΠ²Π΅Π» ваш мСдицинский осмотр, Π²Ρ‹ ΠΌΠΎΠΆΠ΅Ρ‚Π΅:

ΠœΡ‹ привСтствуСм всС ΠΊΠΎΠΌΠΏΠ»ΠΈΠΌΠ΅Π½Ρ‚Ρ‹, ΠΊΠΎΠΌΠΌΠ΅Π½Ρ‚Π°Ρ€ΠΈΠΈ ΠΈΠ»ΠΈ замСчания Ρ‡Π΅Ρ€Π΅Π· Π½Π°ΡˆΡƒ Ρ„ΠΎΡ€ΠΌΡƒ ΠΎΠ±Ρ€Π°Ρ‚Π½ΠΎΠΉ связи.

Когда ΠΎΡ‚ΠΏΡ€Π°Π²Π»ΡΡ‚ΡŒ Ρ€Π΅Π·ΡƒΠ»ΡŒΡ‚Π°Ρ‚Ρ‹ мСдицинского обслСдования

НСзависимо ΠΎΡ‚ Ρ‚ΠΎΠ³ΠΎ, ΠΏΡ€ΠΎΡ…ΠΎΠ΄ΠΈΠ»ΠΈ Π»ΠΈ Π²Ρ‹ ΠΏΡ€Π΅Π΄Π²Π°Ρ€ΠΈΡ‚Π΅Π»ΡŒΠ½ΠΎΠ΅ обслСдованиС ΠΈΠ»ΠΈ Π½Π΅Ρ‚, Π²Ρ€Π°Ρ‡ ΠΎΡ‚ΠΏΡ€Π°Π²ΠΈΡ‚ Π½Π°ΠΌ Ρ€Π΅Π·ΡƒΠ»ΡŒΡ‚Π°Ρ‚Ρ‹.

Π‘ΠΎΡ…Ρ€Π°Π½ΠΈΡ‚Π΅ копию любого Π΄ΠΎΠΊΡƒΠΌΠ΅Π½Ρ‚Π° ΠΈΠ»ΠΈ распСчатку, Π²Ρ‹Π΄Π°Π½Π½ΡƒΡŽ Π²Π°ΠΌ Π²Ρ€Π°Ρ‡ΠΎΠΌ комиссии Π² качСствС Π΄ΠΎΠΊΠ°Π·Π°Ρ‚Π΅Π»ΡŒΡΡ‚Π²Π° вашСго ΠΈΠΌΠΌΠΈΠ³Ρ€Π°Ρ†ΠΈΠΎΠ½Π½ΠΎΠ³ΠΎ мСдицинского осмотра.

Если Π²Ρ‹ ΠΏΡ€ΠΎΡˆΠ»ΠΈ ΠΏΡ€Π΅Π΄Π²Π°Ρ€ΠΈΡ‚Π΅Π»ΡŒΠ½Ρ‹ΠΉ мСдицинский осмотр

Π’Ρ‹, , Π΄ΠΎΠ»ΠΆΠ½Ρ‹ ΠΏΡ€ΠΈΠ»ΠΎΠΆΠΈΡ‚ΡŒ копию Ρ„ΠΎΡ€ΠΌΡ‹ ΠΏΡ€Π΅Π΄Π²Π°Ρ€ΠΈΡ‚Π΅Π»ΡŒΠ½ΠΎΠ³ΠΎ мСдицинского Π·Π°ΠΊΠ»ΡŽΡ‡Π΅Π½ΠΈΡ IMM 1017B, ΠΊΠΎΡ‚ΠΎΡ€ΡƒΡŽ Π²Ρ€Π°Ρ‡ Π΄Π°Π» Π²Π°ΠΌ послС обслСдования. Если Π²Ρ€Π°Ρ‡ Ρ€Π°Π±ΠΎΡ‚Π°Π΅Ρ‚ с eMedical, ΠΎΠ½ выдаст Π²Π°ΠΌ распСчатанный ΠΈΠ½Ρ„ΠΎΡ€ΠΌΠ°Ρ†ΠΈΠΎΠ½Π½Ρ‹ΠΉ лист.

Π’Ρ‹ Π΄ΠΎΠ»ΠΆΠ½Ρ‹ ΠΏΡ€ΠΈΠ»ΠΎΠΆΠΈΡ‚ΡŒ эту Ρ„ΠΎΡ€ΠΌΡƒ ΠΊ своСму Π±ΡƒΠΌΠ°ΠΆΠ½ΠΎΠΌΡƒ заявлСнию. Если Π²Ρ‹ ΠΏΠΎΠ΄Π°Π΅Ρ‚Π΅ заявку ΠΎΠ½Π»Π°ΠΉΠ½, Π²Ρ‹ Π΄ΠΎΠ»ΠΆΠ½Ρ‹ Π·Π°Π³Ρ€ΡƒΠ·ΠΈΡ‚ΡŒ эту Ρ„ΠΎΡ€ΠΌΡƒ, ΠΏΡ€Π΅ΠΆΠ΄Π΅ Ρ‡Π΅ΠΌ смоТСтС ΠΏΠΎΠ΄Π°Ρ‚ΡŒ заявку.

Если Π²Ρ‹ ΠΏΡ€ΠΎΡˆΠ»ΠΈ мСдицинский осмотр послС ΠΏΠΎΠ΄Π°Ρ‡ΠΈ заявлСния

Π‘ вашСй стороны Π½Π΅ трСбуСтся Π½ΠΈΠΊΠ°ΠΊΠΈΡ… Π΄Π°Π»ΡŒΠ½Π΅ΠΉΡˆΠΈΡ… дСйствий.

Как Π΄ΠΎΠ»Π³ΠΎ ваши мСдицинскиС Ρ€Π΅Π·ΡƒΠ»ΡŒΡ‚Π°Ρ‚Ρ‹ ΠΎΡΡ‚Π°ΡŽΡ‚ΡΡ Ρ…ΠΎΡ€ΠΎΡˆΠΈΠΌΠΈ для

Π Π΅Π·ΡƒΠ»ΡŒΡ‚Π°Ρ‚Ρ‹ вашСго мСдицинского обслСдования Π΄Π΅ΠΉΡΡ‚Π²ΠΈΡ‚Π΅Π»ΡŒΠ½Ρ‹ Ρ‚ΠΎΠ»ΡŒΠΊΠΎ Π² Ρ‚Π΅Ρ‡Π΅Π½ΠΈΠ΅ 12 мСсяцСв.

Если Π²Ρ‹ Π½Π΅ ΠΏΡ€ΠΈΠ΅Π΄Π΅Ρ‚Π΅ Π² ΠšΠ°Π½Π°Π΄Ρƒ Π² качСствС посСтитСля, студСнта ΠΈΠ»ΠΈ Ρ€Π°Π±ΠΎΡ‚Π½ΠΈΠΊΠ° Π² Ρ‚Π΅Ρ‡Π΅Π½ΠΈΠ΅ этого Π²Ρ€Π΅ΠΌΠ΅Π½ΠΈ, Π²Π°ΠΌ, Π²ΠΎΠ·ΠΌΠΎΠΆΠ½ΠΎ, придСтся ΡΠ΄Π°Π²Π°Ρ‚ΡŒ Π΅Ρ‰Π΅ ΠΎΠ΄ΠΈΠ½ экзамСн.

ΠŸΠΎΠ»ΡƒΡ‡Π΅Π½ΠΈΠ΅ ΠΊΠΎΠΏΠΈΠΈ вашСго мСдицинского осмотра

Если Π²Π°ΠΌ Π½ΡƒΠΆΠ½Π° копия мСдицинского осмотра, спроситС Ρƒ Π²Ρ€Π°Ρ‡Π°, ΠΊΠΎΠ³Π΄Π° Π²Ρ‹ ΠΏΡ€ΠΈΠ΅Π΄Π΅Ρ‚Π΅.

ΠœΠ΅Π΄ΠΈΡ†ΠΈΠ½ΡΠΊΠΈΠ΅ Π·Π°ΠΊΠ»ΡŽΡ‡Π΅Π½ΠΈΡ ΠΈ рСнтгСновскиС снимки для мСдицинского осмотра становятся нашСй ΡΠΎΠ±ΡΡ‚Π²Π΅Π½Π½ΠΎΡΡ‚ΡŒΡŽ. ΠœΡ‹ Π½Π΅ ΠΌΠΎΠΆΠ΅ΠΌ Π²Π΅Ρ€Π½ΡƒΡ‚ΡŒ ΠΈΡ… Π²Π°ΠΌ.

,

ΠœΠ΅Π΄ΠΈΡ†ΠΈΠ½ΡΠΊΠΈΠΉ Π»ΠΈΡ†Π΅Π½Π·ΠΈΠΎΠ½Π½Ρ‹ΠΉ экзамСн Π² БША

Π£Π²Π΅Π΄ΠΎΠΌΠ»Π΅Π½ΠΈΠ΅ ΠΎ ΠΊΠΎΠ½Ρ„ΠΈΠ΄Π΅Π½Ρ†ΠΈΠ°Π»ΡŒΠ½ΠΎΡΡ‚ΠΈ рСгистрации с использованиСм биомСтричСских Π΄Π°Π½Π½Ρ‹Ρ…

Для ΠΊΠ°Π½Π΄ΠΈΠ΄Π°Ρ‚ΠΎΠ² USMLE


БиомСтрия являСтся Ρ‡Π°ΡΡ‚ΡŒΡŽ систСмы управлСния ΠΈΠ΄Π΅Π½Ρ‚ΠΈΡ„ΠΈΠΊΠ°Ρ†ΠΈΠ΅ΠΉ Prometric ΠΈ ΠΏΡ€Π΅Π΄Π½Π°Π·Π½Π°Ρ‡Π΅Π½Π° для ΠΏΠΎΠ²Ρ‹ΡˆΠ΅Π½ΠΈΡ бСзопасности ΠΈ цСлостности процСсса тСстирования. ΠŸΡ€ΠΎΠ³Ρ€Π°ΠΌΠΌΠ° USMLE ΠΈΡΠΏΠΎΠ»ΡŒΠ·ΡƒΠ΅Ρ‚ Ρ„ΡƒΠ½ΠΊΡ†ΠΈΡŽ рСгистрации с использованиСм биомСтричСских Π΄Π°Π½Π½Ρ‹Ρ… ΠΊΠ°ΠΊ Ρ‡Π°ΡΡ‚ΡŒ стандартных ΠΏΡ€ΠΎΡ†Π΅Π΄ΡƒΡ€ тСстового дня. Π­Ρ‚Π° ΠΏΡ€ΠΎΠ³Ρ€Π°ΠΌΠΌΠ° Π±ΠΈΠΎΠΌΠ΅Ρ‚Ρ€ΠΈΠΈ рСализуСтся ΠΊΠΎΠΌΠΏΠ°Π½ΠΈΠ΅ΠΉ Prometric (ΡˆΡ‚Π°Π±-ΠΊΠ²Π°Ρ€Ρ‚ΠΈΡ€Π° ΠΊΠΎΡ‚ΠΎΡ€ΠΎΠΉ располоТСна ΠΏΠΎ адрСсу 1501 South Clinton Street, Baltimore, MD 21224, U.S.A.), которая дСйствуСт ΠΎΡ‚ ΠΈΠΌΠ΅Π½ΠΈ спонсоров ΠΏΡ€ΠΎΠ³Ρ€Π°ΠΌΠΌΡ‹ USMLE Π² качСствС ΠΎΠ±Ρ€Π°Π±ΠΎΡ‚Ρ‡ΠΈΠΊΠ° Π΄Π°Π½Π½Ρ‹Ρ….

Π’ этом Π£Π²Π΅Π΄ΠΎΠΌΠ»Π΅Π½ΠΈΠΈ ΠΎ ΠΊΠΎΠ½Ρ„ΠΈΠ΄Π΅Π½Ρ†ΠΈΠ°Π»ΡŒΠ½ΠΎΡΡ‚ΠΈ ΠΎΠΏΠΈΡΡ‹Π²Π°ΡŽΡ‚ΡΡ ΠΌΠ΅Ρ‚ΠΎΠ΄Ρ‹ обСспСчСния ΠΊΠΎΠ½Ρ„ΠΈΠ΄Π΅Π½Ρ†ΠΈΠ°Π»ΡŒΠ½ΠΎΡΡ‚ΠΈ ΠΈ бСзопасности, установлСнныС для ΠΏΡ€ΠΎΠ³Ρ€Π°ΠΌΠΌΡ‹ рСгистрации с использованиСм биомСтричСских Π΄Π°Π½Π½Ρ‹Ρ….

О рСгистрации с использованиСм биомСтричСских Π΄Π°Π½Π½Ρ‹Ρ…

ΠŸΡ€ΠΎΠ³Ρ€Π°ΠΌΠΌΠ° Π±ΠΈΠΎΠΌΠ΅Ρ‚Ρ€ΠΈΠΈ ΠΏΡ€Π΅ΠΎΠ±Ρ€Π°Π·ΡƒΠ΅Ρ‚ ΠΎΡ‚ΠΏΠ΅Ρ‡Π°Ρ‚ΠΎΠΊ ΠΏΠ°Π»ΡŒΡ†Π° Π² Ρ†ΠΈΡ„Ρ€ΠΎΠ²ΠΎΠ΅ ΠΈΠ·ΠΎΠ±Ρ€Π°ΠΆΠ΅Π½ΠΈΠ΅, ΠΊΠΎΡ‚ΠΎΡ€ΠΎΠ΅ ΠΈΡΠΏΠΎΠ»ΡŒΠ·ΡƒΠ΅Ρ‚ΡΡ для ΠΏΡ€ΠΎΠ²Π΅Ρ€ΠΊΠΈ личности. Π’ΠΎ врСмя рСгистрации с ΠΏΠΎΠ΄Π΄Π΅Ρ€ΠΆΠΊΠΎΠΉ биомСтричСских Π΄Π°Π½Π½Ρ‹Ρ… Π²Ρ‹ ΠΏΡ€ΠΈΠ»ΠΎΠΆΠΈΡ‚Π΅ ΠΏΠ°Π»Π΅Ρ† ΠΊ сканСру Π² Ρ†Π΅Π½Ρ‚Ρ€Π΅ тСстирования.ΠžΠ±ΠΎΡ€ΡƒΠ΄ΠΎΠ²Π°Π½ΠΈΠ΅ создаст ΠΎΡ†ΠΈΡ„Ρ€ΠΎΠ²Π°Π½Π½ΠΎΠ΅ ΠΈΠ·ΠΎΠ±Ρ€Π°ΠΆΠ΅Π½ΠΈΠ΅ вашСго ΠΎΡ‚ΠΏΠ΅Ρ‡Π°Ρ‚ΠΊΠ° ΠΏΠ°Π»ΡŒΡ†Π° («шаблон»). Π­Ρ‚ΠΎΡ‚ шаблон прСдставлСния Π±ΡƒΠ΄Π΅Ρ‚ связан с Π΄Ρ€ΡƒΠ³ΠΎΠΉ Π»ΠΈΡ‡Π½ΠΎΠΉ ΠΈΠ½Ρ„ΠΎΡ€ΠΌΠ°Ρ†ΠΈΠ΅ΠΉ, ΠΊΠΎΡ‚ΠΎΡ€ΡƒΡŽ Π²Ρ‹ прСдоставляСтС своСй Ρ€Π΅Π³ΠΈΡΡ‚Ρ€ΠΈΡ€ΡƒΡŽΡ‰Π΅ΠΉ ΠΎΡ€Π³Π°Π½ΠΈΠ·Π°Ρ†ΠΈΠΈ, Ρ‡Ρ‚ΠΎ ΠΏΠΎΠ·Π²ΠΎΠ»ΠΈΡ‚ Ρ‚ΠΎΡ‡Π½ΠΎ ΠΈΠ΄Π΅Π½Ρ‚ΠΈΡ„ΠΈΡ†ΠΈΡ€ΠΎΠ²Π°Ρ‚ΡŒ вас Π² процСссС тСстирования. Входя ΠΈ выходя ΠΈΠ· Π»Π°Π±ΠΎΡ€Π°Ρ‚ΠΎΡ€ΠΈΠΈ тСстирования Π² Ρ†Π΅Π½Ρ‚Ρ€Π΅ тСстирования, Π²Ρ‹ Π±ΡƒΠ΄Π΅Ρ‚Π΅ ΠΈΡΠΏΠΎΠ»ΡŒΠ·ΠΎΠ²Π°Ρ‚ΡŒ свой ΠΏΠ°Π»Π΅Ρ† для Π°ΡƒΡ‚Π΅Π½Ρ‚ΠΈΡ„ΠΈΠΊΠ°Ρ†ΠΈΠΈ Π½Π° сканСрах, располоТСнных Π² Ρ†Π΅Π½Ρ‚Ρ€Π΅ тСстирования.

ΠŸΠ΅Ρ€ΡΠΎΠ½Π°Π»ΡŒΠ½Π°Ρ информация, собранная Π² Ρ€Π°ΠΌΠΊΠ°Ρ… ΠΏΡ€ΠΎΠ³Ρ€Π°ΠΌΠΌΡ‹ Π±ΠΈΠΎΠΌΠ΅Ρ‚Ρ€ΠΈΠΈ

БиомСтричСская запись содСрТит Ρ‡Π°ΡΡ‚ΡŒ ΠΈΠ»ΠΈ всю ΡΠ»Π΅Π΄ΡƒΡŽΡ‰ΡƒΡŽ ΠΈΠ½Ρ„ΠΎΡ€ΠΌΠ°Ρ†ΠΈΡŽ: Ρ†ΠΈΡ„Ρ€ΠΎΠ²ΠΎΠ΅ прСдставлСниС вашСго ΠΎΡ‚ΠΏΠ΅Ρ‡Π°Ρ‚ΠΊΠ° ΠΏΠ°Π»ΡŒΡ†Π° (шаблон), ΠΈΠ·ΠΎΠ±Ρ€Π°ΠΆΠ΅Π½ΠΈΠ΅ вашСго ΠΎΡ‚ΠΏΠ΅Ρ‡Π°Ρ‚ΠΊΠ° ΠΏΠ°Π»ΡŒΡ†Π°, Π° Ρ‚Π°ΠΊΠΆΠ΅ вашС имя, адрСс, Π½ΠΎΠΌΠ΅Ρ€ Ρ‚Π΅Π»Π΅Ρ„ΠΎΠ½Π° (Ссли прСдоставлСн), Π΄Π°Ρ‚Ρƒ роТдСния ΠΈΠ»ΠΈ возраст, Π½ΠΎΠΌΠ΅Ρ€ Π΄ΠΎΠΊΡƒΠΌΠ΅Π½Ρ‚Π°, ΡƒΠ΄ΠΎΡΡ‚ΠΎΠ²Π΅Ρ€ΡΡŽΡ‰Π΅Π³ΠΎ Π»ΠΈΡ‡Π½ΠΎΡΡ‚ΡŒ, ΠΈ ΠΎΡ‚ΡΠΊΠ°Π½ΠΈΡ€ΠΎΠ²Π°Π½Π½ΡƒΡŽ копию вашСго Π΄ΠΎΠΊΡƒΠΌΠ΅Π½Ρ‚Π°, ΡƒΠ΄ΠΎΡΡ‚ΠΎΠ²Π΅Ρ€ΡΡŽΡ‰Π΅Π³ΠΎ Π»ΠΈΡ‡Π½ΠΎΡΡ‚ΡŒ (Π½Π°ΠΏΡ€ΠΈΠΌΠ΅Ρ€, Π²ΠΎΠ΄ΠΈΡ‚Π΅Π»ΡŒΡΠΊΠΈΠ΅ ΠΏΡ€Π°Π²Π°), ΠΊΠΎΡ‚ΠΎΡ€Ρ‹ΠΉ Π²ΠΊΠ»ΡŽΡ‡Π°Π΅Ρ‚ Π²Π°ΡˆΡƒ ΠΎΡ†ΠΈΡ„Ρ€ΠΎΠ²Π°Π½Π½ΡƒΡŽ Ρ„ΠΎΡ‚ΠΎΠ³Ρ€Π°Ρ„ΠΈΡŽ.Π—Π°ΠΏΠΈΡΡŒ биомСтричСских Π΄Π°Π½Π½Ρ‹Ρ… сопряТСна с ΠΈΠ½Ρ„ΠΎΡ€ΠΌΠ°Ρ†ΠΈΠ΅ΠΉ ΠΎΠ± ΠΈΠ΄Π΅Π½Ρ‚ΠΈΡ„ΠΈΠΊΠ°Ρ‚ΠΎΡ€Π΅ ΠΈ ΠΈΠ½Ρ„ΠΎΡ€ΠΌΠ°Ρ†ΠΈΠ΅ΠΉ ΠΎ сСансС назначСния тСстирования.

Π¦Π΅Π»ΠΈ ΠΈ использованиС Π»ΠΈΡ‡Π½ΠΎΠΉ ΠΈΠ½Ρ„ΠΎΡ€ΠΌΠ°Ρ†ΠΈΠΈ

Π˜ΡΠΏΠΎΠ»ΡŒΠ·ΡƒΠ΅Ρ‚ΡΡ личная информация, собранная Π² Ρ€Π°ΠΌΠΊΠ°Ρ… БиомСтричСской ΠΏΡ€ΠΎΠ³Ρ€Π°ΠΌΠΌΡ‹:

  1. Для постоянного подтвСрТдСния вашСй личности Π² Ρ‚Π΅Ρ‡Π΅Π½ΠΈΠ΅ всСго тСстового дня
  2. Для выявлСния ΠΈ прСдотвращСния ΠΌΠΎΡˆΠ΅Π½Π½ΠΈΡ‡Π΅ΡΡ‚Π²Π° ΠΏΡ€ΠΈ тСстировании ΠΈ поддСрТания цСлостности процСсса тСстирования ΠΏΡƒΡ‚Π΅ΠΌ обнаруТСния ΠΈ прСдотвращСния сдачи тСстов Π½Π΅ΡƒΠΏΠΎΠ»Π½ΠΎΠΌΠΎΡ‡Π΅Π½Π½Ρ‹ΠΌΠΈ Π»ΠΈΡ†Π°ΠΌΠΈ
  3. Для ΠΏΠΎΠ²Ρ‹ΡˆΠ΅Π½ΠΈΡ бСзопасности Π² ΠΈΡΠΏΡ‹Ρ‚Π°Ρ‚Π΅Π»ΡŒΠ½Ρ‹Ρ… Ρ†Π΅Π½Ρ‚Ρ€Π°Ρ… ΠΏΡƒΡ‚Π΅ΠΌ обнаруТСния ΠΈ прСдотвращСния нСсанкционированного доступа Π»ΠΈΡ† Π² охраняСмыС Π·ΠΎΠ½Ρ‹
  4. Π’ соотвСтствии с трСбованиями Π·Π°ΠΊΠΎΠ½Π°

ΠŸΠ΅Ρ€ΡΠΎΠ½Π°Π»ΡŒΠ½Π°Ρ информация, раскрытая Ρ‚Ρ€Π΅Ρ‚ΡŒΠΈΠΌ Π»ΠΈΡ†Π°ΠΌ

Π’Π°ΡˆΠ° личная информация (Π²ΠΊΠ»ΡŽΡ‡Π°Ρ шаблон ΠΎΡ‚ΠΏΠ΅Ρ‡Π°Ρ‚ΠΊΠ° ΠΏΠ°Π»ΡŒΡ†Π°) ΠΌΠΎΠΆΠ΅Ρ‚ Π±Ρ‹Ρ‚ΡŒ прСдоставлСна β€‹β€‹ΡΠ»Π΅Π΄ΡƒΡŽΡ‰ΠΈΠΌ Π»ΠΈΡ†Π°ΠΌ:

  • ΠžΠ±Ρ€Π°Π±ΠΎΡ‚Ρ‡ΠΈΠΊΠΈ Π΄Π°Π½Π½Ρ‹Ρ…, Ρ‚Π°ΠΊΠΈΠ΅ ΠΊΠ°ΠΊ Prometric, ΠΊΠΎΡ‚ΠΎΡ€Ρ‹Π΅ ΠΎΠ±Ρ€Π°Π±Π°Ρ‚Ρ‹Π²Π°ΡŽΡ‚ Π΄Π°Π½Π½Ρ‹Π΅ ΠΈ ΠΊΠΎΡ‚ΠΎΡ€Ρ‹Π΅ связаны ΠΊΠΎΠ½Ρ‚Ρ€Π°ΠΊΡ‚Π°ΠΌΠΈ, ΠΎΠ³Ρ€Π°Π½ΠΈΡ‡ΠΈΠ²Π°ΡŽΡ‚ использованиС ΠΈΠ½Ρ„ΠΎΡ€ΠΌΠ°Ρ†ΠΈΠΈ цСлями, ΡƒΠΊΠ°Π·Π°Π½Π½Ρ‹ΠΌΠΈ Π² этом ΡƒΠ²Π΅Π΄ΠΎΠΌΠ»Π΅Π½ΠΈΠΈ
  • Π’Ρ€Π΅Ρ‚ΡŒΠΈ стороны, Ссли это трСбуСтся ΠΏΠΎ Π·Π°ΠΊΠΎΠ½Ρƒ ΠΈΠ»ΠΈ ΠΏΠΎ ΠΌΠ΅Ρ€Π΅ нСобходимости для Π·Π°Π²Π΅Ρ€ΡˆΠ΅Π½ΠΈΡ расслСдования ΠΌΠΎΡˆΠ΅Π½Π½ΠΈΡ‡Π΅ΡΡ‚Π²Π° ΠΏΡ€ΠΈ тСстировании, нСпосрСдствСнно связанного с ΡΠΎΠΎΡ‚Π²Π΅Ρ‚ΡΡ‚Π²ΡƒΡŽΡ‰ΠΈΠΌ ΠΊΠ°Π½Π΄ΠΈΠ΄Π°Ρ‚ΠΎΠΌ (Π°ΠΌΠΈ).

Π₯Ρ€Π°Π½Π΅Π½ΠΈΠ΅ ΠΈΠ½Ρ„ΠΎΡ€ΠΌΠ°Ρ†ΠΈΠΈ

Если ΠΈΠ½ΠΎΠ΅ Π½Π΅ прСдусмотрСно Π·Π°ΠΊΠΎΠ½ΠΎΠΌ, личная информация (Π²ΠΊΠ»ΡŽΡ‡Π°Ρ ΠΈΠ·ΠΎΠ±Ρ€Π°ΠΆΠ΅Π½ΠΈΠ΅ вашСго ΠΎΡ‚ΠΏΠ΅Ρ‡Π°Ρ‚ΠΊΠ° ΠΏΠ°Π»ΡŒΡ†Π° ΠΈ шаблон), собранная с ΠΏΠΎΠΌΠΎΡ‰ΡŒΡŽ биомСтричСских Π΄Π°Π½Π½Ρ‹Ρ…, сохраняСтся ΠΎΡ‚ ΠΈΠΌΠ΅Π½ΠΈ спонсоров ΠΏΡ€ΠΎΠ³Ρ€Π°ΠΌΠΌΡ‹ USMLE Π² бСзопасной Ρ†Π΅Π½Ρ‚Ρ€Π°Π»ΠΈΠ·ΠΎΠ²Π°Π½Π½ΠΎΠΉ Π±Π°Π·Π΅ Π΄Π°Π½Π½Ρ‹Ρ… Π² БША Π² дСнь тСстирования ΠΈ Π² Ρ‚Π΅Ρ‡Π΅Π½ΠΈΠ΅ 5 Π»Π΅Ρ‚. с Π΄Π°Ρ‚Ρ‹ вашСго послСднСго посСщСния для тСстирования Π² ΠΊΠΎΠΌΠΏΠ°Π½ΠΈΠΈ Prometric. По истСчСнии этого ΠΏΠ΅Ρ€ΠΈΠΎΠ΄Π° Π²Ρ€Π΅ΠΌΠ΅Π½ΠΈ вся дСмографичСская информация, собранная с использованиСм биомСтричСских Π΄Π°Π½Π½Ρ‹Ρ…, Π° Ρ‚Π°ΠΊΠΆΠ΅ ΠΈΠ·ΠΎΠ±Ρ€Π°ΠΆΠ΅Π½ΠΈΠ΅ ΠΎΡ‚ΠΏΠ΅Ρ‡Π°Ρ‚ΠΊΠ° ΠΏΠ°Π»ΡŒΡ†Π° ΠΈ шаблон ΡƒΠ΄Π°Π»ΡΡŽΡ‚ΡΡ.Π₯отя Π² БША ΠΈ ΠΌΠ½ΠΎΠ³ΠΈΡ… Π΄Ρ€ΡƒΠ³ΠΈΡ… странах Π½Π΅Ρ‚ Π²ΡΠ΅ΠΎΠ±ΡŠΠ΅ΠΌΠ»ΡŽΡ‰ΠΈΡ… Π·Π°ΠΊΠΎΠ½ΠΎΠ² ΠΎ Π·Π°Ρ‰ΠΈΡ‚Π΅ Π΄Π°Π½Π½Ρ‹Ρ…, спонсоры USMLE ΠΈ Prometric Π±ΡƒΠ΄ΡƒΡ‚ ΠΎΠ±Ρ€Π°Π±Π°Ρ‚Ρ‹Π²Π°Ρ‚ΡŒ Π²Π°ΡˆΡƒ Π»ΠΈΡ‡Π½ΡƒΡŽ ΠΈΠ½Ρ„ΠΎΡ€ΠΌΠ°Ρ†ΠΈΡŽ Ρ‚Π°ΠΊΠΈΠΌ ΠΎΠ±Ρ€Π°Π·ΠΎΠΌ, Ρ‡Ρ‚ΠΎΠ±Ρ‹ ΠΎΠ½Π° ΠΏΠΎΠ»ΡƒΡ‡ΠΈΠ»Π° Π½Π°Π΄Π»Π΅ΠΆΠ°Ρ‰ΡƒΡŽ Π·Π°Ρ‰ΠΈΡ‚Ρƒ посрСдством ΠΊΠΎΠ½Ρ‚Ρ€Π°ΠΊΡ‚ΠΎΠ² Π½Π° ΠΏΠ΅Ρ€Π΅Π΄Π°Ρ‡Ρƒ Π΄Π°Π½Π½Ρ‹Ρ… ΠΈ / ΠΈΠ»ΠΈ сСртификации Π² Ρ€Π°ΠΌΠΊΠ°Ρ… ΠΏΡ€ΠΎΠ³Ρ€Π°ΠΌΠΌΡ‹ Safe Harbor ΠΌΠ΅ΠΆΠ΄Ρƒ Π•Π‘ ΠΈ БША.

ΠšΠΎΠ½Ρ„ΠΈΠ΄Π΅Π½Ρ†ΠΈΠ°Π»ΡŒΠ½ΠΎΡΡ‚ΡŒ ΠΈ Π±Π΅Π·ΠΎΠΏΠ°ΡΠ½ΠΎΡΡ‚ΡŒ ΠΈΠ½Ρ„ΠΎΡ€ΠΌΠ°Ρ†ΠΈΠΈ

Π‘Ρ‹Π»ΠΈ Ρ€Π΅Π°Π»ΠΈΠ·ΠΎΠ²Π°Π½Ρ‹ ΡΠΎΠΎΡ‚Π²Π΅Ρ‚ΡΡ‚Π²ΡƒΡŽΡ‰ΠΈΠ΅ тСхничСскиС, физичСскиС ΠΈ административныС ΠΌΠ΅Ρ€Ρ‹ бСзопасности, Ρ‡Ρ‚ΠΎΠ±Ρ‹ ΠΏΠΎΠΌΠΎΡ‡ΡŒ Π·Π°Ρ‰ΠΈΡ‚ΠΈΡ‚ΡŒ Π²Π°ΡˆΡƒ Π»ΠΈΡ‡Π½ΡƒΡŽ ΠΈΠ½Ρ„ΠΎΡ€ΠΌΠ°Ρ†ΠΈΡŽ ΠΎΡ‚ нСсанкционированного доступа ΠΈΠ»ΠΈ ΠΏΠΎΡ‚Π΅Ρ€ΠΈ.Доступ ΠΊ ΠΈΠ½Ρ„ΠΎΡ€ΠΌΠ°Ρ†ΠΈΠΈ, собранной Π² Ρ€Π°ΠΌΠΊΠ°Ρ… ΠΏΡ€ΠΎΠ³Ρ€Π°ΠΌΠΌΡ‹ рСгистрации с использованиСм биомСтричСских Π΄Π°Π½Π½Ρ‹Ρ…, ΠΎΠ³Ρ€Π°Π½ΠΈΡ‡Π΅Π½ Ρ‚Π΅ΠΌΠΈ сотрудниками ΠΈΠ»ΠΈ Π°Π³Π΅Π½Ρ‚Π°ΠΌΠΈ спонсоров ΠΏΡ€ΠΎΠ³Ρ€Π°ΠΌΠΌΡ‹ USMLE, ΠΊΠΎΡ‚ΠΎΡ€Ρ‹Π΅ ΠΈΠΌΠ΅ΡŽΡ‚ ΠΊ Π½Π΅ΠΉ доступ ΠΈ ΠΊΠΎΡ‚ΠΎΡ€Ρ‹Π΅ связаны соглашСниями ΠΎ ΠΊΠΎΠ½Ρ„ΠΈΠ΄Π΅Π½Ρ†ΠΈΠ°Π»ΡŒΠ½ΠΎΡΡ‚ΠΈ.

Бпонсоры ΠΏΡ€ΠΎΠ³Ρ€Π°ΠΌΠΌΡ‹ USMLE ΠΈ Prometric стрСмятся Π·Π°Ρ‰ΠΈΡ‰Π°Ρ‚ΡŒ Π²Π°ΡˆΡƒ ΠΊΠΎΠ½Ρ„ΠΈΠ΄Π΅Π½Ρ†ΠΈΠ°Π»ΡŒΠ½ΠΎΡΡ‚ΡŒ ΠΈ Π±ΡƒΠ΄ΡƒΡ‚ ΠΈΡΠΏΠΎΠ»ΡŒΠ·ΠΎΠ²Π°Ρ‚ΡŒ Π»ΠΈΡ‡Π½ΡƒΡŽ ΠΈΠ½Ρ„ΠΎΡ€ΠΌΠ°Ρ†ΠΈΡŽ, ΡΠΎΠ±Ρ€Π°Π½Π½ΡƒΡŽ Π² Ρ€Π°ΠΌΠΊΠ°Ρ… ΠΏΡ€ΠΎΠ³Ρ€Π°ΠΌΠΌΡ‹ рСгистрации с ΠΏΠΎΠ΄Π΄Π΅Ρ€ΠΆΠΊΠΎΠΉ биомСтричСских Π΄Π°Π½Π½Ρ‹Ρ…, Ρ‚ΠΎΠ»ΡŒΠΊΠΎ для Ρ†Π΅Π»Π΅ΠΉ, описанных Π² этом Π£Π²Π΅Π΄ΠΎΠΌΠ»Π΅Π½ΠΈΠΈ ΠΎ ΠΊΠΎΠ½Ρ„ΠΈΠ΄Π΅Π½Ρ†ΠΈΠ°Π»ΡŒΠ½ΠΎΡΡ‚ΠΈ.

Вопросы

Если Ρƒ вас Π΅ΡΡ‚ΡŒ сомнСния ΠΏΠΎ ΠΏΠΎΠ²ΠΎΠ΄Ρƒ ΠΏΡ€ΠΎΠ³Ρ€Π°ΠΌΠΌΡ‹ рСгистрации с использованиСм биомСтричСских Π΄Π°Π½Π½Ρ‹Ρ…, ΡΠ²ΡΠΆΠΈΡ‚Π΅ΡΡŒ с Π½Π°ΠΌΠΈ.


.

ΠœΠ΅Π΄ΠΈΡ†ΠΈΠ½ΡΠΊΠΈΠΉ осмотр для заявитСлСй Π½Π° постоянноС мСсто ΠΆΠΈΡ‚Π΅Π»ΡŒΡΡ‚Π²Π°

Π’Ρ‹ Π΄ΠΎΠ»ΠΆΠ½Ρ‹ ΠΏΠΎΡΠ΅Ρ‚ΠΈΡ‚ΡŒ Π²Ρ€Π°Ρ‡Π° ΠΈΠ· списка Ρ‚Π΅Ρ€Π°ΠΏΠ΅Π²Ρ‚ΠΎΠ². Π’Π°Ρˆ собствСнный Π²Ρ€Π°Ρ‡ Π½Π΅ ΠΌΠΎΠΆΠ΅Ρ‚ ΠΏΡ€ΠΎΠΉΡ‚ΠΈ мСдицинский осмотр.

Π’Ρ€Π°Ρ‡ Π½Π΅ ΠΏΡ€ΠΈΠ½ΠΈΠΌΠ°Π΅Ρ‚ ΠΎΠΊΠΎΠ½Ρ‡Π°Ρ‚Π΅Π»ΡŒΠ½ΠΎΠ΅ Ρ€Π΅ΡˆΠ΅Π½ΠΈΠ΅ ΠΎ вашСм мСдицинском осмотрС. ΠœΡ‹ ΠΏΡ€ΠΈΠ½ΠΈΠΌΠ°Π΅ΠΌ это Ρ€Π΅ΡˆΠ΅Π½ΠΈΠ΅. Если Π²ΠΎΠ·Π½ΠΈΠΊΠ½ΡƒΡ‚ ΠΏΡ€ΠΎΠ±Π»Π΅ΠΌΡ‹ с мСдицинским осмотром, ΠΌΡ‹ свяТСмся с Π²Π°ΠΌΠΈ Π² письмСнной Ρ„ΠΎΡ€ΠΌΠ΅.

НайдитС Π²Ρ€Π°Ρ‡Π°, ΠΊΠΎΡ‚ΠΎΡ€Ρ‹ΠΉ ΠΏΡ€ΠΎΠ²Π΅Π΄Π΅Ρ‚ экзамСн.

Когда ΠΏΡ€ΠΎΡ…ΠΎΠ΄ΠΈΡ‚ΡŒ мСдицинский осмотр

Π’Ρ‹ ΠΌΠΎΠΆΠ΅Ρ‚Π΅ ΠΏΡ€ΠΎΠΉΡ‚ΠΈ мСдицинский осмотр Π΄ΠΎ ΠΈΠ»ΠΈ послС ΠΏΠΎΠ΄Π°Ρ‡ΠΈ заявлСния, Π² зависимости ΠΎΡ‚ ΠΏΡ€ΠΎΠ³Ρ€Π°ΠΌΠΌΡ‹, Π½Π° ΠΊΠΎΡ‚ΠΎΡ€ΡƒΡŽ Π²Ρ‹ ΠΏΠΎΠ΄Π°Π΅Ρ‚Π΅ заявлСниС.ΠŸΡ€ΠΎΡ‡Ρ‚ΠΈΡ‚Π΅ ΠΈΠ½ΡΡ‚Ρ€ΡƒΠΊΡ†ΠΈΡŽ ΠΊ вашСй ΠΏΡ€ΠΎΠ³Ρ€Π°ΠΌΠΌΠ΅, Ρ‡Ρ‚ΠΎΠ±Ρ‹ ΡƒΠ·Π½Π°Ρ‚ΡŒ, ΠΊΠΎΠ³Π΄Π° ΡΠ΄Π°Π²Π°Ρ‚ΡŒ экзамСн.

ΠŸΠ΅Ρ€Π΅Π΄ ΠΏΠΎΠ΄Π°Ρ‡Π΅ΠΉ заявки

Если Π²Ρ‹ ΠΏΠΎΠ΄Π°Π΅Ρ‚Π΅ заявлСниС ΠΏΠΎ ΠΏΡ€ΠΎΠ³Ρ€Π°ΠΌΠΌΠ΅ Express Entry, Ρƒ вас Π΅ΡΡ‚ΡŒ Π²ΠΎΠ·ΠΌΠΎΠΆΠ½ΠΎΡΡ‚ΡŒ ΡΠ΄Π°Ρ‚ΡŒ экзамСн Π΄ΠΎ ΠΏΠΎΠ΄Π°Ρ‡ΠΈ заявлСния. Π­Ρ‚ΠΎ называСтся ΠΏΡ€Π΅Π΄Π²Π°Ρ€ΠΈΡ‚Π΅Π»ΡŒΠ½Ρ‹ΠΌ мСдицинским осмотром. Π§Ρ‚ΠΎΠ±Ρ‹ ΠΏΠΎΠ»ΡƒΡ‡ΠΈΡ‚ΡŒ Π΅Π³ΠΎ, ΠΎΠ±Ρ€Π°Ρ‚ΠΈΡ‚Π΅ΡΡŒ нСпосрСдствСнно ΠΊ Π»Π΅Ρ‡Π°Ρ‰Π΅ΠΌΡƒ Π²Ρ€Π°Ρ‡Ρƒ.

Π’Ρ‹ Π½Π΅ ΠΌΠΎΠΆΠ΅Ρ‚Π΅ ΠΏΡ€ΠΎΠΉΡ‚ΠΈ мСдицинскоС обслСдованиС Π·Π°Ρ€Π°Π½Π΅Π΅, Ссли вас ΡΠΏΠΎΠ½ΡΠΈΡ€ΡƒΡŽΡ‚ ΠΊΠ°ΠΊ супруг, ΠΏΠ°Ρ€Ρ‚Π½Π΅Ρ€ ΠΈΠ»ΠΈ Ρ€Π΅Π±Π΅Π½ΠΎΠΊ. ΠŸΡ€Π΅ΠΆΠ΄Π΅ Ρ‡Π΅ΠΌ ΠΈΠ΄Ρ‚ΠΈ Π½Π° мСдицинский осмотр, Π΄ΠΎΠΆΠ΄ΠΈΡ‚Π΅ΡΡŒ Π½Π°ΡˆΠΈΡ… инструкций.

ПослС ΠΏΠΎΠ΄Π°Ρ‡ΠΈ заявки

ΠœΡ‹ ΠΏΡ€ΠΈΡˆΠ»Π΅ΠΌ Π²Π°ΠΌ инструкции ΠΎ Ρ‚ΠΎΠΌ, ΠΊΠ°ΠΊ ΠΏΡ€ΠΎΠΉΡ‚ΠΈ мСдицинский осмотр. Π’Ρ‹ Π΄ΠΎΠ»ΠΆΠ½Ρ‹ ΠΏΡ€ΠΎΠΉΡ‚ΠΈ мСдицинский осмотр Π² Ρ‚Π΅Ρ‡Π΅Π½ΠΈΠ΅ 30 Π΄Π½Π΅ΠΉ послС получСния этих инструкций.

Если Π²Ρ‹ ΠΏΠΎΠ΄Π°Π΅Ρ‚Π΅ заявлСниС ΠΎ прСдоставлСнии статуса Π±Π΅ΠΆΠ΅Π½Ρ†Π° Π² ΠΏΡƒΠ½ΠΊΡ‚Π΅ въСзда, сотрудник ΠΏΠΎΠ³Ρ€Π°Π½ΠΈΡ‡Π½ΠΎΠΉ слуТбы скаТСт Π²Π°ΠΌ ΠΏΡ€ΠΎΠΉΡ‚ΠΈ мСдицинский осмотр Π² Ρ‚Π΅Ρ‡Π΅Π½ΠΈΠ΅ 30 Π΄Π½Π΅ΠΉ.

Если Π²Ρ‹ Π½Π΅ Π²Ρ‹ΠΏΠΎΠ»Π½ΠΈΡ‚Π΅ эти инструкции, ΠΌΡ‹ ΠΌΠΎΠΆΠ΅ΠΌ ΠΎΡ‚ΠΊΠ»ΠΎΠ½ΠΈΡ‚ΡŒ Π²Π°ΡˆΡƒ заявку.

Π§Ρ‚ΠΎ Π±Ρ€Π°Ρ‚ΡŒ с собой

Когда Π²Ρ‹ ΠΈΠ΄Π΅Ρ‚Π΅ Π½Π° ΠΏΡ€ΠΈΠ΅ΠΌ для мСдицинского осмотра, Π²Ρ‹ Π΄ΠΎΠ»ΠΆΠ½Ρ‹ принСсти:

  • надлСТащая идСнтификация — ΠΊΠ°ΠΊ ΠΌΠΈΠ½ΠΈΠΌΡƒΠΌ 1 Π΄ΠΎΠΊΡƒΠΌΠ΅Π½Ρ‚ государствСнного ΠΎΠ±Ρ€Π°Π·Ρ†Π° с вашСй Ρ„ΠΎΡ‚ΠΎΠ³Ρ€Π°Ρ„ΠΈΠ΅ΠΉ ΠΈ подписью, Π½Π°ΠΏΡ€ΠΈΠΌΠ΅Ρ€, паспорт ΠΈΠ»ΠΈ Π½Π°Ρ†ΠΈΠΎΠ½Π°Π»ΡŒΠ½ΠΎΠ΅ удостовСрСниС личности
    • Π’Ρ‹ Ρ‚Π°ΠΊΠΆΠ΅ ΠΌΠΎΠΆΠ΅Ρ‚Π΅ ΠΈΡΠΏΠΎΠ»ΡŒΠ·ΠΎΠ²Π°Ρ‚ΡŒ канадскиС Π²ΠΎΠ΄ΠΈΡ‚Π΅Π»ΡŒΡΠΊΠΈΠ΅ ΠΏΡ€Π°Π²Π°, Π½ΠΎ Ρ‚ΠΎΠ»ΡŒΠΊΠΎ Π² КанадС
  • ΠžΡ‡ΠΊΠΈ ΠΈΠ»ΠΈ ΠΊΠΎΠ½Ρ‚Π°ΠΊΡ‚Π½Ρ‹Π΅ Π»ΠΈΠ½Π·Ρ‹, Ссли Π²Ρ‹ ΠΈΡ… носитС
  • Π»ΡŽΠ±Ρ‹Π΅ мСдицинскиС ΠΎΡ‚Ρ‡Π΅Ρ‚Ρ‹ ΠΈΠ»ΠΈ Ρ€Π΅Π·ΡƒΠ»ΡŒΡ‚Π°Ρ‚Ρ‹ Π°Π½Π°Π»ΠΈΠ·ΠΎΠ², ΠΊΠΎΡ‚ΠΎΡ€Ρ‹Π΅ Ρƒ вас Π΅ΡΡ‚ΡŒ Π½Π° ΠΊΠ°ΠΊΠΈΠ΅-Π»ΠΈΠ±ΠΎ ΠΏΡ€Π΅Π΄Ρ‹Π΄ΡƒΡ‰ΠΈΠ΅ ΠΈΠ»ΠΈ ΡΡƒΡ‰Π΅ΡΡ‚Π²ΡƒΡŽΡ‰ΠΈΠ΅ мСдицинскиС условия
  • список Π²Π°ΡˆΠΈΡ… Ρ‚Π΅ΠΊΡƒΡ‰ΠΈΡ… лСкарств
  • Π€ΠΎΡ€ΠΌΠ° мСдицинского Π·Π°ΠΊΠ»ΡŽΡ‡Π΅Π½ΠΈΡ (IMM 1017E), Ссли Π²Ρ‹ Π½Π΅ ΠΏΡ€ΠΎΡˆΠ»ΠΈ ΠΏΡ€Π΅Π΄Π²Π°Ρ€ΠΈΡ‚Π΅Π»ΡŒΠ½Ρ‹ΠΉ мСдицинский осмотр
    • ΠœΡ‹ ΠΎΡ‚ΠΏΡ€Π°Π²ΠΈΠΌ Π²Π°ΠΌ эту Ρ„ΠΎΡ€ΠΌΡƒ

Π’Ρ‹ Ρ‚Π°ΠΊΠΆΠ΅ Π΄ΠΎΠ»ΠΆΠ½Ρ‹ принСсти 4 Π½Π΅Π΄Π°Π²Π½ΠΈΠ΅ Ρ„ΠΎΡ‚ΠΎΠ³Ρ€Π°Ρ„ΠΈΠΈ , Ссли Π²Ρ€Π°Ρ‡ Π³Ρ€ΡƒΠΏΠΏΡ‹ Π½Π΅ ΠΈΡΠΏΠΎΠ»ΡŒΠ·ΡƒΠ΅Ρ‚ eMedical .Π‘Π²ΡΠΆΠΈΡ‚Π΅ΡΡŒ с Π»Π΅Ρ‡Π°Ρ‰ΠΈΠΌ Π²Ρ€Π°Ρ‡ΠΎΠΌ ΠΏΠ΅Ρ€Π΅Π΄ Π½Π°Π·Π½Π°Ρ‡Π΅Π½ΠΈΠ΅ΠΌ, Ρ‡Ρ‚ΠΎΠ±Ρ‹ ΡƒΠ·Π½Π°Ρ‚ΡŒ.

Если вас направят Π½Π° Ρ€Π΅Π½Ρ‚Π³Π΅Π½ ΠΈΠ»ΠΈ Π΄Ρ€ΡƒΠ³ΠΈΠ΅ тСсты, вас ΠΌΠΎΠ³ΡƒΡ‚ ΠΏΠΎΠΏΡ€ΠΎΡΠΈΡ‚ΡŒ снова ΠΏΡ€Π΅Π΄ΡŠΡΠ²ΠΈΡ‚ΡŒ удостовСрСниС личности , ΠΊΠΎΠ³Π΄Π° Π²Ρ‹ ΠΏΠΎΠΉΠ΄Π΅Ρ‚Π΅ Π½Π° эти тСсты.

Бколько Π½ΡƒΠΆΠ½ΠΎ ΠΏΠ»Π°Ρ‚ΠΈΡ‚ΡŒ Π·Π°

Π’Ρ‹ Π΄ΠΎΠ»ΠΆΠ½Ρ‹ ΠΎΠΏΠ»Π°Ρ‚ΠΈΡ‚ΡŒ всС сборы, связанныС с мСдицинским осмотром, ΠΊΠΎΠ³Π΄Π° Π²Ρ‹ Ρ‚Π°ΠΌ Π½Π°Ρ…ΠΎΠ΄ΠΈΡ‚Π΅ΡΡŒ, Π² Ρ‚ΠΎΠΌ числС:

  • ΠΏΠ»Π°Ρ‚Π° Π·Π° Π²Ρ€Π°Ρ‡Π° ΠΈΠ»ΠΈ Ρ€Π°Π΄ΠΈΠΎΠ»ΠΎΠ³Π°
  • Π»ΡŽΠ±Ρ‹Π΅ ΡΠΏΠ΅Ρ†ΠΈΠ°Π»ΡŒΠ½Ρ‹Π΅ тСсты, исслСдования ΠΈΠ»ΠΈ Π»Π΅Ρ‡Π΅Π½ΠΈΠ΅, Π½Π΅ΠΎΠ±Ρ…ΠΎΠ΄ΠΈΠΌΡ‹Π΅
  • Π»ΡŽΠ±Ρ‹Π΅ спСциалисты, ΠΊΠΎΡ‚ΠΎΡ€Ρ‹Π΅ Π²Π°ΠΌ Π½ΡƒΠΆΠ½Ρ‹, Ρ‡Ρ‚ΠΎΠ±Ρ‹ ΡƒΠ²ΠΈΠ΄Π΅Ρ‚ΡŒ

Если ΠΌΡ‹ ΠΎΡ‚ΠΊΠ»ΠΎΠ½ΠΈΠΌ Π²Π°ΡˆΡƒ заявку послС вашСго мСдицинского осмотра, ΠΌΡ‹ Π½Π΅ Π²Π΅Ρ€Π½Π΅ΠΌ Π²Π°ΠΌ эти сборы.

Π‘Π΅ΠΆΠ΅Π½Ρ†Ρ‹ ΠΈ соискатСли ΡƒΠ±Π΅ΠΆΠΈΡ‰Π° освобоТдСны ΠΎΡ‚ ΡƒΠΏΠ»Π°Ρ‚Ρ‹ пошлин.

Π§Π΅Π³ΠΎ ΠΎΠΆΠΈΠ΄Π°Ρ‚ΡŒ Π²ΠΎ врСмя экзамСна

Волько ΡƒΡ‚Π²Π΅Ρ€ΠΆΠ΄Π΅Π½Π½Ρ‹ΠΉ Π²Ρ€Π°Ρ‡ Π³Ρ€ΡƒΠΏΠΏΡ‹ ΠΌΠΎΠΆΠ΅Ρ‚ ΠΏΡ€ΠΎΠΉΡ‚ΠΈ ΠΏΠΎΠ»Π½Ρ‹ΠΉ мСдицинский осмотр ΠΏΠΎ ΠΈΠΌΠΌΠΈΠ³Ρ€Π°Ρ†ΠΈΠΎΠ½Π½Ρ‹ΠΌ ΠΏΡ€ΠΈΡ‡ΠΈΠ½Π°ΠΌ.

Когда Π²Ρ‹ ΠΏΡ€ΠΈΠ΅Π΄Π΅Ρ‚Π΅

Π’Ρ€Π°Ρ‡ Panel ΠΈΠ»ΠΈ пСрсонал ΠΊΠ»ΠΈΠ½ΠΈΠΊΠΈ попросят вас ΠΏΡ€Π΅Π΄ΡŠΡΠ²ΠΈΡ‚ΡŒ удостовСрСниС личности, Ρ‡Ρ‚ΠΎΠ±Ρ‹ ΠΏΠΎΠ΄Ρ‚Π²Π΅Ρ€Π΄ΠΈΡ‚ΡŒ Π²Π°ΡˆΡƒ Π»ΠΈΡ‡Π½ΠΎΡΡ‚ΡŒ. Если вас направят Π½Π° Ρ€Π΅Π½Ρ‚Π³Π΅Π½ ΠΈΠ»ΠΈ Π΄Ρ€ΡƒΠ³ΠΈΠ΅ Π°Π½Π°Π»ΠΈΠ·Ρ‹, вас ΠΌΠΎΠ³ΡƒΡ‚ ΠΏΠΎΠΏΡ€ΠΎΡΠΈΡ‚ΡŒ снова ΠΏΡ€Π΅Π΄ΡŠΡΠ²ΠΈΡ‚ΡŒ удостовСрСниС личности, ΠΊΠΎΠ³Π΄Π° Π²Ρ‹ ΠΏΠΎΠΉΠ΄Π΅Ρ‚Π΅ Π½Π° эти Π°Π½Π°Π»ΠΈΠ·Ρ‹. Π’Π°ΡˆΠ° фотография Ρ‚Π°ΠΊΠΆΠ΅ Π±ΡƒΠ΄Π΅Ρ‚ сдСлана для Π½Π°ΡˆΠΈΡ… записСй.

АнкСта ΠΏΠΎ истории Π±ΠΎΠ»Π΅Π·Π½ΠΈ

Π’Ρ€Π°Ρ‡ Π·Π°ΠΏΠΎΠ»Π½ΠΈΡ‚ с Π²Π°ΠΌΠΈ Π°Π½ΠΊΠ΅Ρ‚Ρƒ ΠΈΠ· истории Π±ΠΎΠ»Π΅Π·Π½ΠΈ. Π­Ρ‚Π° Π°Π½ΠΊΠ΅Ρ‚Π° касаСтся Π»ΡŽΠ±Ρ‹Ρ… ΠΏΡ€Π΅Π΄Ρ‹Π΄ΡƒΡ‰ΠΈΡ… ΠΈΠ»ΠΈ ΡΡƒΡ‰Π΅ΡΡ‚Π²ΡƒΡŽΡ‰ΠΈΡ… Π·Π°Π±ΠΎΠ»Π΅Π²Π°Π½ΠΈΠΉ. Они Ρ‚Π°ΠΊΠΆΠ΅ спросят вас, ΠΊΠ°ΠΊΠΈΠ΅ лСкарства Π²Ρ‹ ΠΏΡ€ΠΈΠ½ΠΈΠΌΠ°Π΅Ρ‚Π΅.

Π’Π°ΠΆΠ½ΠΎ ΡΠΎΠΎΠ±Ρ‰ΠΈΡ‚ΡŒ Π»Π΅Ρ‡Π°Ρ‰Π΅ΠΌΡƒ Π²Ρ€Π°Ρ‡Ρƒ ΠΎ Π»ΡŽΠ±Ρ‹Ρ… ΠΏΡ€Π΅Π΄Ρ‹Π΄ΡƒΡ‰ΠΈΡ… ΠΈΠ»ΠΈ ΡΡƒΡ‰Π΅ΡΡ‚Π²ΡƒΡŽΡ‰ΠΈΡ… заболСваниях. Если Π²Ρ‹ этого Π½Π΅ сдСлаСтС, ΠΎΠ±Ρ€Π°Π±ΠΎΡ‚ΠΊΠ° вашСго мСдицинского осмотра ΠΌΠΎΠΆΠ΅Ρ‚ Π·Π°Π½ΡΡ‚ΡŒ большС Π²Ρ€Π΅ΠΌΠ΅Π½ΠΈ.

ΠœΠ΅Π΄ΠΈΡ†ΠΈΠ½ΡΠΊΠΈΠΉ осмотр

Π’Ρ‹ ΠΏΡ€ΠΎΠΉΠ΄Π΅Ρ‚Π΅ мСдицинский осмотр.

Π’Ρ€Π°Ρ‡ ΠΈΠ»ΠΈ мСдицинский пСрсонал ΠΊΠ»ΠΈΠ½ΠΈΠΊΠΈ Π±ΡƒΠ΄Π΅Ρ‚:

  • вСс
  • Π˜Π·ΠΌΠ΅Ρ€ΡŒΡ‚Π΅ свой рост
  • ΠΏΡ€ΠΎΠ²Π΅Ρ€ΡŒΡ‚Π΅ слух ΠΈ Π·Ρ€Π΅Π½ΠΈΠ΅
  • ΠΈΠ·ΠΌΠ΅Ρ€ΠΈΡ‚ΡŒ Π°Ρ€Ρ‚Π΅Ρ€ΠΈΠ°Π»ΡŒΠ½ΠΎΠ΅ Π΄Π°Π²Π»Π΅Π½ΠΈΠ΅
  • почувствуй ΠΏΡƒΠ»ΡŒΡ
  • ΠΏΡ€ΠΈΡΠ»ΡƒΡˆΠΈΠ²Π°ΠΉΡ‚Π΅ΡΡŒ ΠΊ своСму сСрдцу ΠΈ Π»Π΅Π³ΠΊΠΈΠΌ
  • ΠΏΠΎΡ‰ΡƒΠΏΠ°ΠΉ ΠΆΠΈΠ²ΠΎΡ‚
  • ΠΏΡ€ΠΎΠ²Π΅Ρ€ΡŒΡ‚Π΅, ΠΊΠ°ΠΊ Π΄Π²ΠΈΠ³Π°ΡŽΡ‚ΡΡ ваши конСчности
  • посмотри Π½Π° свою ΠΊΠΎΠΆΡƒ

Π’Ρ€Π°Ρ‡ ΠΈΠ»ΠΈ мСдицинский пСрсонал Π½Π΅ Π±ΡƒΠ΄Π΅Ρ‚ ΠΎΠ±ΡΠ»Π΅Π΄ΠΎΠ²Π°Ρ‚ΡŒ ваши Π³Π΅Π½ΠΈΡ‚Π°Π»ΠΈΠΈ ΠΈΠ»ΠΈ Ρ€Π΅ΠΊΡ‚Π°Π»ΡŒΠ½ΡƒΡŽ ΠΎΠ±Π»Π°ΡΡ‚ΡŒ.Π­Ρ‚ΠΈ части Ρ‚Π΅Π»Π° Π½Π΅ Ρ‚Ρ€Π΅Π±ΡƒΡŽΡ‚ΡΡ для ΠΈΠΌΠΌΠΈΠ³Ρ€Π°Ρ†ΠΈΠΎΠ½Π½ΠΎΠ³ΠΎ мСдицинского осмотра.

Π’ΠΎΠ·ΠΌΠΎΠΆΠ½ΠΎ, Π²Ρ€Π°Ρ‡Ρƒ потрСбуСтся ΠΎΡΠΌΠΎΡ‚Ρ€Π΅Ρ‚ΡŒ Π²Π°ΡˆΡƒ Π³Ρ€ΡƒΠ΄ΡŒ. Если ΠΎΠ½ΠΈ это ΡΠ΄Π΅Π»Π°ΡŽΡ‚, ΠΎΠ½ΠΈ Π±ΡƒΠ΄ΡƒΡ‚:

  • ΠΎΠ±ΡŠΡΡΠ½ΡΡ‚, ΠΏΠΎΡ‡Π΅ΠΌΡƒ ΠΈ ΠΊΠ°ΠΊ проводится обслСдованиС

Π”Ρ€ΡƒΠ³ΠΈΠ΅ Π²ΠΎΠ·ΠΌΠΎΠΆΠ½Ρ‹Π΅ испытания

Π’ зависимости ΠΎΡ‚ вашСго возраста вас ΠΌΠΎΠ³ΡƒΡ‚ ΠΏΠΎΠΏΡ€ΠΎΡΠΈΡ‚ΡŒ ΡΠ΄Π΅Π»Π°Ρ‚ΡŒ Ρ€Π΅Π½Ρ‚Π³Π΅Π½ Π³Ρ€ΡƒΠ΄Π½ΠΎΠΉ ΠΊΠ»Π΅Ρ‚ΠΊΠΈ ΠΈ Π»Π°Π±ΠΎΡ€Π°Ρ‚ΠΎΡ€Π½Ρ‹Π΅ Π°Π½Π°Π»ΠΈΠ·Ρ‹ Π² ΠΊΠ»ΠΈΠ½ΠΈΠΊΠ΅ ΠΈΠ»ΠΈ Π»Π°Π±ΠΎΡ€Π°Ρ‚ΠΎΡ€ΠΈΠΈ. Π­Ρ‚ΠΎ ΠΎΠ±Ρ‹Ρ‡Π½Ρ‹ΠΉ осмотр, ΠΈ Π²Ρ€Π°Ρ‡ обсудит с Π²Π°ΠΌΠΈ Π»ΡŽΠ±Ρ‹Π΅ отклонСния ΠΎΡ‚ Π½ΠΎΡ€ΠΌΡ‹.

Π’ зависимости ΠΎΡ‚ Ρ€Π΅Π·ΡƒΠ»ΡŒΡ‚Π°Ρ‚ΠΎΠ² мСдицинского осмотра вас ΠΌΠΎΠ³ΡƒΡ‚ Π½Π°ΠΏΡ€Π°Π²ΠΈΡ‚ΡŒ ΠΊ спСциалисту для провСдСния Π΄ΠΎΠΏΠΎΠ»Π½ΠΈΡ‚Π΅Π»ΡŒΠ½Ρ‹Ρ… Π°Π½Π°Π»ΠΈΠ·ΠΎΠ². ΠœΡ‹ просим вас Π·Π°ΠΏΠΎΠ»Π½ΠΈΡ‚ΡŒ этот запрос ΠΊΠ°ΠΊ ΠΌΠΎΠΆΠ½ΠΎ скорСС, Ρ‡Ρ‚ΠΎΠ±Ρ‹ ΠΈΠ·Π±Π΅ΠΆΠ°Ρ‚ΡŒ Π·Π°Π΄Π΅Ρ€ΠΆΠ΅ΠΊ Π² ΠΎΠ±Ρ€Π°Π±ΠΎΡ‚ΠΊΠ΅ вашСго мСдицинского обслСдования.

ΠŸΡ€Π°Π²ΠΎ ΠΈΠΌΠ΅Ρ‚ΡŒ ΡΠΎΠΏΡ€ΠΎΠ²ΠΎΠΆΠ΄Π°ΡŽΡ‰Π΅Π³ΠΎ

Π£ вас Π΅ΡΡ‚ΡŒ ΠΏΡ€Π°Π²ΠΎ Π½Π° сопровоТдСниС Π² любоС врСмя Π²ΠΎ врСмя мСдицинского осмотра.

Π’Ρ‹ ΠΌΠΎΠΆΠ΅Ρ‚Π΅:

  • ΠΏΠΎΠΏΡ€ΠΎΡΠΈΡ‚ΡŒ Π² ΠΏΠΎΠ»ΠΈΠΊΠ»ΠΈΠ½ΠΈΠΊΠ΅ сотрудника Π² ΠΏΠ°Π»Π°Ρ‚Π΅
  • Π² любой ΠΌΠΎΠΌΠ΅Π½Ρ‚ ΠΏΡ€Π΅ΠΊΡ€Π°Ρ‚ΠΈΡ‚ΡŒ обслСдованиС, Ρ‡Ρ‚ΠΎΠ±Ρ‹ Π·Π°Π΄Π°Ρ‚ΡŒ вопросы ΠΎ Ρ‚ΠΎΠΌ, Ρ‡Ρ‚ΠΎ Π΄Π΅Π»Π°Π΅Ρ‚ Π²Ρ€Π°Ρ‡
  • ΠΏΡ€Π΅ΠΊΡ€Π°Ρ‚ΠΈΡ‚Π΅ экзамСн ΠΈ попроситС ΡΠΎΠΏΡ€ΠΎΠ²ΠΎΠΆΠ΄Π°ΡŽΡ‰Π΅Π³ΠΎ, Π΄Π°ΠΆΠ΅ Ссли Π²Ρ‹ сначала ΠΎΡ‚ΠΊΠ°Π·Π°Π»ΠΈΡΡŒ ΠΎΡ‚ Π½Π΅Π³ΠΎ

Если Ρƒ вас Π΅ΡΡ‚ΡŒ вопросы ΠΈΠ»ΠΈ Π²Ρ‹ чувствуСтС сСбя Π½Π΅ΠΊΠΎΠΌΡ„ΠΎΡ€Ρ‚Π½ΠΎ Π²ΠΎ врСмя части экзамСна, попроситС Π²Ρ€Π°Ρ‡Π° Π³Ρ€ΡƒΠΏΠΏΡ‹ ΠΎΡΡ‚Π°Π½ΠΎΠ²ΠΈΡ‚ΡŒΡΡ ΠΈ Ρ€Π°ΡΡΠΊΠ°Π·Π°Ρ‚ΡŒ Π΅ΠΌΡƒ ΠΎ своих опасСниях.

ПослС сдачи экзамСна

ПослС Ρ‚ΠΎΠ³ΠΎ, ΠΊΠ°ΠΊ обслСдованиС Π±ΡƒΠ΄Π΅Ρ‚ ΠΏΡ€ΠΎΠ²Π΅Π΄Π΅Π½ΠΎ, Π²Ρ€Π°Ρ‡ ΠΎΡ‚ΠΏΡ€Π°Π²ΠΈΡ‚ Π½Π°ΠΌ Ρ€Π΅Π·ΡƒΠ»ΡŒΡ‚Π°Ρ‚Ρ‹. Π’Ρ€Π°Ρ‡ выдаст Π²Π°ΠΌ Π΄ΠΎΠΊΡƒΠΌΠ΅Π½Ρ‚, ΠΏΠΎΠ΄Ρ‚Π²Π΅Ρ€ΠΆΠ΄Π°ΡŽΡ‰ΠΈΠΉ, Ρ‡Ρ‚ΠΎ Π²Ρ‹ ΠΏΡ€ΠΎΡˆΠ»ΠΈ мСдицинскоС обслСдованиС. Π”Π΅Ρ€ΠΆΠΈΡ‚Π΅ это ΠΏΡ€ΠΈ сСбС ΠΊΠ°ΠΊ Π΄ΠΎΠΊΠ°Π·Π°Ρ‚Π΅Π»ΡŒΡΡ‚Π²ΠΎ вашСго мСдицинского осмотра.

Если Π²Ρ‹ Π½Π΅ ΡƒΠ΄ΠΎΠ²Π»Π΅Ρ‚Π²ΠΎΡ€Π΅Π½Ρ‹ Ρ‚Π΅ΠΌ, ΠΊΠ°ΠΊ Ρ‚Π΅Ρ€Π°ΠΏΠ΅Π²Ρ‚ ΠΈΠ»ΠΈ Ρ€Π΅Π½Ρ‚Π³Π΅Π½ΠΎΠ»ΠΎΠ³ ΠΏΡ€ΠΎΠ²Π΅Π» ваш мСдицинский осмотр, Π²Ρ‹ ΠΌΠΎΠΆΠ΅Ρ‚Π΅:

ΠœΡ‹ привСтствуСм всС ΠΊΠΎΠΌΠΏΠ»ΠΈΠΌΠ΅Π½Ρ‚Ρ‹, ΠΊΠΎΠΌΠΌΠ΅Π½Ρ‚Π°Ρ€ΠΈΠΈ ΠΈΠ»ΠΈ замСчания Ρ‡Π΅Ρ€Π΅Π· Π½Π°ΡˆΡƒ Ρ„ΠΎΡ€ΠΌΡƒ ΠΎΠ±Ρ€Π°Ρ‚Π½ΠΎΠΉ связи.

Как ΠΏΡ€ΠΈΠ»ΠΎΠΆΠΈΡ‚ΡŒ Ρ€Π΅Π·ΡƒΠ»ΡŒΡ‚Π°Ρ‚Ρ‹ мСдицинского обслСдования ΠΊ заявлСнию

Если ΠΏΠ΅Ρ€Π΅Π΄ ΠΏΠΎΠ΄Π°Ρ‡Π΅ΠΉ заявлСния Π²Ρ‹ ΠΏΡ€ΠΎΡˆΠ»ΠΈ ΠΏΡ€Π΅Π΄Π²Π°Ρ€ΠΈΡ‚Π΅Π»ΡŒΠ½Ρ‹ΠΉ мСдицинский осмотр, Ρ‚ΠΎ Π²Ρ‹ Π΄ΠΎΠ»ΠΆΠ½Ρ‹ ΠΏΡ€ΠΈΠ»ΠΎΠΆΠΈΡ‚ΡŒ ΠΊ заявлСнию копию Ρ„ΠΎΡ€ΠΌΡ‹ ΠΏΡ€Π΅Π΄Π²Π°Ρ€ΠΈΡ‚Π΅Π»ΡŒΠ½ΠΎΠ³ΠΎ мСдицинского ΠΎΡ‚Ρ‡Π΅Ρ‚Π° IMM 1017B.

Если Π²Ρ€Π°Ρ‡ Ρ€Π°Π±ΠΎΡ‚Π°Π΅Ρ‚ с eMedical, ΠΎΠ½ даст Π²Π°ΠΌ распСчатанный ΠΈΠ½Ρ„ΠΎΡ€ΠΌΠ°Ρ†ΠΈΠΎΠ½Π½Ρ‹ΠΉ лист, ΠΊΠΎΡ‚ΠΎΡ€Ρ‹ΠΉ Π²Ρ‹ смоТСтС ΠΏΡ€ΠΈΠ»ΠΎΠΆΠΈΡ‚ΡŒ ΠΊ Π²Π°ΡˆΠ΅ΠΌΡƒ заявлСнию.

Если послС ΠΏΠΎΠ΄Π°Ρ‡ΠΈ заявлСния Π²Ρ‹ ΠΏΡ€ΠΎΡˆΠ»ΠΈ мСдицинский осмотр, Π²Π°ΠΌ Π½Π΅ Π½ΡƒΠΆΠ½ΠΎ Π½ΠΈΡ‡Π΅Π³ΠΎ ΠΏΡ€ΠΈΡΡ‹Π»Π°Ρ‚ΡŒ Π½Π°ΠΌ.

Как Π΄ΠΎΠ»Π³ΠΎ ΡΠΎΡ…Ρ€Π°Π½ΡΡŽΡ‚ΡΡ ваши мСдицинскиС Ρ€Π΅Π·ΡƒΠ»ΡŒΡ‚Π°Ρ‚Ρ‹

Π Π΅Π·ΡƒΠ»ΡŒΡ‚Π°Ρ‚Ρ‹ вашСго мСдицинского осмотра Π΄Π΅ΠΉΡΡ‚Π²ΠΈΡ‚Π΅Π»ΡŒΠ½Ρ‹ Ρ‚ΠΎΠ»ΡŒΠΊΠΎ Π² Ρ‚Π΅Ρ‡Π΅Π½ΠΈΠ΅ 12 мСсяцСв. Если Π²Ρ‹ Π½Π΅ ΠΏΡ€ΠΈΠ΅Π΄Π΅Ρ‚Π΅ Π² ΠšΠ°Π½Π°Π΄Ρƒ Π² качСствС постоянного ТитСля Π² Ρ‚Π΅Ρ‡Π΅Π½ΠΈΠ΅ этого Π²Ρ€Π΅ΠΌΠ΅Π½ΠΈ, Π²Π°ΠΌ ΠΌΠΎΠΆΠ΅Ρ‚ ΠΏΠΎΡ‚Ρ€Π΅Π±ΠΎΠ²Π°Ρ‚ΡŒΡΡ Π΅Ρ‰Π΅ ΠΎΠ΄ΠΈΠ½ экзамСн.

ΠŸΠΎΠ»ΡƒΡ‡Π΅Π½ΠΈΠ΅ ΠΊΠΎΠΏΠΈΠΈ вашСго мСдицинского осмотра

Если Π²Π°ΠΌ Π½ΡƒΠΆΠ½Π° копия мСдицинского осмотра, спроситС Ρƒ Π²Ρ€Π°Ρ‡Π°, ΠΊΠΎΠ³Π΄Π° Π²Ρ‹ ΠΏΡ€ΠΈΠ΅Π΄Π΅Ρ‚Π΅.

ΠœΠ΅Π΄ΠΈΡ†ΠΈΠ½ΡΠΊΠΈΠ΅ Π·Π°ΠΊΠ»ΡŽΡ‡Π΅Π½ΠΈΡ ΠΈ рСнтгСновскиС снимки для мСдицинского осмотра становятся нашСй ΡΠΎΠ±ΡΡ‚Π²Π΅Π½Π½ΠΎΡΡ‚ΡŒΡŽ. ΠœΡ‹ Π½Π΅ ΠΌΠΎΠΆΠ΅ΠΌ Π²Π΅Ρ€Π½ΡƒΡ‚ΡŒ ΠΈΡ… Π²Π°ΠΌ.

,

Π”ΠΎΠ±Π°Π²ΠΈΡ‚ΡŒ ΠΊΠΎΠΌΠΌΠ΅Π½Ρ‚Π°Ρ€ΠΈΠΉ

Π’Π°Ρˆ адрСс email Π½Π΅ Π±ΡƒΠ΄Π΅Ρ‚ ΠΎΠΏΡƒΠ±Π»ΠΈΠΊΠΎΠ²Π°Π½. ΠžΠ±ΡΠ·Π°Ρ‚Π΅Π»ΡŒΠ½Ρ‹Π΅ поля ΠΏΠΎΠΌΠ΅Ρ‡Π΅Π½Ρ‹ *